Download as pdf or txt
Download as pdf or txt
You are on page 1of 553

Question 1 of 364

 

A 73 year old man is recovering following an emergency Hartmans procedure


performed for an obstructing sigmoid cancer. The pathology report shows a
moderately differentiated adenocarcinoma that invades the muscularis propria, 3
of 15 lymph nodes are involved with metastatic disease. What is the correct stage
for this?
gathered by dr. elbarky.

Astler Coller Stage B2

Dukes stage A

Dukes stage B

Dukes stage C

Dukes stage D

Remember that the term metastasis simply refers to spread and can include
the lymph nodes. In an examination setting marks can be lost by incorrectly
selecting Dukes D (which would be consistent with liver metastasis) rather
than nodal metastasis (Dukes C).

The involvement of lymph nodes makes this Dukes C. In the Astler Coller system
the B and C subsets are split to B1 and B2 and C1 and C2. Where C2 denotes
involvement of the nodes in conjunction with penetration of the muscularis
propria.

Please rate this question:

 Discuss and give feedback

Next question 

Dukes classification

Gives the extent of spread of colorectal cancer


Dukes Tumour confined to the bowel but not extending beyond it, without
A nodal metastasis (95%)

Dukes Tumour invading bowel wall, but without nodal metastasis (75%)
B

Dukes Lymph node metastases (50%)


C

Dukes Distant metastases (6%)(25% if resectable)


D
gathered by dr. elbarky.

5 year survival in brackets

Next question 

Display my notes on this topic

          

Save my notes

Question stats

A 6.9%
B 5.3%
C 13.6%
D 51.1%
E 23.1%

51.1% of users answered this question correctly

Search eMRCS

Search term Go

Question 2 of 364

 

Brown tumours of bone are associated with which of the following?

Hyperthyroidism

Hypothyroidism
gathered by dr. elbarky.

Hyperparathyroidism

Hypoparathyroidism

Osteopetrosis

Brown tumors are tumors of bone that arise in settings of excess osteoclast
activity, such as hyperparathyroidism, and consist of fibrous tissue, woven bone
and supporting vasculature, but no matrix. They are radiolucent on x-ray. The
osteoclasts consume the trabecular bone that osteoblasts lay down and this front
of reparative bone deposition followed by additional resorption can expand beyond
the usual shape of the bone, involving the periosteum thus causing bone pain.
They appear brown because haemosiderin is deposited at the site.

Please rate this question:

 Discuss and give feedback

Next question 

Primary hyperparathyroidism
In exams, primary hyperparathyroidism is stereotypically seen in elderly females
with an unquenchable thirst and an inappropriately normal or raised parathyroid
hormone level. It is most commonly due to a solitary adenoma

Causes of primary hyperparathyroidism


80%: solitary adenoma
15%: hyperplasia
4%: multiple adenoma
1%: carcinoma

Features - 'bones, stones, abdominal groans and psychic moans'


gathered by dr. elbarky.

Polydipsia, polyuria
Peptic ulceration/constipation/pancreatitis
Bone pain/fracture
Renal stones
Depression
Hypertension

Associations
Hypertension
Multiple endocrine neoplasia: MEN I and II

Investigations
Raised calcium, low phosphate
PTH may be raised or normal
Technetium-MIBI subtraction scan

Treatment
Parathyroidectomy, if imaging suggests target gland then a focused
approach may be used

Next question 

Display my notes on this topic

          

Save my notes

Question stats
A 9.2%
B 6.9%
C 53.4%
D 11.8%
E 18.7%

53.4% of users answered this question correctly


gathered by dr. elbarky.

Search eMRCS

Search term Go

 Open MRCS Part A textbook (../review/textbook.php)

External links

+ Suggest a link

Dashboard

10

11

12

Question 4 of 364

 

Which of the following genes is not implicated in the adenoma-carcinoma


sequence in colorectal cancer?

IGF1 gene
gathered by dr. elbarky.

c-myc

APC

p53

K-ras

IGF1 gene mutation is implicated in some HNPCC tumours but not in the
adenoma- carcinoma sequence.

Other genes involved are:

MCC
DCC
c-yes
bcl-2

Please rate this question:

 Discuss and give feedback

Next question 

Colorectal cancer

Annually, about 150,000 new cases are diagnosed and 50,000 deaths from
the disease
About 75% will have sporadic disease and 25% will have a family history
Colorectal tumours comprise a spectrum of disease ranging from
adenomas through to polyp cancers and frank malignancy.
Polyps may be categorised into: neoplastic polyps, adenomatous polyps and
non neoplastic polyps.
The majority of adenomas are polypoidal lesions, although flat lesions do
occur and may prove to be dysplastic.
Non-neoplastic polyps include hyperplastic, juvenile, hamartomatous,
inflammatory, and lymphoid polyps, which have not generally been thought
of as precursors of cancer.
Three characteristics of adenomas that correlate with malignant potential
have been characterised. These include increased size, villous architecture
and dysplasia. For this reason most polyps identified at colonoscopy should
be removed.
The transformation from polyp to cancer is described by the adenoma -
carcinoma sequence and its principles should be appreciated. Essentially
gathered by dr. elbarky.

genetic changes accompany the transition from adenoma to carcinoma; key


changes include APC, c-myc, K RAS mutations and p53 deletions.

Next question 

Display my notes on this topic

          

Save my notes

Question stats

A 43.8%
B 13.3%
C 9.9%
D 20.3%
E 12.7%

43.8% of users answered this question correctly

Search eMRCS

Search term Go

Question 5 of 364

 

A 55 year old man with a long history of achalasia is successfully treated by a


Hellers Cardiomyotomy. Several years later he develops an oesophageal
malignancy. Which of the following lesions is most likely to be present?

Adenocarcinoma
gathered by dr. elbarky.

Gastrointestinal stromal tumour

Leiomyosarcoma

Rhabdomyosarcoma

Squamous cell carcinoma

Achalasia is a rare condition. However, even once treated there is an increased risk
of malignancy. When it does occur it is most likely to be of squamous cell type.

Please rate this question:

 Discuss and give feedback

Next question 

Oesophageal cancer

Incidence is increasing
In most cases in the Western world this increase is accounted for by a rise
in the number of cases of adenocarcinoma. In the UK adenocarcinomas
account for 65% of cases.
Barretts oesophagus is a major risk factor for most cases of oesophageal
adenocarcinoma.
In other regions of the world squamous cancer is more common and is
linked to smoking, alcohol intake, diets rich in nitrosamines and achalasia.
Surveillance of Barretts is important, as it imparts a 30 fold increase in
cancer risk and if invasive malignancy is diagnosed early then survival may
approach 85% at 5 years.
Diagnosis
Upper GI endoscopy is the first line test
Contrast swallow may be of benefit in classifying benign motility disorders
but has no place in the assessment of tumours
Staging is initially undertaken with CT scanning of the chest, abdomen and
pelvis. If overt metastatic disease is identified using this modality then
further complex imaging is unnecessary
If CT does not show metastatic disease, then local stage may be more
accurately assessed by use of endoscopic ultrasound.
Staging laparoscopy is performed to detect occult peritoneal disease. PET
CT is performed in those with negative laparoscopy. Thoracoscopy is not
gathered by dr. elbarky.

routinely performed.

Treatment
Operable disease is best managed by surgical resection. The most standard
procedure is an Ivor- Lewis type oesophagectomy. This procedure involves the
mobilisation of the stomach and division of the oesophageal hiatus. The abdomen
is closed and a right sided thoracotomy performed. The stomach is brought into
the chest and the oesophagus mobilised further. An intrathoracic
oesophagogastric anastomosis is constructed. Alternative surgical strategies
include a transhiatal resection (for distal lesions), a left thoraco-abdominal
resection (difficult access due to thoracic aorta) and a total oesophagectomy
(McKeown) with a cervical oesophagogastric anastomosis.
The biggest surgical challenge is that of anastomotic leak, with an intrathoracic
anastomosis this will result in mediastinitis. With high mortality. The McKeown
technique has an intrinsically lower systemic insult in the event of anastomotic
leakage.

In addition to surgical resection many patients will be treated with adjuvant


chemotherapy.

Next question 

Display my notes on this topic

          

Save my notes

Question stats
A 34.1%
B 8.5%
41.6% of users answered this question correctly
C 9.3%
D 6.5%
E 41.6%
Search eMRCS

Search term Go
gathered by dr. elbarky.

 Open MRCS Part A textbook (../review/textbook.php)

External links

+ Suggest a link

Dashboard

10

11

12

13

14

15

Question 7 of 364

 

Which of the symptoms below is least typical of pancreatic cancer?

Painless jaundice

Hyperamylasaemia
gathered by dr. elbarky.

Hyperglycaemia

Weight loss

Classical Courvoisier syndrome

Raised serum amylase is relatively uncommon. The typical Courvoisier syndrome


typically occurs in 20% and hyperglycaemia occurs in 15-20%.

Please rate this question:

 Discuss and give feedback

Next question 

Pancreatic cancer

Adenocarcinoma
Risk factors: Smoking, diabetes, adenoma, familial adenomatous polyposis
Mainly occur in the head of the pancreas (70%)
Spread locally and metastasizes to the liver
Carcinoma of the pancreas should be differentiated from other
periampullary tumours with better prognosis

Clinical features
Weight loss
Painless jaundice
Epigastric discomfort (pain usually due to invasion of the coeliac plexus is a
late feature)
Pancreatitis
Trousseau's sign: migratory superficial thrombophlebitis

Investigations
USS: May miss small lesions
CT Scanning (pancreatic protocol). If unresectable on CT then no further
staging needed
PET/CT for those with operable disease on CT alone
ERCP/ MRI for bile duct assessment
Staging laparoscopy to exclude peritoneal disease

Management
gathered by dr. elbarky.

Head of pancreas: Whipple's resection (SE dumping and ulcers). Newer


techniques include pylorus preservation and SMA/ SMV resection
Carcinoma body and tail: poor prognosis, distal pancreatectomy, if operable
Usually adjuvent chemotherapy for resectable disease
ERCP and stent for jaundice and palliation
Surgical bypass may be needed for duodenal obstruction

Next question 

Display my notes on this topic

          

Save my notes

Question stats

A 8.3%
B 40%
C 27.8%
D 6.3%
E 17.5%

40% of users answered this question correctly


Question 8 of 364

 

Which virus is associated with Kaposi's sarcoma?

Human herpes virus 8

Human papillomavirus 16
gathered by dr. elbarky.

Human T-lymphotropic virus 1

Epstein-Barr virus

Human papillomavirus 18

Please rate this question:

 Discuss and give feedback

Next question 

Oncoviruses

Viruses which cause cancer


These may be detected on blood test and prevented by vaccine

These are the main types of oncoviruses and their diseases:

Oncovirus Cancer

Epstein-Barr virus Burkitt's lymphoma


Hodgkin's lymphoma
Post transplant lymphoma
Nasopharyngeal carcinoma
Human papillomavirus 16/18 Cervical cancer
Anal cancer
Penile cancer
Vulval cancer
Oropharyneal cancer

Human herpes virus 8 Kaposi's sarcoma

Hepatitis B virus Hepatocellular carcinoma

Hepatitis C virus Hepatocellular carcinoma


gathered by dr. elbarky.

Human T-lymphotropic virus 1 Tropical spastic paraparesis


Adult T cell leukaemia

Next question 

Display my notes on this topic

          

Save my notes

Question stats

A 44.1%
B 13.3%
C 18.4%
D 15.9%
E 8.3%

44.1% of users answered this question correctly

Search eMRCS

Search term Go

Question 10 of 364

 

A 22 year female who is 24 weeks pregnant presents with brisk frank haematuria.
She is sexually active. She has had a previous pregnancy resulting in caesarean
section. What is the most likely cause?

Renal vein thrombosis


gathered by dr. elbarky.

Placenta percreta

Placenta praevia

Membranous glomerulonephritis

Endometriosis

Pregnancy and frank haematuria, especially if there is a history of placenta previa


or prior caesarean section, should indicate this diagnosis. There is invasive
placental implantation into the myometrium, which can rarely extend into the
bladder causing severe bleeding.

Please rate this question:

 Discuss and give feedback

Next question 

Haematuria

Causes of haematuria

Trauma Injury to renal tract


Renal trauma commonly due to blunt injury
(others penetrating injuries)
Ureter trauma rare: iatrogenic
Bladder trauma: due to RTA or pelvic fractures
Infection Remember TB

Malignancy Renal cell carcinoma (remember paraneoplastic


syndromes): painful or painless
Urothelial malignancies: 90% are transitional cell
carcinoma, can occur anywhere along the urinary
tract. Painless haematuria.
Squamous cell carcinoma and adenocarcinoma:
rare bladder tumours
Prostate cancer
Penile cancers: SCC
gathered by dr. elbarky.

Renal disease Glomerulonephritis

Stones Microscopic haematuria common

Structural Benign prostatic hyperplasia (BPH) causes


abnormalities haematuria due to hypervascularity of the
prostate gland
Cystic renal lesions e.g. polycystic kidney disease
Vascular malformations
Renal vein thrombosis due to renal cell carcinoma

Coagulopathy Causes bleeding of underlying lesions

Drugs Cause tubular necrosis or interstitial nephritis:


aminoglycosides, chemotherapy
Interstitial nephritis: penicillin, sulphonamides,
and NSAIDs
Anticoagulants

Benign Exercise

Gynaecological Endometriosis: flank pain, dysuria, and


haematuria that is cyclical

Iatrogenic Catheterisation
Radiotherapy; cystitis, severe haemorrhage,
bladder necrosis

Pseudohaematuria For example following consumption of beetroot

References
Http://bestpractice.bmj.com/best-practice/monograph/316/overview
/aetiology.html

Next question 

Display my notes on this topic

          
gathered by dr. elbarky.

Save my notes

Question stats

A 11.4%
B 42.8%
C 24.4%
D 12.6%
E 8.8%

42.8% of users answered this question correctly

Search eMRCS

Search term Go

 Open MRCS Part A textbook (../review/textbook.php)

External links

+ Suggest a link

Dashboard

Question 11 of 364

 

A 78 year old man presents with unilateral deafness which has been present for
the past 3 months. On examination, Webers test localises to the contralateral side
and a CT scan of his head shows a thickened calvarium with areas of sclerosis
and radiolucency. His blood tests show an elevated alkaline phosphatase, normal
serum calcium and normal PTH levels. Which of the following is the most likely
gathered by dr. elbarky.

underlying diagnosis?

Multiple myeloma with skull involvement

Osteoporosis

Pagets disease with skull involvement

Lung cancer with skull metastasis

Osteopetrosis with skull involvement

Of the conditions listed, Pagets disease is the most likely diagnosis (skull vault
expansion and sensorineural hearing loss). Multiple myeloma would typically
result in multiple areas of radiolucency and usually raised calcium in this setting.
Osteopetrosis is a recognised cause of the features described. However, it is a rare
inherited disorder and usually presents in children in young adults. Presentation at
this stage with no prior symptoms would be extremely rare and therefore this is
not the most likely diagnosis.

Please rate this question:

 Discuss and give feedback

Next question 

Pagets disease

Paget's disease is a disease of increased but uncontrolled bone turnover and is


characterised by architecturally abnormal bones. It is thought to be primarily a
disorder of osteoclasts, with excessive osteoclastic resorption followed by
increased osteoblastic activity causing areas of sclerosis and deformity. Paget's
disease is common (UK prevalence 5%) but symptomatic in only 1 in 20 patients

Predisposing factors
increasing age
male sex
northern latitude
family history

Clinical features
bone pain (e.g. pelvis, lumbar spine, femur)
classical, untreated features: bowing of tibia, bossing of skull
gathered by dr. elbarky.

raised alkaline phosphatase (ALP) - calcium* and phosphate are typically


normal
skull x-ray: thickened vault, osteoporosis circumscripta

Indications for treatment include bone pain, skull or long bone deformity, fracture,
periarticular Paget's
bisphosphonate (either oral risedronate or IV zoledronate)
calcitonin is less commonly used now

Complications
deafness (cranial nerve entrapment)
bone sarcoma (1% if affected for > 10 years)
fractures
skull thickening
high-output cardiac failure

*usually normal in this condition but hypercalcaemia may occur with prolonged
immobilisation

Next question 

Display my notes on this topic

          

Save my notes

Question stats
A 19.9%
B 7%
50.1% of users answered this question correctly
C 50.1%
D 7.3%
E 15.8%
Search eMRCS

Search term Go
gathered by dr. elbarky.

 Open MRCS Part A textbook (../review/textbook.php)

External links

+ Suggest a link

Dashboard

10

11

12

13

14

15

Question 3 of 364

 

A 63 year old lady is suspected as having sarcoidosis. She is sent to the general
surgeons and a lymph node biopsy is performed. Which histological feature is
most likely to be identified in a lymph node if sarcoid is present?

Psammoma bodies
gathered by dr. elbarky.

Extensive necrosis

Dense eosinophillic infiltrates

Asteroid bodies

None of the above

Asteroid bodies are often found in the granulomas of individuals with sarcoid.
Unlike the granulomata associated with tuberculosis the granulomas of sarcoid
are rarely associated with extensive necrosis.

An Asteroid body in an individual with sarcoid


gathered by dr. elbarky.

(https://d2zgo9qer4wjf4.cloudfront.net/images_eMRCS/swb094b.jpg)
Image sourced from Wikipedia
(https://d2zgo9qer4wjf4.cloudfront.net
(http://en.wikipedia.org
/images_eMRCS/swb094b.jpg)
/wiki/Granuloma)

Please rate this question:

 Discuss and give feedback

Next question 

Chronic inflammation

Overview
Chronic inflammation may occur secondary to acute inflammation.In most cases
chronic inflammation occurs as a primary process. These may be broadly viewed
as being one of three main processes:
Persisting infection with certain organisms such as Mycobacterium
tuberculosis which results in delayed type hypersensitivity reactions and
inflammation.
Prolonged exposure to non-biodegradable substances such as silica or
suture materials which may induce an inflammatory response.
Autoimmune conditions involving antibodies formed against host antigens.

Acute vs. Chronic inflammation

Acute inflammation Chronic inflammation

Changes to existing vascular structure and Angiogenesis predominates


increased permeability of endothelial cells

Infiltration of neutrophils Macrophages, plasma cells


gathered by dr. elbarky.

and lymphocytes predominate

Process may resolve with: Healing by fibrosis is the main


Suppuration result
Complete resolution
Abscess formation
Progression to chronic inflammation
Healing by fibrosis

Granulomatous inflammation
A granuloma consists of a microscopic aggregation of macrophages (with
epithelial type arrangement =epithelioid). Large giant cells may be found at the
periphery of granulomas.

Mediators
Growth factors released by activated macrophages include agents such as
interferon and fibroblast growth factor (plus many more). Some of these such as
interferons may have systemic features resulting in systemic symptoms and signs,
which may be present in individuals with long standing chronic inflammation.

The finding of granulomas is pathognomonic of chronic inflammation, as illustrated


in this biopsy from a patient with colonic Crohns disease
gathered by dr. elbarky.

(https://d2zgo9qer4wjf4.cloudfront.net/images_eMRCS/swb093b.jpg)
Image sourced from Wikipedia
(https://d2zgo9qer4wjf4.cloudfront.net
(http://en.wikipedia.org/wiki/Crohn%27s
/images_eMRCS/swb093b.jpg)
disease)

Next question 

Display my notes on this topic

          

Save my notes

Question stats

A 17.5%
B 9.2%
C 19.6%
D 43.8%
E 9.8%

43.8% of users answered this question correctly

Search eMRCS

Question 6 of 364

 

What is the most common cause of hypercalcaemia in the UK in hospitalised


patients?

Thiazide use
gathered by dr. elbarky.

Metastatic malignancy

Primary hyperparathyroidism

Osteogenic sarcoma

Sarcoidosis

Metastatic cancer accounts for most cases of hypercalcaemia in hospitalised


patients. In the community primary hyperparathyroidism is the commonest cause.

Please rate this question:

 Discuss and give feedback

Next question 

Hypercalcaemia

Main causes
Malignancy (most common cause in hospital in-patients)
Primary hyperparathyroidism (commonest cause in non hospitalised
patients)

Less common
Sarcoidosis (extrarenal synthesis of calcitriol )
Thiazides, lithium
Immobilisation
Pagets disease
Vitamin A/D toxicity
Thyrotoxicosis
MEN
Milk alkali syndrome

Clinical features
Stones, bones, abdominal groans, and psychic moans
High serum calcium levels result in decreased neuronal excitability. Therefore
sluggish reflexes, muscle weakness and constipation may occur.

Next question 
gathered by dr. elbarky.

Display my notes on this topic

          

Save my notes

Question stats

A 24.1%
B 44.2%
C 19%
D 6.5%
E 6.2%

44.2% of users answered this question correctly

Search eMRCS

Search term Go

 Open MRCS Part A textbook (../review/textbook.php)

External links

Question 9 of 364

 

A 43 year old man presents with dyspepsia and undergoes an upper GI endoscopy.
During the procedure diffuse gastric and duodenal ulcers are identified. A Clo test
confirms the presence of Helicobacter pylori infection. What is the most likely
explanation for the ulcers?
gathered by dr. elbarky.

Decreased gastric motility

Increased urease activity

Decreased release of mucous and bicarbonate

Decreased gastrin levels

Increased acid production

H-Pylori has a number of pathological effects. In this question the main issue is by
what mechanism the organism is able to induce both gastric and duodenal
ulceration. Without modestly elevated acid levels, the duodenum would not
undergo gastric metaplasia. H-Pylori cannot colonise duodenal mucosa and
therefore the development of ulcers at this site can only occur in those who have
undergone metaplastic transformation (mediated by increased acidity).

Please rate this question:

 Discuss and give feedback

Next question 

Helicobacter Pylori

Infection with Helicobacter Pylori is implicated in many cases of duodenal


ulceration and up to 60% of patients with gastric ulceration.

It is a gram negative, helical shaped rod with microaerophillic requirements. It has


the ability to produce a urease enzyme that will hydrolyse urea resulting in the
production of ammonia. The effect of ammonia on antral G cells is to cause
release of gastrin via a negative feedback loop.
Once infection is established the organism releases enzymes that disrupt the
gastric mucous layer. Certain subtypes release cytotoxins cag A and vac A gene
products. The organism incites a classical chronic inflammatory process of the
gastric epithelium. This accounts for the development of gastric ulcers. The mildly
increased acidity may induce a process of duodenal gastric metaplasia. Whilst
duodenal mucosa cannot be colonised by H-Pylori, mucosa that has undergone
metaplastic change to the gastric epithelial type may be colonised by H- Pylori with
subsequent inflammation and development of duodenitis and ulcers.

In patients who are colonized, there is a 10-20% risk of peptic ulcer, 1-2% risk
gastric cancer and <1% risk MALT lymphoma.
gathered by dr. elbarky.

Next question 

Display my notes on this topic

          

Save my notes

Question stats

A 6.3%
B 32.2%
C 14.8%
D 7%
E 39.7%

39.7% of users answered this question correctly

Search eMRCS

Search term Go

 Open MRCS Part A textbook (../review/textbook.php)


Question 12 of 364

 

A 59 year old lady is referred from the NHS breast screening program. A recent
mammogram is reported as showing linear, branching microcalcification with
coarse granules. Which disease process is the most likely underlying cause of
these appearances?
gathered by dr. elbarky.

Invasive lobular cancer

Lobular carcinoma in situ

Cribriform type ductal carcinoma in situ

Comedo type ductal carcinoma in situ

Fibroadenosis

Comedo type DCIS is usually associated with microcalcifications. Cribriform


lesions are usually multifocal but less likely to form microcalcifications. Lobular
cancers and in situ lesions rarely form microcalcifications and are difficult to
detect using mammography.

Please rate this question:

 Discuss and give feedback

Next question 

Breast cancer - In situ disease

Breast cancer that has yet to invade the basement membrane is referred to as in
situ disease. Both ductal and lobular in situ variants are recognised.

Ductal carcinoma in situ


Sub types include; comedo, cribriform, micropapillary and solid
Comdeo DCIS is most likely to form microcalcifications
Cribriform and micropapillary are most likely to be multifocal
Most lesions are mixed (composed of multiple subtypes)
High nuclear grade DCIS is associated with more malignant characteristics
(loss of p53, increased erbB2 expression)
Local excision of low nuclear grade DCIS will usually produce satisfactory
outcomes.
Multifocal lesions, large and high nuclear grade lesions will usually require
mastectomy
Whole breast irradiation improves locoregional control when breast
conserving surgery is performed

Lobular carcinoma in situ


Much rarer than DCIS
Does not form microcalcifications
gathered by dr. elbarky.

Usually single growth pattern


When an invasive component is found it is less likely to be associated with
axillary nodal metastasis than with invasive foci found within DCIS
Low grade LCIS is usually treated by monitoring rather than excision

Next question 

Display my notes on this topic

          

Save my notes

Question stats

A 15.4%
B 14%
C 22.2%
D 37%
E 11.3%

37% of users answered this question correctly

Search eMRCS

Search term Go

Question 14 of 364

 

Which of the following disorders is associated with massive splenomegaly?

Acute lymphoblastic leukaemia

Acute myeloblastic leukaemia


gathered by dr. elbarky.

Acute myelomonocytic leukaemia

Acute monoblastic leukaemia

Chronic granulocytic leukaemia

Chronic leukaemia is more likely to be associated with splenomegaly than acute


leukaemia.

Please rate this question:

 Discuss and give feedback

Next question 

Spleen

The spleen is located in the left upper quadrant of the abdomen and its size can
vary depending upon the amount of blood it contains. The typical adult spleen is
12.5cm long and 7.5cm wide. The usual weight of the adult spleen is 150g.
The exact position of the spleen can vary with respiratory activity, posture and the
state of surrounding viscera. It usually lies obliquely with its long axis aligned to
the 9th, 10th and 11th ribs. It is separated from these ribs by both diaphragm and
pleural cavity. The normal spleen is not palpable.

The shape of the spleen is influenced by the state of the colon and stomach.
Gastric distension will cause the spleen to resemble the shape of an orange
segment. Colonic distension will cause it to become more tetrahedral.

The spleen is almost entirely covered by peritoneum, which adheres firmly to its
capsule. Recesses of the greater sac separate it from the stomach and kidney. It
develops from the upper dorsal mesogastrium, remaining connected to the
posterior abdominal wall and stomach by two folds of peritoneum; the lienorenal
ligament and gastrosplenic ligament. The lienorenal ligament is derived from
peritoneum where the wall of the general peritoneum meets the omental bursa
between the left kidney and spleen; the splenic vessels lie in its layers. The
gastrosplenic ligament also has two layers, formed by the meeting of the walls of
the greater sac and omental bursa between spleen and stomach, the short gastric
and left gastroepiploic branches of the splenic artery pass in its layers. Laterally,
the spleen is in contact with the phrenicocolic ligament.

Relations
gathered by dr. elbarky.

Superiorly Diaphragm

Anteriorly Gastric impression

Posteriorly Kidney

Inferiorly Colon

Tail of pancreas and splenic vessels (splenic artery divides here, branches pass
Hilum
to the white pulp transporting plasma)

Contents
White Immune function. Contains central trabecular artery. The germinal centres are
pulp supplied by arterioles called penicilliary radicles.

Red
Filters abnormal red blood cells.
pulp

Function
Filtration of abnormal blood cells and foreign bodies such as bacteria.
Immunity: IgM. Production of properdin, and tuftsin which help target fungi
and bacteria for phagocytosis.
Haematopoiesis: up to 5th month gestation or in haematological disorders.
Pooling: storage of 40% platelets.
Iron reutilisation
Storage monocytes

Disorders of the spleen


Massive splenomegaly
Myelofibrosis
Chronic myeloid leukaemia
Visceral leishmaniasis (kala-azar)
Malaria
Gaucher's syndrome

Other causes (as above plus)


Portal hypertension e.g. secondary to cirrhosis
Lymphoproliferative disease e.g. CLL, Hodgkin's
Haemolytic anaemia
Infection: hepatitis, glandular fever
Infective endocarditis
Sickle-cell*, thalassaemia
Rheumatoid arthritis (Felty's syndrome)

*the majority of adult patients with sickle-cell will have an atrophied spleen due to
repeated infarction
gathered by dr. elbarky.

Next question 

Display my notes on this topic

          

Save my notes

Question stats

A 21.1%
B 30.2%
C 7.6%
D 6.4%
E 34.7%

34.7% of users answered this question correctly

Search eMRCS

Search term Go

 Open MRCS Part A textbook (../review/textbook.php)


Question 15 of 364

 

A 43 year old man from Greece presents with colicky right upper quadrant pain,
jaundice and an urticarial rash. He is initially treated with ciprofloxacin, but does
not improve. What is the most likely diagnosis?

Infection with Wucheria bancrofti


gathered by dr. elbarky.

Infection with Echinococcus granulosus

Type III hypersensitivity reaction

Allergy to ciprofloxacin

Common bile duct stones

Infection with Echinococcus granulosus will typically produce a type I


hypersensitivity reaction which is characterised by an urticarial rash. With biliary
rupture a classical triad of biliary colic, jaundice and urticaria occurs. Whilst
jaundice and biliary colic may be a feature of CBD stones they do not produce an
urticarial rash. Antibiotic sensitivity with ciprofloxacin may produce jaundice and a
rash, however it was not present at the outset and does not cause biliary colic.

Please rate this question:

 Discuss and give feedback

Next question 

Hydatid cysts

Hydatid cysts are endemic in Mediterranean and Middle Eastern countries. They
are caused by the tapeworm parasite Echinococcus granulosus. An outer fibrous
capsule is formed containing multiple small daughter cysts. These cysts are
allergens which precipitate a type 1 hypersensitivity reaction.

Clinical features are as follows:


Up to 90% cysts occur in the liver and lungs
Can be asymtomatic, or symptomatic if cysts > 5cm in diameter
Morbidity caused by cyst bursting, infection and organ dysfunction (biliary,
bronchial, renal and cerebrospinal fluid outflow obstruction)
In biliary ruputure there may be the classical triad of; biliary colic, jaundice,
and urticaria

CT is the best investigation to differentiate hydatid cysts from amoebic and


pyogenic cysts.
Surgery is the mainstay of treatment (the cyst walls must not be ruptured during
removal and the contents sterilised first).
gathered by dr. elbarky.

Next question 

Display my notes on this topic

          

Save my notes

Question stats

A 17.4%
B 45%
C 9.1%
D 7.3%
E 21.2%

45% of users answered this question correctly

Search eMRCS

Search term Go

 Open MRCS Part A textbook (../review/textbook.php)

External links

Question 13 of 364

 

A 30 year old male presents with a painless swelling of the testis. Histologically
the stroma has a lymphocytic infiltrate. The most likely diagnosis is :

Differentiated teratoma
gathered by dr. elbarky.

Malignant undifferentiated teratoma

Classical seminoma

Spermatocytic seminoma

Anaplastic seminoma

Seminoma is the commonest type of testicular tumour and is more common in


males aged between 30-40 years. Classical seminoma is the commonest subtype
and histology shows lymphocytic stromal infiltrate. Other subtypes include:
1. Spermatocytic: tumour cells resemble spermatocytes. Excellent prognosis.
2. Anaplastic
3. Syncytiotrophoblast giant cells: β HCG present in cells
A teratoma is more common in males aged 20-30 years.

Please rate this question:

 Discuss and give feedback

Next question 

Testicular disorders

Testicular cancer
Testicular cancer is the most common malignancy in men aged 20-30 years.
Around 95% of cases of testicular cancer are germ-cell tumours. Germ cell
tumours may essentially be divided into:

Tumour
Tumour type Key features markers Pathology
Tumour
Tumour type Key features markers Pathology

Seminoma Commonest AFP usually Sheet like


subtype normal lobular
(50%) HCG elevated patterns of
Average in 10% cells with
age at seminomas substantial
diagnosis = Lactate fibrous
40 dehydrogenase; component.
Even elevated in Fibrous septa
advanced 10-20% contain
gathered by dr. elbarky.

disease seminomas lymphocytic


associated (but also in inclusions
with 5 year many other and
survival of conditions) granulomas
73% may be seen.

Non seminomatous Younger age at AFP elevated in Heterogenous


germ cell tumours presentation up to 70% of texture with
(42%) =20-30 years cases occasional
Teratoma Advanced disease HCG elevated ectopic tissue
Yolk sac tumour carries worse in up to 40% of such as hair
Choriocarcinoma prognosis (48% at cases
Mixed germ cell 5 years) Other markers
tumours (10%) Retroperitoneal rarely helpful
lymph node
dissection may be
needed for
residual disease
after
chemotherapy

Image demonstrating a classical seminoma, these tumours are typically more


uniform than teratomas
gathered by dr. elbarky.

(https://d2zgo9qer4wjf4.cloudfront.net/images_eMRCS/swb090b.jpg)
Image sourced from Wikipedia
(https://d2zgo9qer4wjf4.cloudfront.net
(http://en.wikipedia.org
/images_eMRCS/swb090b.jpg)
/wiki/Seminoma)

Risk factors for testicular cancer


Cryptorchidism
Infertility
Family history
Klinefelter's syndrome
Mumps orchitis

Features
A painless lump is the most common presenting symptom
Pain may also be present in a minority of men
Other possible features include hydrocele, gynaecomastia
Diagnosis
Ultrasound is first-line
CT scanning of the chest/ abdomen and pelvis is used for staging
Tumour markers (see above) should be measured

Management
Orchidectomy (Inguinal approach)
Chemotherapy and radiotherapy may be given depending on staging
Abdominal lesions >1cm following chemotherapy may require
retroperitoneal lymph node dissection.
gathered by dr. elbarky.

Prognosis is generally excellent


5 year survival for seminomas is around 95% if Stage I
5 year survival for teratomas is around 85% if Stage I

Benign disease

Epididymo-orchitis
Acute epididymitis is an acute inflammation of the epididymis, often involving the
testis and usually caused by bacterial infection.
Infection spreads from the urethra or bladder. In men <35 years, gonorrhoea
or chlamydia are the usual infections.
Amiodarone is a recognised non infective cause of epididymitis, which
resolves on stopping the drug.
Tenderness is usually confined to the epididymis, which may facilitate
differentiating it from torsion where pain usually affects the entire testis.

Testicular torsion
Twist of the spermatic cord resulting in testicular ischaemia and necrosis.
Most common in males aged between 10 and 30 (peak incidence 13-15
years)
Pain is usually severe and of sudden onset.
Cremasteric reflex is lost and elevation of the testis does not ease the pain.
Treatment is with surgical exploration. If a torted testis is identified then
both testis should be fixed as the condition of bell clapper testis is often
bilateral.

Hydrocele
Presents as a mass that transilluminates, usually possible to 'get above' it
on examination.
In younger men it should be investigated with USS to exclude tumour.
In children it may occur as a result of a patent processus vaginalis.
Treatment in adults is with a Lords or Jabouley procedure.
Treatment in children is with trans inguinal ligation of PPV.
Next question 

Display my notes on this topic

          

Save my notes
gathered by dr. elbarky.

Question stats

A 13.6%
B 10.7%
C 51.9%
D 13.3%
E 10.5%

51.9% of users answered this question correctly

Search eMRCS

Search term Go

 Open MRCS Part A textbook (../review/textbook.php)

External links

+ Suggest a link

Dashboard

3

Question 13 of 364

 

A 30 year old male presents with a painless swelling of the testis. Histologically
the stroma has a lymphocytic infiltrate. The most likely diagnosis is :

Differentiated teratoma
gathered by dr. elbarky.

Malignant undifferentiated teratoma

Classical seminoma

Spermatocytic seminoma

Anaplastic seminoma

Seminoma is the commonest type of testicular tumour and is more common in


males aged between 30-40 years. Classical seminoma is the commonest subtype
and histology shows lymphocytic stromal infiltrate. Other subtypes include:
1. Spermatocytic: tumour cells resemble spermatocytes. Excellent prognosis.
2. Anaplastic
3. Syncytiotrophoblast giant cells: β HCG present in cells
A teratoma is more common in males aged 20-30 years.

Please rate this question:

 Discuss and give feedback

Next question 

Testicular disorders

Testicular cancer
Testicular cancer is the most common malignancy in men aged 20-30 years.
Around 95% of cases of testicular cancer are germ-cell tumours. Germ cell
tumours may essentially be divided into:

Tumour
Tumour type Key features markers Pathology
Tumour
Tumour type Key features markers Pathology

Seminoma Commonest AFP usually Sheet like


subtype normal lobular
(50%) HCG elevated patterns of
Average in 10% cells with
age at seminomas substantial
diagnosis = Lactate fibrous
40 dehydrogenase; component.
Even elevated in Fibrous septa
advanced 10-20% contain
gathered by dr. elbarky.

disease seminomas lymphocytic


associated (but also in inclusions
with 5 year many other and
survival of conditions) granulomas
73% may be seen.

Non seminomatous Younger age at AFP elevated in Heterogenous


germ cell tumours presentation up to 70% of texture with
(42%) =20-30 years cases occasional
Teratoma Advanced disease HCG elevated ectopic tissue
Yolk sac tumour carries worse in up to 40% of such as hair
Choriocarcinoma prognosis (48% at cases
Mixed germ cell 5 years) Other markers
tumours (10%) Retroperitoneal rarely helpful
lymph node
dissection may be
needed for
residual disease
after
chemotherapy

Image demonstrating a classical seminoma, these tumours are typically more


uniform than teratomas
gathered by dr. elbarky.

(https://d2zgo9qer4wjf4.cloudfront.net/images_eMRCS/swb090b.jpg)
Image sourced from Wikipedia
(https://d2zgo9qer4wjf4.cloudfront.net
(http://en.wikipedia.org
/images_eMRCS/swb090b.jpg)
/wiki/Seminoma)

Risk factors for testicular cancer


Cryptorchidism
Infertility
Family history
Klinefelter's syndrome
Mumps orchitis

Features
A painless lump is the most common presenting symptom
Pain may also be present in a minority of men
Other possible features include hydrocele, gynaecomastia
Diagnosis
Ultrasound is first-line
CT scanning of the chest/ abdomen and pelvis is used for staging
Tumour markers (see above) should be measured

Management
Orchidectomy (Inguinal approach)
Chemotherapy and radiotherapy may be given depending on staging
Abdominal lesions >1cm following chemotherapy may require
retroperitoneal lymph node dissection.
gathered by dr. elbarky.

Prognosis is generally excellent


5 year survival for seminomas is around 95% if Stage I
5 year survival for teratomas is around 85% if Stage I

Benign disease

Epididymo-orchitis
Acute epididymitis is an acute inflammation of the epididymis, often involving the
testis and usually caused by bacterial infection.
Infection spreads from the urethra or bladder. In men <35 years, gonorrhoea
or chlamydia are the usual infections.
Amiodarone is a recognised non infective cause of epididymitis, which
resolves on stopping the drug.
Tenderness is usually confined to the epididymis, which may facilitate
differentiating it from torsion where pain usually affects the entire testis.

Testicular torsion
Twist of the spermatic cord resulting in testicular ischaemia and necrosis.
Most common in males aged between 10 and 30 (peak incidence 13-15
years)
Pain is usually severe and of sudden onset.
Cremasteric reflex is lost and elevation of the testis does not ease the pain.
Treatment is with surgical exploration. If a torted testis is identified then
both testis should be fixed as the condition of bell clapper testis is often
bilateral.

Hydrocele
Presents as a mass that transilluminates, usually possible to 'get above' it
on examination.
In younger men it should be investigated with USS to exclude tumour.
In children it may occur as a result of a patent processus vaginalis.
Treatment in adults is with a Lords or Jabouley procedure.
Treatment in children is with trans inguinal ligation of PPV.
Next question 

Display my notes on this topic

          

Save my notes
gathered by dr. elbarky.

Question stats

A 13.6%
B 10.7%
C 51.9%
D 13.3%
E 10.5%

51.9% of users answered this question correctly

Search eMRCS

Search term Go

 Open MRCS Part A textbook (../review/textbook.php)

External links

+ Suggest a link

Dashboard

3

Question 14 of 364

 

Which of the following disorders is associated with massive splenomegaly?

Acute lymphoblastic leukaemia

Acute myeloblastic leukaemia


gathered by dr. elbarky.

Acute myelomonocytic leukaemia

Acute monoblastic leukaemia

Chronic granulocytic leukaemia

Chronic leukaemia is more likely to be associated with splenomegaly than acute


leukaemia.

Please rate this question:

 Discuss and give feedback

Next question 

Spleen

The spleen is located in the left upper quadrant of the abdomen and its size can
vary depending upon the amount of blood it contains. The typical adult spleen is
12.5cm long and 7.5cm wide. The usual weight of the adult spleen is 150g.
The exact position of the spleen can vary with respiratory activity, posture and the
state of surrounding viscera. It usually lies obliquely with its long axis aligned to
the 9th, 10th and 11th ribs. It is separated from these ribs by both diaphragm and
pleural cavity. The normal spleen is not palpable.

The shape of the spleen is influenced by the state of the colon and stomach.
Gastric distension will cause the spleen to resemble the shape of an orange
segment. Colonic distension will cause it to become more tetrahedral.

The spleen is almost entirely covered by peritoneum, which adheres firmly to its
capsule. Recesses of the greater sac separate it from the stomach and kidney. It
develops from the upper dorsal mesogastrium, remaining connected to the
posterior abdominal wall and stomach by two folds of peritoneum; the lienorenal
ligament and gastrosplenic ligament. The lienorenal ligament is derived from
peritoneum where the wall of the general peritoneum meets the omental bursa
between the left kidney and spleen; the splenic vessels lie in its layers. The
gastrosplenic ligament also has two layers, formed by the meeting of the walls of
the greater sac and omental bursa between spleen and stomach, the short gastric
and left gastroepiploic branches of the splenic artery pass in its layers. Laterally,
the spleen is in contact with the phrenicocolic ligament.

Relations
gathered by dr. elbarky.

Superiorly Diaphragm

Anteriorly Gastric impression

Posteriorly Kidney

Inferiorly Colon

Tail of pancreas and splenic vessels (splenic artery divides here, branches pass
Hilum
to the white pulp transporting plasma)

Contents
White Immune function. Contains central trabecular artery. The germinal centres are
pulp supplied by arterioles called penicilliary radicles.

Red
Filters abnormal red blood cells.
pulp

Function
Filtration of abnormal blood cells and foreign bodies such as bacteria.
Immunity: IgM. Production of properdin, and tuftsin which help target fungi
and bacteria for phagocytosis.
Haematopoiesis: up to 5th month gestation or in haematological disorders.
Pooling: storage of 40% platelets.
Iron reutilisation
Storage monocytes

Disorders of the spleen


Massive splenomegaly
Myelofibrosis
Chronic myeloid leukaemia
Visceral leishmaniasis (kala-azar)
Malaria
Gaucher's syndrome

Other causes (as above plus)


Portal hypertension e.g. secondary to cirrhosis
Lymphoproliferative disease e.g. CLL, Hodgkin's
Haemolytic anaemia
Infection: hepatitis, glandular fever
Infective endocarditis
Sickle-cell*, thalassaemia
Rheumatoid arthritis (Felty's syndrome)

*the majority of adult patients with sickle-cell will have an atrophied spleen due to
repeated infarction
gathered by dr. elbarky.

Next question 

Display my notes on this topic

          

Save my notes

Question stats

A 21.1%
B 30.2%
C 7.6%
D 6.4%
E 34.7%

34.7% of users answered this question correctly

Search eMRCS

Search term Go

 Open MRCS Part A textbook (../review/textbook.php)


Question 15 of 364

 

A 43 year old man from Greece presents with colicky right upper quadrant pain,
jaundice and an urticarial rash. He is initially treated with ciprofloxacin, but does
not improve. What is the most likely diagnosis?

Infection with Wucheria bancrofti


gathered by dr. elbarky.

Infection with Echinococcus granulosus

Type III hypersensitivity reaction

Allergy to ciprofloxacin

Common bile duct stones

Infection with Echinococcus granulosus will typically produce a type I


hypersensitivity reaction which is characterised by an urticarial rash. With biliary
rupture a classical triad of biliary colic, jaundice and urticaria occurs. Whilst
jaundice and biliary colic may be a feature of CBD stones they do not produce an
urticarial rash. Antibiotic sensitivity with ciprofloxacin may produce jaundice and a
rash, however it was not present at the outset and does not cause biliary colic.

Please rate this question:

 Discuss and give feedback

Next question 

Hydatid cysts

Hydatid cysts are endemic in Mediterranean and Middle Eastern countries. They
are caused by the tapeworm parasite Echinococcus granulosus. An outer fibrous
capsule is formed containing multiple small daughter cysts. These cysts are
allergens which precipitate a type 1 hypersensitivity reaction.

Clinical features are as follows:


Up to 90% cysts occur in the liver and lungs
Can be asymtomatic, or symptomatic if cysts > 5cm in diameter
Morbidity caused by cyst bursting, infection and organ dysfunction (biliary,
bronchial, renal and cerebrospinal fluid outflow obstruction)
In biliary ruputure there may be the classical triad of; biliary colic, jaundice,
and urticaria

CT is the best investigation to differentiate hydatid cysts from amoebic and


pyogenic cysts.
Surgery is the mainstay of treatment (the cyst walls must not be ruptured during
removal and the contents sterilised first).
gathered by dr. elbarky.

Next question 

Display my notes on this topic

          

Save my notes

Question stats

A 17.4%
B 45%
C 9.1%
D 7.3%
E 21.2%

45% of users answered this question correctly

Search eMRCS

Search term Go

 Open MRCS Part A textbook (../review/textbook.php)

External links

Question 16 of 364

 

A 60-year-old man presents with lower urinary tract symptoms and is offered a
PSA test. Which one of the following could interfere with the PSA level?

Vigorous exercise in the past 48 hours


gathered by dr. elbarky.

Poorly controlled diabetes mellitus

Drinking more than 4 units of alcohol in the past 48 hours

Smoking

Recent cholecystectomy

Please rate this question:

 Discuss and give feedback

Next question 

PSA testing

Prostate specific antigen (PSA) is a serine protease enzyme produced by normal


and malignant prostate epithelial cells. It has become an important tumour marker
but much controversy still exists regarding its usefulness as a screening tool.

The NHS Prostate Cancer Risk Management Programme (PCRMP) has published
updated guidelines in 2009 on how to handle requests for PSA testing in
asymptomatic men. A recent European trial (ERSPC) showed a statistically
significant reduction in the rate of death prostate cancer by 20% in men aged 55 to
69 years but this was associated with a high risk of over-diagnosis and over-
treatment. Having reviewed this and other data the National Screening Committee
have decided not to introduce a prostate cancer screening programme yet but
rather allow men to make an informed choice.

Age-adjusted upper limits for PSA were recommended by the PCRMP*:


{Age} {PSA level (ng/ml)}

50-59 years 3.0

60-69 years 4.0

> 70 years 5.0

PSA levels may also be raised by**:


benign prostatic hyperplasia (BPH)
gathered by dr. elbarky.

prostatitis and urinary tract infection (NICE recommend to postpone the


PSA test for at least 1 month after treatment)
ejaculation (ideally not in the previous 48 hours)
vigorous exercise (ideally not in the previous 48 hours)
urinary retention
instrumentation of the urinary tract

Poor specificity and sensitivity


around 33% of men with a PSA of 4-10 ng/ml will be found to have prostate
cancer. With a PSA of 10-20 ng/ml this rises to 60% of men
around 20% with prostate cancer have a normal PSA
various methods are used to try and add greater meaning to a PSA level
including age-adjusted upper limits and monitoring change in PSA level with
time (PSA velocity or PSA doubling time)

*aide memoire for upper PSA limit: (age - 20) / 10

**whether digital rectal examination actually causes a rise in PSA levels is a matter
of debate

Next question 

Display my notes on this topic

          

Save my notes

Question stats
A 59.9%
B 8.5%
59.9% of users answered this question correctly
C 12.7%
D 10.3%
E 8.6%
Search eMRCS

Search term Go
gathered by dr. elbarky.

 Open MRCS Part A textbook (../review/textbook.php)

External links

+ Suggest a link

Dashboard

10

11

12

13

14

15

Question 17 of 364

 

A 32 year old woman presents with an episode of haemoptysis and is found to


have metastatic tumour present within the parenchyma of the lungs. This is
biopsied and subsequent histology shows clear cells. What is the most likely
primary site?
gathered by dr. elbarky.

Kidney

Breast

Liver

Adrenal

Bone

Clear cell tumours are a sub type of renal cell cancer it is associated with specific
genetic changes localised to chromosome 3.

Please rate this question:

 Discuss and give feedback

Next question 

Renal lesions

Lesion Disease specific features Treatment


Lesion Disease specific features Treatment

Renal cell Most present with Usually radical or partial


carcinoma haematuria (50%) nephrectomy
Common renal tumour
(85% cases)
Paraneoplastic
features include
hypertension and
polycythaemia
Most commonly has
haematogenous
gathered by dr. elbarky.

mestastasis

Nephroblastoma Rare childhood tumour Surgical resection


It accounts for 80% of combined with
all genitourinary chemotherapy (usually
malignancies in those vincristine, actinomycin D
under the age of 15 and doxorubicin)
years
Up to 90% will have a
mass
50% will be
hypertensive
Diagnostic work up
includes ultrasound
and CT scanning

Neuroblastoma Most common Surgical resection,


extracranial tumour of radiotherapy and
childhood chemotherapy
80% occur in those
under 4 years of age
Tumour of neural crest
origin (up to 50% occur
in the adrenal gland)
The tumour is usually
calcified and may be
diagnosed using MIBG
scanning
Staging is with CT
Lesion Disease specific features Treatment

Transitional cell Accounts for 90% of Radical


carcinoma lower urinary tract nephroureterectomy
tumours, but only 10%
of renal tumours

Males affected 3x
more than females
Occupational exposure
to industrial dyes and
rubber chemicals may
gathered by dr. elbarky.

increase risk
Up to 80% present with
painless haematuria
Diagnosis and staging
is with CT IVU

Angiomyolipoma 80% of these 50% of patients with


hamartoma type lesions >4cm will have
lesions occur symptoms and will require
sporadically, the surgical resection
remainder are seen in
those with tuberous
sclerosis
Tumour is composed
of blood vessels,
smooth muscle and fat
Massive bleeding may
occur in 10% of cases

Next question 

Display my notes on this topic

          

Save my notes

Question stats
A 54.3%
B 18.4%
54.3% of users answered this question correctly
C 6.4%
D 11.1%
E 9.7%
Search eMRCS

Search term Go
gathered by dr. elbarky.

 Open MRCS Part A textbook (../review/textbook.php)

External links

+ Suggest a link

Dashboard

10

11

12

13

14

15

Question 18 of 364

 

A 38 year old lady presents with right upper quadrant pain and nausea. She is
otherwise well and her only medical therapy is the oral contraceptive pill which she
has taken for many years with no ill effects. Her liver function tests are normal. An
ultrasound examination demonstrates a hyperechoic well defined lesion in the left
lobe of the liver which measures 14 cm in diameter. What is the most probable
gathered by dr. elbarky.

underlying cause?

Liver cell adenoma

Cavernous haemangioma

Mesenchymal hamartoma

Cystadenoma

Cholangiocarcinoma

It is unusual for liver adenomas to reach very large sizes. It is the large size of
this lesion that makes a haemangioma the most likely answer.

Cavernous haemangioma often presents with vague symptoms and signs. They
may grow to considerable size. Liver function tests are usually normal. The lesions
are typically well defined and hyperechoic on ultrasound. A causative link between
OCP use and haemangiomata has yet to be established, but is possible.

Please rate this question:

 Discuss and give feedback

Next question 

Benign liver lesions

Benign liver lesions


Haemangioma Most common benign tumours of mesenchymal
origin
Incidence in autopsy series is 8%
Cavernous haemangiomas may be enormous
Clinically they are reddish purple hypervascular
lesions
Lesions are normally separated from normal liver by
ring of fibrous tissue
On ultrasound they are typically hyperechoic

Liver cell 90% develop in women in their third to fifth decade


gathered by dr. elbarky.

adenoma Linked to use of oral contraceptive pill


Lesions are usually solitary
They are usually sharply demarcated from normal
liver although they usually lack a fibrous capsule
On ultrasound the appearances are of mixed echoity
and heterogeneous texture. On CT most lesions are
hypodense when imaged prior to administration of
IV contrast agents
In patients with haemorrhage or symptoms removal
of the adenoma may be required

Mesenchymal Congential and benign, usually present in infants. May


hamartomas compress normal liver

Liver abscess Biliary sepsis is a major predisposing factor


Structures drained by the portal venous system form
the second largest source
Common symptoms include fever, right upper
quadrant pain. Jaundice may be seen in 50%
Ultrasound will usually show a fluid filled cavity,
hyperechoic walls may be seen in chronic abscesses

Amoebic Liver abscess is the most common extra intestinal


abscess manifestation of amoebiasis
Between 75 and 90% lesions occur in the right lobe
Presenting complaints typically include fever and
right upper quadrant pain
Ultrasonography will usually show a fluid filled
structure with poorly defined boundaries
Aspiration yield sterile odourless fluid which has an
anchovy paste consistency
Treatment is with metronidazole
Hyatid cysts Seen in cases of Echinococcus infection
Typically an intense fibrotic reaction occurs around
sites of infection
The cyst has no epithelial lining
Cysts are commonly unilocular and may grow to
20cm in size. The cyst wall is thick and has an
external laminated hilar membrane and an internal
enucleated germinal layer
Typically presents with malaise and right upper
quadrant pain. Secondary bacterial infection occurs
in 10%.
gathered by dr. elbarky.

Liver function tests are usually abnormal and


eosinophilia is present in 33% cases
Ultrasound may show septa and hyatid sand or
daughter cysts.
Percutaneous aspiration was previously contra
indicated, it is not incorporated into some treatment
regimens
Treatment is by sterilisation of the cyst with
mebendazole and may be followed by surgical
resection. Hypertonic swabs are packed around the
cysts during surgery

Polycystic liver Usually occurs in association with polycystic kidney


disease disease
Autosomal dominant disorder
Symptoms may occur as a result of capsular stretch

Cystadenoma Rare lesions with malignant potential


Usually solitary multiloculated lesions
Liver function tests usually normal
Ultrasonography typically shows a large anechoic,
fluid filled area with irregular margins. Internal echos
may result from septa
Surgical resection is indicated in all cases

Next question 

Display my notes on this topic

          
Save my notes

Question stats

A 41.5%
B 32.4%
C 8.3%
D 12.1%
gathered by dr. elbarky.

E 5.7%

32.4% of users answered this question correctly

Search eMRCS

Search term Go

 Open MRCS Part A textbook (../review/textbook.php)

External links

+ Suggest a link

Dashboard

8

Question 19 of 364

 

A 48 year old women presents with recurrent loin pain and fevers. Investigation
reveals a staghorn calculus of the left kidney. Infection with which of the following
organisms is most likely?

Staphylococcus saprophyticus
gathered by dr. elbarky.

Proteus mirabilis

Klebsiella

E-Coli

Staphylococcus epidermidis

Infection with Proteus mirabilis accounts for 90% of all proteus infections. It has a
urease producing enzyme. This will tend to favor urinary alkalinisation which is a
relative prerequisite for the formation of staghorn calculi.

Please rate this question:

 Discuss and give feedback

Next question 

Renal stones

Type of Percentage of
stones Features all calculi
Type of Percentage of
stones Features all calculi

Calcium Hypercalciuria is a major risk factor (various 85%


oxalate causes)
Hyperoxaluria may also increase risk
Hypocitraturia increases risk because citrate
forms complexes with calcium making it more
soluble
Stones are radio-opaque (though less than
calcium phosphate stones)
Hyperuricosuria may cause uric acid stones to
gathered by dr. elbarky.

which calcium oxalate binds

Cystine Inherited recessive disorder of 1%


transmembrane cystine transport leading to
decreased absorption of cystine from
intestine and renal tubule
Multiple stones may form
Relatively radiodense because they contain
sulphur

Uric acid Uric acid is a product of purine metabolism 5-10%


May precipitate when urinary pH low
May be caused by diseases with extensive
tissue breakdown e.g. malignancy
More common in children with inborn errors of
metabolism
Radiolucent

Calcium May occur in renal tubular acidosis, high 10%


phosphate urinary pH increases supersaturation of urine
with calcium and phosphate
Renal tubular acidosis types 1 and 3 increase
risk of stone formation (types 2 and 4 do not)
Radio-dense stones (composition similar to
bone)

Struvite Stones formed from magnesium, ammonium 2-20%


and phosphate
Occur as a result of urease producing bacteria
(and are thus associated with chronic
infections)
Under the alkaline conditions produced, the
crystals can precipitate
Slightly radio-opaque
Effect of urinary pH on stone formation
Urine pH will show individual variation (from pH 5-7). Post prandially the pH falls as
purine metabolism will produce uric acid. Then the urine becomes more alkaline
(alkaline tide). When the stone is not available for analysis the pH of urine may
help to determine which stone was present.

Stone type Urine acidity Mean urine pH

Calcium phosphate Normal- alkaline >5.5

Calcium oxalate Variable 6


gathered by dr. elbarky.

Uric acid Acid 5.5

Struvate Alkaline >7.2

Cystine Normal 6.5

Next question 

Display my notes on this topic

          

Save my notes

Question stats

A 7.9%
B 55.5%
C 9.7%
D 20%
E 6.9%

55.5% of users answered this question correctly

Search eMRCS

Question 20 of 364

 

A 52 year old lady presents with an episode of nipple discharge. It is usually clear
in nature. On examination, the discharge is seen to originate from a single duct and
although it appears clear, when the discharge is tested with a labstix it is shown to
contain blood. Imaging and examination shows no obvious mass lesion. Which of
the following lesions is most likely?
gathered by dr. elbarky.

Duct ectasia

DCIS

Intraductal papilloma

Fat necrosis

Breast abscess

Intraductal papilloma usually cause single duct discharge. The fluid is often clear,
although it may be blood stained. If the fluid is tested with a labstix (little point in
routine practice) then it will usually contain small amounts of blood. A
microdochectomy may be performed.

Please rate this question:

 Discuss and give feedback

Next question 

Non malignant breast disease

Duct ectasia
Mammary duct ectasia may be seen in up to 25% of normal female breasts
Patients usually present with nipple discharge, which may be from single or
multiple ducts (usually present age >50 years)
The discharge is often thick and green
Duct ectasia is a normal variant of breast involution and is not the same
condition as periductal mastitis
Periductal mastitis
Present at younger age than duct ectasia
May present with features of inflammation, abscess or mammary duct
fistula
Strongly associated with smoking
Usually treated with antibiotics, abscess will require drainage

Intraductal papilloma
Growth of papilloma in a single duct
Usually presents with clear or blood stained discharge originating from a
gathered by dr. elbarky.

single duct
No increase in risk of malignancy

Breast abscess
Lactational mastitis is common
Infection is usually with Staphylococcus aureus
On examination there is usually a tender fluctuant mass
Treatment is with antibiotics and ultrasound guided aspiration
Overlying skin necrosis is an indication for surgical debridement, which may
be complicated by the development of a subsequent mammary duct fistula.

Tuberculosis
Rare in western countries, usually secondary TB
Affects women later in child bearing period
Chronic breast or axillary sinus is present in up to 50% cases
Diagnosis is by biopsy culture and histology

Next question 

Display my notes on this topic

          

Save my notes

Question stats

A 26%
B 15.4%
C 47.6%
D 5.8%
E 5.1%

47.6% of users answered this question correctly

Search eMRCS
gathered by dr. elbarky.

Search term Go

 Open MRCS Part A textbook (../review/textbook.php)

External links

+ Suggest a link

Dashboard

10

11

12

13

Question 21 of 364

 

Causes of primary chronic inflammation do not include which of the following?

Sarcoidosis

Tuberculosis
gathered by dr. elbarky.

Ulcerative colitis

Hip prostheses

Chronic cholecystitis

Chronic cholecystitis is caused by recurrent episodes of acute inflammation.


Prosthetic implants may be the site of primary chronic inflammation. A common
example clinically is breast implants which may become encapsulated. The
subsequent fibrosis then results in distortion and may be painful. Hip implants
may be affected in a similar fashion but this is less common.

Please rate this question:

 Discuss and give feedback

Next question 

Chronic inflammation

Overview
Chronic inflammation may occur secondary to acute inflammation.In most cases
chronic inflammation occurs as a primary process. These may be broadly viewed
as being one of three main processes:
Persisting infection with certain organisms such as Mycobacterium
tuberculosis which results in delayed type hypersensitivity reactions and
inflammation.
Prolonged exposure to non-biodegradable substances such as silica or
suture materials which may induce an inflammatory response.
Autoimmune conditions involving antibodies formed against host antigens.
Acute vs. Chronic inflammation

Acute inflammation Chronic inflammation

Changes to existing vascular structure and Angiogenesis predominates


increased permeability of endothelial cells

Infiltration of neutrophils Macrophages, plasma cells


and lymphocytes predominate

Process may resolve with: Healing by fibrosis is the main


Suppuration result
gathered by dr. elbarky.

Complete resolution
Abscess formation
Progression to chronic inflammation
Healing by fibrosis

Granulomatous inflammation
A granuloma consists of a microscopic aggregation of macrophages (with
epithelial type arrangement =epithelioid). Large giant cells may be found at the
periphery of granulomas.

Mediators
Growth factors released by activated macrophages include agents such as
interferon and fibroblast growth factor (plus many more). Some of these such as
interferons may have systemic features resulting in systemic symptoms and signs,
which may be present in individuals with long standing chronic inflammation.

The finding of granulomas is pathognomonic of chronic inflammation, as illustrated


in this biopsy from a patient with colonic Crohns disease

(https://d2zgo9qer4wjf4.cloudfront.net/images_eMRCS/swb093b.jpg)
Image sourced from Wikipedia
(https://d2zgo9qer4wjf4.cloudfront.net
(http://en.wikipedia.org/wiki/Crohn%27s
/images_eMRCS/swb093b.jpg)
disease)

Next question 

Display my notes on this topic

          
gathered by dr. elbarky.

Save my notes

Question stats

A 9.1%
B 9.4%
C 9.1%
D 39%
E 33.3%

33.3% of users answered this question correctly

Search eMRCS

Search term Go

 Open MRCS Part A textbook (../review/textbook.php)

External links

+ Suggest a link

Dashboard

Question 22 of 364

 

A 20 year old man develops acute appendicitis, his appendix is removed and he
makes a full recovery. Which of the following pathological processes is least likely
to be present in the acutely inflamed tissues?

Altered Starlings forces


gathered by dr. elbarky.

Sequestration of neutrophils

Formation of fluid exudate

Formation of granulomas

None of the above

Neutrophil polymorphs=Acute inflammation.


Granuloma = Chronic inflammation.

Acute inflammation:
3 phases
1. Changes in blood vessel and flow: flush, flare, wheal
2. Fluid exudates (rich in protein i.e. Ig, coagulation factors) produced via
increased vascular permeability
3. Cellular exudates mainly containing neutrophil polymorphs pass into
extravascular space.

Neutrophils are then transported to tissues via:

a. Margination of neutrophils to the peripheral plasmatic of the vessel rather than


the central axial stream
b. Pavementing: Adhesion of neutrophils to endothelial cells in venules at site of
acute inflammation
c. Emigration: neutrophils pass between endothelial cells into the tissue

Please rate this question:


 Discuss and give feedback

Next question 

Acute inflammation

Inflammation is the reaction of the tissue elements to injury. Vascular changes


occur, resulting in the generation of a protein rich exudate. So long as the injury
does not totally destroy the existing tissue architecture, the episode may resolve
with restoration of original tissue architecture.
gathered by dr. elbarky.

Vascular changes
Vasodilation occurs and persists throughout the inflammatory phase.
Inflammatory cells exit the circulation at the site of injury.
The equilibrium that balances Starlings forces within capillary beds is
disrupted and a protein rich exudate will form as the vessel walls also
become more permeable to proteins.
The high fibrinogen content of the fluid may form a fibrin clot. This has
several important immunomodulatory functions.

Sequelae

Resolution Typically occurs with minimal initial injury


Stimulus removed and normal tissue
architecture results

Organisation Delayed removal of exudate


Tissues undergo organisation and usually
fibrosis

Suppuration Typically formation of an abscess or an


empyema
Sequestration of large quantities of dead
neutrophils

Progression to chronic Coupled inflammatory and reparative


inflammation activities
Usually occurs when initial infection or
suppuration has been inadequately managed

Causes
Infections e.g. Viruses, exotoxins or endotoxins released by bacteria
Chemical agents
Physical agents e.g. Trauma
Hypersensitivity reactions
Tissue necrosis

Presence of neutrophil polymorphs is a histological diagnostic feature of acute


inflammation

Next question 
gathered by dr. elbarky.

Display my notes on this topic

          

Save my notes

Question stats

A 15.2%
B 11.7%
C 7.3%
D 58.6%
E 7.2%

58.6% of users answered this question correctly

Search eMRCS

Search term Go

 Open MRCS Part A textbook (../review/textbook.php)

External links

+ Suggest a link
360

361

362

363

364
gathered by dr. elbarky.

All contents of this site are © 2016 E-Medical Revision Ltd - Terms and Back to top
Conditions (../help/termsandconditions.php) Privacy policy (../help
/privacypolicy.php)

Question 23 of 364

 

A 24 year old man presents with symptoms of malaise, weight loss and
lymphadenopathy. A lymph node biopsy is performed and the subsequent
histology report states that there is evidence of granuloma formation and central
necrosis. What is the most likely underlying cause?
gathered by dr. elbarky.

Non Hodgkins lymphoma

Churg Strauss syndrome

Epstein Barr Virus infection

Rheumatoid nodule

Infection with Mycobacterium tuberculosis

These histological features are typically seen in TB. Necrosis occurring in


granulomas is usually indicative of an underlying infective cause. Churg Strauss
syndrome is a form of vasculitis, which is the usual histological finding.
Granulomas are reported in the condition, but it is rare for them to demonstrate
necrosis.

Please rate this question:

 Discuss and give feedback

Next question 

Tuberculosis pathology

Is a form of primary chronic inflammation, caused by the inability of


macrophages to kill the Mycobacterium tuberculosis.
The macrophages often migrate to regional lymph nodes, the lung lesion
plus affected lymph nodes is referred to as a Ghon complex.
This leads to the formation of a granuloma which is a collection of
epithelioid histiocytes.
There is the presence of caseous necrosis in the centre.
The inflammatory response is mediated by a type 4 hypersensitivity
reaction.
In healthy individuals the disease may be contained, in the
immunocompromised disseminated (miliary TB) may occur.

Diagnosis
Waxy membrane of mycobacteria prevents binding with normal stains. Ziehl
- Neelsen staining is typically used.
Culture based methods take far longer.

Image showing acid- alcohol fast mycobacteria stained using the Ziehl- Neelsen
method
gathered by dr. elbarky.

(https://d2zgo9qer4wjf4.cloudfront.net/images_eMRCS/swb107b.jpg)
Image sourced from Wikipedia
(https://d2zgo9qer4wjf4.cloudfront.net
(http://en.wikipedia.org/wiki/Ziehl%E2%80
/images_eMRCS/swb107b.jpg)
%93Neelsen stain)

Next question 

Display my notes on this topic

          

Save my notes

Question stats

A 11.1%
B 6.8%
C 8.5%
D 6%
E 67.7%

67.7% of users answered this question correctly

Search eMRCS

Search term Go
gathered by dr. elbarky.

 Open MRCS Part A textbook (../review/textbook.php)

External links

+ Suggest a link

Dashboard

10

11

12

13

14

Question 24 of 364

 

A male infant is born prematurely at 34 weeks gestation by emergency cesarean


section. He initially appears to be stable. However, over the ensuing 24 hours he
develops worsening neurological function. Which of the following processes is
most likely to have occurred?
gathered by dr. elbarky.

Extra dural haemorrhage

Sub dural haemorrhage

Sub arachnoid haemorrhage

Intraventricular haemorrhage

Arteriovenous malformation

Premature neonates= Intra ventricular haemorrhage


Non accidental injury= Sub dural bleed

Please rate this question:

 Discuss and give feedback

Next question 

Intraventricular haemorrhage

Intraventricular haemorrhage is a haemorrhage that occurs into the ventricular


system of the brain. It is relatively rare in adult surgical practice and when it does
occur, it is typically associated with severe head injuries. In premature neonates it
may occur spontaneously. The blood may clot and occlude CSF flow,
hydrocephalus may result.
In neonatal practice the vast majority of IVH occur in the first 72 hours after birth,
the aetiology is not well understood and it is suggested to occur as a result of birth
trauma combined with cellular hypoxia, together the with the delicate neonatal
CNS.

Treatment
Is largely supportive, therapies such as intraventricular thrombolysis and
prophylactic CSF drainage have been trialled and not demonstrated to show
benefit. Hydrocephalus and rising ICP is an indication for shunting.

Next question 
gathered by dr. elbarky.

Display my notes on this topic

          

Save my notes

Question stats

A 9.2%
B 9.1%
C 8.1%
D 63.7%
E 10%

63.7% of users answered this question correctly

Search eMRCS

Search term Go

 Open MRCS Part A textbook (../review/textbook.php)


Question 25 of 364

 

A 22 year old lady presents with an episode of renal colic and following
investigation is suspected of suffering from MEN IIa. Which of the following
abnormalities of the parathyroid glands are most often found in this condition?

Hypertrophy
gathered by dr. elbarky.

Hyperplasia

Adenoma

Carcinoma

Metaplasia

MEN IIa
Medullary thyroid cancer
Hyperparathyroidism (usually hyperplasia)
Phaeochromocytoma

In MEN IIa the commonest lesion is medullary thyroid cancer, with regards to the
parathyroid glands the most common lesion is hyperplasia. In MEN I a parathyroid
adenoma is the most common lesion.

Please rate this question:

 Discuss and give feedback

Next question 

Multiple Endocrine Neoplasia

Multiple endocrine neoplasia (MEN) is inherited as an autosomal dominant


disorder.

The table below summarises the three main types of MEN:


MEN type I MEN type IIa MEN type IIb

Mnemonic 'three P's': Phaeochromocytoma Same as MEN IIa


Medullary thyroid with addition of:
Parathyroid (95%): Parathyroid cancer (70%) Marfanoid body
adenoma Hyperparathyroidism habitus
Pituitary (70%): (60%) Mucosal
Prolactinoma/ACTH/Growth neuromas
Hormone secreting adenoma
Pancreas (50%): Islet cell
tumours/Zollinger Ellison
gathered by dr. elbarky.

syndrome

also: Adrenal (adenoma) and


thyroid (adenoma)

MENIN gene (chromosome 11) RET oncogene RET oncogene


(chromosome 10) (chromosome 10)
Most common presentation =
hypercalcaemia

Next question 

Display my notes on this topic

          

Save my notes

Question stats

A 7.7%
B 40.1%
C 27.8%
D 16.2%
E 8.3%

Question 26 of 364

 

A 67 year old lady is suspected of having Pagets disease of the nipple.


Mammography and USS are normal. What is the most appropriate next step in her
management?

Arrange a core biopsy


gathered by dr. elbarky.

Arrange FNAC of the area

Arrange a punch biopsy

Undertake a mastectomy

Arrange for focused radiotherapy

This is a relatively clear indication for a punch biopsy. If cellular atypia is present
on punch biopsy then any in situ malignancy should be considered. FNAC would be
unsuitable.

Please rate this question:

 Discuss and give feedback

Next question 

Tissue sampling

Tissue sampling is an important surgical process. Biopsy modalities vary


according to the site, experience and subsequent planned therapeutic outcome

The modalities comprise:


-Fine needle aspiration cytology
-Core biopsy
-Excision biopsy
-Tru cut biopsy
-Punch biopsy
-Cytological smears
-Endoscopic or laparoscopic biopsy
When the lesion is superficial the decision needs to be taken as to whether
complete excision is desirable or whether excision biopsy is acceptable. In
malignant melanoma for example the need for safe margins will mean that a more
radical surgical approach needs to be adopted after diagnostic confirmation from
excision biopsy than would be the case in basal cell carcinoma. Punch biopsies
are useful in gaining histological diagnosis of unclear skin lesions where excision
biopsy is undesirable such as in establishing whether a skin lesion is vasculitic or
not.

Fine needle aspiration cytology (FNAC) is an operator dependent procedure that


may or may not be image guided and essentially involves passing a needle through
gathered by dr. elbarky.

a lesion whilst suction is applied to a syringe. The material thus obtained is


expressed onto a slide and sent for cytological assessment. This test can be
limited by operator inexperience and also by the lack of histological architectural
information (e.g. Follicular carcinoma of the thyroid). Where a discharge is present
a sample may be sent for cytology although in some sites (e.g. Nipple discharge )
the information gleaned may be meaningless.

Tissue samples may be obtained by both core and tru cut biopsy. A core biopsy is
obtained by use of a spring loaded gun with a needle passing quickly through the
lesion of interest. A tru cut biopsy achieves the same objective but the needle
moved by hand. When performing these techniques image guidance may be
desirable (e.g. In breast lesions). Consideration needs to be given to any planned
surgical resection as it may be necessary to resect the biopsy tract along with the
specimen (e.g. In sarcoma surgery).

Visceral lesions may be accessed percutaneously under image guidance such as


ultrasound guided biopsy of liver metastases. Or under direct vision such as a
colonoscopic biopsy.

Next question 

Display my notes on this topic

          

Save my notes

Question stats

A 25.3%
B 14.9%
C 45.1%
D 8.5%
E 6.3%

45.1% of users answered this question correctly

Search eMRCS
gathered by dr. elbarky.

Search term Go

 Open MRCS Part A textbook (../review/textbook.php)

External links

+ Suggest a link

Dashboard

10

11

12

13

Question 27 of 364

 

A 63 year old male presents with several episodes of haematuria. He suffers from
COPD secondary to long term smoking. What is the most likely underlying cause?

Renal cortical adenoma


gathered by dr. elbarky.

Renal adenocarcinoma

Nephroblastoma

Transitional cell carcinoma of the bladder

Adenocarcinoma of the bladder

TCC is the most common subtype and is strongly linked to smoking. The important
point to note in this question is the term most likely as renal adenocarcinoma may
produce similar symptoms but is less likely.

Please rate this question:

 Discuss and give feedback

Next question 

Bladder cancer

Bladder cancer is the second most common urological cancer. It most commonly
affects males aged between 50 and 80 years of age. Those who are current, or
previous (within 20 years), smokers have a 2-5 fold increased risk of the disease.
Exposure to hydrocarbons such as 2-Naphthylamine increases the risk. Although
rare in the UK, chronic bladder inflammation arising from Schistosomiasis
infection remains a common cause of squamous cell carcinomas, in those
countries where the disease is endemic.

Benign tumours
Benign tumours of the bladder including inverted urothelial papilloma and
nephrogenic adenoma are uncommon.
Bladder malignancies
Transitional cell carcinoma (>90% of cases)
Squamous cell carcinoma ( 1-7% -except in regions affected by
schistosomiasis)
Adenocarcinoma (2%)

Transitional cell carcinomas may arise as solitary lesions, or may be multifocal,


owing to the effect of 'field change' within the urothelium. Up to 70% of TCC's will
have a papillary growth pattern. These tumours are usually superficial in location
and accordingly have a better prognosis. The remaining tumours show either
mixed papillary and solid growth or pure solid growths. These tumours are typically
gathered by dr. elbarky.

more prone to local invasion and may be of higher grade, the prognosis is therefore
worse. Those with T3 disease or worse have a 30% (or higher) risk of regional or
distant lymph node metastasis.

TNM Staging

Stage Description

T0 No evidence of tumour

Ta Non invasive papillary carcinoma

T1 Tumour invades sub epithelial connective tissue

T2a Tumor invades superficial muscularis propria (inner half)

T2b Tumor invades deep muscularis propria (outer half)

T3 Tumour extends to perivesical fat

T4 Tumor invades any of the following: prostatic stroma, seminal vesicles,


uterus, vagina

T4a Invasion of uterus, prostate or bowel

T4b Invasion of pelvic sidewall or abdominal wall

N0 No nodal disease

N1 Single regional lymph node metastasis in the true pelvis (hypogastric,


obturator, external iliac, or presacral lymph node)

N2 Multiple regional lymph node metastasis in the true pelvis (hypogastric,


obturator, external iliac, or presacral lymph node metastasis)

N3 Lymph node metastasis to the common iliac lymph nodes

M0 No distant metastasis
Stage Description

M1 Distant disease

Presentation
Most patients (85%) will present with painless, macroscopic haematuria. In those
patients with incidental microscopic haematuria, up to 10% of females aged over
50 will be found to have a malignancy (once infection excluded).

Staging
gathered by dr. elbarky.

Most will undergo a cystoscopy and biopsies or TURBT, this provides histological
diagnosis and information relating to depth of invasion. Locoregional spread is
best determined using pelvic MRI and distant disease CT scanning. Nodes of
uncertain significance may be investigated using PET CT.

Treatment
Those with superficial lesions may be managed using TURBT in isolation. Those
with recurrences or higher grade/ risk on histology may be offered intravesical
chemotherapy. Those with T2 disease are usually offered either surgery (radical
cystectomy and ileal conduit) or radical radiotherapy.

Prognosis

T1 90%

T2 60%

T3 35%

T4a 10-25%

Any T, N1-N2 30%

Next question 

Display my notes on this topic

          

Save my notes
Question stats

A 5.8%
B 17.8%
C 6.3%
D 59.7%
E 10.4%

59.7% of users answered this question correctly


gathered by dr. elbarky.

Search eMRCS

Search term Go

 Open MRCS Part A textbook (../review/textbook.php)

External links

+ Suggest a link

Dashboard

10

Question 28 of 364

 

Which of the following is least likely to occur in association with severe atrophic
gastritis?

Gastric ulcers
gathered by dr. elbarky.

Gastric cancer

Anaemia

Duodenal ulcers

Gastric polyps

Achlorhydria would make the formation of duodenal ulcers unlikely. Note the
question states 'least likely'.

Due to the loss of gastric acid a duodenal ulcer is unlikely. Note that gastric polyps
may form (see below).

Please rate this question:

 Discuss and give feedback

Next question 

Gastritis

Type of
gastritis Features
Type of
gastritis Features

Type A Autoimmune
Circulating antibodies to parietal cells, causes reduction in
cell mass and hypochlorhydria
Loss of parietal cells = loss of intrinsic factor = B12
malabsorption
Absence of antral involvement
Hypochlorhydria causes elevated gastrin levels- stimulating
enterochromaffin cells and adenomas may form
gathered by dr. elbarky.

Type B Antral gastritis


Associated with infection with helicobacter pylori infection
Intestinal metaplasia may occur in stomach and require
surveillance endoscopy
Peptic ulceration may occur

Reflux Bile refluxes into stomach, either post surgical or due to


gastritis failure of pyloric function
Histologically, evidence of chronic inflammation, and foveolar
hyperplasia
May respond to therapy with prokinetics

Erosive Agents disrupt the gastric mucosal barrier


gastritis Most commonly due to NSAIDs and alcohol
With NSAIDs the effects occur secondary to COX 1 inhibition

Stress This occurs as a result of mucosal ischaemia during


ulceration hypotension or hypovolaemia
The stomach is the most sensitive organ in the GI tract to
ischaemia following hypovolaemia
Diffuse ulceration may occur
Prophylaxis with acid lowering therapy and sucralfate may
minimise complications

Menetriers Gross hypertrophy of the gastric mucosal folds, excessive


disease mucous production and hypochlorhydria
Pre malignant condition

References
Whiting J et al. The long term results of endoscopic surveillance of premalignant
gastric lesions. Gut 2002; 50 :378381.

Dixon M et al. Reflux gastritis: distinct histopathological entity?J Clin Pathol 1986;
39 : 524-530.
Next question 

Display my notes on this topic

          

Save my notes
gathered by dr. elbarky.

Question stats

A 8.9%
B 9.1%
C 9.3%
D 38.3%
E 34.3%

38.3% of users answered this question correctly

Search eMRCS

Search term Go

 Open MRCS Part A textbook (../review/textbook.php)

External links

+ Suggest a link

Dashboard

3

Question 29 of 364

 

A 45 year old man presents with symptoms of urinary colic. In the history he has
suffered from recurrent episodes of frank haematuria over the past week or so. On
examination he has a left loin mass and a varicocele. The most likely diagnosis is:

Renal adenocarcinoma
gathered by dr. elbarky.

Renal cortical adenoma

Squamous cell carcinoma of the renal pelvis

Retroperitoneal fibrosis

Nephroblastoma

Renal adenocarcinoma are the most common renal malignancy and account
for 75% cases.
Patients may develop frank haematuria and have episodes of clot colic.
A Grawitz tumour is an eponymous name for Renal Adenocarcinoma.
May metastasise to bone.

Please rate this question:

 Discuss and give feedback

Next question 

Renal tumours

Renal cell carcinoma


Renal cell carcinoma is an adenocarcinoma of the renal cortex and is believed to
arise from the proximal convoluted tubule. They are usually solid lesions, up to
20% may be multifocal, 20% may be calcified and 20% may have either a cystic
component or be wholly cystic. They are often circumscribed by a pseudocapsule
of compressed normal renal tissue. Spread may occur either by direct extension
into the adrenal gland, renal vein or surrounding fascia. More distant disease
usually occurs via the haematogenous route to lung, bone or brain.
Renal cell carcinoma comprise up to 85% of all renal malignancies. Males are
more commonly affected than females and sporadic tumours typically affect
patients in their sixth decade.
Patients may present with a variety of symptoms including; haematuria (50%), loin
pain (40%), mass (30%) and up to 25% may have symptoms of metastasis.Less
than 10% have the classic triad of haematuria, pain and mass.

Investigation
Many cases will present as haematuria and be discovered during diagnostic work
up. Benign renal tumours are rare, so renal masses should be investigated with
multislice CT scanning. Some units will add an arterial and venous phase to the
scan to demonstrate vascularity and evidence of caval ingrowth.
gathered by dr. elbarky.

CT scanning of the chest and abdomen to detect distant disease should also be
undertaken.

Routine bone scanning is not indicated in the absence of symptoms.

Biopsy should not be performed when a nephrectomy is planned but is mandatory


before any ablative therapies are undertaken.

Assessment of the functioning of the contra lateral kidney.

Management
T1 lesions may be managed by partial nephrectomy and this gives equivalent
oncological results to total radical nephrectomy. Partial nephrectomy may also be
performed when there is inadequate reserve in the remaining kidney.

For T2 lesions and above a radical nephrectomy is standard practice and this may
be performed via a laparoscopic or open approach. Preoperative embolisation is
not indicated nor is resection of uninvolved adrenal glands. During surgery early
venous control is mandatory to avoid shedding of tumour cells into the circulation.

Patients with completely resected disease do not benefit from adjuvant therapy
with either chemotherapy or biological agents. These should not be administered
outside the setting of clinical trials.

Patients with transitional cell cancer will require a nephroureterectomy with


disconnection of the ureter at the bladder.

References
Lungberg B et al. EAU guidelines on renal cell carcinoma: The 2010 update.
European Urology 2010 (58): 398-406.

Next question 
Display my notes on this topic

          

Save my notes

Question stats

A 60.4%
gathered by dr. elbarky.

B 8%
C 16.1%
D 7.6%
E 7.8%

60.4% of users answered this question correctly

Search eMRCS

Search term Go

 Open MRCS Part A textbook (../review/textbook.php)

External links

+ Suggest a link

Dashboard

5

Question 30 of 364

 

Which of the following diseases is not considered a risk factor for gastric cancer?

Polya gastrectomy for antral ulcer

Atrophic gastritis
gathered by dr. elbarky.

Intestinal metaplasia of columnar type at the gastric cardia

Patient with polyp showing medium grade dysplasia

Long term therapy with sucralfate

Although some acid lowering procedures increase the risk of gastric cancer the
use of sucralfate does not, at the present time, seem to increase the risk.

Please rate this question:

 Discuss and give feedback

Next question 

Gastric cancer

Overview
There are 700,000 new cases of gastric cancer worldwide each year. It is most
common in Japan and less common in western countries. It is more common in
men and incidence rises with increasing age. The exact cause of many sporadic
cancer is not known, however, familial cases do occur in HNPCC families. In
addition, smoking and smoked or preserved foods increase the risk. Japanese
migrants retain their increased risk (decreased in subsequent generations). The
distribution of the disease in western countries is changing towards a more
proximal location (perhaps due to rising obesity).

Pathology
There is some evidence of support a stepwise progression of the disease through
intestinal metaplasia progressing to atrophic gastritis and subsequent dysplasia,
through to cancer. The favoured staging system is TNM. The risk of lymph node
involvement is related to size and depth of invasion; early cancers confined to
submucosa have a 20% incidence of lymph node metastasis. Tumours of the
gastro-oesophageal junction are classified as below:

Type True oesophageal cancers and may be associated with Barrett's


1 oesophagus.

Type Carcinoma of the cardia, arising from cardiac type epithelium


2 or short segments with intestinal metaplasia at the oesophagogastric
junction.

Type Sub cardial cancers that spread across the junction. Involve similar
gathered by dr. elbarky.

3 nodal stations to gastric cancer.

Groups for close endoscopic monitoring


Intestinal metaplasia of columnar type
Atrophic gastritis
Low to medium grade dysplasia
Patients who have previously undergone resections for benign peptic ulcer
disease (except highly selective vagotomy).

Referral to endoscopy

Patients of any age with Patients without Worsening dyspepsia


dyspepsia and any of the dyspepsia
following

Chronic gastrointestinal Dysphagia Barretts oesophagus


bleeding

Dysphagia Unexplained Intestinal metaplasia


abdominal pain or
weight loss

Weight loss Vomiting Dysplasia

Iron deficiency anaemia Upper abdominal Atrophic gastritis


mass

Upper abdominal mass Jaundice Patient aged over 55 years


with unexplained or
persistent dyspepsia

Upper GI endoscopy performed for dyspepsia. The addition of dye spraying (as
shown in the bottom right) may facilitate identification of smaller tumours
gathered by dr. elbarky.

(https://d2zgo9qer4wjf4.cloudfront.net/images_eMRCS/swb067b.jpg)
Image sourced from Wikipedia
(https://d2zgo9qer4wjf4.cloudfront.net
(http://en.wikipedia.org/wiki/Gastric
/images_eMRCS/swb067b.jpg)
cancer)

Staging
CT scanning of the chest abdomen and pelvis is the routine first line staging
investigation in most centres.
Laparoscopy to identify occult peritoneal disease
PET CT (particularly for junctional tumours)

Treatment
Proximally sited disease greater than 5-10cm from the OG junction may be
treated by sub total gastrectomy
Total gastrectomy if tumour is <5cm from OG junction
For type 2 junctional tumours (extending into oesophagus)
oesophagogastrectomy is usual
Endoscopic sub mucosal resection may play a role in early gastric cancer
confined to the mucosa and perhaps the sub mucosa (this is debated)
Lymphadenectomy should be performed. A D2 lymphadenectomy is widely
advocated by the Japanese, the survival advantages of extended
lymphadenectomy have been debated. However, the overall
recommendation is that a D2 nodal dissection be undertaken.
Most patients will receive chemotherapy either pre or post operatively.

Prognosis
UK Data

Disease extent Percentage 5 year survival

All RO resections 54%

Early gastric cancer 91%

Stage 1 87%

Stage 2 65%
gathered by dr. elbarky.

Stage 3 18%

Operative procedure

Total Gastrectomy , lymphadenectomy and Roux en Y anastomosis

General anaesthesia
Prophylactic intravenous antibiotics
Incision: Rooftop.
Perform a thorough laparotomy to identify any occult disease.
Mobilise the left lobe of the liver off the diaphragm and place a large pack over it.
Insert a large self retaining retractor e.g. omnitract or Balfour (take time with this,
the set up should be perfect). Pack the small bowel away.
Begin by mobilising the omentum off the transverse colon.
Proceed to detach the short gastric vessels.
Mobilise the pylorus and divide it at least 2cm distally using a linear cutter stapling
device.
Continue the dissection into the lesser sac taking the lesser omentum and left
gastric artery flush at its origin.
The lymph nodes should be removed en bloc with the specimen where possible.
Place 2 stay sutures either side of the distal oesophagus. Ask the anaesthetist to
pull back on the nasogastric tube. Divide the distal oesophagus and remove the
stomach.
The oesphago jejunal anastomosis should be constructed. Identify the DJ flexure
and bring a loop of jejunum up to the oesophagus (to check it will reach). Divide
the jejunum at this point. Bring the divided jejunum either retrocolic or antecolic to
the oesophagus. Anastamose the oesophagus to the jejunum, using either
interrupted 3/0 vicryl or a stapling device. Then create the remainder of the Roux
en Y reconstruction distally.
Place a jejunostomy feeding tube.
Wash out the abdomen and insert drains (usually the anastomosis and duodenal
stump). Help the anaesthetist insert the nasogastric tube (carefully!)
Close the abdomen and skin.
Enteral feeding may commence on the first post-operative day. However, most
surgeons will leave patients on free NG drainage for several days and keep them nil
by mouth.

Next question 

Display my notes on this topic

          
gathered by dr. elbarky.

Save my notes

Question stats

A 15.9%
B 9.7%
C 8.5%
D 7.3%
E 58.6%

58.6% of users answered this question correctly

Search eMRCS

Search term Go

 Open MRCS Part A textbook (../review/textbook.php)

External links

+ Suggest a link

Dashboard

Question 30 of 364

 

Which of the following diseases is not considered a risk factor for gastric cancer?

Polya gastrectomy for antral ulcer

Atrophic gastritis
gathered by dr. elbarky.

Intestinal metaplasia of columnar type at the gastric cardia

Patient with polyp showing medium grade dysplasia

Long term therapy with sucralfate

Although some acid lowering procedures increase the risk of gastric cancer the
use of sucralfate does not, at the present time, seem to increase the risk.

Please rate this question:

 Discuss and give feedback

Next question 

Gastric cancer

Overview
There are 700,000 new cases of gastric cancer worldwide each year. It is most
common in Japan and less common in western countries. It is more common in
men and incidence rises with increasing age. The exact cause of many sporadic
cancer is not known, however, familial cases do occur in HNPCC families. In
addition, smoking and smoked or preserved foods increase the risk. Japanese
migrants retain their increased risk (decreased in subsequent generations). The
distribution of the disease in western countries is changing towards a more
proximal location (perhaps due to rising obesity).

Pathology
There is some evidence of support a stepwise progression of the disease through
intestinal metaplasia progressing to atrophic gastritis and subsequent dysplasia,
through to cancer. The favoured staging system is TNM. The risk of lymph node
involvement is related to size and depth of invasion; early cancers confined to
submucosa have a 20% incidence of lymph node metastasis. Tumours of the
gastro-oesophageal junction are classified as below:

Type True oesophageal cancers and may be associated with Barrett's


1 oesophagus.

Type Carcinoma of the cardia, arising from cardiac type epithelium


2 or short segments with intestinal metaplasia at the oesophagogastric
junction.

Type Sub cardial cancers that spread across the junction. Involve similar
gathered by dr. elbarky.

3 nodal stations to gastric cancer.

Groups for close endoscopic monitoring


Intestinal metaplasia of columnar type
Atrophic gastritis
Low to medium grade dysplasia
Patients who have previously undergone resections for benign peptic ulcer
disease (except highly selective vagotomy).

Referral to endoscopy

Patients of any age with Patients without Worsening dyspepsia


dyspepsia and any of the dyspepsia
following

Chronic gastrointestinal Dysphagia Barretts oesophagus


bleeding

Dysphagia Unexplained Intestinal metaplasia


abdominal pain or
weight loss

Weight loss Vomiting Dysplasia

Iron deficiency anaemia Upper abdominal Atrophic gastritis


mass

Upper abdominal mass Jaundice Patient aged over 55 years


with unexplained or
persistent dyspepsia

Upper GI endoscopy performed for dyspepsia. The addition of dye spraying (as
shown in the bottom right) may facilitate identification of smaller tumours
gathered by dr. elbarky.

(https://d2zgo9qer4wjf4.cloudfront.net/images_eMRCS/swb067b.jpg)
Image sourced from Wikipedia
(https://d2zgo9qer4wjf4.cloudfront.net
(http://en.wikipedia.org/wiki/Gastric
/images_eMRCS/swb067b.jpg)
cancer)

Staging
CT scanning of the chest abdomen and pelvis is the routine first line staging
investigation in most centres.
Laparoscopy to identify occult peritoneal disease
PET CT (particularly for junctional tumours)

Treatment
Proximally sited disease greater than 5-10cm from the OG junction may be
treated by sub total gastrectomy
Total gastrectomy if tumour is <5cm from OG junction
For type 2 junctional tumours (extending into oesophagus)
oesophagogastrectomy is usual
Endoscopic sub mucosal resection may play a role in early gastric cancer
confined to the mucosa and perhaps the sub mucosa (this is debated)
Lymphadenectomy should be performed. A D2 lymphadenectomy is widely
advocated by the Japanese, the survival advantages of extended
lymphadenectomy have been debated. However, the overall
recommendation is that a D2 nodal dissection be undertaken.
Most patients will receive chemotherapy either pre or post operatively.

Prognosis
UK Data

Disease extent Percentage 5 year survival

All RO resections 54%

Early gastric cancer 91%

Stage 1 87%

Stage 2 65%
gathered by dr. elbarky.

Stage 3 18%

Operative procedure

Total Gastrectomy , lymphadenectomy and Roux en Y anastomosis

General anaesthesia
Prophylactic intravenous antibiotics
Incision: Rooftop.
Perform a thorough laparotomy to identify any occult disease.
Mobilise the left lobe of the liver off the diaphragm and place a large pack over it.
Insert a large self retaining retractor e.g. omnitract or Balfour (take time with this,
the set up should be perfect). Pack the small bowel away.
Begin by mobilising the omentum off the transverse colon.
Proceed to detach the short gastric vessels.
Mobilise the pylorus and divide it at least 2cm distally using a linear cutter stapling
device.
Continue the dissection into the lesser sac taking the lesser omentum and left
gastric artery flush at its origin.
The lymph nodes should be removed en bloc with the specimen where possible.
Place 2 stay sutures either side of the distal oesophagus. Ask the anaesthetist to
pull back on the nasogastric tube. Divide the distal oesophagus and remove the
stomach.
The oesphago jejunal anastomosis should be constructed. Identify the DJ flexure
and bring a loop of jejunum up to the oesophagus (to check it will reach). Divide
the jejunum at this point. Bring the divided jejunum either retrocolic or antecolic to
the oesophagus. Anastamose the oesophagus to the jejunum, using either
interrupted 3/0 vicryl or a stapling device. Then create the remainder of the Roux
en Y reconstruction distally.
Place a jejunostomy feeding tube.
Wash out the abdomen and insert drains (usually the anastomosis and duodenal
stump). Help the anaesthetist insert the nasogastric tube (carefully!)
Close the abdomen and skin.
Enteral feeding may commence on the first post-operative day. However, most
surgeons will leave patients on free NG drainage for several days and keep them nil
by mouth.

Next question 

Display my notes on this topic

          
gathered by dr. elbarky.

Save my notes

Question stats

A 15.9%
B 9.7%
C 8.5%
D 7.3%
E 58.6%

58.6% of users answered this question correctly

Search eMRCS

Search term Go

 Open MRCS Part A textbook (../review/textbook.php)

External links

+ Suggest a link

Dashboard

Question 31 of 364

 

A 45-year-old man presents to surgical outpatients with a long history of recurrent


abdominal pain and vomiting. He is noted to have a peripheral motor neuropathy
on examination. What is the most likely diagnosis?

Huntington's disease
gathered by dr. elbarky.

Myeloma

Acute intermittent porphyria

Lawrence-Moon-Biedl syndrome

Friedreich's ataxia

Neurological signs combined with abdominal pain is acute intermittent porphyria


or lead poisoning until proven otherwise.

Please rate this question:

 Discuss and give feedback

Next question 

Acute intermittent porphyria


Acute intermittent porphyria (AIP) is a rare autosomal dominant condition caused
by a defect in porphobilinogen deaminase, an enzyme involved in the biosynthesis
of haem. The results in the toxic accumulation of delta aminolaevulinic acid and
porphobilinogen. It characteristically presents with abdominal and
neuropsychiatric symptoms in 20-40 year olds. AIP is more common in females
(5:1)

Features
abdominal: abdominal pain, vomiting
neurological: motor neuropathy
psychiatric: e.g. depression
hypertension and tachycardia common
gathered by dr. elbarky.

Diagnosis
classically urine turns deep red on standing
raised urinary porphobilinogen (elevated between attacks and to a greater
extent during acute attacks)
assay of red cells for porphobilinogen deaminase
raised serum levels of delta aminolaevulinic acid and porphobilinogen

Next question 

Display my notes on this topic

          

Save my notes

Question stats

A 13.2%
B 9.3%
C 51%
D 13%
E 13.5%

51% of users answered this question correctly


Question 32 of 364

 

Infection with which of the following micro-organisms may result in a clinical


picture resembling achalasia of the oesphagus?

Epstein Barr virus


gathered by dr. elbarky.

Wuchereria Bancrofti

Candida Spp

Trypanosoma Cruzi

Helicobacter Pylori

Infection with Trypanosoma Cruzi may result in destruction of the ganglion cells of
the myenteric plexus, resulting in a clinical picture similar to achalasia.

Please rate this question:

 Discuss and give feedback

Next question 

Trypanosoma Cruzi

Protozoan
Causes Chagas disease
Carried by bugs which infect the skin whilst feeding
Penetrate through open wounds and mucous membranes
Intracellular proliferation
Major infective sites include CNS, intestinal myenteric plexus, spleen, lymph
nodes and cardiac muscle
Chronic disease is irreversible, nifurtimox is used to treat acute infection

Next question 
Display my notes on this topic

          

Save my notes

Question stats
gathered by dr. elbarky.

A 9.4%
B 12.4%
C 22.1%
D 41.8%
E 14.3%

41.8% of users answered this question correctly

Search eMRCS

Search term Go

 Open MRCS Part A textbook (../review/textbook.php)

External links

+ Suggest a link

Dashboard

5

Question 33 of 364

 

A 16 year old boy develops a painful swelling of his distal femur. An osteoblastic
sarcoma is diagnosed. To which of the following sites is this lesion most likely to
metastasise?

Inguinal lymph nodes


gathered by dr. elbarky.

Common iliac lymph nodes

Liver

Brain

Lung

Sarcomas in which Lymphatic Metastasis is seen:

'RACE For MS'

R: Rhabdomyosarcoma
A: Angiosarcoma
C: Clear cell sarcoma
E: Epithelial cell sarcoma

For: Fibrosarcoma

M: Malignant fibrous histiocytoma


S: Synovial cell sarcoma

Or

'SCARE'

Synovial sarcoma
Clear cell sarcoma
Angiosarcoma
Rhabdomyosarcoma
Epithelioid sarcoma
Sarcomas often metastasise via the haematogenous route and the lung is a
common site for sarcoma metastasis. The liver and brain are often spared (at
least initially). A smaller number may develop lymphatic metastasis (see above).

Please rate this question:

 Discuss and give feedback

Next question 
gathered by dr. elbarky.

Sarcomas

Malignant tumours of mesenchymal origin

Types
May be either bone or soft tissue in origin.
Bone sarcoma include:
Osteosarcoma
Ewings sarcoma (although non bony sites recognised)
Chondrosarcoma - originate from Chondrocytes

Soft tissue sarcoma are a far more heterogeneous group and include:
Liposarcoma-adipocytes
Rhabdomyosarcoma-striated muscle
Leiomyosarcoma-smooth muscle
Synovial sarcomas- close to joints (cell of origin not known but not
synovium)

Malignant fibrous histiocytoma is a sarcoma that may arise in both soft tissue and
bone.

Features
Certain features of a mass or swelling should raise suspicion for a sarcoma these
include:
Large >5cm soft tissue mass
Deep tissue location or intra muscular location
Rapid growth
Painful lump

Assessment
Imaging of suspicious masses should utilise a combination of MRI, CT and USS.
Blind biopsy should not be performed prior to imaging and where required should
be done in such a way that the biopsy tract can be subsequently included in any
resection.

Ewings sarcoma
Commoner in males
Incidence of 0.3 / 1, 000, 000
Onset typically between 10 and 20 years of age
Location by femoral diaphysis is commonest site
Histologically it is a small round tumour
Blood borne metastasis is common and chemotherapy is often combined
with surgery
gathered by dr. elbarky.

Osteosarcoma
Mesenchymal cells with osteoblastic differentiation
20% of all primary bone tumours
Incidence of 5 per 1,000,000
Peak age 15-30, commoner in males
Limb preserving surgery may be possible and many patients will receive
chemotherapy

Liposarcoma
Malignancy of adipocytes
Rare, approximately 2.5 per 1,000,000. They are the most common soft
tissue sarcoma
Typically located in deep locations such as retroperitoneum
Affect older age group usually >40 years of age
May be well differentiated and thus slow growing although may undergo de-
differentiation and disease progression
Many tumours will have a pseudocapsule that can misleadingly allow
surgeons to feel that they can 'shell out' these lesions. In reality, tumour may
invade at the edge of the pseudocapsule and result in local recurrence if this
strategy is adopted
Usually resistant to radiotherapy, although this is often used in a palliative
setting

Malignant Fibrous Histiocytoma


Tumour with large number of histiocytes
Also described as undifferentiated pleomorphic sarcoma NOS (i.e. Cell of
origin is not known)
Four major subtypes are recognised: storiform-pleomorphic (70% cases),
myxoid (less aggressive), giant cell and inflammatory
Treatment is usually with surgical resection and adjuvant radiotherapy as
this reduces the likelihood of local recurrence

Next question 
Display my notes on this topic

          

Save my notes

Question stats
gathered by dr. elbarky.

A 17.6%
B 15.2%
C 12.5%
D 10.8%
E 44%

44% of users answered this question correctly

Search eMRCS

Search term Go

 Open MRCS Part A textbook (../review/textbook.php)

External links

+ Suggest a link

Dashboard

5

Question 34 of 364

 

Which of the following best describes the main pathological feature of generalized
osteoporosis?

Reduction in volume of normally mineralized osteoid matrix


gathered by dr. elbarky.

Increase in volume of normally mineralized osteoid matrix

Reduction in volume of abnormally mineralized osteoid matrix

Increase in volume of abnormally mineralized osteoid matrix

Significant increase in bony trabeculae that are excessively mineralised

Osteoporosis is characterized by reduction of normally mineralized osteoid


matrix

Please rate this question:

 Discuss and give feedback

Next question 

Osteoporosis

Osteoporosis is a condition of bone atrophy. It is a lesion in which the volume of


bone tissue per unit volume of anatomical bone is reduced. It may be generalized
(most common) or localized (following pressure or disuse). The hallmark is a
reduction in the amount of osteoid matrix, which, however, remains normally
mineralized. It is therefore distinct from osteomalacia in which there is abundant
osteoid which is poorly calcified. In osteoporosis the bony trabeculae are greatly
thinned which significantly reduces the tensile strength of the bone and renders it
at increased risk of pathological fracture.
It is only visible on plain films when the calcium content is approximately halved.
More subtle changes can be appreciated by use of DEXA scanning.
Treatment
Fractures are treated according to their site and mechanism of injury. Those
deemed to occur as a result of osteoporosis should also be addressed medically
to treat the underlying osteoporosis.
Drugs that can be prescribed to prevent fragility fractures include bisphosphonates
(alendronate, ibandronate, risedronate and zoledronic acid) and non-
bisphosphonates (raloxifene, denosumab, teriparatide, calcitriol and hormone
replacement therapy).
Calcium and vitamin D supplements are also administered.

Individuals at risk of fragility fractures should also be considered for prophylactic


gathered by dr. elbarky.

medical treatment as outlined above.

Next question 

Display my notes on this topic

          

Save my notes

Question stats

A 65.1%
B 6.5%
C 11.3%
D 11.2%
E 5.9%

65.1% of users answered this question correctly

Search eMRCS

Search term Go

Question 35 of 364

 

Which of the following is the most common childhood brain tumour?

Glioblastoma multiforme

Astrocytoma
gathered by dr. elbarky.

Medulloblastoma

Ependymoma

Meningioma

Glioblastoma multiforme is rare in childhood. In contrast, astrocytoma is the


commonest brain tumour in children. Medulloblastoma is no longer the
commonest CNS tumour in children (Cancer research UK)

Please rate this question:

 Discuss and give feedback

Next question 

CNS tumours

60% = Glioma and metastatic disease


20% = Meningioma
10% = Pituitary lesions

In paediatric practice medulloblastomas (neuroectodermal tumours) were the


commonest lesions, astrocytomas now account for the majority.
Tumours arising in right temporal and frontal lobe may reach considerable size
before becoming symptomatic. Whereas tumours in the speech and visual areas
will typically produce early symptoms.

Most paediatric CNS tumours are infratentorial


Most adult CNS tumours are supratentorial

Diagnosis
MRI Scanning provides the best resolution.

Treatment
Usually surgery, even if tumour cannot be completely resected conditions such as
rising ICP can be addressed with tumour debulking and survival and quality of life
prolonged.
Curative surgery can usually be undertaken with lesions such as meningiomas.
gathered by dr. elbarky.

Gliomas have a marked propensity to invade normal brain and resection of these
lesions is nearly always incomplete.

Next question 

Display my notes on this topic

          

Save my notes

Question stats

A 21.1%
B 37.2%
C 25.4%
D 7.2%
E 9.1%

37.2% of users answered this question correctly

Search eMRCS

Search term Go

Question 36 of 364

 

A 35 year old type 1 diabetic presents with difficulty mobilising and back pain
radiating to the thigh. He has a temperature of 39 oC and has pain on extension of
the hip. He is diagnosed with an iliopsoas abscess. Which of the following
statements is false in relation to his diagnosis?
gathered by dr. elbarky.

Staphylococcus aureus is the most likely primary cause

Recurrence occurs in 60% cases

More common in males

Crohn's is the most likely secondary cause

CT guided drainage is preferable first line management

Classical features include: a limp, back pain and fever. Recurrence rates are about
15-20%. In the UK, Staphylococcus is the commonest primary cause, others include
Streptococcus and E.coli. Management is ideally by CT guided drainage.

Please rate this question:

 Discuss and give feedback

Next question 

Iliopsoas abscess

Collection of pus in iliopsoas compartment (iliopsoas and iliacus)


Causes:

Primary

Haematogenous spread of bacteria


Staphylococcus aureus: most common

Secondary
Crohn's (commonest cause in this category)
Diverticulitis, Colorectal cancer
UTI, GU cancers
Vertebral osteomyelitis
Femoral catheter, lithotripsy
Endocarditis

Note the mortality rate can be up to 19-20% in secondary iliopsoas abscesses


compared with 2.4% in primary abscesses.
gathered by dr. elbarky.

Clinical features

Fever
Back/flank pain
Limp
Weight loss

Clinical examination

Patient in the supine position with the knee flexed and the hip mildly
externally rotated

Specific tests to diagnose iliopsoas inflammation:

Place hand proximal to the patient's ipsilateral knee and ask patient to lift thigh
against your hand. This will cause pain due to contraction of the psoas muscle.

Lie the patient on the normal side and hyperextend the affected hip. In
inflammation this should elicit pain as the psoas muscle is stretched.

Investigation

CT is gold standard

Management

Antibiotics
Percutaneous drainage
Surgery is indicated if:

1. Failure of percutaneous drainage


2. Presence of an another intra-abdominal pathology which requires surgery

Surgical approach
The authors technique for draining these collections is given here.
Review the CT scans and plan surgical approach. An extraperitoneal approach is
important.
The collection usually extends inferiorly and can be accessed from an incision at a
level of L4 on the affected side.

GA
Transverse laterally placed incision.
Incise external oblique.
Split the subsequent muscle layers.
As you approach the peritoneum use blunt dissection to pass laterally around it.
Remember the ureter and gonadal veins lie posterior at this level.
gathered by dr. elbarky.

Eventually you will enter the abscess cavity, a large amount of pus is usually
released at this point. Drain the area with suction and washout with saline.
Place a corrugated drain well into the abscess cavity.
If you have made a small skin incision it is reasonable to bring the drain up through
the skin wound. Otherwise place a lateral exit site and close the skin and external
oblique. If you do this ensure that you use interrupted sutures.

Anchor the drain with strong securely tied silk sutures (it is extremely tiresome if it
falls out!)

Reference

Iliopsoas abscesses
I H Mallick, M H Thoufeeq, T P Rajendran
Postgrad Med J 2004;80:459-462

Next question 

Display my notes on this topic

          

Save my notes

Question stats

A 17.3%
B 32.5%
C 10.8%

Question 37 of 364

 

A 34 year old lady undergoes an elective cholecystectomy for attacks of recurrent


cholecystitis due to gallstones. Microscopic assessment of the gallbladder is most
likely to show which of the following?

Dysplasia of the fundus


gathered by dr. elbarky.

Widespread necrosis

Aschoff-Rokitansky sinuses

Metaplasia of the fundus

None of the above

Aschoff-Rokitansky sinuses are the result of hyperplasia and herniation of


epithelial cells through the fibromuscular layer of the gallbladder wall. They may be
macroscopic or microscopic. Ashoff-Rokitansky sinuses may be identified in
cases of chronic cholecystitis and gallstones. Although gallstones may predispose
to the development of gallbladder cancer the actual incidence of dysplasia and
metaplastic change is rare. In the elective setting described above necrosis would
be rare.

Please rate this question:

 Discuss and give feedback

Next question 

Gallbladder

Fibromuscular sac with capacity of 50ml


Columnar epithelium

Relations of the gallbladder

Anterior Liver
Posterior Covered by peritoneum
Transverse colon
1st part of the duodenum

Laterally Right lobe of liver

Medially Quadrate lobe of liver

Arterial supply
Cystic artery (branch of Right hepatic artery)
gathered by dr. elbarky.

Venous drainage
Directly to the liver

Nerve supply
Sympathetic- mid thoracic spinal cord, Parasympathetic- anterior vagal trunk

Common bile duct

Origin Confluence of cystic and common hepatic ducts

Relations at Medially - Hepatic artery


origin Posteriorly- Portal vein

Relations Duodenum - anteriorly


distally Pancreas - medially and laterally
Right renal vein - posteriorly

Arterial supply Branches of hepatic artery and retroduodenal branches of


gastroduodenal artery

Hepatobiliary triangle

Medially Common hepatic duct

Inferiorly Cystic duct

Superiorly Inferior edge of liver

Contents Cystic artery

Relations of the gallbladder


gathered by dr. elbarky.

(https://d2zgo9qer4wjf4.cloudfront.net/images_eMRCS/bbb021b.jpg)
© Image provided by the (https://d2zgo9qer4wjf4.cloudfront.net/images_eMRCS
University of Sheffield /bbb021b.jpg)

Next question 

Display my notes on this topic

          

Save my notes

Question stats

A 14%
B 9.6%
C 48.5%
D 17.6%
E 10.4%

48.5% of users answered this question correctly

Search eMRCS

Search term Go

Question 38 of 364

 

A 56 year old man has undergone a radical nephrectomy. The pathologist bisects
the kidney and identifies a pink fleshy tumour in the renal pelvis. What is the most
likely disease?

Renal cell carcinoma


gathered by dr. elbarky.

Transitional cell carcinoma

Angiomyolipoma

Phaeochromocytoma

Renal adenoma

Most renal tumours are yellow or brown in colour. TCC's are one of the few
tumours to appear pink.

The finding of a TCC in the renal pelvis mandates a nephroureterectomy.

Please rate this question:

 Discuss and give feedback

Next question 

Renal lesions

Lesion Disease specific features Treatment


Lesion Disease specific features Treatment

Renal cell Most present with Usually radical or partial


carcinoma haematuria (50%) nephrectomy
Common renal tumour
(85% cases)
Paraneoplastic
features include
hypertension and
polycythaemia
Most commonly has
haematogenous
gathered by dr. elbarky.

mestastasis

Nephroblastoma Rare childhood tumour Surgical resection


It accounts for 80% of combined with
all genitourinary chemotherapy (usually
malignancies in those vincristine, actinomycin D
under the age of 15 and doxorubicin)
years
Up to 90% will have a
mass
50% will be
hypertensive
Diagnostic work up
includes ultrasound
and CT scanning

Neuroblastoma Most common Surgical resection,


extracranial tumour of radiotherapy and
childhood chemotherapy
80% occur in those
under 4 years of age
Tumour of neural crest
origin (up to 50% occur
in the adrenal gland)
The tumour is usually
calcified and may be
diagnosed using MIBG
scanning
Staging is with CT
Lesion Disease specific features Treatment

Transitional cell Accounts for 90% of Radical


carcinoma lower urinary tract nephroureterectomy
tumours, but only 10%
of renal tumours

Males affected 3x
more than females
Occupational exposure
to industrial dyes and
rubber chemicals may
gathered by dr. elbarky.

increase risk
Up to 80% present with
painless haematuria
Diagnosis and staging
is with CT IVU

Angiomyolipoma 80% of these 50% of patients with


hamartoma type lesions >4cm will have
lesions occur symptoms and will require
sporadically, the surgical resection
remainder are seen in
those with tuberous
sclerosis
Tumour is composed
of blood vessels,
smooth muscle and fat
Massive bleeding may
occur in 10% of cases

Next question 

Display my notes on this topic

          

Save my notes

Question stats
A 24.3%
B 40.6%
40.6% of users answered this question correctly
C 20.9%
D 6.5%
E 7.6%
Search eMRCS

Search term Go
gathered by dr. elbarky.

 Open MRCS Part A textbook (../review/textbook.php)

External links

+ Suggest a link

Dashboard

10

11

12

13

14

15

Question 39 of 364

 

Which of the following is not an oncogene?

ras

myc
gathered by dr. elbarky.

sis

Ki 67

erb-B

Ki 67 is a nuclear proliferation marker (used in immunohistochemistry). Although,


Ki67 positivity is a marker of malignancy, it is not itself, an oncogene.

Please rate this question:

 Discuss and give feedback

Next question 

Oncogenes

Oncogenes are cancer promoting genes that are derived from normal genes
(proto-oncogenes). Proto-oncogenes play an important physiological role in
cellular growth. They are implicated in the development of up to 20% of human
cancers.

Proto-oncogenes may become oncogenes via the following processes:


Mutation (point mutation)
Chromosomal translocation
Increased protein expression

Only one mutated copy of the gene is needed for cancer to occur - a dominant
effect

Classification of oncogenes
Growth factors e.g. Sis
Transcription factors e.g. Myc
Receptor tyrosine kinase e.g. RET
Cytoplasmic tyrosine kinase e.g. Src
Regulatory GTPases e.g. Ras

Tumour suppressor genes


Tumour suppressor genes restrict or repress cellular proliferation in normal cells.
Their inactivation through mutation or germ line incorporation is implicated in
renal, colonic, breast, bladder and many other cancers. One of the best known
tumour suppressor genes is p53. p53 gene offers protection by causing apoptosis
gathered by dr. elbarky.

of damaged cells. Other well known genes include BRCA 1 and 2.

Next question 

Display my notes on this topic

          

Save my notes

Question stats

A 10.7%
B 11.6%
C 18.5%
D 42.3%
E 16.8%

42.3% of users answered this question correctly

Search eMRCS

Search term Go

 Open MRCS Part A textbook (../review/textbook.php)


Question 40 of 364

 

Which of the following statements relating to parathyroid neoplasms is incorrect?

15% of cases are due to parathyroid carcinoma

80% of cases are due to parathyroid adenomas


gathered by dr. elbarky.

Parathyroid adenomas are often encapsulated

10% of parathyroid adenomas develop in ectopically located glands

85% of cases of primary hyperparathyroidism are due to solitary


adenomas

Parathyroid carcinomas account for up to 5% of tumours. Adenomas are often


encapsulated. Lesions that are fibrotic and densely adherent to the gland may be a
carcinoma. 85% cases of primary hyperparathyroidism are due to a single
adenoma and this is the reason some surgeons favour a focused
parathyroidectomy.

Please rate this question:

 Discuss and give feedback

Next question 

Parathyroid glands and disorders of calcium metabolism

Hyperparathyroidism

Clinical
Disease type Hormone profile features Cause
Disease type Hormone profile Clinical features Cause

Primary PTH May be Most cases


hyperparathyroidism (Elevated) asymptomatic if due to
Ca2+ mild solitary
(Elevated) Recurrent adenoma
Phosphate abdominal pain (80%),
(Low) (pancreatitis, multifocal
Urine renal colic) disease
calcium : Changes to occurs in
creatinine emotional or 10-15% and
clearance cognitive state parathyroid
gathered by dr. elbarky.

ratio > 0.01 carcinoma in


1% or less

Secondary PTH May have few Parathyroid


hyperparathyroidism (Elevated) symptoms gland
Ca2+ (Low or Eventually may hyperplasia
normal) develop bone occurs as a
Phosphate disease, osteitis result of low
(Elevated) fibrosa cystica calcium,
Vitamin D and soft tissue almost
levels (Low) calcifications always in a
setting of
chronic renal
failure

Tertiary Ca2+ (Normal Metastatic Occurs as a


hyperparathyroidism or high) calcification result of
PTH Bone pain and / ongoing
(Elevated) or fracture hyperplasia of
Phosphate Nephrolithiasis the
levels Pancreatitis parathyroid
(Decreased glands after
or Normal) correction of
Vitamin D underlying
(Normal or renal disorder,
decreased) hyperplasia of
Alkaline all 4 glands is
phosphatase usually the
(Elevated) cause

Differential diagnoses
It is important to consider the rare but relatively benign condition of benign familial
hypocalciuric hypercalcaemia, caused by an autosomal dominant genetic disorder.
Diagnosis is usually made by genetic testing and concordant biochemistry (urine
calcium : creatinine clearance ratio <0.01-distinguished from primary
hyperparathyroidism).

Treatment

Primary hyperparathyroidism
Indications for surgery
Elevated serum Calcium > 1mg/dL above normal
Hypercalciuria > 400mg/day
Creatinine clearance < 30% compared with normal
Episode of life threatening hypercalcaemia
Nephrolithiasis
gathered by dr. elbarky.

Age < 50 years


Neuromuscular symptoms
Reduction in bone mineral density of the femoral neck, lumbar spine, or
distal radius of more than 2.5 standard deviations below peak bone mass (T
score lower than -2.5)

Secondary hyperparathyroidism
Usually managed with medical therapy.

Indications for surgery in secondary (renal) hyperparathyroidism:


Bone pain
Persistent pruritus
Soft tissue calcifications

Tertiary hyperparathyroidism
Allow 12 months to elapse following transplant as many cases will resolve
The presence of an autonomously functioning parathyroid gland may require
surgery. If the culprit gland can be identified then it should be excised. Otherwise
total parathyroidectomy and re-implantation of part of the gland may be required.

References
1. Pitt S et al. Secondary and Tertiary Hyperparathyroidism, State of the Art
Surgical Management. Surg Clin North Am 2009 Oct;89(5):1227-39.

2. MacKenzie-Feder J et al. Primary Hyperparathyroidism: An Overview. Int J


Endocrinol 2011; 2011: 251410.

Next question 

Display my notes on this topic


          

Save my notes

Question stats

A 33.8%
B 12.7%
C 21.4%
gathered by dr. elbarky.

D 15.7%
E 16.3%

33.8% of users answered this question correctly

Search eMRCS

Search term Go

 Open MRCS Part A textbook (../review/textbook.php)

External links

+ Suggest a link

Dashboard

7

Question 45 of 364

 

A 10 year old boy who has learning difficulties, is reported as having a difference in
size between his two legs. What is the underlying disorder?

Neurofibromatosis Type I
gathered by dr. elbarky.

Neurofibromatosis Type II

Kartagener's syndrome

Multiple endocrine neoplasia type II

Multiple endocrine neoplasia type I

Neurofibromatosis type I. A hallmark finding is a plexiform neurofibroma, which is


a sheet of neurofibromatosis tissue which encases major nerves. In children this
attracts extra blood circulation, which can accelerate growth of the affected limb.
Other features include:
Schwannoma, > 6
Cafe au lait spots, axillary freckling, Lisch nodules, Optic glioma. Meningiomas,
Glioma, or Schwannoma.

Please rate this question:

 Discuss and give feedback

Next question 

Genetics and surgical disease


Some of the more commonly occurring genetic conditions occurring in surgical
patients are presented here.

Li-Fraumeni Syndrome
Autosomal dominant
Consists of germline mutations to p53 tumour suppressor gene
High incidence of malignancies particularly sarcomas and leukaemias
Diagnosed when:

*Individual develops sarcoma under 45 years


*First degree relative diagnosed with any cancer below age 45 years and another
gathered by dr. elbarky.

family member develops malignancy under 45 years or sarcoma at any age

BRCA 1 and 2
Carried on chromosome 17 (BRCA 1) and Chromosome 13 (BRCA 2)
Linked to developing breast cancer (60%) risk.
Associated risk of developing ovarian cancer (55% with BRCA 1 and 25%
with BRCA 2).

Lynch Syndrome
Autosomal dominant
Develop colonic cancer and endometrial cancer at young age
80% of affected individuals will get colonic and/ or endometrial cancer
High risk individuals may be identified using the Amsterdam criteria

Amsterdam criteria
Three or more family members with a confirmed diagnosis of colorectal cancer,
one of whom is a first degree (parent, child, sibling) relative of the other two.
Two successive affected generations.
One or more colon cancers diagnosed under age 50 years.
Familial adenomatous polyposis (FAP) has been excluded.

Gardners syndrome
Autosomal dominant familial colorectal polyposis
Multiple colonic polyps
Extra colonic diseases include: skull osteoma, thyroid cancer and
epidermoid cysts
Desmoid tumours are seen in 15%
Mutation of APC gene located on chromosome 5
Due to colonic polyps most patients will undergo colectomy to reduce risk of
colorectal cancer
Now considered a variant of familial adenomatous polyposis coli

Next question 
Display my notes on this topic

          

Save my notes

Question stats
gathered by dr. elbarky.

A 34.4%
B 26.3%
C 23.3%
D 9.1%
E 6.9%

34.4% of users answered this question correctly

Search eMRCS

Search term Go

 Open MRCS Part A textbook (../review/textbook.php)

External links

+ Suggest a link

Dashboard

5

Question 42 of 364

 

An obese 40 year old male presents with episodes of anxiety, confusion and one
convulsive episode. CT brain is normal. An abdominal CT scan shows a small
1.5cm lesion in the head of the pancreas. What is the most likely diagnosis?

Glucagonoma
gathered by dr. elbarky.

Insulinoma

Somatostatinoma

Adenocarcinoma

Squamous cell carcinoma

These episodes are due to hypoglycaemia. Insulinomas are normally solitary


tumours and may not be seen by radiological imaging. Resection is the treatment
of choice.

Please rate this question:

 Discuss and give feedback

Next question 

Insulinoma

Insulin producing tumours of the pancreatic β cells


Incidence of 1 per 1,000,000 per year
90% of lesions are benign
Most tumours less than 2cm in size
Between 5 and 10% have MEN type 1
75% of patients with MEN 1 will develop pancreatic islet cell tumours

Typical features of insulinoma


Symptomatic hypoglycaemia during fasting
Concomitant blood glucose of less than 3mmol/L
Relief of hypoglycaemia by use of glucose

Testing
When neuroglycopenic symptoms occur blood is taken for serum insulin levels,
serum glucose, C-peptide and pro insulin concentrations. The plasma insulin
concentration is >10 micro U/ml in patients with the disorder.

Tumour localisation
USS (25% accuracy), endoscopic USS better (75% accuracy)
CT scanning (pancreatic protocol=40% accuracy)
gathered by dr. elbarky.

Malignant insulinomas are larger and diagnostic accuracy with MRI is nearly
100% in such cases
Somatostatin receptor scintigraphy (50% accuracy)

Treatment
Since the majority of tumours are benign; the blind segmental resection of the
pancreas (e.g. Whipples) cannot be justified, this may be considered acceptable
for malignant lesions. The best approach at laparotomy is to corroborate pre
operative imaging with intraoperative ultrasonography to identify the lesion.
Tumours may be close to the pancreatic duct and this must be appreciated by the
operating surgeon. The perioperative use of octreotide reduces the amount of
pancreatic drainage, but not overall complications.

Next question 

Display my notes on this topic

          

Save my notes

Question stats

A 13.5%
B 60.8%
C 9.6%
D 11.1%
E 5%

Question 41 of 364

 

Which of the following are not true of Li-Fraumeni syndrome?

It consists of mutations to the p53 tumour suppressor gene

Is likely to be present in a teenager presenting with a liposarcoma


gathered by dr. elbarky.

It has an autosomal dominant inheritance pattern

Affected individuals are unlikely to develop acute myeloid leukaemia

Adrenal malignancies are more common than in normal population

The classical LFS malignancies - sarcoma, cancers of the breast, brain and
adrenal glands - comprise about 80% of all cancers that occur in this
syndrome.

They are at high risk of developing leukaemia.

Please rate this question:

 Discuss and give feedback

Next question 

Genetics and surgical disease

Some of the more commonly occurring genetic conditions occurring in surgical


patients are presented here.

Li-Fraumeni Syndrome
Autosomal dominant
Consists of germline mutations to p53 tumour suppressor gene
High incidence of malignancies particularly sarcomas and leukaemias
Diagnosed when:
*Individual develops sarcoma under 45 years
*First degree relative diagnosed with any cancer below age 45 years and another
family member develops malignancy under 45 years or sarcoma at any age

BRCA 1 and 2
Carried on chromosome 17 (BRCA 1) and Chromosome 13 (BRCA 2)
Linked to developing breast cancer (60%) risk.
Associated risk of developing ovarian cancer (55% with BRCA 1 and 25%
with BRCA 2).
gathered by dr. elbarky.

Lynch Syndrome
Autosomal dominant
Develop colonic cancer and endometrial cancer at young age
80% of affected individuals will get colonic and/ or endometrial cancer
High risk individuals may be identified using the Amsterdam criteria

Amsterdam criteria
Three or more family members with a confirmed diagnosis of colorectal cancer,
one of whom is a first degree (parent, child, sibling) relative of the other two.
Two successive affected generations.
One or more colon cancers diagnosed under age 50 years.
Familial adenomatous polyposis (FAP) has been excluded.

Gardners syndrome
Autosomal dominant familial colorectal polyposis
Multiple colonic polyps
Extra colonic diseases include: skull osteoma, thyroid cancer and
epidermoid cysts
Desmoid tumours are seen in 15%
Mutation of APC gene located on chromosome 5
Due to colonic polyps most patients will undergo colectomy to reduce risk of
colorectal cancer
Now considered a variant of familial adenomatous polyposis coli

Next question 

Display my notes on this topic

          
Save my notes

Question stats

A 10.4%
B 13.5%
C 15.5%
D 46.3%
E 14.3%
gathered by dr. elbarky.

46.3% of users answered this question correctly

Search eMRCS

Search term Go

 Open MRCS Part A textbook (../review/textbook.php)

External links

+ Suggest a link

Dashboard

9

Question 44 of 364

 

A 28 year old man has a long history of recurrent chest infections. On examination,
he is noted to have no palpable vas deferens. However, both testes are located
within the scrotum. What is the most likely underlying disease association?

Kleinfelters syndrome
gathered by dr. elbarky.

Kallmann syndrome

Cystic fibrosis

Coeliac disease

Gardners syndrome

99% of males with cystic fibrosis will have absent vas

Please rate this question:

 Discuss and give feedback

Next question 

Absence of the vas deferens

Absence of the vas may be unilateral or bilateral


Cystic fibrosis CFTR gene mutations are the cause in 40% of cases
Some non CF cases are due to unilateral renal agenesis
Sperm harvesting may allow for assisted conception

Next question 
Display my notes on this topic

          

Save my notes

Question stats

A 14.9%
gathered by dr. elbarky.

B 15.8%
C 56.5%
D 6.1%
E 6.7%

56.5% of users answered this question correctly

Search eMRCS

Search term Go

 Open MRCS Part A textbook (../review/textbook.php)

External links

+ Suggest a link

Dashboard

5

Question 43 of 364

 

A 25 year old man is injured in a road traffic accident. His right tibia is fractured
and is managed by fasciotomies and application of an external fixator. Over the
next 48 hours his serum creatinine rises and urine is sent for microscopy, muddy
brown casts are identified. What is the most likely underlying diagnosis?
gathered by dr. elbarky.

Acute interstitial nephritis

Acute tubular necrosis

Glomerulonephritis

IgA Nephropathy

Thin basement membrane disease

This patient is likely to have had compartment syndrome (tibial fracture +


fasciotomies) which may produce myoglobinuria. The presence of worsening renal
function, together with muddy brown casts is strongly suggestive of acute tubular
necrosis. Acute interstitial nephritis usually arises from drug toxicity and does not
usually produce urinary muddy brown casts. Thin basement membrane disease is
an autosomal dominant condition that causes persistent microscopic haematuria,
but not worsening renal function.

Please rate this question:

 Discuss and give feedback

Next question 

Acute Renal Failure

Final pathway is tubular cell death.


Renal medulla is a relatively hypoxic environment making it susceptible to
renal tubular hypoxia.
Renovascular autoregulation maintains renal blood flow across a range of
arterial pressures.
Estimates of GFR are best indices of level of renal function. Useful clinical
estimates can be obtained by considering serum creatinine, age, race,
gender and body size. eGFR calculations such as the Cockcroft and Gault
equation are less reliable in populations with high GFR's.
Nephrotoxic stimuli such as aminoglycosides and radiological contrast
media induce apoptosis. Myoglobinuria and haemolysis result in necrosis.
Overlap exists and proinflammatory cytokines play an important role in
potentiating ongoing damage.
Post-operative renal failure is more likely to occur in patients who are elderly,
have peripheral vascular disease, high BMI, have COPD, receive
vasopressors, are on nephrotoxic medication or undergo emergency
surgery.
Avoiding hypotension will reduce risk of renal tubular damage.
gathered by dr. elbarky.

There is no evidence that administration of ACE inhibitors or dopamine


reduces the incidence of post-operative renal failure.

Next question 

Display my notes on this topic

          

Save my notes

Question stats

A 10.6%
B 67.4%
C 8.2%
D 7.2%
E 6.5%

67.4% of users answered this question correctly

Search eMRCS

Search term Go

Question 45 of 364

 

A 10 year old boy who has learning difficulties, is reported as having a difference in
size between his two legs. What is the underlying disorder?

Neurofibromatosis Type I
gathered by dr. elbarky.

Neurofibromatosis Type II

Kartagener's syndrome

Multiple endocrine neoplasia type II

Multiple endocrine neoplasia type I

Neurofibromatosis type I. A hallmark finding is a plexiform neurofibroma, which is


a sheet of neurofibromatosis tissue which encases major nerves. In children this
attracts extra blood circulation, which can accelerate growth of the affected limb.
Other features include:
Schwannoma, > 6
Cafe au lait spots, axillary freckling, Lisch nodules, Optic glioma. Meningiomas,
Glioma, or Schwannoma.

Please rate this question:

 Discuss and give feedback

Next question 

Genetics and surgical disease


Some of the more commonly occurring genetic conditions occurring in surgical
patients are presented here.

Li-Fraumeni Syndrome
Autosomal dominant
Consists of germline mutations to p53 tumour suppressor gene
High incidence of malignancies particularly sarcomas and leukaemias
Diagnosed when:

*Individual develops sarcoma under 45 years


*First degree relative diagnosed with any cancer below age 45 years and another
gathered by dr. elbarky.

family member develops malignancy under 45 years or sarcoma at any age

BRCA 1 and 2
Carried on chromosome 17 (BRCA 1) and Chromosome 13 (BRCA 2)
Linked to developing breast cancer (60%) risk.
Associated risk of developing ovarian cancer (55% with BRCA 1 and 25%
with BRCA 2).

Lynch Syndrome
Autosomal dominant
Develop colonic cancer and endometrial cancer at young age
80% of affected individuals will get colonic and/ or endometrial cancer
High risk individuals may be identified using the Amsterdam criteria

Amsterdam criteria
Three or more family members with a confirmed diagnosis of colorectal cancer,
one of whom is a first degree (parent, child, sibling) relative of the other two.
Two successive affected generations.
One or more colon cancers diagnosed under age 50 years.
Familial adenomatous polyposis (FAP) has been excluded.

Gardners syndrome
Autosomal dominant familial colorectal polyposis
Multiple colonic polyps
Extra colonic diseases include: skull osteoma, thyroid cancer and
epidermoid cysts
Desmoid tumours are seen in 15%
Mutation of APC gene located on chromosome 5
Due to colonic polyps most patients will undergo colectomy to reduce risk of
colorectal cancer
Now considered a variant of familial adenomatous polyposis coli

Next question 
Display my notes on this topic

          

Save my notes

Question stats
gathered by dr. elbarky.

A 34.4%
B 26.3%
C 23.3%
D 9.1%
E 6.9%

34.4% of users answered this question correctly

Search eMRCS

Search term Go

 Open MRCS Part A textbook (../review/textbook.php)

External links

+ Suggest a link

Dashboard

5

Question 46 of 364

 

The pathogenicity of the tubercle bacillus is due to which of the following?

Necrosis caused by expanding granulomas

Ability to multiply within fibroblasts


gathered by dr. elbarky.

Delayed hypersensitivity reaction against bacteria

Effect of antibody response

Direct toxic effect on host cells

Mycobacteria stimulate a specific T cell response of cell mediated immunity. This


is effective in reducing the infection, the delayed hypersensitivity also damages
tissues. Necrosis occurs in TB but is usually within the granuloma.

Please rate this question:

 Discuss and give feedback

Next question 

Tuberculosis pathology

Is a form of primary chronic inflammation, caused by the inability of


macrophages to kill the Mycobacterium tuberculosis.
The macrophages often migrate to regional lymph nodes, the lung lesion
plus affected lymph nodes is referred to as a Ghon complex.
This leads to the formation of a granuloma which is a collection of
epithelioid histiocytes.
There is the presence of caseous necrosis in the centre.
The inflammatory response is mediated by a type 4 hypersensitivity
reaction.
In healthy individuals the disease may be contained, in the
immunocompromised disseminated (miliary TB) may occur.
Diagnosis
Waxy membrane of mycobacteria prevents binding with normal stains. Ziehl
- Neelsen staining is typically used.
Culture based methods take far longer.

Image showing acid- alcohol fast mycobacteria stained using the Ziehl- Neelsen
method
gathered by dr. elbarky.

(https://d2zgo9qer4wjf4.cloudfront.net/images_eMRCS/swb107b.jpg)
Image sourced from Wikipedia
(https://d2zgo9qer4wjf4.cloudfront.net
(http://en.wikipedia.org/wiki/Ziehl%E2%80
/images_eMRCS/swb107b.jpg)
%93Neelsen stain)

Next question 

Display my notes on this topic

          

Save my notes

Question stats

A 26%
B 12.1%
C 45.9%
D 8%
E 8%

Question 47 of 364

 

A 46 year old lady presents with symptoms of diarrhoea, weight loss of 10 Kg and
a skin rash of erythematous blisters involving the abdomen and buttocks. The
blisters have an irregular border and both intact and ruptured vesicles. What is the
most likely diagnosis?
gathered by dr. elbarky.

Colonic adenocarcinoma

Pancreatic adenocarcinoma

Tropical sprue

Glucagonoma

Insulinoma

Glucagonoma is strongly associated with necrolytic migratory erythema.

Please rate this question:

 Discuss and give feedback

Next question 

Glucagonoma

Rare pancreatic tumours arising from the alpha cells of the pancreas.
Glucagon levels markedly elevated.
Symptoms include diarrhoea, weight loss and necrolytic migratory
erythema.
A serum level of glucagon >1000pg/ml usually suggests the diagnosis,
imaging with CT scanning is also required.
Treatment is with surgical resection. However, careful staging is required for
these tumours are usually malignant and non resectable.

Next question 
Display my notes on this topic

          

Save my notes

Question stats
gathered by dr. elbarky.

A 10.9%
B 15.6%
C 24.3%
D 36.3%
E 12.8%

36.3% of users answered this question correctly

Search eMRCS

Search term Go

 Open MRCS Part A textbook (../review/textbook.php)

External links

+ Suggest a link

Dashboard

5

Question 48 of 364

 

A 22 year old man presents with dyspnoea. He is previously well and apart from an
orchidopexy 17 years previously has no medical history. A chest x-ray
demonstrates some opacities affecting both lung fields. What is the most likely
underlying diagnosis?
gathered by dr. elbarky.

Renal cell carcinoma

Testicular seminoma

Testicular teratoma

Sarcoidosis

Rhadomyosarcoma

Undescended testis is associated with increased risk of testicular cancer.

Testicular tumours are one of the commonest solid tumours to affect young males
and there is a peak in incidence of teratoma in this age group. The need for
orchidopexy is associated with an increased risk in developing testicular cancer.

Please rate this question:

 Discuss and give feedback

Next question 

Testicular disorders

Testicular cancer
Testicular cancer is the most common malignancy in men aged 20-30 years.
Around 95% of cases of testicular cancer are germ-cell tumours. Germ cell
tumours may essentially be divided into:
Tumour
Tumour type Key features markers Pathology

Seminoma Commonest AFP usually Sheet like


subtype normal lobular
(50%) HCG elevated patterns of
Average in 10% cells with
age at seminomas substantial
diagnosis = Lactate fibrous
40 dehydrogenase; component.
Even elevated in Fibrous septa
advanced 10-20% contain
gathered by dr. elbarky.

disease seminomas lymphocytic


associated (but also in inclusions
with 5 year many other and
survival of conditions) granulomas
73% may be seen.

Non seminomatous Younger age at AFP elevated in Heterogenous


germ cell tumours presentation up to 70% of texture with
(42%) =20-30 years cases occasional
Teratoma Advanced disease HCG elevated ectopic tissue
Yolk sac tumour carries worse in up to 40% of such as hair
Choriocarcinoma prognosis (48% at cases
Mixed germ cell 5 years) Other markers
tumours (10%) Retroperitoneal rarely helpful
lymph node
dissection may be
needed for
residual disease
after
chemotherapy

Image demonstrating a classical seminoma, these tumours are typically more


uniform than teratomas
gathered by dr. elbarky.

(https://d2zgo9qer4wjf4.cloudfront.net/images_eMRCS/swb090b.jpg)
Image sourced from Wikipedia
(https://d2zgo9qer4wjf4.cloudfront.net
(http://en.wikipedia.org
/images_eMRCS/swb090b.jpg)
/wiki/Seminoma)

Risk factors for testicular cancer


Cryptorchidism
Infertility
Family history
Klinefelter's syndrome
Mumps orchitis

Features
A painless lump is the most common presenting symptom
Pain may also be present in a minority of men
Other possible features include hydrocele, gynaecomastia
Diagnosis
Ultrasound is first-line
CT scanning of the chest/ abdomen and pelvis is used for staging
Tumour markers (see above) should be measured

Management
Orchidectomy (Inguinal approach)
Chemotherapy and radiotherapy may be given depending on staging
Abdominal lesions >1cm following chemotherapy may require
retroperitoneal lymph node dissection.
gathered by dr. elbarky.

Prognosis is generally excellent


5 year survival for seminomas is around 95% if Stage I
5 year survival for teratomas is around 85% if Stage I

Benign disease

Epididymo-orchitis
Acute epididymitis is an acute inflammation of the epididymis, often involving the
testis and usually caused by bacterial infection.
Infection spreads from the urethra or bladder. In men <35 years, gonorrhoea
or chlamydia are the usual infections.
Amiodarone is a recognised non infective cause of epididymitis, which
resolves on stopping the drug.
Tenderness is usually confined to the epididymis, which may facilitate
differentiating it from torsion where pain usually affects the entire testis.

Testicular torsion
Twist of the spermatic cord resulting in testicular ischaemia and necrosis.
Most common in males aged between 10 and 30 (peak incidence 13-15
years)
Pain is usually severe and of sudden onset.
Cremasteric reflex is lost and elevation of the testis does not ease the pain.
Treatment is with surgical exploration. If a torted testis is identified then
both testis should be fixed as the condition of bell clapper testis is often
bilateral.

Hydrocele
Presents as a mass that transilluminates, usually possible to 'get above' it
on examination.
In younger men it should be investigated with USS to exclude tumour.
In children it may occur as a result of a patent processus vaginalis.
Treatment in adults is with a Lords or Jabouley procedure.
Treatment in children is with trans inguinal ligation of PPV.
Next question 

Display my notes on this topic

          

Save my notes
gathered by dr. elbarky.

Question stats

A 9.2%
B 28.8%
C 41%
D 13.4%
E 7.6%

41% of users answered this question correctly

Search eMRCS

Search term Go

 Open MRCS Part A textbook (../review/textbook.php)

External links

+ Suggest a link

Dashboard

3

Question 49 of 364

 

The cell of origin in virtually all pancreatic carcinomas is which of the following?

The acinar cells

The islet beta cells


gathered by dr. elbarky.

The islet alpha cells

The interstitial fibroblasts

The ductular epithelium

Over 90% of pancreatic carcinomas are adenocarcinomas and are thus of ductular
epithelial origin.

Please rate this question:

 Discuss and give feedback

Next question 

Pancreatic cancer

Adenocarcinoma
Risk factors: Smoking, diabetes, adenoma, familial adenomatous polyposis
Mainly occur in the head of the pancreas (70%)
Spread locally and metastasizes to the liver
Carcinoma of the pancreas should be differentiated from other
periampullary tumours with better prognosis

Clinical features
Weight loss
Painless jaundice
Epigastric discomfort (pain usually due to invasion of the coeliac plexus is a
late feature)
Pancreatitis
Trousseau's sign: migratory superficial thrombophlebitis

Investigations
USS: May miss small lesions
CT Scanning (pancreatic protocol). If unresectable on CT then no further
staging needed
PET/CT for those with operable disease on CT alone
ERCP/ MRI for bile duct assessment
Staging laparoscopy to exclude peritoneal disease

Management
gathered by dr. elbarky.

Head of pancreas: Whipple's resection (SE dumping and ulcers). Newer


techniques include pylorus preservation and SMA/ SMV resection
Carcinoma body and tail: poor prognosis, distal pancreatectomy, if operable
Usually adjuvent chemotherapy for resectable disease
ERCP and stent for jaundice and palliation
Surgical bypass may be needed for duodenal obstruction

Next question 

Display my notes on this topic

          

Save my notes

Question stats

A 37%
B 10.9%
C 6.9%
D 6.8%
E 38.4%

38.4% of users answered this question correctly


Question 50 of 364

 

A 28 year old lady presents with a pigmented lesion on her calf. Excisional biopsy
confirms a diagnosis of melanoma measuring 1cm in diameter with a Breslow
thickness of 0.1mm. The lesion is less than 1 mm at all resection margins. Which
of the following surgical resection margins is acceptable for this lesion?
gathered by dr. elbarky.

5 cm

1 cm

0.5 cm

2 cm

3 cm

Please rate this question:

 Discuss and give feedback

Next question 

Malignant melanoma

The main diagnostic features (major Secondary features (minor


criteria): criteria)
Change in size Diameter >6mm
Change in shape Inflammation
Change in colour Oozing or bleeding
Altered sensation

Treatment
Suspicious lesions should undergo excision biopsy. The lesion should be
removed in completely as incision biopsy can make subsequent
histopathological assessment difficult.
Once the diagnosis is confirmed the pathology report should be reviewed to
determine whether further re-excision of margins is required (see below):

Margins of excision-Related to Breslow thickness

Lesions 0-1mm thick 1cm

Lesions 1-2mm thick 1- 2cm (Depending upon site and pathological features)

Lesions 2-4mm thick 2-3 cm (Depending upon site and pathological features)
gathered by dr. elbarky.

Lesions >4 mm thick 3cm

Marsden J et al Revised UK guidelines for management of Melanoma. Br J


Dermatol 2010 163:238-256.

Further treatments such as sentinel lymph node mapping, isolated limb perfusion
and block dissection of regional lymph node groups should be selectively applied.

Next question 

Display my notes on this topic

          

Save my notes

Question stats

A 7.9%
B 48.2%
C 18.5%
D 16.2%
E 9.2%

48.2% of users answered this question correctly

Search eMRCS

Question 51 of 364

 

A baby is born by normal vaginal delivery at 39 weeks gestation. Initially all


appears well and then the clinical staff become concerned because the baby
develops recurrent episodes of cyanosis. These are worse during feeding and
improve dramatically when the baby cries. The most likely underlying diagnosis is:
gathered by dr. elbarky.

Choanal atresia

Oesophageal reflux

Tetralogy of Fallot

Oesophageal atresia

Congenital diaphragmatic hernia

In Choanal atresia the episodes of cyanosis are usually worst during feeding.
Improvement may be seen when the baby cries as the oropharyngeal airway is
used.

Please rate this question:

 Discuss and give feedback

Next question 

Choanal atresia

Congenital disorder with an incidence of 1 in 7000 births.


Posterior nasal airway occluded by soft tissue or bone.
Associated with other congenital malformations e.g. coloboma
Babies with unilateral disease may go unnoticed.
Babies with bilateral disease will present early in life as they can then only
breathe through their mouth.
Treatment is with fenestration procedures designed to restore patency.
Next question 

Display my notes on this topic

          

Save my notes
gathered by dr. elbarky.

Question stats

A 38.7%
B 6%
C 25.8%
D 18.5%
E 11%

38.7% of users answered this question correctly

Search eMRCS

Search term Go

 Open MRCS Part A textbook (../review/textbook.php)

External links

+ Suggest a link

Dashboard

3

Question 52 of 364

 

A 42 year old man presents with a painless lump in the left testicle that he noticed
on self examination. Clinically there is a firm nodule in the left testicle, ultrasound
appearances show an irregular mass lesion. His serum AFP and HCG levels are
both within normal limits. What is the most likely diagnosis?
gathered by dr. elbarky.

Yolk sack tumour

Seminoma

Testicular teratoma

Epididymo-orchitis

Adenomatoid tumour

Seminomas typically have normal AFP and HCG. These are usually raised in
teratomas and yolk sac tumours

This man's age, presenting symptoms and normal tumour markers make a
seminoma the most likely diagnosis. Epididymo-orchitis does not produce irregular
mass lesions which are painless.

Please rate this question:

 Discuss and give feedback

Next question 

Testicular disorders

Testicular cancer
Testicular cancer is the most common malignancy in men aged 20-30 years.
Around 95% of cases of testicular cancer are germ-cell tumours. Germ cell
tumours may essentially be divided into:
Tumour
Tumour type Key features markers Pathology

Seminoma Commonest AFP usually Sheet like


subtype normal lobular
(50%) HCG elevated patterns of
Average in 10% cells with
age at seminomas substantial
diagnosis = Lactate fibrous
40 dehydrogenase; component.
Even elevated in Fibrous septa
advanced 10-20% contain
gathered by dr. elbarky.

disease seminomas lymphocytic


associated (but also in inclusions
with 5 year many other and
survival of conditions) granulomas
73% may be seen.

Non seminomatous Younger age at AFP elevated in Heterogenous


germ cell tumours presentation up to 70% of texture with
(42%) =20-30 years cases occasional
Teratoma Advanced disease HCG elevated ectopic tissue
Yolk sac tumour carries worse in up to 40% of such as hair
Choriocarcinoma prognosis (48% at cases
Mixed germ cell 5 years) Other markers
tumours (10%) Retroperitoneal rarely helpful
lymph node
dissection may be
needed for
residual disease
after
chemotherapy

Image demonstrating a classical seminoma, these tumours are typically more


uniform than teratomas
gathered by dr. elbarky.

(https://d2zgo9qer4wjf4.cloudfront.net/images_eMRCS/swb090b.jpg)
Image sourced from Wikipedia
(https://d2zgo9qer4wjf4.cloudfront.net
(http://en.wikipedia.org
/images_eMRCS/swb090b.jpg)
/wiki/Seminoma)

Risk factors for testicular cancer


Cryptorchidism
Infertility
Family history
Klinefelter's syndrome
Mumps orchitis

Features
A painless lump is the most common presenting symptom
Pain may also be present in a minority of men
Other possible features include hydrocele, gynaecomastia
Diagnosis
Ultrasound is first-line
CT scanning of the chest/ abdomen and pelvis is used for staging
Tumour markers (see above) should be measured

Management
Orchidectomy (Inguinal approach)
Chemotherapy and radiotherapy may be given depending on staging
Abdominal lesions >1cm following chemotherapy may require
retroperitoneal lymph node dissection.
gathered by dr. elbarky.

Prognosis is generally excellent


5 year survival for seminomas is around 95% if Stage I
5 year survival for teratomas is around 85% if Stage I

Benign disease

Epididymo-orchitis
Acute epididymitis is an acute inflammation of the epididymis, often involving the
testis and usually caused by bacterial infection.
Infection spreads from the urethra or bladder. In men <35 years, gonorrhoea
or chlamydia are the usual infections.
Amiodarone is a recognised non infective cause of epididymitis, which
resolves on stopping the drug.
Tenderness is usually confined to the epididymis, which may facilitate
differentiating it from torsion where pain usually affects the entire testis.

Testicular torsion
Twist of the spermatic cord resulting in testicular ischaemia and necrosis.
Most common in males aged between 10 and 30 (peak incidence 13-15
years)
Pain is usually severe and of sudden onset.
Cremasteric reflex is lost and elevation of the testis does not ease the pain.
Treatment is with surgical exploration. If a torted testis is identified then
both testis should be fixed as the condition of bell clapper testis is often
bilateral.

Hydrocele
Presents as a mass that transilluminates, usually possible to 'get above' it
on examination.
In younger men it should be investigated with USS to exclude tumour.
In children it may occur as a result of a patent processus vaginalis.
Treatment in adults is with a Lords or Jabouley procedure.
Treatment in children is with trans inguinal ligation of PPV.
Next question 

Display my notes on this topic

          

Save my notes
gathered by dr. elbarky.

Question stats

A 10.2%
B 51%
C 17.8%
D 6.8%
E 14.1%

51% of users answered this question correctly

Search eMRCS

Search term Go

 Open MRCS Part A textbook (../review/textbook.php)

External links

+ Suggest a link

Dashboard

3

Question 53 of 364

 

Which of the following is not characteristic of a granuloma?

Altered macrophages

Fused macrophages
gathered by dr. elbarky.

Epithelioid cells

Mixture of chronic inflammatory cells

Polymorphnuclear leucocytes, cellular debris and fibrin

These are typical components of an abscess cavity. Polymorphonuclear


leucocytes may be found in a granuloma if there is a focus of suppuration.

Please rate this question:

 Discuss and give feedback

Next question 

Chronic inflammation

Overview
Chronic inflammation may occur secondary to acute inflammation.In most cases
chronic inflammation occurs as a primary process. These may be broadly viewed
as being one of three main processes:
Persisting infection with certain organisms such as Mycobacterium
tuberculosis which results in delayed type hypersensitivity reactions and
inflammation.
Prolonged exposure to non-biodegradable substances such as silica or
suture materials which may induce an inflammatory response.
Autoimmune conditions involving antibodies formed against host antigens.

Acute vs. Chronic inflammation


Acute inflammation Chronic inflammation

Changes to existing vascular structure and Angiogenesis predominates


increased permeability of endothelial cells

Infiltration of neutrophils Macrophages, plasma cells


and lymphocytes predominate

Process may resolve with: Healing by fibrosis is the main


Suppuration result
Complete resolution
Abscess formation
gathered by dr. elbarky.

Progression to chronic inflammation


Healing by fibrosis

Granulomatous inflammation
A granuloma consists of a microscopic aggregation of macrophages (with
epithelial type arrangement =epithelioid). Large giant cells may be found at the
periphery of granulomas.

Mediators
Growth factors released by activated macrophages include agents such as
interferon and fibroblast growth factor (plus many more). Some of these such as
interferons may have systemic features resulting in systemic symptoms and signs,
which may be present in individuals with long standing chronic inflammation.

The finding of granulomas is pathognomonic of chronic inflammation, as illustrated


in this biopsy from a patient with colonic Crohns disease

(https://d2zgo9qer4wjf4.cloudfront.net/images_eMRCS/swb093b.jpg)
Image sourced from Wikipedia
(https://d2zgo9qer4wjf4.cloudfront.net
(http://en.wikipedia.org/wiki/Crohn%27s
/images_eMRCS/swb093b.jpg)
disease)
Next question 

Display my notes on this topic

          

Save my notes
gathered by dr. elbarky.

Question stats

A 15.4%
B 12.5%
C 20%
D 8.6%
E 43.5%

43.5% of users answered this question correctly

Search eMRCS

Search term Go

 Open MRCS Part A textbook (../review/textbook.php)

External links

+ Suggest a link

Dashboard

3

Question 54 of 364

 

A 22 year old woman presents with macroscopic haematuria. She is sexually


active. She is known to have renal calculi and had a berry aneurysm clipped. What
is the most likely cause?

Interstitial nephritis
gathered by dr. elbarky.

Membranous glomerulonephritis

Renal vein thrombosis

Endometriosis

Adult polycystic kidney disease

APKD is associated with liver cysts (70%), berry aneurysms (25%) and pancreatic
cysts (10%). Patients may have a renal mass, hypertension, renal calculi and
macroscopic haematuria.

Please rate this question:

 Discuss and give feedback

Next question 

Haematuria

Causes of haematuria

Trauma Injury to renal tract


Renal trauma commonly due to blunt injury
(others penetrating injuries)
Ureter trauma rare: iatrogenic
Bladder trauma: due to RTA or pelvic fractures

Infection Remember TB
Malignancy Renal cell carcinoma (remember paraneoplastic
syndromes): painful or painless
Urothelial malignancies: 90% are transitional cell
carcinoma, can occur anywhere along the urinary
tract. Painless haematuria.
Squamous cell carcinoma and adenocarcinoma:
rare bladder tumours
Prostate cancer
Penile cancers: SCC

Renal disease Glomerulonephritis


gathered by dr. elbarky.

Stones Microscopic haematuria common

Structural Benign prostatic hyperplasia (BPH) causes


abnormalities haematuria due to hypervascularity of the
prostate gland
Cystic renal lesions e.g. polycystic kidney disease
Vascular malformations
Renal vein thrombosis due to renal cell carcinoma

Coagulopathy Causes bleeding of underlying lesions

Drugs Cause tubular necrosis or interstitial nephritis:


aminoglycosides, chemotherapy
Interstitial nephritis: penicillin, sulphonamides,
and NSAIDs
Anticoagulants

Benign Exercise

Gynaecological Endometriosis: flank pain, dysuria, and


haematuria that is cyclical

Iatrogenic Catheterisation
Radiotherapy; cystitis, severe haemorrhage,
bladder necrosis

Pseudohaematuria For example following consumption of beetroot

References
Http://bestpractice.bmj.com/best-practice/monograph/316/overview
/aetiology.html
Next question 

Display my notes on this topic

          

Save my notes
gathered by dr. elbarky.

Question stats

A 11.1%
B 12.6%
C 10.2%
D 7.5%
E 58.6%

58.6% of users answered this question correctly

Search eMRCS

Search term Go

 Open MRCS Part A textbook (../review/textbook.php)

External links

+ Suggest a link

Dashboard

3

Question 55 of 364

 

A 56 year old man presents with symptoms of neuropathic facial pain and some
weakness of the muscles of facial expression on the right side. On examination he
has a hard mass approximately 6cm anterior to the right external auditory meatus.
What is the most likely diagnosis?
gathered by dr. elbarky.

Pleomorphic adenoma

Adenocarcinoma

Mucoepidermoid carcinoma

Adenoid cystic carcinoma

Lymphoma

The patient is most likely to have a malignant lesion within the parotid. Of the
malignancies listed; adenoid cystic carcinoma has the greatest tendency to
perineural invasion.

Please rate this question:

 Discuss and give feedback

Next question 

Parotid gland malignancy

Most parotid neoplasms (80%) are benign lesions


Most commonly present with painless mass around the mandible
Up to 30% may present with pain, when this is associated with a discrete
mass lesion in the parotid it usually indicates perineural invasion.
Perineural invasion is very unlikely to occur in association with benign
lesions
80% of patients with facial nerve weakness caused by parotid malignancies
will have nodal metastasis and a 5 year survival of 25%
Types of malignancy

Mucoepidermoid 30% of all parotid malignancies


carcinoma Usually low potential for local invasiveness and
metastasis (depends mainly on grade)

Adenoid cystic Unpredictable growth pattern


carcinoma Tendency for perineural spread
Nerve growth may display skip lesions resulting in
incomplete excision
Distant metastasis more common (visceral rather than
nodal spread)
5 year survival 35%
gathered by dr. elbarky.

Mixed tumours Often a malignancy occurring in a previously benign


parotid lesion

Acinic cell Intermediate grade malignancy


carcinoma May show perineural invasion
Low potential for distant metastasis
5 year survival 80%

Adenocarcinoma Develops from secretory portion of gland


Risk of regional nodal and distant metastasis
5 year survival depends upon stage at presentation,
may be up to 75% with small lesions with no nodal
involvement

Lymphoma Large rubbery lesion, may occur in association with


Warthins tumours
Diagnosis should be based on regional nodal biopsy
rather than parotid resection Treatment is with
chemotherapy (and radiotherapy)

Next question 

Display my notes on this topic

          

Save my notes

Question stats
A 25.5%
B 16.6%
C 13.9%
D 38.4%
E 5.6%

38.4% of users answered this question correctly


gathered by dr. elbarky.

Search eMRCS

Search term Go

 Open MRCS Part A textbook (../review/textbook.php)

External links

+ Suggest a link

Dashboard

10

11

12

Question 56 of 364

 

A 62 year old man is admitted with dull lower back pain and abdominal discomfort.
On examination, he is hypertensive and a lower abdominal fullness is elicited on
examination. An abdominal ultrasound demonstrates hydronephrosis and
intravenous urography demonstrated medially displaced ureters. A CT scan shows
a periaortic mass. What is the most likely cause?
gathered by dr. elbarky.

Abdominal liposarcoma

Metastatic pancreatic cancer

Abdominal aortic aneurysm

Retroperitoneal fibrosis

Colonic cancer

Retroperitoneal fibrosis is an uncommon condition and its aetiology is poorly


understood. In a significant proportion the ureters are displaced medially. In most
retroperitoneal malignancies they are displaced laterally. Hypertension is another
common finding. A CT scan will often show a para-aortic mass

Please rate this question:

 Discuss and give feedback

Next question 

Retroperitoneal fibrosis

Retroperitoneal fibrosis or Ormond's disease is a disease featuring the


proliferation of fibrous tissue in the retroperitoneum, the compartment of
the body containing the kidneys, aorta, renal tract, and various other
structures. It may present with lower back pain, kidney failure, hypertension,
deep vein thrombosis, and other obstructive symptoms. It is named after
John Kelso Ormond, who rediscovered the condition in 1948
Imaged with CT scanning of the abdomen
Biopsy is usually performed to exclude other causes
Treatment includes steroids and occasionally surgery, ureteric stent
insertion is commonly required

Next question 

Display my notes on this topic

          
gathered by dr. elbarky.

Save my notes

Question stats

A 14.5%
B 8.1%
C 28.4%
D 42.2%
E 6.8%

42.2% of users answered this question correctly

Search eMRCS

Search term Go

 Open MRCS Part A textbook (../review/textbook.php)

External links

+ Suggest a link

Dashboard

Question 57 of 364

 

A 45 year old lady has recently undergone a thyroidectomy for treatment of


medullary thyroid cancer. Which of the following tumour markers is used clinically
to screen for recurrence?

Free T3
gathered by dr. elbarky.

Thyroglobulin

Calcitonin

Free T4

Thyroid stimulating hormone

Calcitonin is clinically utilised to screen for medullary thyroid cancer recurrence.


Thyroid function testing does not form part of either diagnosis or follow up from a
malignancy perspective. However, routine assessment of TSH may be needed in
patients on thyroxine.

Please rate this question:

 Discuss and give feedback

Next question 

Thyroid malignancy

Papillary carcinoma
Commonest sub-type
Accurately diagnosed on fine needle aspiration cytology
Histologically, they may demonstrate psammoma bodies (areas of
calcification) and so called 'orphan Annie' nuclei
They typically metastasise via the lymphatics and thus laterally located
apparently ectopic thyroid tissue is usually a metastasis from a well
differentiated papillary carcinoma
Follicular carcinoma
Are less common than papillary lesions
Like papillary tumours, they may present as a discrete nodule. Although they
appear to be well encapsulated macroscopically there is invasion on
microscopic evaluation
Lymph node metastases are uncommon and these tumours tend to spread
haematogenously. This translates into a higher mortality rate
Follicular lesions cannot be accurately diagnosed on fine needle aspiration
cytology and thus all follicular FNA's (THY 3f) will require at least a hemi
thyroidectomy
gathered by dr. elbarky.

Anaplastic carcinoma
Less common and tend to occur in elderly females
Disease is usually advanced at presentation and often only palliative
decompression and radiotherapy can be offered.

Medullary carcinoma
These are tumours of the parafollicular cells ( C Cells) and are of neural
crest origin.
The serum calcitonin may be elevated which is of use when monitoring for
recurrence.
They may be familial and occur as part of the MEN -2A disease spectrum.
Spread may be either lymphatic or haematogenous and as these tumours
are not derived primarily from thyroid cells they are not responsive to
radioiodine.

Lymphoma
These respond well to combined chemoradiotherapy
Radical surgery is unnecessary once the disease has been diagnosed on
biopsy material. Such biopsy material is not generated by an FNA and thus a
core biopsy has to be obtained (with care!).

Next question 

Display my notes on this topic

          

Save my notes

Question stats
A 8.6%
B 19.9%
C 56.1%
D 7.6%
E 7.8%

56.1% of users answered this question correctly


gathered by dr. elbarky.

Search eMRCS

Search term Go

 Open MRCS Part A textbook (../review/textbook.php)

External links

+ Suggest a link

Dashboard

10

11

12

Question 58 of 364

 

A 42 year old man from Southern India presents with chronic swelling of both
lower legs, they are brawny and indurated with marked skin trophic changes.
Which of the following organisms is the most likely origin of this disease process?

Loa loa
gathered by dr. elbarky.

Wuchereria bancrofti

Trypanosoma cruzi

Trypanosoma gambiense

None of the above

W. Bancrofti is the commonest cause of filariasis leading to lymphatic obstruction.


Infection with Loa loa typically occurs in the African sub continent and usually
results in generalised sub cutaneous infections without lymphatic obstruction.
Trypanosomal infections would not produce this clinical picture.

Please rate this question:

 Discuss and give feedback

Next question 

Wuchereria bancrofti

Parasitic filarial nematode


Accounts for 90% of cases of filariasis
Usually diagnosed by blood smears
Usually transmitted by mosquitos
Treatment is with diethylcarbamazine

Next question 
Display my notes on this topic

          

Save my notes

Question stats
gathered by dr. elbarky.

A 10.9%
B 54.3%
C 17.2%
D 11.3%
E 6.3%

54.3% of users answered this question correctly

Search eMRCS

Search term Go

 Open MRCS Part A textbook (../review/textbook.php)

External links

+ Suggest a link

Dashboard

5

Question 59 of 364

 

A 38 year old sheep farmer presents to the clinic with a 3 month history of malaise
and right upper quadrant pain. On examination, he is mildly jaundiced. His liver
function tests demonstrate a mild elevation in bilirubin and transaminases, his full
blood count shows an elevated eosinophil level. An abdominal x-ray is performed
by the senior house officer and demonstrates a calcified lesion in the right upper
gathered by dr. elbarky.

quadrant of the abdomen. What is the most probably underlying diagnosis?

Mesenchymal hamartoma

Clonorchiasis

Fasciolopsiasis

Liver cell adenoma

Hydatid cyst

Hydatid disease is more common in those who work with sheep or dogs. Liver
function tests may be abnormal and an eosinophilia is often present. Plain
radiographs may reveal a calcified cyst wall. Fasciolopsiasis infection is confined
to intestinal wall in most cases.

Please rate this question:

 Discuss and give feedback

Next question 

Benign liver lesions

Benign liver lesions


Haemangioma Most common benign tumours of mesenchymal
origin
Incidence in autopsy series is 8%
Cavernous haemangiomas may be enormous
Clinically they are reddish purple hypervascular
lesions
Lesions are normally separated from normal liver by
ring of fibrous tissue
On ultrasound they are typically hyperechoic

Liver cell 90% develop in women in their third to fifth decade


gathered by dr. elbarky.

adenoma Linked to use of oral contraceptive pill


Lesions are usually solitary
They are usually sharply demarcated from normal
liver although they usually lack a fibrous capsule
On ultrasound the appearances are of mixed echoity
and heterogeneous texture. On CT most lesions are
hypodense when imaged prior to administration of
IV contrast agents
In patients with haemorrhage or symptoms removal
of the adenoma may be required

Mesenchymal Congential and benign, usually present in infants. May


hamartomas compress normal liver

Liver abscess Biliary sepsis is a major predisposing factor


Structures drained by the portal venous system form
the second largest source
Common symptoms include fever, right upper
quadrant pain. Jaundice may be seen in 50%
Ultrasound will usually show a fluid filled cavity,
hyperechoic walls may be seen in chronic abscesses

Amoebic Liver abscess is the most common extra intestinal


abscess manifestation of amoebiasis
Between 75 and 90% lesions occur in the right lobe
Presenting complaints typically include fever and
right upper quadrant pain
Ultrasonography will usually show a fluid filled
structure with poorly defined boundaries
Aspiration yield sterile odourless fluid which has an
anchovy paste consistency
Treatment is with metronidazole
Hyatid cysts Seen in cases of Echinococcus infection
Typically an intense fibrotic reaction occurs around
sites of infection
The cyst has no epithelial lining
Cysts are commonly unilocular and may grow to
20cm in size. The cyst wall is thick and has an
external laminated hilar membrane and an internal
enucleated germinal layer
Typically presents with malaise and right upper
quadrant pain. Secondary bacterial infection occurs
in 10%.
gathered by dr. elbarky.

Liver function tests are usually abnormal and


eosinophilia is present in 33% cases
Ultrasound may show septa and hyatid sand or
daughter cysts.
Percutaneous aspiration was previously contra
indicated, it is not incorporated into some treatment
regimens
Treatment is by sterilisation of the cyst with
mebendazole and may be followed by surgical
resection. Hypertonic swabs are packed around the
cysts during surgery

Polycystic liver Usually occurs in association with polycystic kidney


disease disease
Autosomal dominant disorder
Symptoms may occur as a result of capsular stretch

Cystadenoma Rare lesions with malignant potential


Usually solitary multiloculated lesions
Liver function tests usually normal
Ultrasonography typically shows a large anechoic,
fluid filled area with irregular margins. Internal echos
may result from septa
Surgical resection is indicated in all cases

Next question 

Display my notes on this topic

          
Save my notes

Question stats

A 6.3%
B 12.8%
C 15%
D 6.3%
gathered by dr. elbarky.

E 59.6%

59.6% of users answered this question correctly

Search eMRCS

Search term Go

 Open MRCS Part A textbook (../review/textbook.php)

External links

+ Suggest a link

Dashboard

8

Question 60 of 364

 

A pathologist is examining a histological section and identifies Hassall's


corpuscles. With what are they most commonly associated?

Follicular carcinoma of the thyroid


gathered by dr. elbarky.

Medulla of the thymus

Medulla of the spleen

Medulla of the kidney

Fundus of the stomach

Hassall's corpuscles are the concentric ring of epithelial cells seen in the medulla
of the thymus.

Please rate this question:

 Discuss and give feedback

Next question 

Thymus

The thymus develops from the third and fourth pharyngeal pouches. It descends to
lie in the anterior superior mediastinum. It is encapsulated and is subdivided into
lobules, these consist of a cortex and a medulla. The cortex is composed of tightly
packed lymphocytes, the medulla consists largely of epithelial cells. The medullary
epithelial cells are concentrically arranged and may surround a keratinised centre,
known as Hassall's corpuscles.
The inferior parathyroid glands also develop from the third pharyngeal pouch and
may also be located with the thymus gland.
Its arterial supply is from the internal mammary artery or pericardiophrenic
arteries. Venous drainage is to the left brachiocephalic vein.

Hassall's corpuscles stained with H+E


(https://d2zgo9qer4wjf4.cloudfront.net
gathered by dr. elbarky.

/images_eMRCS/swb116b.png)
Image sourced from Wikipedia
(https://d2zgo9qer4wjf4.cloudfront.net
(http://en.wikipedia.org/wiki/Hassall%27s
/images_eMRCS/swb116b.png)
corpuscles)

Next question 

Display my notes on this topic

          

Save my notes

Question stats
A 14.8%
B 47.8%
C 19.7%
D 10.4%
E 7.3%

47.8% of users answered this question correctly


gathered by dr. elbarky.

Search eMRCS

Search term Go

 Open MRCS Part A textbook (../review/textbook.php)

External links

+ Suggest a link

Dashboard

10

11

12

Question 61 of 364

 

A cachectic 32 year old man with severe perineal Crohns disease is receiving
treatment with intravenous antibiotics. Over the past 72 hours he has complained
of intermittent dysphagia and odynophagia. What is the most likely cause?

Oesophageal Crohns disease


gathered by dr. elbarky.

Oesophageal candidiasis

Plummer Vinson syndrome

Globus

Achalasia

Treatment with systemic antibiotics may result in development of candidiasis.


Patients may present with odynophagia and episodic dysphagia. Endoscopic
appearances are usually diagnostic and treatment is with an oral anti fungal agent.

Please rate this question:

 Discuss and give feedback

Next question 

Dysphagia

Causes of dysphagia

Extrinsic Mediastinal masses


Cervical spondylosis

Oesophageal wall Achalasia


Diffuse oesophageal spasm
Hypertensive lower oesophageal sphincter
Intrinsic Tumours
Strictures
Oesophageal web
Schatzki rings

Neurological CVA
Parkinson's disease
Multiple Sclerosis
Brainstem pathology
Myasthenia Gravis
gathered by dr. elbarky.

Investigation
All patients require an upper GI endoscopy unless there are compelling reasons for
this not to be performed. Motility disorders may be best appreciated by
undertaking fluoroscopic swallowing studies.

A full blood count should be performed.

Ambulatory oesophageal pH and manometry studies will be required to evaluate


conditions such as achalasia and patients with GORD being considered for
fundoplication surgery.

Next question 

Display my notes on this topic

          

Save my notes

Question stats

A 13.7%
B 56.4%
C 14.3%
D 7.8%
E 7.8%

Question 62 of 364

 

A 32 year old man is involved in a house fire and sustains extensive partial
thickness burns to his torso and thigh. Two weeks post incident he develops
oedema of both lower legs. The most likely cause of this is:

Iliofemoral deep vein thrombosis


gathered by dr. elbarky.

Venous obstruction due to scarring

Hypoalbuminaemia

Excessive administration of intravenous fluids

None of the above

Loss of plasma proteins is the most common cause of oedema developing in this
time frame.

Please rate this question:

 Discuss and give feedback

Next question 

Burns pathology
Extensive burns
Haemolysis due to damage of erythrocytes by heat and microangiopathy
Loss of capillary membrane integrity causing plasma leakage into interstitial
space
Extravasation of fluids from the burn site causing hypovolaemic shock (up
to 48h after injury)- decreased blood volume and increased haematocrit
Protein loss
Secondary infection e.g. Staphylococcus aureus
ARDS
Risk of Curlings ulcer (acute peptic stress ulcers)
Danger of full thickness circumferential burns in an extremity as these may
develop compartment syndrome
gathered by dr. elbarky.

Healing
Superficial burns: keratinocytes migrate to form a new layer over the burn
site
Full thickness burns: dermal scarring. Usually need keratinocytes from skin
grafts to provide optimal coverage.

Next question 

Display my notes on this topic

          

Save my notes

Question stats

A 9.2%
B 20.1%
C 57.1%
D 6.9%
E 6.8%

57.1% of users answered this question correctly

Search eMRCS

Question 63 of 364

 

A 2 year old boy is brought to the clinic by his mother who has noticed that he has
developed a small mass. On examination; a small smooth cyst is identified which
is located above the hyoid bone. On ultrasound the lesion appears to be a
heterogenous and multiloculated mass. What is the most likely diagnosis?
gathered by dr. elbarky.

Cystic hygroma

Thyroglossal cyst

Dermoid

Branchial cyst

Rhabdomyosarcoma

Dermoid cysts are usually multiloculated and heterogeneous. Most are located
above the hyoid, and their appearances on imaging differentiate them from
thyroglossal cysts.

Please rate this question:

 Discuss and give feedback

Next question 

Neck Masses in Children

Thyroglossal cyst Located in the anterior triangle, usually in the


midline and below the hyoid (65% cases)
Derived from remnants of the thyroglossal duct
Thin walled and anechoic on USS (echogenicity
suggests infection of cyst)
Branchial cyst Six branchial arches separated by branchial clefts
Incomplete obliteration of the branchial

apparatus may result in cysts, sinuses or fistulae


75% of branchial cysts originate from the second
branchial cleft
Usually located anterior to the
sternocleidomastoid near the angle of the
mandible
Unless infected the fluid of the cyst has a similar
consistency to water and is anechoic on USS
gathered by dr. elbarky.

Dermoids Derived from pleuripotent stem cells and are


located in the midline
Most commonly in a suprahyoid location
They have heterogeneous appearances on
imaging and contain variable amounts of calcium
and fat

Thyroid gland True thyroid lesions are rare in children and


usually represent thyroglossal cysts or tumours
like lymphoma

Lymphatic Usually located posterior to the


malformations sternocleidomastoid
Cystic hygroma result from occlusion of
lymphatic channels
The painless, fluid filled, lesions usually present
prior to the age of 2
They are often closely linked to surrounding
structures and surgical removal is difficult
They are typically hypoechoic on USS

Infantile May present in either triangle of the neck


haemangioma Grow rapidly initially and then will often
spontaneously regress
Plain x-rays will show a mass lesion, usually
containing calcified phleboliths
As involution occurs the fat content of the
lesions increases

Lymphadenopathy Located in either triangle of the neck


May be reactive or neoplastic
Generalised lymphadenopathy usually secondary
to infection in children (very common)
Next question 

Display my notes on this topic

          

Save my notes
gathered by dr. elbarky.

Question stats

A 18%
B 27.6%
C 33.8%
D 14.7%
E 5.9%

33.8% of users answered this question correctly

Search eMRCS

Search term Go

 Open MRCS Part A textbook (../review/textbook.php)

External links

+ Suggest a link

Dashboard

3

Question 64 of 364

 

A 70 year old lady who has never smoked is identified as having a peripherally
sited mass in her left lung. What is the most likely underlying diagnosis?

Small cell lung cancer


gathered by dr. elbarky.

Gastric cancer metastasis

Adenocarcinoma

Squamous cell carcinoma

Lymphoma

Adenocarcinomas are the most common tumour type present in never smokers.
They are usually located at the periphery.

Please rate this question:

 Discuss and give feedback

Next question 

Lung cancer

Lung cancers may be classified according to histological subtypes. The main


distinction is between small cell and non small cell lung cancer. Non small cell
lung cancer is the most common variant and accounts for 80% of all lung cancers.

Non small cell lung cancer


These share common features of prognosis and management. They comprise the
following tumours:
Squamous cell carcinoma (25% cases)
Adenocarcinoma (40% cases)
Large cell carcinoma (10% cases)

Paraneoplastic features and early disease dissemination are less likely than with
small cell lung carcinoma. Adenocarcinoma is the most common lung cancer type
encountered in never smokers.

Small cell lung carcinoma


Small cell lung carcinomas are comprised of cells with a neuro endocrine
differentiation. The neuroendocrine hormones may be released from these cells
with a wide range of paraneoplastic associations. These tumours are strongly
associated with smoking and will typically arise in the larger airways. They
disseminate early in the course of the disease and although they are usually
chemosensitive this seldom results in long lasting remissions.

Next question 
gathered by dr. elbarky.

Display my notes on this topic

          

Save my notes

Question stats

A 24%
B 7.2%
C 47.6%
D 14.4%
E 6.8%

47.6% of users answered this question correctly

Search eMRCS

Search term Go

 Open MRCS Part A textbook (../review/textbook.php)

External links

Question 65 of 364

 

A 40 year old man undergoes a complex appendicectomy and the wound fails to
heal satisfactorily. The wound site itself is associated with multiple sinuses and
fistulas. Pus is sent for microbiology and shows gram positive organisms and
sulphur granules. What is the most likely underlying diagnosis?
gathered by dr. elbarky.

Infection with Staphylococcus aureus

Infection with Bacteroides fragilis

Actinomycosis

Crohns disease

Ulcerative colitis

The presence of chronic sinuses together with gram positive organisms and
sulphur granules is highly suggestive of Actinomycosis. Crohns disease is
associated with multiple fistulae, but not gram positive organisms with sulphur
granules.

Please rate this question:

 Discuss and give feedback

Next question 

Actinomycosis

Chronic, progressive granulomatous disease caused by filamentous gram positive


anaerobic bacteria from the Actinomycetaceae family.

Actinomyces are commensal bacteria that become pathogenic when a mucosal


barrier is breached.

The disease most commonly occurs in the head and neck, although it may also
occur in the abdominal cavity and in the thorax.
The mass will often enlarge across tissue planes with the formation of multiple
sinus tracts.

Abdominopelvic actinomycosis occurs most frequently in individuals that have had


appendicitis (65%) cases.

Pathology
On histological examination gram positive organisms and evidence of
sulphur granules.
Sulphur granules are colonies of organisms that appear as round or oval
basophilic masses.
They are also seen in other conditions such as nocardiosis.
gathered by dr. elbarky.

Treatment
Long term antibiotic therapy usually with penicillin.
Surgical resection is indicated for extensive necrotic tissue, non healing
sinus tracts, abscesses or where biopsy is needed to exclude malignancy.

References
Wong V, Turmezei T and Weston V. Actinomycosis. BMJ 2011;343d6099.

Next question 

Display my notes on this topic

          

Save my notes

Question stats

A 18.4%
B 16.1%
C 48.9%
D 10.2%
E 6.3%

48.9% of users answered this question correctly


Question 66 of 364

 

A 28 year old man develops an acute paronychia and subsequent spreading


sepsis. The tissue exudate has a higher protein content than normal tissue
because?

Breakdown of tissue cells release protein


gathered by dr. elbarky.

Capillary walls are more permeable

Increased blood flow transports more protein into the area

Intracapillary pressure is raised

Plasma cells release gamma globulin

The increased permeability allows the exudation of plasma proteins.

Please rate this question:

 Discuss and give feedback

Next question 

Acute inflammation
Inflammation is the reaction of the tissue elements to injury. Vascular changes
occur, resulting in the generation of a protein rich exudate. So long as the injury
does not totally destroy the existing tissue architecture, the episode may resolve
with restoration of original tissue architecture.

Vascular changes
Vasodilation occurs and persists throughout the inflammatory phase.
Inflammatory cells exit the circulation at the site of injury.
The equilibrium that balances Starlings forces within capillary beds is
disrupted and a protein rich exudate will form as the vessel walls also
become more permeable to proteins.
The high fibrinogen content of the fluid may form a fibrin clot. This has
gathered by dr. elbarky.

several important immunomodulatory functions.

Sequelae

Resolution Typically occurs with minimal initial injury


Stimulus removed and normal tissue
architecture results

Organisation Delayed removal of exudate


Tissues undergo organisation and usually
fibrosis

Suppuration Typically formation of an abscess or an


empyema
Sequestration of large quantities of dead
neutrophils

Progression to chronic Coupled inflammatory and reparative


inflammation activities
Usually occurs when initial infection or
suppuration has been inadequately managed

Causes
Infections e.g. Viruses, exotoxins or endotoxins released by bacteria
Chemical agents
Physical agents e.g. Trauma
Hypersensitivity reactions
Tissue necrosis

Presence of neutrophil polymorphs is a histological diagnostic feature of acute


inflammation
Next question 

Display my notes on this topic

          

Save my notes
gathered by dr. elbarky.

Question stats

A 19.4%
B 54.3%
C 9%
D 8.3%
E 9.1%

54.3% of users answered this question correctly

Search eMRCS

Search term Go

 Open MRCS Part A textbook (../review/textbook.php)

External links

+ Suggest a link

Dashboard

3

Question 67 of 364

 

An 18 year old male presents with lethargy, night sweats and on examination is
found to have left supraclavicular lymphadenopathy. A surgical registrar performs
a left supraclavicular lymph node biopsy. The pathologist identifies Reed-
Sternberg cells on the subsequent histology sections, what is the most likely
diagnosis?
gathered by dr. elbarky.

Metastatic gastric cancer

Hodgkins lymphoma

Non Hodgkins lymphoma

Tuberculosis

None of the above

Reed-Sternberg cells are characteristic histological cell type found in Hodgkins


disease.

Please rate this question:

 Discuss and give feedback

Next question 

Lymphadenopathy

Lymphadenopathy in the neck, axillae, groins and abdomen


Need to note: solitary/multiple, defined/indistinct, hard/rubbery/soft,
tender/painless

Causes of lymphadenopathy

Mnemonic: Hodgkins disease

H aematological: Hodgkins lymphoma, NHL, Leukaemia


O ncological: metastases
D ermatopathic lymphadenitis
G aucher's disease
K awasaki disease
I nfections: TB, glandular fever, Syphilis
N iemann Pick disease
S erum sickness
D rug reaction (phenytoin)
I mmunological (SLE)
S arcoidosis
E ndocrinological (Hyperthyroidism)
A ngioimmunoplastic lymphadenopathy
gathered by dr. elbarky.

S LE
E osinophilic granulomatosis

Next question 

Display my notes on this topic

          

Save my notes

Question stats

A 6.3%
B 66.4%
C 14.3%
D 6.3%
E 6.7%

66.4% of users answered this question correctly

Search eMRCS

Search term Go

Question 68 of 364

 

An unusually tall 43 year old lady presents to the surgical clinic with bilateral
inguinal hernias. She develops chest pain and collapses. As part of her
investigations a chest x-ray shows evidence of mediastinal widening. What is the
most likely underlying diagnosis?
gathered by dr. elbarky.

Pulmonary embolus

Aortic dissection

Tietze syndrome

Boerhaaves syndrome

Myocardial infarct

Marfans syndrome may present with a variety of connective tissue disorders such
as bilateral inguinal hernia. They are at high risk of aortic dissection, as in this
case.

Please rate this question:

 Discuss and give feedback

Next question 

Aortic dissection

More common than rupture of the abdominal aorta


33% of patients die within the first 24 hours, and 50% die within 48 hours if
no treatment received
Associated with hypertension
Features of aortic dissection: tear in the intimal layer, followed by formation
and propagation of a subintimal hematoma. Cystic medial necrosis
(Marfan's)
Most common site of dissection: 90% occurring within 10 centimetres of the
aortic valve
Stanford Classification

Type Location Treatment

A Ascending aorta/ aortic root Surgery- aortic root replacement

B Descending aorta Medical therapy with antihypertensives

DeBakey classification

Type Site affected


gathered by dr. elbarky.

I Ascending aorta, aortic arch, descending aorta

II Ascending aorta only

III Descending aorta distal to left subclavian artery

Clinical features
Tearing, sudden onset chest pain (painless 10%)
Hypertension or Hypotension
A blood pressure difference (in each arm) greater than 20 mm Hg
Neurologic deficits (20%)

Investigations
CXR: widened mediastinum, abnormal aortic knob, ring sign, deviation of the
trachea/oesophagus
CT angiography of the thoracic aorta
MRI angiography
Conventional angiography (now rarely used diagnostically)

Management
Beta-blockers: aim HR 60-80 bpm and systolic BP 100-120 mm Hg
For type A dissections the standard of care is aortic root replacement

Next question 

Display my notes on this topic

          
Save my notes

Question stats

A 7.5%
B 66.4%
C 9.4%
D 9.7%
gathered by dr. elbarky.

E 7%

66.4% of users answered this question correctly

Search eMRCS

Search term Go

 Open MRCS Part A textbook (../review/textbook.php)

External links

+ Suggest a link

Dashboard

8

Question 69 of 364

 

A 34 year old man presents to the surgical clinic 8 months following a laparotomy
for a ruptured spleen. He complains of a nodule in the centre of his laparotomy
wound. This is explored surgically and a stitch granuloma is found and excised.
From which of the following cell types do granulomata arise?
gathered by dr. elbarky.

Polymorpho nucleocytes

Plasma cells

Reed- Sternberg cells

Platelets

Macrophages

Granulomas are organised collections of macrophages

Macrophages give origin to granulomas.

Please rate this question:

 Discuss and give feedback

Next question 

Chronic inflammation

Overview
Chronic inflammation may occur secondary to acute inflammation.In most cases
chronic inflammation occurs as a primary process. These may be broadly viewed
as being one of three main processes:
Persisting infection with certain organisms such as Mycobacterium
tuberculosis which results in delayed type hypersensitivity reactions and
inflammation.
Prolonged exposure to non-biodegradable substances such as silica or
suture materials which may induce an inflammatory response.
Autoimmune conditions involving antibodies formed against host antigens.

Acute vs. Chronic inflammation

Acute inflammation Chronic inflammation

Changes to existing vascular structure and Angiogenesis predominates


increased permeability of endothelial cells

Infiltration of neutrophils Macrophages, plasma cells


and lymphocytes predominate
gathered by dr. elbarky.

Process may resolve with: Healing by fibrosis is the main


Suppuration result
Complete resolution
Abscess formation
Progression to chronic inflammation
Healing by fibrosis

Granulomatous inflammation
A granuloma consists of a microscopic aggregation of macrophages (with
epithelial type arrangement =epithelioid). Large giant cells may be found at the
periphery of granulomas.

Mediators
Growth factors released by activated macrophages include agents such as
interferon and fibroblast growth factor (plus many more). Some of these such as
interferons may have systemic features resulting in systemic symptoms and signs,
which may be present in individuals with long standing chronic inflammation.

The finding of granulomas is pathognomonic of chronic inflammation, as illustrated


in this biopsy from a patient with colonic Crohns disease
gathered by dr. elbarky.

(https://d2zgo9qer4wjf4.cloudfront.net/images_eMRCS/swb093b.jpg)
Image sourced from Wikipedia
(https://d2zgo9qer4wjf4.cloudfront.net
(http://en.wikipedia.org/wiki/Crohn%27s
/images_eMRCS/swb093b.jpg)
disease)

Next question 

Display my notes on this topic

          

Save my notes

Question stats

A 13.3%
B 9.3%
C 8.3%
D 7%
E 62%

62% of users answered this question correctly

Search eMRCS

Question 70 of 364

 

A 38 year old man has been suffering from mechanical back pain for several years.
One morning he awakes from sleep and feels a sudden onset of pain in his back
radiating down his left leg. Which of the following events is most likely to account
for his symptoms?
gathered by dr. elbarky.

Prolapse of inner annulus fibrosus

Prolapse of outer annulus fibrosus

Prolapse of nucleus pulposus

Rupture of the ligamentum flavum

None of the above

The symptoms would be most likely the result of intervertebral disk prolapse. In
disk prolapse the nucleus pulposus is the structure which usually herniates.

Please rate this question:

 Discuss and give feedback

Next question 

Intervertebral discs

Consist of an outer annulus fibrosus and an inner nucleus pulposus.


The anulus fibrosus consists of several layers of fibrocartilage.
The nucleus pulposus contains loose fibres suspended in a mucoprotein gel
with the consistency of jelly. The nucleus of the disc acts as a shock
absorber.
Pressure on the disc causes posterior protrusion of the nucleus pulposus.
Most commonly in the lumbrosacral and lower cervical areas.
The discs are separated by hyaline cartilage.
There is one disc between each pair of vertebrae, except for C1/2 and the
sacrococcygeal vertebrae.
Next question 

Display my notes on this topic

          

Save my notes
gathered by dr. elbarky.

Question stats

A 9.7%
B 14.7%
C 59.2%
D 8.9%
E 7.4%

59.2% of users answered this question correctly

Search eMRCS

Search term Go

 Open MRCS Part A textbook (../review/textbook.php)

External links

+ Suggest a link

Dashboard

3

Question 71 of 364

 

A 65 year old male attends surgical out patients with epigastric discomfort. He has
recently been diagnosed with diabetes by the GP and is a heavy smoker. An OGD is
normal. What is the most likely diagnosis?

Pancreatic adenocarcinoma
gathered by dr. elbarky.

Pancreatic squamous cell carcinoma

Pancreatic insulinoma

Pancreatic glucagonoma

Pancreatic gastrinoma

The dominant differential diagnosis should be of pancreatic adenocarcinoma in


this setting. Glucagonomas are very rare and may be associated with a bullous
rash.

Please rate this question:

 Discuss and give feedback

Next question 

Pancreatic cancer

Adenocarcinoma
Risk factors: Smoking, diabetes, adenoma, familial adenomatous polyposis
Mainly occur in the head of the pancreas (70%)
Spread locally and metastasizes to the liver
Carcinoma of the pancreas should be differentiated from other
periampullary tumours with better prognosis

Clinical features
Weight loss
Painless jaundice
Epigastric discomfort (pain usually due to invasion of the coeliac plexus is a
late feature)
Pancreatitis
Trousseau's sign: migratory superficial thrombophlebitis

Investigations
USS: May miss small lesions
CT Scanning (pancreatic protocol). If unresectable on CT then no further
staging needed
PET/CT for those with operable disease on CT alone
ERCP/ MRI for bile duct assessment
gathered by dr. elbarky.

Staging laparoscopy to exclude peritoneal disease

Management
Head of pancreas: Whipple's resection (SE dumping and ulcers). Newer
techniques include pylorus preservation and SMA/ SMV resection
Carcinoma body and tail: poor prognosis, distal pancreatectomy, if operable
Usually adjuvent chemotherapy for resectable disease
ERCP and stent for jaundice and palliation
Surgical bypass may be needed for duodenal obstruction

Next question 

Display my notes on this topic

          

Save my notes

Question stats
A 41.5%
B 7.9%
C 15.4%
D 21.5%
E 13.7%

41.5% of users answered this question correctly


gathered by dr. elbarky.

Search eMRCS

Search term Go

 Open MRCS Part A textbook (../review/textbook.php)

External links

+ Suggest a link

Dashboard

10

11

12

Question 72 of 364

 

A 20 year old lady is referred to the vascular clinic. She has been feeling generally
unwell for the past six weeks. She works as a typist and has noticed increasing
pain in her forearms whilst working. On examination, she has absent upper limb
pulses. Her ESR is measured and mildly elevated. What is the most likely
diagnosis?
gathered by dr. elbarky.

Polyarteritis nodosa

Wegeners granulomatosis

Giant cell arteritis

Takayasu's arteritis

Buergers disease

Takayasus arteritis may be divided into acute systemic phases and the chronic
pulseless phase. In the latter part of the disease process the patient may complain
of symptoms such as upper limb claudication. In the later stages of the condition
the vessels will typically show changes of intimal proliferation, together with band
fibrosis of the intima and media.

Please rate this question:

 Discuss and give feedback

Next question 

Vasculitis

The vasculitides are a group of conditions characterised by inflammation of the


blood vessel walls. This may, in turn, compromise vessel integrity. Constitutional
symptoms may be present. Whilst certain disease subtypes are reported to affect
specific vessels, there is often a degree of overlap clinically.

Vessel diameter and vasculitis classification


Aorta and branches Takayasu's arteritis
Buergers disease
Giant cell arteritis

Large and medium sized arteries Buergers disease


Giant cell arteritis
Polyarteritis nodosa

Medium sized muscular arteries Polyarteritis nodosa


Wegeners granulomatosis
gathered by dr. elbarky.

Small muscular arteries Wegeners granulomatosis


Rheumatoid vasculitis

Specific conditions

Takyasu's arteritis Inflammatory, obliterative arteritis affecting aorta


and branches
Females> Males
Symptoms may include upper limb claudication
Clinical findings include diminished or absent
pulses
ESR often affected during the acute phase

Buergers disease Segmental thrombotic occlusions of the small


and medium sized lower limb vessels
Commonest in young male smokers
Proximal pulses usually present, but pedal pulses
are lost
An acuter hypercellular occlusive thrombus is
often present
Tortuous corkscrew shaped collateral vessels
may be seen on angiography

Giant cell arteritis Systemic granulomatous arteritis that usually


affects large and medium sized vessels
Females > Males
Temporal arteritis is commonest type
Granulomatous lesions may be seen on biopsy
(although up to 50% are normal)
Polyarteritis nodosa Systemic necrotising vasculitis affecting small
and medium sized muscular arteries

Most common in populations with high


prevalence of hepatitis B
Renal disease is seen in 70% cases
Angiography may show saccular or fusiform
aneurysms and arterial stenoses

Wegeners Predominantly affects small and medium sized


granulomatosis arteries
gathered by dr. elbarky.

Systemic necrotising granulomatous vasculitis


Cutaneous vascular lesions may be seen
(ulceration, nodules and purpura)
Sinus imaging may show mucosal thickening and
air fluid levels

Treatment
Conditions such as Buergers disease are markedly helped by smoking cessation.
Immunosupression is the main treatment for vasculitides.

Next question 

Display my notes on this topic

          

Save my notes

Question stats
A 14.9%
B 8.9%
C 10.2%
D 54.2%
E 11.9%

54.2% of users answered this question correctly


gathered by dr. elbarky.

Search eMRCS

Search term Go

 Open MRCS Part A textbook (../review/textbook.php)

External links

+ Suggest a link

Dashboard

10

11

12

Question 73 of 364

 

A 40 year old man presents with obstructive jaundice and dysphagia. Twenty years
previously he underwent a right hemicolectomy for a mucinous right sided colonic
carcinoma. He was subsequently diagnosed as having Lynch syndrome. A recent
colonoscopy was normal. What is the most likely cause of his jaundice?
gathered by dr. elbarky.

Hepatocellular carcinoma

Liver metastasis from colonic cancer

Pancreatic carcinoma

Duodenal carcinoma

Metastatic gastric carcinoma

Lynch syndrome usually results in colonic cancer which is right sided and
mucinous. The next most common site to be affected is the uterus. The stomach
is at particular risk and this risk is up to 10 times greater in HNPCC (Lynch)
patients than the general population. Duodenal adenomas (and rarely carcinoma)
are usually seen in association with FAP. Whilst pancreatic carcinoma is
associated with HNPCC it is far less likely to occur than gastric cancer.
We are often asked how these patients become jaundiced, this occurs as a result
of nodal spread along the hepatoduodenal ligament nodes to occlude the porta
hepatis resulting in jaundice.

Please rate this question:

 Discuss and give feedback

Next question 

Genetics of colorectal cancer

The lifetime risk of colorectal cancer in the UK population is 5%. Up to 5% of newly


diagnosed bowel cancers will be in those individuals who have a high genetically
acquired risk of bowel cancer. Cancers arising in the low-moderate genetic risk
group comprise approximately 30% of newly diagnosed bowel cancer.
Genetics of inherited colorectal cancer syndromes

Genes
Syndrome Features implicated

FAP More than 100 adenomatous polyps APC (over 90%)


affecting the colon and rectum. Duodenal
and fundic glandular polyps

Gardner As FAP but with desmoid tumours and APC


syndrome mandibular osteomas
gathered by dr. elbarky.

Turcots Polyposis and colonic tumours and CNS APC +MLH1


syndrome tumours and PMS2

HNPCC Colorectal cancer without extensive MSH2, MLH1,


polyposis. Endometrial cancer, renal and PMS2 and
CNS GTBP

Peutz-Jeghers Hamartomatous polyps in GI tract and LKB1 andSTK11


syndrome increased risk of GI malignancy (in up to 70%)

Cowden Multiple hamartomas (see below) PTEN (85%)


disease

MYH Autosomal recessive, multiple MYH


associated adenomatous polyps in GI tract, those in
polyposis colon having somatic KRAS mutations

FAP
Autosomal dominant condition, affects 1 in 12,000. Accounts for 0.5% of all CRCs.
Lifetime incidence of colorectal cancer in untreated FAP =100%. Up to 25% cases
are caused by de-novo germ line mutations and show no prior family history. The
APC tumour suppressor gene is affected in most cases.

APC in non inherited colorectal cancer


Up to 80% of sporadic colorectal cancers will have somatic mutations that
inactivate APC[1]. Both alleles are usually affected. Although the APC protein more
than likely has multiple critical cellular functions, the best-established role for APC
in the cancer process is as a major binding partner and regulator of the β- catenin
protein in the so-called canonical or β- catenin dependent Wnt signaling pathway.

HNPCC (Lynch syndrome)


HNPCC cancers differ from conventional tumours in a number of respects. In the
colon the tumours are more likely to be right sided, histologically they are more
likely to be mucinous and have dense lymphocytic infiltrates. To be diagnosed as
having HNPCC individuals must show typically HNPCC tumours in at least three
individuals, (one of whom must be a first degree relative to the other two). In at
least two successive generations. At least one cancer must be diagnosed under
the age of 50. FAP must be excluded and tumours should be verified by
pathological identification (Amsterdam criteria). The genetic changes in HNPCC
stem primarily from microsatellite instability affecting DNA mismatch repair
genes. In HNPCC the mismatch repair genes most commonly implicated include;
MSH2 and MLH1 and these occur in up to 70% of people with HNPCC. The finding
of microsatellite instability is unusual in sporadic colorectal cancers.
Approximately 60% of individuals who fulfill the Amsterdam criteria will not be
found to have evidence of mismatch repair gene defects on genetic testing. The
risk of developing colorectal cancer in those who have not demonstrated mutation
of the mis match repair genes is increased if they fulfill the Amsterdam criteria, but
gathered by dr. elbarky.

not
the extent that it is increased in those who fulfill the criteria AND have evidence of
mis match repair gene defects.

KRAS Mutations
The RAS family of small G proteins act as molecular switches downstream of
growth factor receptors. KRAS and the other two members of the family; HRAS and
NRAS, are the site of mutation in approximately 40% of colorectal cancers. When
adenomas are examined the proportion of adenomas less than 1cm showing
KRAS mutations was only 10% which contrasts with 50% in those lesions greater
than 1cm.

p53 mutations
The p53 protein functions as a key transcriptional regulator of genes that encode
proteins with functions in cell-cycle checkpoints at the G1/S and G2/M boundaries,
in promoting apoptosis, and in restricting angiogenesis . As such, selection for p53
defects at the adenoma-carcinoma transition may reflect the fact that stresses on
tumor cells activate cell-cycle arrest, apoptotic, and antiangiogenic pathways in
cells with wild-type p53 function. Many colonic tumours will demonstrate changes
in the p53 gene that may facilitate tumour progression through from adenoma to
carcinoma.

Cowden syndrome
Also known as multiple hamartoma syndrome. Rare autosomal dominant
condition with incidence of 1 in 200,000.. It is characterised by multiple
mucocutaneous lesions, trichilemmomas, oral papillomas and acral keratosis.
Most often diagnosed in third decade of life. Breast carcinoma may occur in up to
50% of patients and conditions such as fibrocystic disease of the breast may
occur in 75% of women. Thyroid disease occurs in 75% and may include
malignancy. Endoscopic screening will identify disease in up to 85% although the
small bowel is rarely involved. There is a 15-20% risk of developing colorectal
cancer and regular colonoscopic screening from age 45 is recommended.

Terminology
Oncogene Oncogenes are genes which have the potential to induce
cellular proliferation and avoid apoptosis. Oncogene mutations
are general gain of function and are therefore dominant.
Increased expression of oncogenes are found in most tumours

Tumour These genes generally inhibit cellular proliferation or induce


suppressor apoptosis. Mutations in tumour suppressor genes are
gene generally loss of function mutations, and are therefore
recessive. Mutations in both tumour suppressor gene alleles
allow cells to proliferate without restraint
gathered by dr. elbarky.

References
1. Fearon, E.R. and B. Vogelstein, A genetic model for colorectal tumorigenesis.
Cell, 1990. 61(5): p. 759-67.

Next question 

Display my notes on this topic

          

Save my notes

Question stats

A 11.8%
B 20.5%
C 18.6%
D 18.4%
E 30.7%

30.7% of users answered this question correctly

Search eMRCS

Search term Go

Question 74 of 364

 

Which syndrome is likely to be present in a 28 year old man who presents with a
locally advanced mucinous carcinoma of the caecum. There are scanty polyps in the
remaining colon. His father died from colorectal cancer aged 34.

FAP
gathered by dr. elbarky.

Lynch syndrome

Cowden disease

MYH associated polyposis

Peutz-Jeghers syndrome

Lynch syndrome is likely when right sided colonic cancers occur at a young age.
These tumours are often poorly differentiated and mucinous. The Amsterdam
criteria can be used to identify families at risk who may benefit from genetic testing.

Please rate this question:

 Discuss and give feedback

Next question 

Polyposis syndromes

Genetic Screening and Associated


Syndrome defect Features management disorders
Genetic Screening and Associated
Syndrome defect Features management disorders

Familial Mutation of Typically over If known to be Gastric


adenomatous APC gene 100 colonic at risk then fundal
polyposis (80%) cases, adenomas predictive polyps
dominant Cancer risk of genetic testing (50%).
100% as teenager Duodenal
20% are new Annual flexible polyps 90%.
mutations sigmoidoscopy If severe
from 15 years duodenal
If no polyps polyposis
gathered by dr. elbarky.

found then 5 cancer risk


yearly of 30% at 10
colonoscopy years.
started at age Abdominal
20 desmoid
Polyps found = tumours.
resectional
surgery
(resection and
pouch Vs sub
total
colectomy and
IRA)

MYH Biallelic Multiple colonic Once identified Duodenal


associated mutation of polyps resection and polyposis in
polyposis mut Y Later onset ileoanal pouch 30%
human right sided reconstruction Associated
homologue cancers more is with
(MYH) on common than in recommended increased
chromosome FAP Attenuated risk of breast
1p, recessive 100% cancer phenotype - cancer (self
risk by age 60 regular examination)
colonoscopy
Genetic Screening and Associated
Syndrome defect Features management disorders

Peutz STK11 Multiple benign Annual Malignancies


-Jeghers (LKB1) intestinal examination at other sites
syndrome mutation on hamartomas Pan intestinal Classical
chromosome Episodic endoscopy pigmentation
19 in some obstruction and every 2-3 years pattern
(but not all) intussceception
cases, Increased risk
dominant of GI cancers
(colorectal
gathered by dr. elbarky.

cancer 20%,
gastric 5%)
Increased risk
of breast,
ovarian, cervical
pancreatic and
testicular
cancers

Cowden Mutation of Macrocephaly Targeted Breast


disease PTEN gene Multiple individualised cancer (81%
on intestinal screening risk)
chromosome hamartomas Thyroid
10q22, Multiple cancer and
dominant trichilemmomas non toxic
89% risk of goitre
cancer at any Uterine
site cancer
16% risk of
colorectal
cancer

HNPCC Germline Colo rectal Colonoscopy Extra colonic


(Lynch mutations of cancer 30-70% every 1-2 years cancers
syndrome) DNA Endometrial from age 25
mismatch cancer 30-70% Consideration
repair genes Gastric cancer of prophylactic
5-10% surgery
Scanty colonic Extra colonic
polyps may be surveillance
present recommended
Colonic
tumours likely
to be right sided
and mucinous
Next question 

Display my notes on this topic

          

Save my notes
gathered by dr. elbarky.

Question stats

A 26.6%
B 47.8%
C 9.5%
D 7.8%
E 8.2%

47.8% of users answered this question correctly

Search eMRCS

Search term Go

 Open MRCS Part A textbook (../review/textbook.php)

External links

+ Suggest a link

Dashboard

3

Question 75 of 364

 

A 22 year old man is investigated for weight loss. A duodenal biopsy taken as part
of his investigations shows total villous atrophy and lymphocytic infiltrate. He has
a skin lesion that has small itchy papules. What is the most likely underlying
diagnosis?
gathered by dr. elbarky.

Pyoderma gangrenosum

Dermatitis herpetiformis

Bullous pemphigoid

Acanthosis nigricans

Pemphigus vulgaris

The patient has coeliac disease and this is classically associated with dermatitis
herpetiformis.

Please rate this question:

 Discuss and give feedback

Next question 

Skin Diseases

Skin lesions may be referred for surgical assessment, but more commonly will
come via a dermatologist for definitive surgical management.

Skin malignancies include basal cell carcinoma, squamous cell carcinoma and
malignant melanoma.

Basal Cell Carcinoma


Most common form of skin cancer.
Commonly occur on sun exposed sites apart from the ear.
Sub types include nodular, morphoeic, superficial and pigmented.
Typically slow growing with low metastatic potential.
Standard surgical excision, topical chemotherapy and radiotherapy are all
successful.
As a minimum a diagnostic punch biopsy should be taken if treatment other
than standard surgical excision is planned.

Squamous Cell Carcinoma


Again related to sun exposure.
May arise in pre - existing solar keratoses.
May metastasize if left.
Immunosupression (e.g. following transplant), increases risk.
Wide local excision is the treatment of choice and where a diagnostic
gathered by dr. elbarky.

excision biopsy has demonstrated SCC, repeat surgery to gain adequate


margins may be required.

Malignant Melanoma

The main diagnostic features (major Secondary features (minor


criteria): criteria)
Change in size Diameter >6mm
Change in shape Inflammation
Change in colour Oozing or bleeding
Altered sensation

Treatment
Suspicious lesions should undergo excision biopsy. The lesion should be
removed in completely as incision biopsy can make subsequent
histopathological assessment difficult.
Once the diagnosis is confirmed the pathology report should be reviewed to
determine whether further re-excision of margins is required (see below):

Margins of excision-Related to Breslow thickness

Lesions 0-1mm thick 1cm

Lesions 1-2mm thick 1- 2cm (Depending upon site and pathological features)

Lesions 2-4mm thick 2-3 cm (Depending upon site and pathological features)

Lesions >4 mm thick 3cm

Marsden J et al. Revised UK guidelines for management of Melanoma. Br J


Dermatol 2010 163:238-256.

Further treatments such as sentinel lymph node mapping, isolated limb perfusion
and block dissection of regional lymph node groups should be selectively applied.
Kaposi Sarcoma
Tumour of vascular and lymphatic endothelium.
Purple cutaneous nodules.
Associated with immuno supression.
Classical form affects elderly males and is slow growing.
Immunosupression form is much more aggressive and tends to affect those
with HIV related disease.

Non malignant skin disease

Dermatitis Herpetiformis
gathered by dr. elbarky.

Chronic itchy clusters of blisters.


Linked to underlying gluten enteropathy (coeliac disease).

Dermatofibroma
Benign lesion.
Firm elevated nodules.
Usually history of trauma.
Lesion consists of histiocytes, blood vessels and fibrotic changes.

Pyogenic granuloma
Overgrowth of blood vessels.
Red nodules.
Usually follow trauma.
May mimic amelanotic melanoma.

Acanthosis nigricans
Brown to black, poorly defined, velvety hyperpigmentation of the skin.
Usually found in body folds such as the posterior and lateral folds of the
neck, the axilla, groin, umbilicus, forehead, and other areas.
The most common cause of acanthosis nigricans is insulin resistance,
which leads to increased circulating insulin levels. Insulin spillover into the
skin results in its abnormal increase in growth (hyperplasia of the skin).
In the context of a malignant disease, acanthosis nigricans is a
paraneoplastic syndrome and is then commonly referred to as acanthosis
nigricans maligna. Involvement of mucous membranes is rare and suggests
a coexisting malignant condition.

Next question 

Display my notes on this topic


          

Save my notes

Question stats

A 21.6%
B 51.2%
C 8.4%
gathered by dr. elbarky.

D 9.7%
E 9.1%

51.2% of users answered this question correctly

Search eMRCS

Search term Go

 Open MRCS Part A textbook (../review/textbook.php)

External links

+ Suggest a link

Dashboard

7

Question 76 of 364

 

A 52 year old man is having a blood transfusion after losing blood from a total
knee replacement. He is normally fit and well. Three hours into the transfusion he
complains of sudden onset abdominal pain and nausea. His temperature is 39
degrees, Blood pressure 98/42 mmHg, HR 105 bpm and saturations 94% air. His
urine appears dark. What is the most likely diagnosis?
gathered by dr. elbarky.

Delayed haemolytic transfusion reaction

Neutrophilic febrile reaction

Acute haemolytic transfusion reaction

Sickle cell crisis

Transfusion associated lung injury

Rapid intravascular haemolysis leading to shock, DIC and death can occur with this
reaction.

Please rate this question:

 Discuss and give feedback

Next question 

Blood transfusion reactions

Acute transfusion reactions present as adverse signs or symptoms during or


within 24 hours of a blood transfusion. The most frequent reactions are fever,
chills, pruritus, or urticaria, which typically resolve promptly without specific
treatment or complications. Other signs occurring in temporal relationship with a
blood transfusion, such as severe dyspnoea, pyrexia, or loss of consciousness may
be the first indication of a more severe potentially fatal reaction.
The causes of adverse reactions are multi-factorial. Immune mediated reactions,
some of the most feared, occur as a result of component mismatch, the
commonest cause of which is clerical error. More common, non immune mediated,
complications may occur as a result of product contamination, this may be
bacterial or viral.
Transfusion related lung injury is well recognised and there are two proposed
mechanisms which underpin this. One involves the sequestration of primed
neutrophils within the recipient pulmonary capillary bed. The other proposed
mechanism suggests that HLA mismatches between donor neutrophils and
recipient lung tissue is to blame.
The table below summarises the main types of transfusion reaction.

Immune mediated Non immune mediated

Pyrexia Hypocalcaemia
gathered by dr. elbarky.

Alloimmunization CCF

Thrombocytopaenia Infections

Transfusion associated lung injury Hyperkalaemia

Graft vs Host disease

Urticaria

Acute or delayed haemolysis

ABO incompatibility

Rhesus incompatibility

Next question 

Display my notes on this topic

          

Save my notes

Question stats

A 19.7%
B 7.6%
C 58.6%

Question 77 of 364

 

A 66 year old lady presents with pain in her right hip. It has been increasing over
the previous three weeks and waking her from sleep. On examination, she is tender
on internal rotation. Blood tests reveal a markedly elevated serum calcium and
alkaline phosphatase levels. What is the likely cause?
gathered by dr. elbarky.

Metastatic breast cancer

Chondrosarcoma

Osteoporosis

Pagets disease

Rickets

Increasing pain at rest, together with increased serum calcium and alkaline
phosphatase are most likely to represent metastatic tumour to bone.
Chondrosarcomas do occur in the pelvis but are not associated with increased
serum calcium and typically have a longer history.

Please rate this question:

 Discuss and give feedback

Next question 

Bone disease

Disease Features Treatment


Disease Features Treatment

Pagets Focal bone resorption Bisphosphonates


followed by excessive and
chaotic bone deposition
Affects (in order): spine,
skull, pelvis and femur
Serum alkaline
phosphatase raised (other
parameters normal)
Abnormal thickened,
sclerotic bone on x-rays
gathered by dr. elbarky.

Risk of cardiac failure with


>15% bony involvement
Small risk of sarcomatous
change

Osteoporosis Excessive bone resorption Bisphosphonates,


resulting in demineralised calcium and vitamin D
bone
Commoner in old age
Increased risk of
pathological fracture,
otherwise asymptomatic
Alkaline phosphatase
normal, calcium normal

Secondary Bone destruction and Radiotherapy,


bone tumours tumour infiltration prophylactic fixation
Mirel scoring used to and analgesia
predict risk of fracture
Appearances depend on
primary (e.g.sclerotic -
prostate, lytic - breast)
Elevated serum calcium
and alkaline phosphatase
may be seen

Next question 

Display my notes on this topic

          
Save my notes

Question stats

A 42.3%
B 11.3%
C 13.3%
D 27.8%
gathered by dr. elbarky.

E 5.3%

42.3% of users answered this question correctly

Search eMRCS

Search term Go

 Open MRCS Part A textbook (../review/textbook.php)

External links

+ Suggest a link

Dashboard

8

Question 78 of 364

 

A 65 year old man presents with a history of progressive dysphagia over the past 4
weeks. For the preceding 5 years he had regularly attended his general practitioner
with symptoms of dyspepsia and reflux. What is the most likely cause?

Leiomyoma of the oesophagus


gathered by dr. elbarky.

Plummer Vinson syndrome

Squamous cell carcinoma of the oesophagus

Adenocarcinoma of the oesophagus

Achalasia

A short history of progressive dysphagia in a middle aged man who has a


background history of reflux is strongly suggestive of malignancy. Long standing
reflux symptoms may be suggestive of a increased risk of developing Barretts
oesophagus. Note that not all patients with Barretts transformation alone are
symptomatic. In such cases, the tumours are adenocarcinoma.

Please rate this question:

 Discuss and give feedback

Next question 

Dysphagia

Causes of dysphagia

Extrinsic Mediastinal masses


Cervical spondylosis
Oesophageal wall Achalasia
Diffuse oesophageal spasm

Hypertensive lower oesophageal sphincter

Intrinsic Tumours
Strictures
Oesophageal web
Schatzki rings

Neurological CVA
gathered by dr. elbarky.

Parkinson's disease
Multiple Sclerosis
Brainstem pathology
Myasthenia Gravis

Investigation
All patients require an upper GI endoscopy unless there are compelling reasons for
this not to be performed. Motility disorders may be best appreciated by
undertaking fluoroscopic swallowing studies.

A full blood count should be performed.

Ambulatory oesophageal pH and manometry studies will be required to evaluate


conditions such as achalasia and patients with GORD being considered for
fundoplication surgery.

Next question 

Display my notes on this topic

          

Save my notes

Question stats

A 6.1%
B 8.8%
C 16.7%
D 57.2%
E 11.2%

57.2% of users answered this question correctly

Search eMRCS

Search term Go
gathered by dr. elbarky.

 Open MRCS Part A textbook (../review/textbook.php)

External links

+ Suggest a link

Dashboard

10

11

12

13

14

Question 78 of 364

 

A 65 year old man presents with a history of progressive dysphagia over the past 4
weeks. For the preceding 5 years he had regularly attended his general practitioner
with symptoms of dyspepsia and reflux. What is the most likely cause?

Leiomyoma of the oesophagus


gathered by dr. elbarky.

Plummer Vinson syndrome

Squamous cell carcinoma of the oesophagus

Adenocarcinoma of the oesophagus

Achalasia

A short history of progressive dysphagia in a middle aged man who has a


background history of reflux is strongly suggestive of malignancy. Long standing
reflux symptoms may be suggestive of a increased risk of developing Barretts
oesophagus. Note that not all patients with Barretts transformation alone are
symptomatic. In such cases, the tumours are adenocarcinoma.

Please rate this question:

 Discuss and give feedback

Next question 

Dysphagia

Causes of dysphagia

Extrinsic Mediastinal masses


Cervical spondylosis
Oesophageal wall Achalasia
Diffuse oesophageal spasm

Hypertensive lower oesophageal sphincter

Intrinsic Tumours
Strictures
Oesophageal web
Schatzki rings

Neurological CVA
gathered by dr. elbarky.

Parkinson's disease
Multiple Sclerosis
Brainstem pathology
Myasthenia Gravis

Investigation
All patients require an upper GI endoscopy unless there are compelling reasons for
this not to be performed. Motility disorders may be best appreciated by
undertaking fluoroscopic swallowing studies.

A full blood count should be performed.

Ambulatory oesophageal pH and manometry studies will be required to evaluate


conditions such as achalasia and patients with GORD being considered for
fundoplication surgery.

Next question 

Display my notes on this topic

          

Save my notes

Question stats

A 6.1%
B 8.8%
C 16.7%
D 57.2%
E 11.2%

57.2% of users answered this question correctly

Search eMRCS

Search term Go
gathered by dr. elbarky.

 Open MRCS Part A textbook (../review/textbook.php)

External links

+ Suggest a link

Dashboard

10

11

12

13

14

Question 79 of 364

 

A 28 year old female attends the gynaecology unit for a D+C following an
incomplete miscarriage. She has previously had recurrent pulmonary embolic
events. After the procedure she is persistently bleeding. Her APTT is 52
(increased). What is the most likely cause?
gathered by dr. elbarky.

Antiphospholipid syndrome

DIC

Haemophilia B

von Willebrand disease

Factor V Leiden deficiency

A combination of thromboembolism and bleeding in a young woman should raise


the possibility of antiphospholipid syndrome. Other features may include foetal
loss, venous and arterial thrombosis and thrombocytopenia. A Lupus
anticoagulant may be present and the APTT is prolonged.

Please rate this question:

 Discuss and give feedback

Next question 

Abnormal coagulation

Cause Factors affected

Heparin Prevents activation factors 2,9,10,11

Warfarin Affects synthesis of factors 2,7,9,10

DIC Factors 1,2,5,8,11


Cause Factors affected

Liver disease Factors 1,2,5,7,9,10,11

Interpretation blood clotting test results

Disorder APTT PT Bleeding time

Haemophilia Increased Normal Normal

von Willebrand's disease Increased Normal Increased


gathered by dr. elbarky.

Vitamin K deficiency Increased Increased Normal

Next question 

Display my notes on this topic

          

Save my notes

Question stats

A 53.7%
B 17.5%
C 7.4%
D 11.8%
E 9.6%

53.7% of users answered this question correctly

Search eMRCS

Search term Go

Question 80 of 364

 

A 28 year old man has a carcinoid tumour identified in his appendix. Blood testing
for which of the substances listed below is likely to be helpful during follow up?

CA19-9
gathered by dr. elbarky.

Alkaline phosphatase

AFP

CEA

Chromogranin A

It is important to distinguish between blood and urine tests for carcinoid


syndrome. Blood tests usually measure chromogranin A,neuron-specific enolase
(NSE), substance P, and gastrin. Urine tests usually measure 5 HIAA, which is a
metabolite of serotonin. Sometimes blood tests for 5 hydroxytryptamine
(serotonin) are also performed.

Please rate this question:

 Discuss and give feedback

Next question 

Carcinoid syndrome

Carcinoid tumours secrete serotonin


Originate in neuroendocrine cells mainly in the intestine (midgut-distal
ileum/appendix)
Can occur in the rectum, bronchi
Hormonal symptoms mainly occur when disease spreads outside the bowel

Clinical features
Onset: insidious over many years
Flushing face
Palpitations
Pulmonary valve stenosis and tricuspid regurgitation causing dyspnoea
Asthma
Severe diarrhoea (secretory, persists despite fasting)

Investigation
5-HIAA in a 24-hour urine collection
Somatostatin receptor scintigraphy
CT scan
Blood testing for chromogranin A
gathered by dr. elbarky.

Treatment
Octreotide
Surgical removal

Next question 

Display my notes on this topic

          

Save my notes

Question stats

A 11%
B 9%
C 10.4%
D 19.6%
E 50.1%

50.1% of users answered this question correctly

Search eMRCS

Search term Go

Question 81 of 364

 

A 12 year old child is admitted with a 12 hour history of colicky right upper
quadrant pain. On examination the child is afebrile and is jaundiced. The abdomen
is soft and non tender at the time of examination. What is the most likely cause?

Infectious hepatitis
gathered by dr. elbarky.

Acute cholecystitis

Cholangitis

Hereditary spherocytosis

Gilberts syndrome

Cholecystitis is unlikely in the absence of focal tenderness

The child is most likely to have hereditary spherocytosis. In these individuals there
may be disease flares precipitated by acute illness. They form small pigment
stones. These may cause biliary colic and some may require cholecystectomy.
Gilbert's syndrome is an inherited condition causing unconjugated
hyperbilirubinaemia. Patients may have jaundice or be asymptomatic. The other
LFTs are normal and Gilbert's may be confirmed with a fasting test or Nicotinic
acid test. There is no need for treatment as it is a benign condition.

Please rate this question:

 Discuss and give feedback

Next question 

Hereditary Spherocytosis

Most common disorder of the red cell membrane, it has an incidence of 1 in 5000.
The abnormally shaped erythrocytes are prone to splenic sequestration and
destruction. This can result in hyperbilirubinaemia, jaundice and splenomegaly. In
older patients an intercurrent illness may increase the rate of red cell destruction
resulting in more acute symptoms.
Severe cases may benefit from splenectomy.

Next question 

Display my notes on this topic

          
gathered by dr. elbarky.

Save my notes

Question stats

A 13.2%
B 12.3%
C 9.1%
D 42.4%
E 23.1%

42.4% of users answered this question correctly

Search eMRCS

Search term Go

 Open MRCS Part A textbook (../review/textbook.php)

External links

+ Suggest a link

Dashboard

Question 82 of 364

 

A 32 year old man who has suffered from Crohns disease for many years presents
with intermittent jaundice. When it occurs, it is obstructive in nature. It then usually
resolves spontaneously. What is the most likely cause?

Crigler Najjar syndrome


gathered by dr. elbarky.

Parasitic infection of the liver

Bile duct stones

Gilberts syndrome

Multi cystic liver disease

Bile salts are absorbed in the terminal ileum. When this process is impaired as in
Crohns the patient may develop gallstones, if these pass into the CBD then
obstructive jaundice will result.

Please rate this question:

 Discuss and give feedback

Next question 

Surgical jaundice

Jaundice can present in a manner of different surgical situations. As with all types
of jaundice a careful history and examination will often give clues as to the most
likely underlying cause. Liver function tests whilst conveying little in the way of
information about liver synthetic function, will often facilitate classification as to
whether the jaundice is pre hepatic, hepatic or post hepatic. The typical LFT
patterns are given below:

Location Bilirubin ALT/ AST Alkaline phosphatase


Location Bilirubin ALT/ AST Alkaline phosphatase

Pre Normal or Normal Normal


hepatic high

Hepatic High Elevated (often Elevated but seldom to very


very high) high levels

Post High-very Moderate elevation High- very high


hepatic high
gathered by dr. elbarky.

In post hepatic jaundice the stools are often of pale colour and this feature should
be specifically addressed in the history.

Modes of presentation
These are addressed in the table below:

Diagnosis Typical features Pathogenesis

Gallstones Typically history of Usually small calibre gallstones


biliary colic or which can pass through the
episodes of cystic duct. In Mirizzi syndrome
chlolecystitis. the stone may compress the bile
Obstructive type duct directly- one of the rare
history and test times that cholecystitis may
results. present with jaundice

Cholangitis Usually obstructive Ascending infection of the bile


and will have ducts usually by E. coli and by
Charcots triad of definition occurring in a pool of
symptoms (pain, stagnant bile.
fever, jaundice)

Pancreatic cancer Typically painless Direct occlusion of distal bile


jaundice with duct or pancreatic duct by
palpable tumour. Sometimes nodal
gallbladder disease at the portal hepatis may
(Courvoisier's Law) be the culprit in which case the
bile duct may be of normal
calibre.
Diagnosis Typical features Pathogenesis

TPN associated Usually follows Often due to hepatic dysfunction


jaundice long term use and and fatty liver which may occur
is usually painless with long term TPN usage.
with non
obstructive
features

Bile duct injury Depending upon Often due to a difficult


the type of injury cholecystectomy when anatomy
may be of sudden in Calots triangle is not
gathered by dr. elbarky.

or gradual onset appreciated. In the worst


and is usually of scenario the bile duct is excised
obstructive type and jaundice offers rapidly post
operatively. More insidious is that
of bile duct stenosis which may
be caused by clips or diathermy
injury.

Cholangiocarcinoma Gradual onset Direct occlusion by disease and


obstructive pattern also extrinsic compression by
nodal disease at the porta
hepatis.

Septic surgical Usually hepatic Combination of impaired biliary


patient features excretion and drugs such as
ciprofloxacin which may cause
cholestasis.

Metastatic disease Mixed hepatic and Combination of liver synthetic


post hepatic failure (late) and extrinsic
compression by nodal disease
and anatomical compression of
intra hepatic structures (earlier)

Diagnosis
An ultrasound of the liver and biliary tree is the most commonly used first line test.
This will establish bile duct calibre, often ascertain the presence of gallstones, may
visualise pancreatic masses and other lesions. The most important clinical
question is essentially the extent of biliary dilatation and its distribution.

Where pancreatic neoplasia is suspected, the next test should be a pancreatic


protocol CT scan. With liver tumours and cholangiocarcinoma an MRI/ MRCP is
often the preferred option. PET scans may be used to stage a number of
malignancies but do not routinely form part of first line testing.
Where MRCP fails to give adequate information an ERCP may be necessary. In
many cases this may form part of patient management. It is however, invasive and
certainly not without risk and highly operator dependent.

Management
Clearly this will depend to an extent upon the underlying cause but relief of
jaundice is important, even if surgery forms part of the planned treatment. Patients
with unrelieved jaundice have a much higher incidence of septic complications,
bleeding and death.

Screen for and address any clotting irregularities


gathered by dr. elbarky.

In patients with malignancy a stent will need to be inserted. These come in two
main types; metal and plastic. Plastic stents are cheap and easy to replace and
should be used if any surgical intervention (e.g. Whipples) is planned. However,
they are prone to displacement and blockage. Metal stents are much more
expensive and may compromise a surgical resection. However, they are far less
prone to displacement and to a lesser extent blockage than their plastic
counterparts.

If malignancy is in bile duct/ pancreatic head and stenting has been attempted and
has failed, then an alternative strategy is to drain the biliary system percutaneously
via a transhepatic route. It may also be possible to insert a stent in this way. One of
the main problems with temporary PTC's is their propensity to displacement, which
may result in a bile leak.

In patients who have a bile duct injury surgery will be required to repair the defect.
If the bile duct has been inadvertently excised then a hepatico-jejunostomy will
need to be created (difficult!)

If gallstones are the culprit, then these may be removed by ERCP and a
cholecystectomy performed. Where there is doubt about the efficacy of the ERCP
an operative cholangiogram should be performed and bile duct exploration
undertaken where stones remain. When the bile duct has been formally opened the
options are between closure over a T tube, a choledochoduodenostomy or
choledochojejunostomy.

Patients with cholangitis should receive high dose broad spectrum antibiotics via
the intravenous route. Biliary decompression should follow soon afterwards,
instrumenting the bile duct of these patients will often provoke a septic episode
(but should be done anyway).

Next question 
Display my notes on this topic

          

Save my notes

Question stats

A 13.6%
gathered by dr. elbarky.

B 7%
C 54.5%
D 17.2%
E 7.8%

54.5% of users answered this question correctly

Search eMRCS

Search term Go

 Open MRCS Part A textbook (../review/textbook.php)

External links

+ Suggest a link

Dashboard

5

Question 83 of 364

 

A 24 year old man is diagnosed as having Hodgkins lymphoma. Which subtype is


associated with the most favorable prognosis?

Anaplastic
gathered by dr. elbarky.

Lymphocyte depleted

Lymphocyte rich

Nodular sclerosing

Mixed cellularity

Nodular sclerosis cHL (NSCHL) is the most common subtype, accounting for
about 70% of cHL cases in the developed world and characterized by neoplastic
lacunar type HRS cells in an inflammatory background of band-forming sclerosis.
Mediastinal adenopathy is seen in 80% of cases and bulky nodes (>10cm in
diameter) are present in about half the patients15. Association with Epstein-Barr
virus is less frequent and NSCHL has a better prognosis overall than other types of
cHL.

Please rate this question:

 Discuss and give feedback

Next question 

Hodgkins lymphoma

Presenting features
Asymptomatic lympadenopathy
Cough, Pel Ebstein fever, haemoptysis, dyspnoea
B Symptoms - 10% weight loss, fever, night sweats

Staging
All patients are staged with CT scanning of the chest, abdomen and pelvis
The Ann Arbor staging system is commonly used

Stage Features

I Single lymph node region

II Two or more regions on the same side of the diaphragm

III Involvement of lymph node regions on both sides of the diaphragm

IV Involvement of extra nodal sites


gathered by dr. elbarky.

Sub types
Classical Hodgkin lymphoma is classified into the following 4 types:

Nodular sclerosing Hodgkin lymphoma (NSHL)


Mixed-cellularity Hodgkin lymphoma (MCHL)
Lymphocyte-depleted Hodgkin lymphoma (LDHL)
Lymphocyte-rich classical Hodgkin lymphoma (LRHL)

Reed Sternberg cells may be identified histologically.

Treatment
This may be multimodal and both chemo and radiotherapy are used.

Diagnosis
This is made by excision of a complete lymph node that is then submitted for
detailed histological evaluation.

Pathogenesis
Infection with Ebstein Barr virus is linked to the condition (particularly mixed
cellularity lymphoma).

Prognosis
Stage I disease is associated with survival figures of up to 85% at 5 years. Nodular
sclerosing has the best prognosis. Lymphocyte depleted Hodgkins lymphoma,
advancing age, male sex and stage IV disease are all associated with a worsening
of prognosis.

Reference
Shanbhag S and Ambinder R. Hodgkin Lymphoma: a review and update on recent
progress. CA Cancer J Clin . 2018 March ; 68(2): 116132

Next question 
Display my notes on this topic

          

Save my notes

Question stats
gathered by dr. elbarky.

A 15.3%
B 11.2%
C 19.2%
D 46.7%
E 7.7%

46.7% of users answered this question correctly

Search eMRCS

Search term Go

 Open MRCS Part A textbook (../review/textbook.php)

External links

+ Suggest a link

Dashboard

5

Question 84 of 364

 

A 60 year old man presents with recurrent renal stones. He is found to have a
calcium of 2.72 (elevated) and a PTH of 12 (elevated). What is the most
appropriate long term management plan?

Neck exploration and parathyroidectomy


gathered by dr. elbarky.

CT scanning of the neck

MRI scanning of the neck

Medical treatment alone

External beam radiotherapy to the neck

This patient has primary hyperparathyroidism and nephrolithiasis, which is an


indication for parathyroidectomy.

Please rate this question:

 Discuss and give feedback

Next question 

Parathyroid glands and disorders of calcium metabolism

Hyperparathyroidism

Clinical
Disease type Hormone profile features Cause
Disease type Hormone profile Clinical features Cause

Primary PTH May be Most cases


hyperparathyroidism (Elevated) asymptomatic if due to
Ca2+ mild solitary
(Elevated) Recurrent adenoma
Phosphate abdominal pain (80%),
(Low) (pancreatitis, multifocal
Urine renal colic) disease
calcium : Changes to occurs in
creatinine emotional or 10-15% and
clearance cognitive state parathyroid
gathered by dr. elbarky.

ratio > 0.01 carcinoma in


1% or less

Secondary PTH May have few Parathyroid


hyperparathyroidism (Elevated) symptoms gland
Ca2+ (Low or Eventually may hyperplasia
normal) develop bone occurs as a
Phosphate disease, osteitis result of low
(Elevated) fibrosa cystica calcium,
Vitamin D and soft tissue almost
levels (Low) calcifications always in a
setting of
chronic renal
failure

Tertiary Ca2+ (Normal Metastatic Occurs as a


hyperparathyroidism or high) calcification result of
PTH Bone pain and / ongoing
(Elevated) or fracture hyperplasia of
Phosphate Nephrolithiasis the
levels Pancreatitis parathyroid
(Decreased glands after
or Normal) correction of
Vitamin D underlying
(Normal or renal disorder,
decreased) hyperplasia of
Alkaline all 4 glands is
phosphatase usually the
(Elevated) cause

Differential diagnoses
It is important to consider the rare but relatively benign condition of benign familial
hypocalciuric hypercalcaemia, caused by an autosomal dominant genetic disorder.
Diagnosis is usually made by genetic testing and concordant biochemistry (urine
calcium : creatinine clearance ratio <0.01-distinguished from primary
hyperparathyroidism).

Treatment

Primary hyperparathyroidism
Indications for surgery
Elevated serum Calcium > 1mg/dL above normal
Hypercalciuria > 400mg/day
Creatinine clearance < 30% compared with normal
Episode of life threatening hypercalcaemia
Nephrolithiasis
gathered by dr. elbarky.

Age < 50 years


Neuromuscular symptoms
Reduction in bone mineral density of the femoral neck, lumbar spine, or
distal radius of more than 2.5 standard deviations below peak bone mass (T
score lower than -2.5)

Secondary hyperparathyroidism
Usually managed with medical therapy.

Indications for surgery in secondary (renal) hyperparathyroidism:


Bone pain
Persistent pruritus
Soft tissue calcifications

Tertiary hyperparathyroidism
Allow 12 months to elapse following transplant as many cases will resolve
The presence of an autonomously functioning parathyroid gland may require
surgery. If the culprit gland can be identified then it should be excised. Otherwise
total parathyroidectomy and re-implantation of part of the gland may be required.

References
1. Pitt S et al. Secondary and Tertiary Hyperparathyroidism, State of the Art
Surgical Management. Surg Clin North Am 2009 Oct;89(5):1227-39.

2. MacKenzie-Feder J et al. Primary Hyperparathyroidism: An Overview. Int J


Endocrinol 2011; 2011: 251410.

Next question 

Display my notes on this topic


          

Save my notes

Question stats

A 52.4%
B 15.5%
C 17.6%
gathered by dr. elbarky.

D 8%
E 6.5%

52.4% of users answered this question correctly

Search eMRCS

Search term Go

 Open MRCS Part A textbook (../review/textbook.php)

External links

+ Suggest a link

Dashboard

7

Question 85 of 364

 

A 23 year old lady presents with a nodule in the right lobe of the thyroid.
Examination of the neck is otherwise unremarkable and clinically she is euthyroid.
Imaging shows a solid nodule at the site. What is the correct course of action?

Image guided core biopsy


gathered by dr. elbarky.

Image guided fine needle aspiration for cytology

Arrange a hemithyroidectomy

Perform an incision biopsy

Perform an excision biopsy

FNAC is the first line investigation in this setting. Whereas FNAC has declined in
popularity recently (in breast investigation), it remain a very popular option in the
investigation of thyroid masses. It cannot reliably diagnose a follicular tumour.

Please rate this question:

 Discuss and give feedback

Next question 

Tissue sampling

Tissue sampling is an important surgical process. Biopsy modalities vary


according to the site, experience and subsequent planned therapeutic outcome

The modalities comprise:


-Fine needle aspiration cytology
-Core biopsy
-Excision biopsy
-Tru cut biopsy
-Punch biopsy
-Cytological smears
-Endoscopic or laparoscopic biopsy
When the lesion is superficial the decision needs to be taken as to whether
complete excision is desirable or whether excision biopsy is acceptable. In
malignant melanoma for example the need for safe margins will mean that a more
radical surgical approach needs to be adopted after diagnostic confirmation from
excision biopsy than would be the case in basal cell carcinoma. Punch biopsies
are useful in gaining histological diagnosis of unclear skin lesions where excision
biopsy is undesirable such as in establishing whether a skin lesion is vasculitic or
not.

Fine needle aspiration cytology (FNAC) is an operator dependent procedure that


may or may not be image guided and essentially involves passing a needle through
gathered by dr. elbarky.

a lesion whilst suction is applied to a syringe. The material thus obtained is


expressed onto a slide and sent for cytological assessment. This test can be
limited by operator inexperience and also by the lack of histological architectural
information (e.g. Follicular carcinoma of the thyroid). Where a discharge is present
a sample may be sent for cytology although in some sites (e.g. Nipple discharge )
the information gleaned may be meaningless.

Tissue samples may be obtained by both core and tru cut biopsy. A core biopsy is
obtained by use of a spring loaded gun with a needle passing quickly through the
lesion of interest. A tru cut biopsy achieves the same objective but the needle
moved by hand. When performing these techniques image guidance may be
desirable (e.g. In breast lesions). Consideration needs to be given to any planned
surgical resection as it may be necessary to resect the biopsy tract along with the
specimen (e.g. In sarcoma surgery).

Visceral lesions may be accessed percutaneously under image guidance such as


ultrasound guided biopsy of liver metastases. Or under direct vision such as a
colonoscopic biopsy.

Next question 

Display my notes on this topic

          

Save my notes

Question stats

A 14.6%
B 61.2%
C 10.7%
D 5.6%
E 7.9%

61.2% of users answered this question correctly

Search eMRCS
gathered by dr. elbarky.

Search term Go

 Open MRCS Part A textbook (../review/textbook.php)

External links

+ Suggest a link

Dashboard

10

11

12

13

Question 86 of 364

 

A 74 year old woman presents with an erythematous rash originating in the nipple.
It is spreading to the surrounding areolar area and the associated normal tissue.
What is the most likely cause?

Nipple eczema
gathered by dr. elbarky.

Pagets disease of the nipple

Fibroadenosis

Invasive lobular carcinoma

Radial scar

In most patients with Pagets disease, an underlying mammary carcinoma will


be found.

Paget's is associated with DCIS or invasive carcinoma. Unlike eczema of the nipple
which predominantly affects the areolar region, Pagets will usually affect the
nipple first and then spread to the areolar area. Diagnosis is made by punch
biopsy.

Please rate this question:

 Discuss and give feedback

Next question 

Breast cancer

Commoner in the older age group


Invasive ductal carcinomas are the most common type. Some may arise as
a result of ductal carcinoma in situ (DCIS). There are associated
carcinomas of special type e.g. Tubular that may carry better prognosis.
The pathological assessment involves assessment of the tumour and lymph
nodes, sentinel lymph node biopsy is often used to minimise the morbidity
of an axillary dissection.
Treatment, typically this is either wide local excision or mastectomy. There
are many sub types of both of these that fall outside of the MRCS. Some key
rules to bear in mind.
Whatever operation is contemplated the final cosmetic outcome does have
a bearing. A woman with small breasts and a large tumour will tend to fare
better with mastectomy, even if clear pathological and clinical margins can
be obtained. Conversely a women with larger breasts may be able to
undergo breast conserving surgery even with a relatively large primary lesion
(NB tumours >4cm used to attract recommendation for mastectomy). For
screen detected and impalpable tumour image guidance will be necessary.
gathered by dr. elbarky.

Reconstruction is always an option following any resectional procedure.


However, its exact type must be tailored to age and co-morbidities of the
patient. The main operations in common use include latissimus dorsi
myocutaneous flap and sub pectoral implants. Women wishing to avoid a
prosthesis may be offered TRAM or DIEP flaps.

Surgical options
Mastectomy vs Wide local excision

Mastectomy Wide Local Excision

Multifocal tumour Solitary lesion

Central tumour Peripheral tumour

Large lesion in small breast Small lesion in large breast

DCIS >4cm DCIS <4cm

Patient Choice Patient choice

Central lesions may be managed using breast conserving surgery where an


acceptable cosmetic result may be obtained, this is rarely the case in small breasts

A compelling indication for mastectomy, a larger tumour that would be unsuitable for
breast conserving surgery
gathered by dr. elbarky.

(https://d2zgo9qer4wjf4.cloudfront.net/images_eMRCS/swb069b.jpg)
Image sourced from Wikipedia
(https://d2zgo9qer4wjf4.cloudfront.net
(http://en.wikipedia.org
/images_eMRCS/swb069b.jpg)
/wiki/Mastectomy)

Whatever surgical option is chosen the aim should be to have a local recurrence
rate of 5% or less at 5 years [1].

Nottingham Prognostic Index


The Nottingham Prognostic Index can be used to give an indication of survival. In
this system the tumour size is weighted less heavily than other major prognostic
parameters.

Calculation of NPI
Tumour Size x 0.2 + Lymph node score(From table below)+Grade score(From
table below).

Score Lymph nodes involved Grade

1 0 1

2 1-3 2

3 >3 3

Prognosis

Score Percentage 5 year survival


Score Percentage 5 year survival

2.0 to 2.4 93%

2.5 to 3.4 85%

3.5 to 5.4 70%

>5.4 50%

This data was originally published in 1992. It should be emphasised that other
gathered by dr. elbarky.

factors such as vascular invasion and receptor status also impact on survival and
are not included in this data and account for varying prognoses often cited in the
literature.

References
Surgical guidelines for the management of breast cancer, Association of Breast
Surgery at BASO 2009, Eur J Surg Oncol (2009), doi:10.1016/j.ejso.2009.01.008

Next question 

Display my notes on this topic

          

Save my notes

Question stats

A 18.9%
B 61.3%
C 6.8%
D 7.6%
E 5.5%

61.3% of users answered this question correctly


Question 87 of 364

 

In which of the following conditions is a Ghon complex most likely to be found?

Actinomycosis

Infection with mycobacterium tuberculosis


gathered by dr. elbarky.

Mycosis fungoides

Lewy body dementia

Chronic lymphocytic leukaemia

Ghon focus is primary infection


Ghon complex is primary infection and regional nodal involvement

Please rate this question:

 Discuss and give feedback

Next question 

Tuberculosis pathology
Is a form of primary chronic inflammation, caused by the inability of
macrophages to kill the Mycobacterium tuberculosis.
The macrophages often migrate to regional lymph nodes, the lung lesion
plus affected lymph nodes is referred to as a Ghon complex.
This leads to the formation of a granuloma which is a collection of
epithelioid histiocytes.
There is the presence of caseous necrosis in the centre.
The inflammatory response is mediated by a type 4 hypersensitivity
reaction.
In healthy individuals the disease may be contained, in the
immunocompromised disseminated (miliary TB) may occur.
gathered by dr. elbarky.

Diagnosis
Waxy membrane of mycobacteria prevents binding with normal stains. Ziehl
- Neelsen staining is typically used.
Culture based methods take far longer.

Image showing acid- alcohol fast mycobacteria stained using the Ziehl- Neelsen
method

(https://d2zgo9qer4wjf4.cloudfront.net/images_eMRCS/swb107b.jpg)
Image sourced from Wikipedia
(https://d2zgo9qer4wjf4.cloudfront.net
(http://en.wikipedia.org/wiki/Ziehl%E2%80
/images_eMRCS/swb107b.jpg)
%93Neelsen stain)

Next question 

Display my notes on this topic

          
Save my notes

Question stats

A 10.2%
B 64.2%
C 9.1%
D 8.3%
gathered by dr. elbarky.

E 8.2%

64.2% of users answered this question correctly

Search eMRCS

Search term Go

 Open MRCS Part A textbook (../review/textbook.php)

External links

+ Suggest a link

Dashboard

8

Question 88 of 364

 

A 40 year old women presented with a mass on her forehead. On examination, she
had a fluctuant pulsatile mass on her head. Examination of her neck revealed a
mass inferior to the hyoid with a positive Berry's sign. What is the most likely
underlying diagnosis?
gathered by dr. elbarky.

Follicular thyroid cancer

Medullary thyroid cancer

Papillary thyroid cancer

Anaplastic thyroid cancer

Parathyroid gland cancer

Berrys sign= Absence of carotid pulse due to malignant thyromegaly.

Papillary thyroid cancers will tend to spread via lymphatics and present with
disease that is nearly always confined to the neck. Follicular carcinomas may
metastasise haematogenously and the skull may be the presenting site of disease
in between 2 and 8% of patients.

Please rate this question:

 Discuss and give feedback

Next question 

Thyroid malignancy

Papillary carcinoma
Commonest sub-type
Accurately diagnosed on fine needle aspiration cytology
Histologically, they may demonstrate psammoma bodies (areas of
calcification) and so called 'orphan Annie' nuclei
They typically metastasise via the lymphatics and thus laterally located
apparently ectopic thyroid tissue is usually a metastasis from a well
differentiated papillary carcinoma

Follicular carcinoma
Are less common than papillary lesions
Like papillary tumours, they may present as a discrete nodule. Although they
appear to be well encapsulated macroscopically there is invasion on
microscopic evaluation
Lymph node metastases are uncommon and these tumours tend to spread
haematogenously. This translates into a higher mortality rate
gathered by dr. elbarky.

Follicular lesions cannot be accurately diagnosed on fine needle aspiration


cytology and thus all follicular FNA's (THY 3f) will require at least a hemi
thyroidectomy

Anaplastic carcinoma
Less common and tend to occur in elderly females
Disease is usually advanced at presentation and often only palliative
decompression and radiotherapy can be offered.

Medullary carcinoma
These are tumours of the parafollicular cells ( C Cells) and are of neural
crest origin.
The serum calcitonin may be elevated which is of use when monitoring for
recurrence.
They may be familial and occur as part of the MEN -2A disease spectrum.
Spread may be either lymphatic or haematogenous and as these tumours
are not derived primarily from thyroid cells they are not responsive to
radioiodine.

Lymphoma
These respond well to combined chemoradiotherapy
Radical surgery is unnecessary once the disease has been diagnosed on
biopsy material. Such biopsy material is not generated by an FNA and thus a
core biopsy has to be obtained (with care!).

Next question 

Display my notes on this topic

          
Save my notes

Question stats

A 40.6%
B 17.9%
C 16.8%
D 15.5%
gathered by dr. elbarky.

E 9.2%

40.6% of users answered this question correctly

Search eMRCS

Search term Go

 Open MRCS Part A textbook (../review/textbook.php)

External links

+ Suggest a link

Dashboard

8

Question 89 of 364

 

A term neonate is undergoing an open inguinal herniotomy. During the operation,


the operating surgeon identifies a small round yellow nodule within the inguinal
canal. What is this most likely to represent?

Ectopic lymphatic tissue


gathered by dr. elbarky.

Adrenal rest

Encysted hydrocele of the cord

Yolk sac tumour

Hamartoma

Adrenal rests are a very common finding in term neonates (50%) and most regress
so that by adulthood the population incidence is 1%. The adrenal glands and
gonads both develop from the urogenital ridge. In early embryogenesis, the adrenal
cortical tissue can migrate with the descending gonads.

Please rate this question:

 Discuss and give feedback

Next question 

Adrenal gland embryology

First detected at 6 weeks' gestation, the adrenal cortex is derived from the
mesoderm of the posterior abdominal wall. Steroid secretion from the fetal cortex
begins shortly thereafter. Adult-type zona glomerulosa and fasciculata are
detected in fetal life but make up only a small proportion of the gland, and the zona
reticularis is not present at all. The fetal cortex predominates throughout fetal life.
The adrenal medulla is of ectodermal origin, arising from neural crest cells that
migrate to the medial aspect of the developing cortex.

The fetal adrenal gland is relatively large. At 4 months' gestation, it is 4 times the
size of the kidney; however, at birth, it is a third of the size of the kidney. This
occurs because of the rapid regression of the fetal cortex at birth. It disappears
almost completely by age 1 year; by age 4-5 years, the permanent adult-type
adrenal cortex has fully developed.

Anatomic anomalies of the adrenal gland may occur. Because the development of
the adrenals is closely associated with that of the kidneys, agenesis of an adrenal
gland is usually associated with ipsilateral agenesis of the kidney, and fused
adrenal glands (whereby the 2 glands join across the midline posterior to the
aorta) are also associated with a fused kidney.

Adrenal hypoplasia occurs in the following 2 forms: (1) hypoplasia or absence of


the fetal cortex with a poorly formed medulla and (2) disorganized fetal cortex and
gathered by dr. elbarky.

medulla with no permanent cortex present. Adrenal heterotopia describes a


normal adrenal gland in an abnormal location, such as within the renal or hepatic
capsules. Accessory adrenal tissue (adrenal rests), which is usually comprised
only of cortex but seen combined with medulla in some cases, is most commonly
located in the broad ligament or spermatic cord but can be found anywhere within
the abdomen. Even intracranial adrenal rests have been reported

Next question 

Display my notes on this topic

          

Save my notes

Question stats

A 15.1%
B 35.8%
C 26.6%
D 14.7%
E 7.8%

35.8% of users answered this question correctly

Search eMRCS

Question 89 of 364

 

A term neonate is undergoing an open inguinal herniotomy. During the operation,


the operating surgeon identifies a small round yellow nodule within the inguinal
canal. What is this most likely to represent?

Ectopic lymphatic tissue


gathered by dr. elbarky.

Adrenal rest

Encysted hydrocele of the cord

Yolk sac tumour

Hamartoma

Adrenal rests are a very common finding in term neonates (50%) and most regress
so that by adulthood the population incidence is 1%. The adrenal glands and
gonads both develop from the urogenital ridge. In early embryogenesis, the adrenal
cortical tissue can migrate with the descending gonads.

Please rate this question:

 Discuss and give feedback

Next question 

Adrenal gland embryology

First detected at 6 weeks' gestation, the adrenal cortex is derived from the
mesoderm of the posterior abdominal wall. Steroid secretion from the fetal cortex
begins shortly thereafter. Adult-type zona glomerulosa and fasciculata are
detected in fetal life but make up only a small proportion of the gland, and the zona
reticularis is not present at all. The fetal cortex predominates throughout fetal life.
The adrenal medulla is of ectodermal origin, arising from neural crest cells that
migrate to the medial aspect of the developing cortex.

The fetal adrenal gland is relatively large. At 4 months' gestation, it is 4 times the
size of the kidney; however, at birth, it is a third of the size of the kidney. This
occurs because of the rapid regression of the fetal cortex at birth. It disappears
almost completely by age 1 year; by age 4-5 years, the permanent adult-type
adrenal cortex has fully developed.

Anatomic anomalies of the adrenal gland may occur. Because the development of
the adrenals is closely associated with that of the kidneys, agenesis of an adrenal
gland is usually associated with ipsilateral agenesis of the kidney, and fused
adrenal glands (whereby the 2 glands join across the midline posterior to the
aorta) are also associated with a fused kidney.

Adrenal hypoplasia occurs in the following 2 forms: (1) hypoplasia or absence of


the fetal cortex with a poorly formed medulla and (2) disorganized fetal cortex and
gathered by dr. elbarky.

medulla with no permanent cortex present. Adrenal heterotopia describes a


normal adrenal gland in an abnormal location, such as within the renal or hepatic
capsules. Accessory adrenal tissue (adrenal rests), which is usually comprised
only of cortex but seen combined with medulla in some cases, is most commonly
located in the broad ligament or spermatic cord but can be found anywhere within
the abdomen. Even intracranial adrenal rests have been reported

Next question 

Display my notes on this topic

          

Save my notes

Question stats

A 15.1%
B 35.8%
C 26.6%
D 14.7%
E 7.8%

35.8% of users answered this question correctly

Search eMRCS

Question 90 of 364

 

What is the most common cause of mesenteric infarction?

Mesenteric vein thrombosis

Acute embolism affecting the superior mesenteric artery


gathered by dr. elbarky.

Acute on chronic thrombus of the superior mesenteric artery

Sub intimal dissection of the superior mesenteric artery

Proximal migration of abdominal aortic aneurysm

Acute embolic events account for up to 50% of cases of mesenteric infarcts.


These may occur as a result of long standing atrial fibrillation, ventricular
anuerysms and post myocardial infarction.

Please rate this question:

 Discuss and give feedback

Next question 

Mesenteric vessel disease

Mesenteric ischaemia accounts for 1 in 1000 acute surgical admissions. It is


primarily caused by arterial embolism resulting in infarction of the colon. It is more
likely to occur in areas such as the splenic flexure that are located at the borders
of the territory supplied by the superior and inferior mesenteric arteries.

Types
Acute mesenteric Sudden onset abdominal pain followed by profuse
embolus diarrhoea.
(commonest 50%) May be associated with vomiting.
Rapid clinical deterioration.
Serological tests: WCC, lactate, amylase may all be
abnormal particularly in established disease. These
can be normal in the early phases.

Acute on chronic Usually longer prodromal history.


mesenteric Post prandial abdominal discomfort and weight
ischaemia loss are dominant features. Patients will usually
gathered by dr. elbarky.

present with an acute on chronic event, but


otherwise will tend not to present until mesenteric
flow is reduced by greater than 80%.
When acute thrombosis occurs presentation may
be as above. In the chronic setting the symptoms
will often be those of ischaemic colitis (mucosa is
the most sensitive area to this insult).

Mesenteric vein Usually a history over weeks.


thrombosis Overt abdominal signs and symptoms will not
occur until venous thrombosis has reached a stage
to compromise arterial inflow.
Thrombophilia accounts for 60% of cases.

Low flow This occurs in patients with multiple co morbidities


mesenteric in whom mesenteric perfusion is significantly
infarction compromised by overuse of inotropes or
background cardiovascular compromise.
The end result is that the bowel is not adequately
perfused and infarcts occur from the mucosa
outwards.

Diagnosis
Serological tests: WCC, lactate, CRP, amylase (can be normal in early
disease).
Cornerstone for diagnosis of arterial AND venous mesenteric disease is CT
angiography scanning in the arterial phase with thin slices (<5mm). Venous
phase contrast is not helpful.
SMA duplex USS is useful in the evaluation of proximal SMA disease in
patients with chronic mesenteric ischaemia.
MRI is of limited use due to gut peristalsis and movement artefact.

Management
Overt signs of peritonism: Laparotomy
Mesenteric vein thrombosis: If no peritonism: Medical management with IV
heparin
At operation limited resection of frankly necrotic bowel with view to relook
laparotomy at 24-48h. In the interim urgent bowel revascularisation via
endovascular (preferred) or surgery.

Prognosis
Overall poor. Best outlook is from an acute ischaemia from an embolic event
where surgery occurs within 12h. Survival may be 50%. This falls to 30% with
treatment delay. The other conditions carry worse survival figures.
gathered by dr. elbarky.

Next question 

Display my notes on this topic

          

Save my notes

Question stats

A 19.2%
B 49.3%
C 18.4%
D 6.3%
E 6.9%

49.3% of users answered this question correctly

Search eMRCS

Search term Go

 Open MRCS Part A textbook (../review/textbook.php)


Question 91 of 364

 

Which is the characteristic finding on a blood film post splenectomy?

Stipple cell

Tear drop cell


gathered by dr. elbarky.

Reticulocytes

Howell-Jolly bodies

Schistocyte

Blood film in hyposplenism:

Howell-Jolly bodies
Pappenheimer bodies
Poikilocytes (Target cells)
Erythrocyte containing siderotic granules
Heinz bodies

Please rate this question:

 Discuss and give feedback

Next question 

Splenectomy

Indications
Trauma: 1/4 are iatrogenic
Spontaneous rupture: EBV
Hypersplenism: hereditary spherocytosis or elliptocytosis etc
Malignancy: lymphoma or leukaemia
Splenic cysts, hydatid cysts, splenic abscesses

Splenectomy
Technique
Trauma
GA
Long midline incision
If time permits insert a self retaining retractor (e.g. Balfour/ omnitract)
Large amount of free blood is usually present. Pack all 4 quadrants of the
abdomen. Allow the anaesthetist to 'catch up'
Remove the packs and assess the viability of the spleen. Hilar injuries and
extensive parenchymal lacerations will usually require splenectomy.
Divide the short gastric vessels and ligate them.
Clamp the splenic artery and vein. Two clamps on the patient side are better
and allow for double ligation and serve as a safety net if your assistant does
gathered by dr. elbarky.

not release the clamp smoothly.


Be careful not to damage the tail of the pancreas, if you do then this will
need to be formally removed and the pancreatic duct closed.
Wash out the abdomen and place a tube drain to the splenic bed.
Some surgeons implant a portion of spleen into the omentum, whether you
decide to do this is a matter of personal choice.
Post operatively the patient will require prophylactic penicillin V and
pneumococcal vaccine.

Elective
Elective splenectomy is a very different operation from that performed in the
emergency setting. The spleen is often large (sometimes massive). Most cases
can be performed laparoscopically. The spleen will often be macerated inside a
specimen bag to facilitate extraction.

Complications
Haemorrhage (may be early and either from short gastrics or splenic hilar
vessels
Pancreatic fistula (from iatrogenic damage to pancreatic tail)
Thrombocytosis: prophylactic aspirin
Encapsulated bacteria infection e.g. Strep. pneumoniae, Haemophilus
influenzae and Neisseria meningitidis

Post splenectomy changes


Platelets will rise first (therefore in ITP should be given after splenic artery
clamped)
Blood film will change over following weeks, Howell Jolly bodies will appear
Other blood film changes include target cells and Pappenheimer bodies
Increased risk of post splenectomy sepsis, therefore prophylactic antibiotics
and pneumococcal vaccine should be given.

Post splenectomy sepsis


Typically occurs with encapsulated organisms
Opsonisation occurs but then not recognised
Next question 

Display my notes on this topic

          

Save my notes
gathered by dr. elbarky.

Question stats

A 6.7%
B 5.4%
C 7.9%
D 75%
E 5%

75% of users answered this question correctly

Search eMRCS

Search term Go

 Open MRCS Part A textbook (../review/textbook.php)

External links

+ Suggest a link

Dashboard

3

Question 150 of 364

 

A 33 year old man is involved in a road traffic accident. He is initially stable and
transferred to the accident and emergency department. On arrival he is
catheterised. One minute later he becomes hypotensive, with evidence of
angioedema surrounding his penis. What is the most likely explanation for this
event?
gathered by dr. elbarky.

Type V latex hypersensitivity reaction

Type IV latex hypersensitivity reaction

Type III latex hypersensitivity reaction

Type I latex hypersensitivity reaction

Type II latex hypersensitivity reaction

Sudden collapse and angioedema following exposure to latex (of which most
urinary catheters are manufactured) suggests a type I hypersensitivity reaction.

Please rate this question:

 Discuss and give feedback

Next question 

Hypersensitivity reactions

The Gell and Coombs classification divides hypersensitivity reactions into 4 types

Type I Type II Type III Type IV

Description Anaphylactic Cytotoxic Immune Delayed


complex type

Mediator IgE IgG, IgM IgG, Ig A, IgM T-cells


Type I Type II Type III Type IV

Antigen Exogenous Cell surface Soluble Tissues

Response Minutes Hours Hours 2-3 days


time

Examples Asthma Autoimmune Serum Graft versus


Hay fever haemolytic sickness host
anaemia SLE disease
Pemphigus Aspergillosis Contact
Goodpasture's dermatitis
gathered by dr. elbarky.

Next question 

Display my notes on this topic

          

Save my notes

Question stats

A 6.6%
B 12.8%
C 6.2%
D 67.4%
E 7%

67.4% of users answered this question correctly

Search eMRCS

Search term Go

 Open MRCS Part A textbook (../review/textbook.php)


Question 93 of 364

 

A 50 year old female presents with bilateral parotid gland swelling and symptoms
of a dry mouth. On examination, she has bilateral facial nerve palsies. This
improved following steroid treatment. What is the likely underlying diagnosis?

Pleomorphic adenoma
gathered by dr. elbarky.

Sarcoid

Warthins tumour

Sialolithiasis

Adenoid cystic carcinoma

Sarcoid occurs bilaterally in 70% of cases and facial nerve involvement is


recognised. Treatment is conservative in most cases although individuals with
facial nerve palsy will usually receive steroids with good effect.

Please rate this question:

 Discuss and give feedback

Next question 

Parotid gland clinical

Benign neoplasms
Up to 80% of all salivary gland tumours occur in the parotid gland and up to 80% of
these are benign. There is no consistent correlation between the rate of growth
and the malignant potential of the lesion. However, benign tumours should not
invade structures such as the facial nerve.
With the exception of Warthins tumours, they are commoner in women than men.
The median age of developing a lesion is in the 5th decade of life.

Benign tumour types

Tumour type Features


Tumour type Features

Benign pleomorphic Most common parotid neoplasm (80%)


adenoma or benign mixed Proliferation of epithelial and myoepithelial cells
tumor of the ducts and an increase in stromal
components
Slow growing, lobular, and not well encapsulated
Recurrence rate of 1-5% with appropriate
excision (parotidectomy)
Recurrence possibly secondary to capsular
disruption during surgery
Malignant degeneration occurring in 2-10% of
gathered by dr. elbarky.

adenomas observed for long periods, with


carcinoma ex-pleomorphic adenoma occurring
most frequently as adenocarcinoma

Warthin tumor (papillary Second most common benign parotid tumor


cystadenoma lymphoma or (5%)
adenolymphoma) Most common bilateral benign neoplasm of the
parotid
Marked male as compared to female
predominance
Occurs later in life (sixth and seventh decades)
Presents as a lymphocytic infiltrate and cystic
epithelial proliferation
May represent heterotopic salivary gland
epithelial tissue trapped within intraparotid
lymph nodes
Incidence of bilaterality and multicentricity of
10%
Malignant transformation rare (almost unheard
of)

Monomorphic adenoma Account for less than 5% of tumours


Slow growing
Consist of only one morphological cell type
(hence term mono)
Include; basal cell adenoma, canalicular
adenoma, oncocytoma, myoepitheliomas

Haemangioma Should be considered in the differential of a


parotid mass in a child
Accounts for 90% of parotid tumours in children
less than 1 year of age
Hypervascular on imaging
Spontaneous regression may occur and
malignant transformation is almost unheard of
Malignant salivary gland tumours
Types of malignancy

Mucoepidermoid 30% of all parotid malignancies


carcinoma Usually low potential for local invasiveness and
metastasis (depends mainly on grade)

Adenoid cystic Unpredictable growth pattern


carcinoma Tendency for perineural spread
Nerve growth may display skip lesions resulting in
incomplete excision
gathered by dr. elbarky.

Distant metastasis more common (visceral rather than


nodal spread)
5 year survival 35%

Mixed tumours Often a malignancy occurring in a previously benign


parotid lesion

Acinic cell carcinoma Intermediate grade malignancy


May show perineural invasion
Low potential for distant metastasis
5 year survival 80%

Adenocarcinoma Develops from secretory portion of gland


Risk of regional nodal and distant metastasis
5 year survival depends upon stage at presentation,
may be up to 75% with small lesions with no nodal
involvement

Lymphoma Large rubbery lesion, may occur in association with


Warthins tumours
Diagnosis should be based on regional nodal biopsy
rather than parotid resection
Treatment is with chemotherapy (and radiotherapy)

Diagnostic evaluation
Plain x-rays may be used to exclude calculi
Sialography may be used to delineate ductal anatomy
FNAC is used in most cases
Superficial parotidectomy may be either diagnostic of therapeutic
depending upon the nature of the lesion
Where malignancy is suspected the primary approach should be definitive
resection rather than excisional biopsy
CT/ MRI may be used in cases of malignancy for staging primary disease
Treatment
For nearly all lesions this consists of surgical resection, for benign disease this will
usually consist of a superficial parotidectomy. For malignant disease a radical or
extended radical parotidectomy is performed. The facial nerve is included in the
resection if involved. The need for neck dissection is determined by the potential
for nodal involvement.

Other parotid disorders


HIV infection
Lymphoepithelial cysts associated with HIV occur almost exclusively in the
parotid
Typically presents as bilateral, multicystic, symmetrical swelling
gathered by dr. elbarky.

Risk of malignant transformation is low and management usually


conservative

Sjogren syndrome
Autoimmune disorder characterised by parotid enlargement, xerostomia and
keratoconjunctivitis sicca
90% of cases occur in females
Second most common connective tissue disorder
Bilateral, non tender enlargement of the gland is usual
Histologically, the usual findings are of a lymphocytic infiltrate in acinar
units and epimyoepithelial islands surrounded by lymphoid stroma
Treatment is supportive
There is an increased risk of subsequent lymphoma

Sarcoid
Parotid involvement occurs in 6% of patients with sarcoid
Bilateral in most cases
Gland is not tender
Xerostomia may occur
Management of isolated parotid disease is usually conservative

Next question 

Display my notes on this topic

          

Save my notes
Question stats

A 7.5%
B 67.5%
C 8.4%
D 8%
E 8.6%

67.5% of users answered this question correctly


gathered by dr. elbarky.

Search eMRCS

Search term Go

 Open MRCS Part A textbook (../review/textbook.php)

External links

+ Suggest a link

Dashboard

10

Question 94 of 364

 

A 20 year old male is referred to the clinic. He has undergone genetic testing
because his father died from colorectal cancer at the age of 21. His testing revealed
a mutation of the APC gene. A colonoscopy is proposed. What is the most likely
finding?
gathered by dr. elbarky.

Multiple colonic hamartomas

Carpet villous adenoma of the rectum

Caecal carcinoma

Multiple colonic adenomas

Multiple colonic hyperplastic polyps

APC mutations are found in familial adenomatous polyposis coli. These have
multiple colonic adenomas.

Please rate this question:

 Discuss and give feedback

Next question 

Polyposis syndromes

Genetic Screening and Associated


Syndrome defect Features management disorders
Genetic Screening and Associated
Syndrome defect Features management disorders

Familial Mutation of Typically over If known to be Gastric


adenomatous APC gene 100 colonic at risk then fundal
polyposis (80%) cases, adenomas predictive polyps
dominant Cancer risk of genetic testing (50%).
100% as teenager Duodenal
20% are new Annual flexible polyps 90%.
mutations sigmoidoscopy If severe
from 15 years duodenal
If no polyps polyposis
gathered by dr. elbarky.

found then 5 cancer risk


yearly of 30% at 10
colonoscopy years.
started at age Abdominal
20 desmoid
Polyps found = tumours.
resectional
surgery
(resection and
pouch Vs sub
total
colectomy and
IRA)

MYH Biallelic Multiple colonic Once identified Duodenal


associated mutation of polyps resection and polyposis in
polyposis mut Y Later onset ileoanal pouch 30%
human right sided reconstruction Associated
homologue cancers more is with
(MYH) on common than in recommended increased
chromosome FAP Attenuated risk of breast
1p, recessive 100% cancer phenotype - cancer (self
risk by age 60 regular examination)
colonoscopy
Genetic Screening and Associated
Syndrome defect Features management disorders

Peutz STK11 Multiple benign Annual Malignancies


-Jeghers (LKB1) intestinal examination at other sites
syndrome mutation on hamartomas Pan intestinal Classical
chromosome Episodic endoscopy pigmentation
19 in some obstruction and every 2-3 years pattern
(but not all) intussceception
cases, Increased risk
dominant of GI cancers
(colorectal
gathered by dr. elbarky.

cancer 20%,
gastric 5%)
Increased risk
of breast,
ovarian, cervical
pancreatic and
testicular
cancers

Cowden Mutation of Macrocephaly Targeted Breast


disease PTEN gene Multiple individualised cancer (81%
on intestinal screening risk)
chromosome hamartomas Thyroid
10q22, Multiple cancer and
dominant trichilemmomas non toxic
89% risk of goitre
cancer at any Uterine
site cancer
16% risk of
colorectal
cancer

HNPCC Germline Colo rectal Colonoscopy Extra colonic


(Lynch mutations of cancer 30-70% every 1-2 years cancers
syndrome) DNA Endometrial from age 25
mismatch cancer 30-70% Consideration
repair genes Gastric cancer of prophylactic
5-10% surgery
Scanty colonic Extra colonic
polyps may be surveillance
present recommended
Colonic
tumours likely
to be right sided
and mucinous
Next question 

Display my notes on this topic

          

Save my notes
gathered by dr. elbarky.

Question stats

A 14.1%
B 8.7%
C 6%
D 43.2%
E 28%

43.2% of users answered this question correctly

Search eMRCS

Search term Go

 Open MRCS Part A textbook (../review/textbook.php)

External links

+ Suggest a link

Dashboard

3

Question 95 of 364

 

A 32 year old man undergoes an appendicectomy. A large carcinoid tumour is


identified and a completion right hemicolectomy is performed. He is well for
several months and then develops symptoms of palpitations and facial flushing.
Which of the following diagnostic markers should be requested?
gathered by dr. elbarky.

Alpha feto protein

Urinary 5-Hydroxyindoleacetic acid measurements

Urinary catecholamines

Urinary VMA measurements

None of the above

5 HIAA is the most commonly used diagnostic marker for carcinoid syndrome, it is
measured in a 24 hour urine collection.

Please rate this question:

 Discuss and give feedback

Next question 

Carcinoid syndrome

Carcinoid tumours secrete serotonin


Originate in neuroendocrine cells mainly in the intestine (midgut-distal
ileum/appendix)
Can occur in the rectum, bronchi
Hormonal symptoms mainly occur when disease spreads outside the bowel

Clinical features
Onset: insidious over many years
Flushing face
Palpitations
Pulmonary valve stenosis and tricuspid regurgitation causing dyspnoea
Asthma
Severe diarrhoea (secretory, persists despite fasting)

Investigation
5-HIAA in a 24-hour urine collection
Somatostatin receptor scintigraphy
CT scan
Blood testing for chromogranin A

Treatment
gathered by dr. elbarky.

Octreotide
Surgical removal

Next question 

Display my notes on this topic

          

Save my notes

Question stats

A 6.4%
B 68.9%
C 6.6%
D 12.7%
E 5.4%

68.9% of users answered this question correctly

Search eMRCS

Search term Go

Question 96 of 364

 

A 56 year old man presents with jaundice. He has a long history of alcohol misuse.
On examination he is jaundiced and ultrasound shows multiple echo dense lesions
in both lobes of the liver. His alpha feto protein is elevated 6 times the normal
range. What is the most likely diagnosis?
gathered by dr. elbarky.

Gallstones

Hepatocellular carcinoma

Metastatic pancreatic adenocarcinoma

Mirizzi syndrome

Benign bile duct stricture

HCC may complicate cirrhosis. AFP is often raised in HCC.

Please rate this question:

 Discuss and give feedback

Next question 

Hepatocellular carcinoma

Hepatocellular carcinoma is the second leading cause of cancer deaths globally.


Up to 750,000 cases are reported annually. Unfortunately the incidence
approximates to the death rate so there are few long term survivors[1]. The
disease occurs most commonly in those with chronic hepatitis and established
liver cirrhosis. Therefore, these individuals should be closely screened for the
development of HCC with serum AFP and liver USS every 6-12 months. Rising AFP
and liver USS showing a nodule greater than 1cm in diameter makes HCC much
more likely and such patients should then undergo MRI scanning.
The presence of adenomas in an otherwise healthy liver is a recognised risk factor
for HCC [2, 3] and many surgeons will remove liver adenomas for this reason[4].

Diagnosis
The aim is to avoid unnecessary percutaneous biopsy. Radiologically on CT the
classical feature is a suspicious lesion which is highlighted during the arterial
phase with washout during the venous phase, this reflects the hypervascularity of
the lesions.The risk of tumour seeding as a result of a liver biopsy is 2.7% with a
median time interval between biopsy and seeding of 17 months[5].

Barcelona Clinic Liver Classification


There are many classification systems for addressing the management and
prognosis, the BCLC system has the convenience of categorising disease extent
with treatment and prognostic outcomes. In determining the ideal treatment
modality for HCC the key points are not just disease extent, but also the functional
state of the liver and patient.
gathered by dr. elbarky.

Prognosis-5 yr
Stage Features Treatment survival

Stage Child-Pugh A Resection 40-70%


0 Single lesion (less than
2cm)
Normal portal
pressures

Stage Single nodule greater If associated disease May be up to


A than 3cm or multiple then radiofrequency 70% in some
nodules (no more than ablation
3) If no associated disease
Child Pugh A/ B then transplantation

Stage Multiple nodules Trans arterial chemo- 26% at 3 years


B Child Pugh A/B embolisation (usually
with doxorubicin)

Stage Advanced tumours Sorafenib Usually survive


C Invasion of portal vein 10.7 months
Child Pugh A/B

Stage Child Pugh stage C Best supportive care Less than 6


D Advanced tumours months survival

[6]
In selected patients the best outcomes are achieved with surgical resection, or
transplantation where surgical resection is precluded. Anatomical resections with
minimum 2cm margins provide the best outcomes.
At the present time there is no evidence to recommend treatment with adjuvant
chemotherapy[6].

Sorafenib
This is an oral multi tyrosine kinase inhibitor. It is the only drug that has been
currently demonstrated to extend survival in individuals with advanced
hepatocellular cancer[7]. The improvement in survival is from a median of 7
months to 10 months.

References
1. Jemal, A., et al., Global cancer statistics. CA Cancer J Clin, 2011. 61(2): p. 69-90.
2. Leese, T., O. Farges, and H. Bismuth, Liver cell adenomas. A 12-year surgical
experience from a specialist hepato-biliary unit. Ann Surg, 1988. 208(5): p. 558-64.
3. Farges, O. and S. Dokmak, Malignant transformation of liver adenoma: an
analysis of the literature. Dig Surg, 2010. 27(1): p. 32-8.
4. Ehrl, D., et al., 'Incidentaloma' of the liver: management of a diagnostic and
therapeutic dilemma. HPB Surg, 2012. 2012: p. 891787.
gathered by dr. elbarky.

5. Silva, M.A., et al., Needle track seeding following biopsy of liver lesions in the
diagnosis of hepatocellular cancer: a systematic review and meta-analysis. Gut,
2008. 57(11): p. 1592-6.
6. EASL-EORTC clinical practice guidelines: management of hepatocellular
carcinoma. J Hepatol, 2012. 56(4): p. 908-43.
7. Abou-Alfa, G.K., et al., Phase II study of sorafenib in patients with advanced
hepatocellular carcinoma. J Clin Oncol, 2006. 24(26): p. 4293-300.

Next question 

Display my notes on this topic

          

Save my notes

Question stats

A 4.6%
B 77%
C 9.5%
D 4.8%
E 4.1%

77% of users answered this question correctly

Search eMRCS

Question 97 of 364

 

A 22 year old is found to have bilateral acoustic neuromas. Which of the disorders
listed below is most likely to be present?

Neurofibromatosis Type I
gathered by dr. elbarky.

Neurofibromatosis Type II

MEN I

MEN II

Gardner's syndrome

In NF2 bilateral acoustic neuromas are characteristic with a family history of


Neurofibroma.

Please rate this question:

 Discuss and give feedback

Next question 

Genetics and surgical disease

Some of the more commonly occurring genetic conditions occurring in surgical


patients are presented here.

Li-Fraumeni Syndrome
Autosomal dominant
Consists of germline mutations to p53 tumour suppressor gene
High incidence of malignancies particularly sarcomas and leukaemias
Diagnosed when:

*Individual develops sarcoma under 45 years


*First degree relative diagnosed with any cancer below age 45 years and another
family member develops malignancy under 45 years or sarcoma at any age
BRCA 1 and 2
Carried on chromosome 17 (BRCA 1) and Chromosome 13 (BRCA 2)
Linked to developing breast cancer (60%) risk.
Associated risk of developing ovarian cancer (55% with BRCA 1 and 25%
with BRCA 2).

Lynch Syndrome
Autosomal dominant
Develop colonic cancer and endometrial cancer at young age
80% of affected individuals will get colonic and/ or endometrial cancer
gathered by dr. elbarky.

High risk individuals may be identified using the Amsterdam criteria

Amsterdam criteria
Three or more family members with a confirmed diagnosis of colorectal cancer,
one of whom is a first degree (parent, child, sibling) relative of the other two.
Two successive affected generations.
One or more colon cancers diagnosed under age 50 years.
Familial adenomatous polyposis (FAP) has been excluded.

Gardners syndrome
Autosomal dominant familial colorectal polyposis
Multiple colonic polyps
Extra colonic diseases include: skull osteoma, thyroid cancer and
epidermoid cysts
Desmoid tumours are seen in 15%
Mutation of APC gene located on chromosome 5
Due to colonic polyps most patients will undergo colectomy to reduce risk of
colorectal cancer
Now considered a variant of familial adenomatous polyposis coli

Next question 

Display my notes on this topic

          

Save my notes

Question stats
A 26.7%
B 50.4%
50.4% of users answered this question correctly
C 5.6%
D 10%
E 7.3%
Search eMRCS

Search term Go
gathered by dr. elbarky.

 Open MRCS Part A textbook (../review/textbook.php)

External links

+ Suggest a link

Dashboard

10

11

12

13

14

15

Question 98 of 364

 

In patients with multiple endocrine neoplasia type IIb which of the following clinical
appearances is the patient most likely to display?

Acromegalic facies
gathered by dr. elbarky.

Turners type features

Profound kyphoscoliosis

Multiple bony exostoses

Marfanoid features

Patients with MEN IIb may display Marfanoid features. It is unclear at the present
time whether they have discrete changes in the microfibrils of elastic fibres that
are present in Marfans.

Please rate this question:

 Discuss and give feedback

Next question 

Multiple Endocrine Neoplasia

Multiple endocrine neoplasia (MEN) is inherited as an autosomal dominant


disorder.

The table below summarises the three main types of MEN:

MEN type I MEN type IIa MEN type IIb


Mnemonic 'three P's': Phaeochromocytoma Same as MEN IIa
Medullary thyroid with addition of:
Parathyroid (95%): Parathyroid cancer (70%) Marfanoid body
adenoma Hyperparathyroidism habitus
Pituitary (70%): (60%) Mucosal
Prolactinoma/ACTH/Growth neuromas
Hormone secreting adenoma
Pancreas (50%): Islet cell
tumours/Zollinger Ellison
syndrome

also: Adrenal (adenoma) and


gathered by dr. elbarky.

thyroid (adenoma)

MENIN gene (chromosome 11) RET oncogene RET oncogene


(chromosome 10) (chromosome 10)
Most common presentation =
hypercalcaemia

Next question 

Display my notes on this topic

          

Save my notes

Question stats

A 11.2%
B 6%
C 6.5%
D 9.5%
E 66.8%

66.8% of users answered this question correctly


Question 99 of 364

 

A 39 year old lady has undergone surgery for breast cancer. As part of the
histopathology report the pathologist provides the surgeon with a Nottingham
Prognostic Index score of 6.4. He also states that the tumour size is 2cm. Which of
the following inferences can be made in relation to this statement?
gathered by dr. elbarky.

The tumour is likely to be grade 1

Vascular invasion is present

Lymph node metastasis are definitely present

The tumour is oestrogen receptor positive

None of the above

A score of this value is unlikely to be reached with a grade 1 tumour and a size of
2cm. Therefore lymph node metastasis are definitely present. In addition since the
maximal score for lymph node metastasis is 3 the tumour is likely be of a higher
grade (see below). The Nottingham Prognostic Index provides no information
about oestrogen receptor status or the presence or absence of vascular invasion.

Please rate this question:

 Discuss and give feedback

Next question 

Nottingham prognostic index

The Nottingham Prognostic Index can be used to give an indication of survival


following breast cancer surgery. In this system, the tumour size is weighted less
heavily than other major prognostic parameters.

Calculation of NPI
Tumour Size x 0.2 + Lymph node score(From table below)+Grade score(From
table below).
Lymph nodes involved Grade
Score

1 0 1

2 1-3 2

3 >3 3

Prognosis
gathered by dr. elbarky.

Score Percentage 5 year survival

2.0 to 2.4 93%

2.5 to 3.4 85%

3.5 to 5.4 70%

>5.4 50%

This data was originally published in 1992. It should be emphasised that other
factors such as vascular invasion and receptor status also impact on survival and
are not included in this data and account for varying prognoses often cited in the
literature.

Reference
Galea, M.H., et al., The Nottingham Prognostic Index in primary breast cancer.
Breast Cancer Res Treat, 1992. 22(3): p. 207-19.

Next question 

Display my notes on this topic

          

Save my notes

Question stats
A 15.9%
B 13.4%
C 54.7%
D 6.8%
E 9.2%

54.7% of users answered this question correctly


gathered by dr. elbarky.

Search eMRCS

Search term Go

 Open MRCS Part A textbook (../review/textbook.php)

External links

+ Suggest a link

Dashboard

10

11

12

Question 100 of 364

 

A 3 year old boy is brought to the clinic by his mother who has noticed a mass in
his neck. On examination; he has a smooth mass located on the lateral aspect of
his anterior triangle, near to the angle of the mandible. On ultrasound; it has a fluid
filled, anechoic, appearance. What is the most likely cause?
gathered by dr. elbarky.

Cystic hygroma

Dermoid cyst

Thyroglossal cyst

Branchial cyst

Lymphoma

Branchial cysts are usually located laterally and derived from the second branchial
cleft. Unless infection has occurred they will usually have an anechoic appearance
on ultrasound.

Please rate this question:

 Discuss and give feedback

Next question 

Neck Masses in Children

Thyroglossal cyst Located in the anterior triangle, usually in the


midline and below the hyoid (65% cases)
Derived from remnants of the thyroglossal duct
Thin walled and anechoic on USS (echogenicity
suggests infection of cyst)
Branchial cyst Six branchial arches separated by branchial clefts
Incomplete obliteration of the branchial

apparatus may result in cysts, sinuses or fistulae


75% of branchial cysts originate from the second
branchial cleft
Usually located anterior to the
sternocleidomastoid near the angle of the
mandible
Unless infected the fluid of the cyst has a similar
consistency to water and is anechoic on USS
gathered by dr. elbarky.

Dermoids Derived from pleuripotent stem cells and are


located in the midline
Most commonly in a suprahyoid location
They have heterogeneous appearances on
imaging and contain variable amounts of calcium
and fat

Thyroid gland True thyroid lesions are rare in children and


usually represent thyroglossal cysts or tumours
like lymphoma

Lymphatic Usually located posterior to the


malformations sternocleidomastoid
Cystic hygroma result from occlusion of
lymphatic channels
The painless, fluid filled, lesions usually present
prior to the age of 2
They are often closely linked to surrounding
structures and surgical removal is difficult
They are typically hypoechoic on USS

Infantile May present in either triangle of the neck


haemangioma Grow rapidly initially and then will often
spontaneously regress
Plain x-rays will show a mass lesion, usually
containing calcified phleboliths
As involution occurs the fat content of the
lesions increases

Lymphadenopathy Located in either triangle of the neck


May be reactive or neoplastic
Generalised lymphadenopathy usually secondary
to infection in children (very common)
Next question 

Display my notes on this topic

          

Save my notes
gathered by dr. elbarky.

Question stats

A 30.4%
B 8%
C 5.5%
D 52.2%
E 3.9%

52.2% of users answered this question correctly

Search eMRCS

Search term Go

 Open MRCS Part A textbook (../review/textbook.php)

External links

+ Suggest a link

Dashboard

3

Question 101 of 364

 

A 28 year old lady presents with benign cyclical mastalgia. Which of the following
is not a recognised treatment for the condition?

Evening primrose oil


gathered by dr. elbarky.

Bromocriptine

Methotrexate

Danazol

Tamoxifen

Surgical excision of tender breast tissue is inappropriate

Methotrexate is used for the treatment of breast cancer. Whilst the use of
tamoxifen is of benefit other agents such as flaxseed oil or evening primrose oil
should be tried first. Danazol is effective, but many women dislike the side effects.

Please rate this question:

 Discuss and give feedback

Next question 

Benign cyclical mastalgia

Benign cyclical mastalgia is a common cause of breast pain in younger females. It


varies in intensity according to the phase of the menstrual cycle. It is not
associated with point tenderness of the chest wall (more likely to be Tietze's
syndrome).

The underlying cause is difficult to pinpoint, examination should focus on


identifying focal lesions (such as cysts) that may be treated to provide
symptomatic benefit. Women should be advised to wear a supportive bra.
Conservative treatments include flax seed oil and evening primrose oil. There is
slightly more evidence in favor of flax seed oil, though neither has performed much
better than placebo in RCT's.

Hormonal agents such as bromocriptine and danazol may be more effective.


However, many women discontinue these therapies due to adverse effects.

Next question 
gathered by dr. elbarky.

Display my notes on this topic

          

Save my notes

Question stats

A 9.9%
B 8.5%
C 56%
D 7%
E 18.7%

56% of users answered this question correctly

Search eMRCS

Search term Go

 Open MRCS Part A textbook (../review/textbook.php)

External links

+ Suggest a link

Question 102 of 364

 

A 22 year old man undergoes a splenectomy for an iatrogenic splenic injury. On the
second post operative day a full blood count is performed. Which of the following
components of the full blood count is the first to be affected ?

Erythrocyte count
gathered by dr. elbarky.

Reticulocyte count

Eosinophil count

Monocyte count

Lymphocyte count

The granulocyte (the eosinophil component is seldom raised) and platelet count
are the first to be affected following splenectomy. Then reticulocytes increase.
Although a lymphocytosis and monocytosis are reported, these take several weeks
to develop.

Please rate this question:

 Discuss and give feedback

Next question 

Post splenectomy blood film changes

The loss of splenic tissue results in the inability to readily remove immature or
abnormal red blood cells from the circulation. The red cell count does not alter
significantly. However, cytoplasmic inclusions may be seen e.g. Howell-Jolly
bodies.
In the first few days after splenectomy target cells, siderocytes and reticulocytes
will appear in the circulation. Immediately following splenectomy a granulocytosis
(mainly composed of neutrophils) is seen, this is replaced by a lymphocytosis and
monocytosis over the following weeks.
The platelet count is usually increased and this may be persistent, oral antiplatelet
agents may be needed in some patients.
Image showing Howell Jolly bodies (arrowed)
gathered by dr. elbarky.

(https://d2zgo9qer4wjf4.cloudfront.net/images_eMRCS/swb115b.jpg)
Image sourced from Wikipedia
(https://d2zgo9qer4wjf4.cloudfront.net
(http://en.wikipedia.org/wiki/Howell-
/images_eMRCS/swb115b.jpg)
Jolly body)

Next question 

Display my notes on this topic

          

Save my notes

Question stats

A 17.6%
B 54.8%
C 7%
D 9.9%
E 10.7%

54.8% of users answered this question correctly


Question 103 of 364

 

A 28 year old man presents with hypertension and haematuria. Haematological


investigations show polycythaemia but otherwise no abnormality. CT scanning
shows a left renal mass. What is the most likely cause?

Wilms tumour
gathered by dr. elbarky.

Renal adenocarcinoma

Renal transitional cell carcinoma

Staghorn calculus

Renal cyst

Renal adenocarcinoma is the most common variant and is associated with


polycythaemia.

Please rate this question:

 Discuss and give feedback

Next question 

Haematuria

Causes of haematuria

Trauma Injury to renal tract


Renal trauma commonly due to blunt injury
(others penetrating injuries)
Ureter trauma rare: iatrogenic
Bladder trauma: due to RTA or pelvic fractures

Infection Remember TB
Malignancy Renal cell carcinoma (remember paraneoplastic
syndromes): painful or painless
Urothelial malignancies: 90% are transitional cell
carcinoma, can occur anywhere along the urinary
tract. Painless haematuria.
Squamous cell carcinoma and adenocarcinoma:
rare bladder tumours
Prostate cancer
Penile cancers: SCC

Renal disease Glomerulonephritis


gathered by dr. elbarky.

Stones Microscopic haematuria common

Structural Benign prostatic hyperplasia (BPH) causes


abnormalities haematuria due to hypervascularity of the
prostate gland
Cystic renal lesions e.g. polycystic kidney disease
Vascular malformations
Renal vein thrombosis due to renal cell carcinoma

Coagulopathy Causes bleeding of underlying lesions

Drugs Cause tubular necrosis or interstitial nephritis:


aminoglycosides, chemotherapy
Interstitial nephritis: penicillin, sulphonamides,
and NSAIDs
Anticoagulants

Benign Exercise

Gynaecological Endometriosis: flank pain, dysuria, and


haematuria that is cyclical

Iatrogenic Catheterisation
Radiotherapy; cystitis, severe haemorrhage,
bladder necrosis

Pseudohaematuria For example following consumption of beetroot

References
Http://bestpractice.bmj.com/best-practice/monograph/316/overview
/aetiology.html
Next question 

Display my notes on this topic

          

Save my notes
gathered by dr. elbarky.

Question stats

A 10.3%
B 71.2%
C 8.4%
D 4.9%
E 5.2%

71.2% of users answered this question correctly

Search eMRCS

Search term Go

 Open MRCS Part A textbook (../review/textbook.php)

External links

+ Suggest a link

Dashboard

3

Question 103 of 364

 

A 28 year old man presents with hypertension and haematuria. Haematological


investigations show polycythaemia but otherwise no abnormality. CT scanning
shows a left renal mass. What is the most likely cause?

Wilms tumour
gathered by dr. elbarky.

Renal adenocarcinoma

Renal transitional cell carcinoma

Staghorn calculus

Renal cyst

Renal adenocarcinoma is the most common variant and is associated with


polycythaemia.

Please rate this question:

 Discuss and give feedback

Next question 

Haematuria

Causes of haematuria

Trauma Injury to renal tract


Renal trauma commonly due to blunt injury
(others penetrating injuries)
Ureter trauma rare: iatrogenic
Bladder trauma: due to RTA or pelvic fractures

Infection Remember TB
Malignancy Renal cell carcinoma (remember paraneoplastic
syndromes): painful or painless
Urothelial malignancies: 90% are transitional cell
carcinoma, can occur anywhere along the urinary
tract. Painless haematuria.
Squamous cell carcinoma and adenocarcinoma:
rare bladder tumours
Prostate cancer
Penile cancers: SCC

Renal disease Glomerulonephritis


gathered by dr. elbarky.

Stones Microscopic haematuria common

Structural Benign prostatic hyperplasia (BPH) causes


abnormalities haematuria due to hypervascularity of the
prostate gland
Cystic renal lesions e.g. polycystic kidney disease
Vascular malformations
Renal vein thrombosis due to renal cell carcinoma

Coagulopathy Causes bleeding of underlying lesions

Drugs Cause tubular necrosis or interstitial nephritis:


aminoglycosides, chemotherapy
Interstitial nephritis: penicillin, sulphonamides,
and NSAIDs
Anticoagulants

Benign Exercise

Gynaecological Endometriosis: flank pain, dysuria, and


haematuria that is cyclical

Iatrogenic Catheterisation
Radiotherapy; cystitis, severe haemorrhage,
bladder necrosis

Pseudohaematuria For example following consumption of beetroot

References
Http://bestpractice.bmj.com/best-practice/monograph/316/overview
/aetiology.html
Next question 

Display my notes on this topic

          

Save my notes
gathered by dr. elbarky.

Question stats

A 10.3%
B 71.2%
C 8.4%
D 4.9%
E 5.2%

71.2% of users answered this question correctly

Search eMRCS

Search term Go

 Open MRCS Part A textbook (../review/textbook.php)

External links

+ Suggest a link

Dashboard

3

Question 104 of 364

 

A 75 year old lady presents with weight loss, pain and a swelling over her left knee.
She has been treated for Pagets disease of the bone for some time. What is the
most likely diagnosis?

Ewings sarcoma
gathered by dr. elbarky.

Osteosarcoma

Myeloma

Septic arthritis

Osteoclastoma

Osteosarcoma may complicate Pagets disease of bone in up to 10% cases.


Radiological appearances include bone destruction coupled with new bone
formation, periosteal elevation may also occur. Surgical resection is the main
treatment.

Please rate this question:

 Discuss and give feedback

Next question 

Sarcomas

Malignant tumours of mesenchymal origin

Types
May be either bone or soft tissue in origin.
Bone sarcoma include:
Osteosarcoma
Ewings sarcoma (although non bony sites recognised)
Chondrosarcoma - originate from Chondrocytes
Soft tissue sarcoma are a far more heterogeneous group and include:
Liposarcoma-adipocytes
Rhabdomyosarcoma-striated muscle
Leiomyosarcoma-smooth muscle
Synovial sarcomas- close to joints (cell of origin not known but not
synovium)

Malignant fibrous histiocytoma is a sarcoma that may arise in both soft tissue and
bone.

Features
gathered by dr. elbarky.

Certain features of a mass or swelling should raise suspicion for a sarcoma these
include:
Large >5cm soft tissue mass
Deep tissue location or intra muscular location
Rapid growth
Painful lump

Assessment
Imaging of suspicious masses should utilise a combination of MRI, CT and USS.
Blind biopsy should not be performed prior to imaging and where required should
be done in such a way that the biopsy tract can be subsequently included in any
resection.

Ewings sarcoma
Commoner in males
Incidence of 0.3 / 1, 000, 000
Onset typically between 10 and 20 years of age
Location by femoral diaphysis is commonest site
Histologically it is a small round tumour
Blood borne metastasis is common and chemotherapy is often combined
with surgery

Osteosarcoma
Mesenchymal cells with osteoblastic differentiation
20% of all primary bone tumours
Incidence of 5 per 1,000,000
Peak age 15-30, commoner in males
Limb preserving surgery may be possible and many patients will receive
chemotherapy

Liposarcoma
Malignancy of adipocytes
Rare, approximately 2.5 per 1,000,000. They are the most common soft
tissue sarcoma
Typically located in deep locations such as retroperitoneum
Affect older age group usually >40 years of age
May be well differentiated and thus slow growing although may undergo de-
differentiation and disease progression
Many tumours will have a pseudocapsule that can misleadingly allow
surgeons to feel that they can 'shell out' these lesions. In reality, tumour may
invade at the edge of the pseudocapsule and result in local recurrence if this
strategy is adopted
Usually resistant to radiotherapy, although this is often used in a palliative
setting

Malignant Fibrous Histiocytoma


gathered by dr. elbarky.

Tumour with large number of histiocytes


Also described as undifferentiated pleomorphic sarcoma NOS (i.e. Cell of
origin is not known)
Four major subtypes are recognised: storiform-pleomorphic (70% cases),
myxoid (less aggressive), giant cell and inflammatory
Treatment is usually with surgical resection and adjuvant radiotherapy as
this reduces the likelihood of local recurrence

Next question 

Display my notes on this topic

          

Save my notes

Question stats

A 9.9%
B 53.8%
C 10%
D 8.1%
E 18.3%

53.8% of users answered this question correctly

Search eMRCS

Question 105 of 364

 

An 82 year old lady presents with a carcinoma of the caecum. Approximately what
proportion of patients presenting with this diagnosis will have synchronous
lesions?

<1%
gathered by dr. elbarky.

60%

50%

20%

5%

Synchronous colonic tumours are seen in 5% cases and all patients having a
flexible sigmoidoscopy should have completion colonoscopy if tumours or
polyps are found

Synchronous lesions may occur in up to 5% of patients with colorectal cancer. A


full and complete lumenal study with either colonoscopy, CT cologram or barium
enema is mandatory in all patients being considered for surgery.

Please rate this question:

 Discuss and give feedback

Next question 

Colorectal cancer

Annually, about 150,000 new cases are diagnosed and 50,000 deaths from
the disease
About 75% will have sporadic disease and 25% will have a family history
Colorectal tumours comprise a spectrum of disease ranging from
adenomas through to polyp cancers and frank malignancy.
Polyps may be categorised into: neoplastic polyps, adenomatous polyps and
non neoplastic polyps.
The majority of adenomas are polypoidal lesions, although flat lesions do
occur and may prove to be dysplastic.
Non-neoplastic polyps include hyperplastic, juvenile, hamartomatous,
inflammatory, and lymphoid polyps, which have not generally been thought
of as precursors of cancer.
Three characteristics of adenomas that correlate with malignant potential
have been characterised. These include increased size, villous architecture
and dysplasia. For this reason most polyps identified at colonoscopy should
be removed.
The transformation from polyp to cancer is described by the adenoma -
gathered by dr. elbarky.

carcinoma sequence and its principles should be appreciated. Essentially


genetic changes accompany the transition from adenoma to carcinoma; key
changes include APC, c-myc, K RAS mutations and p53 deletions.

Next question 

Display my notes on this topic

          

Save my notes

Question stats

A 12.6%
B 9.3%
C 9.2%
D 26.1%
E 42.9%

42.9% of users answered this question correctly

Search eMRCS

Search term Go

Question 106 of 364

 

An enthusiastic medical student approaches you with a list of questions about


blood transfusion reactions. Which of her following points is incorrect?

Graft versus host disease involves neutrophil proliferation


gathered by dr. elbarky.

Thrombocytopaenia may occur in women with a prior pregnancy

IgA antibodies may cause blood pressure compromise during transfusion

Hypocalcaemia can occur

Iron overload can be avoided by chelation therapy

Mnemonic for transfusion reactions:

Got a bad unit

G raft vs. Host disease


O verload
T hrombocytopaenia

A lloimmunization

B lood pressure unstable


A cute haemolytic reaction
D elayed haemolytic reaction

U rticaria
N eutrophilia
I nfection
T ransfusion associated lung injury

GVHD results from lymphocytic proliferation. The patient's own lymphocytes are
similar to the donor's lymphocytes, therefore don't perceive them as being foreign.
The donor lymphocytes, however, sees the recipient lymphocytes as being foreign.
Therefore they proliferate causing severe complications.

Thrombocytopaenia occurs a few days after transfusion and may resolve


spontaneously.

Patients with IGA antibodies need IgA deficient blood transfusions.

Please rate this question:

 Discuss and give feedback

Next question 
gathered by dr. elbarky.

Blood transfusion reactions

Acute transfusion reactions present as adverse signs or symptoms during or


within 24 hours of a blood transfusion. The most frequent reactions are fever,
chills, pruritus, or urticaria, which typically resolve promptly without specific
treatment or complications. Other signs occurring in temporal relationship with a
blood transfusion, such as severe dyspnoea, pyrexia, or loss of consciousness may
be the first indication of a more severe potentially fatal reaction.
The causes of adverse reactions are multi-factorial. Immune mediated reactions,
some of the most feared, occur as a result of component mismatch, the
commonest cause of which is clerical error. More common, non immune mediated,
complications may occur as a result of product contamination, this may be
bacterial or viral.
Transfusion related lung injury is well recognised and there are two proposed
mechanisms which underpin this. One involves the sequestration of primed
neutrophils within the recipient pulmonary capillary bed. The other proposed
mechanism suggests that HLA mismatches between donor neutrophils and
recipient lung tissue is to blame.
The table below summarises the main types of transfusion reaction.

Immune mediated Non immune mediated

Pyrexia Hypocalcaemia

Alloimmunization CCF

Thrombocytopaenia Infections

Transfusion associated lung injury Hyperkalaemia

Graft vs Host disease

Urticaria

Acute or delayed haemolysis


Immune mediated Non immune mediated

ABO incompatibility

Rhesus incompatibility

Next question 
gathered by dr. elbarky.

Display my notes on this topic

          

Save my notes

Question stats

A 39.2%
B 11.2%
C 25.9%
D 16%
E 7.7%

39.2% of users answered this question correctly

Search eMRCS

Search term Go

 Open MRCS Part A textbook (../review/textbook.php)

External links

+ Suggest a link

Question 107 of 364

 

A 32 year old man is admitted with a distended tense abdomen. He previously


underwent a difficult appendicectomy 1 year previously and was discharged. At
laparotomy the abdomen is filled with a gelatinous substance. What is the most
likely diagnosis?
gathered by dr. elbarky.

Appendiceal sarcoma

Pseudomyxoma peritoneii

Appendiceal adenocarcinoma

Appendiceal carcinoid

Caecal adenocarcinoma

Pseudomyxoma is classically associated with mucin production and the appendix


is the commonest source.

Please rate this question:

 Discuss and give feedback

Next question 

Pseudomyxoma Peritonei

Rare mucinous tumour


Most commonly arising from the appendix (other abdominal viscera are also
recognised as primary sites)
Incidence of 1-2/1,000,000 per year
The disease is characterised by the accumulation of large amounts of
mucinous material in the abdominal cavity

Treatment
Is usually surgical and consists of cytoreductive surgery (and often peritonectomy
c.f Sugarbaker procedure) combined with intra peritoneal chemotherapy with
mitomycin C.

Survival is related to the quality of primary treatment and in Sugarbakers own


centre 5 year survival rates of 75% have been quoted. Patients with disseminated
intraperitoneal malignancy from another source fare far worse.
In selected patients a second look laparotomy is advocated and some practice this
routinely.

Next question 
gathered by dr. elbarky.

Display my notes on this topic

          

Save my notes

Question stats

A 5%
B 77.3%
C 5%
D 8%
E 4.7%

77.3% of users answered this question correctly

Search eMRCS

Search term Go

 Open MRCS Part A textbook (../review/textbook.php)


Question 108 of 364

 

A 23 year old man suffers a thermal injury to his left hand. It becomes red and
painful. Which of the following mediators are not involved in this process?

Histamine
gathered by dr. elbarky.

Free radicals

Prostaglandins

Leukotrienes

Serotonin

Acute inflammation is not mediated by free radicals

Chemical mediators facilitate the spread of inflammation into normal tissue


Chemical mediators include:
Lysosomal compounds
Chemokines such as serotinin and histamine (released by platelets and
mast cells)

Other enzyme cascades producing inflammatory mediators include:


Complement, kinin, coagulation system and fibrinolytic system

Please rate this question:

 Discuss and give feedback

Next question 

Acute inflammation

Inflammation is the reaction of the tissue elements to injury. Vascular changes


occur, resulting in the generation of a protein rich exudate. So long as the injury
does not totally destroy the existing tissue architecture, the episode may resolve
with restoration of original tissue architecture.

Vascular changes
Vasodilation occurs and persists throughout the inflammatory phase.
Inflammatory cells exit the circulation at the site of injury.
The equilibrium that balances Starlings forces within capillary beds is
disrupted and a protein rich exudate will form as the vessel walls also
become more permeable to proteins.
The high fibrinogen content of the fluid may form a fibrin clot. This has
several important immunomodulatory functions.
gathered by dr. elbarky.

Sequelae

Resolution Typically occurs with minimal initial injury


Stimulus removed and normal tissue
architecture results

Organisation Delayed removal of exudate


Tissues undergo organisation and usually
fibrosis

Suppuration Typically formation of an abscess or an


empyema
Sequestration of large quantities of dead
neutrophils

Progression to chronic Coupled inflammatory and reparative


inflammation activities
Usually occurs when initial infection or
suppuration has been inadequately managed

Causes
Infections e.g. Viruses, exotoxins or endotoxins released by bacteria
Chemical agents
Physical agents e.g. Trauma
Hypersensitivity reactions
Tissue necrosis

Presence of neutrophil polymorphs is a histological diagnostic feature of acute


inflammation

Next question 
Display my notes on this topic

          

Save my notes

Question stats
gathered by dr. elbarky.

A 9.8%
B 32.7%
C 7.4%
D 7.9%
E 42.2%

32.7% of users answered this question correctly

Search eMRCS

Search term Go

 Open MRCS Part A textbook (../review/textbook.php)

External links

+ Suggest a link

Dashboard

5

Question 109 of 364

 

Which of the following are not typical of Lynch syndrome?

It is inherited in an autosomal recessive manner

Affected patients are more likely to develop right colon mucinous tumours
gathered by dr. elbarky.

than the general population

Affected individuals have an 80% lifetime risk of colon cancer

Endometrial cancer is seen in 80% of women

Gastric cancers are more common

Lynch syndrome is inherited in an autosomal dominant fashion. It is characterised


by microsatellite instability in the DNA mismatch repair genes. Colonic tumours in
patients with Lynch syndrome are more likely to be right sided tumours and to be
poorly differentiated.

Please rate this question:

 Discuss and give feedback

Next question 

Genetics and surgical disease

Some of the more commonly occurring genetic conditions occurring in surgical


patients are presented here.

Li-Fraumeni Syndrome
Autosomal dominant
Consists of germline mutations to p53 tumour suppressor gene
High incidence of malignancies particularly sarcomas and leukaemias
Diagnosed when:

*Individual develops sarcoma under 45 years


*First degree relative diagnosed with any cancer below age 45 years and another
family member develops malignancy under 45 years or sarcoma at any age

BRCA 1 and 2
Carried on chromosome 17 (BRCA 1) and Chromosome 13 (BRCA 2)
Linked to developing breast cancer (60%) risk.
Associated risk of developing ovarian cancer (55% with BRCA 1 and 25%
with BRCA 2).

Lynch Syndrome
Autosomal dominant
gathered by dr. elbarky.

Develop colonic cancer and endometrial cancer at young age


80% of affected individuals will get colonic and/ or endometrial cancer
High risk individuals may be identified using the Amsterdam criteria

Amsterdam criteria
Three or more family members with a confirmed diagnosis of colorectal cancer,
one of whom is a first degree (parent, child, sibling) relative of the other two.
Two successive affected generations.
One or more colon cancers diagnosed under age 50 years.
Familial adenomatous polyposis (FAP) has been excluded.

Gardners syndrome
Autosomal dominant familial colorectal polyposis
Multiple colonic polyps
Extra colonic diseases include: skull osteoma, thyroid cancer and
epidermoid cysts
Desmoid tumours are seen in 15%
Mutation of APC gene located on chromosome 5
Due to colonic polyps most patients will undergo colectomy to reduce risk of
colorectal cancer
Now considered a variant of familial adenomatous polyposis coli

Next question 

Display my notes on this topic

          

Save my notes
Question stats

A 56.1%
B 8.3%
C 7.7%
D 11.9%
E 16.1%

56.1% of users answered this question correctly


gathered by dr. elbarky.

Search eMRCS

Search term Go

 Open MRCS Part A textbook (../review/textbook.php)

External links

+ Suggest a link

Dashboard

10

Question 110 of 364

 

A 34-year-old man is taken immediately to theatre with aortic dissection. You note
he is tall with pectus excavatum and arachnodactyly. His condition is primarily due
to a defect in which one of the following proteins?

Polycystin-1
gathered by dr. elbarky.

Fibrillin

Type IV collagen

Type I collagen

Elastin

Although fibrillin is the primary protein affected (due to a defect in the fibrillin-1
gene) it should be noted that fibrillin is used as a substrate of elastin.

Please rate this question:

 Discuss and give feedback

Next question 

Marfan's syndrome

Marfan's syndrome is an autosomal dominant connective tissue disorder. It is


caused by a defect in the fibrillin-1 gene on chromosome 15 and affects around 1
in 3,000 people.

Features
tall stature with arm span to height ratio > 1.05
high-arched palate
arachnodactyly
pectus excavatum
pes planus
scoliosis of > 20 degrees
heart: dilation of the aortic sinuses (seen in 90%) which may lead to aortic
aneurysm, aortic dissection, aortic regurgitation, mitral valve prolapse (75%),
lungs: repeated pneumothoraces
eyes: upwards lens dislocation (superotemporal ectopia lentis), blue sclera,
myopia
dural ectasia (ballooning of the dural sac at the lumbosacral level)

The life expectancy of patients used to be around 40-50 years. With the advent of
regular echocardiography monitoring and beta-blocker/ACE-inhibitor therapy this
has improved significantly over recent years. Aortic dissection and other
cardiovascular problems remain the leading cause of death however.
gathered by dr. elbarky.

Next question 

Display my notes on this topic

          

Save my notes

Question stats

A 6.5%
B 36.2%
C 20.6%
D 18.2%
E 18.5%

36.2% of users answered this question correctly

Search eMRCS

Search term Go

 Open MRCS Part A textbook (../review/textbook.php)


Question 92 of 364

 

Which of the following breast tumours is most commonly associated with a risk of
metastasis to the contralateral breast?

Invasive ductal carcinoma


gathered by dr. elbarky.

Invasive lobular carcinoma

Phyllodes tumour

Pagets disease of the breast

Atypical ductal hyperplasia

Risk of metastasis to the contralateral breast is a classical feature of invasive


lobular carcinoma.

Please rate this question:

 Discuss and give feedback

Next question 

Lobular carcinoma of the breast

Lobular breast cancers are less common than their ductal counterparts. They
typically present differently, the mass is usually more diffuse and less obvious on
the usual imaging modalities of ultrasound and mammography. This is significant
since the disease may be understaged resulting in inadequate treatment when
wide local excision is undertaken.
In women with invasive lobular carcinoma it is usually safest to perform an MRI
scan of the breast, if breast conserving surgery is planned.
Lobular carcinomas are also more likely to be multifocal and metastasise to the
contralateral breast.
Lobular carcinoma in situ is occasionally diagnosed incidentally on core biopsies.
Unlike DCIS, lobular carcinoma in situ is far less strongly associated with foci of
invasion and is usually managed by close monitoring.
Lobular carcinoma stained using haematoxylin and eosin
gathered by dr. elbarky.

(https://d2zgo9qer4wjf4.cloudfront.net/images_eMRCS/swb217b.png)
Image sourced from (https://d2zgo9qer4wjf4.cloudfront.net/images_eMRCS
Wikipedia () /swb217b.png)

Next question 

Display my notes on this topic

          

Save my notes

Question stats

A 15.7%
B 61.4%
C 10.1%
D 6.9%
E 5.8%

61.4% of users answered this question correctly


Question 111 of 364

 

A 25 year old junior doctor has a chest x-ray performed as part of a routine
insurance medical examination. The x-ray shows evidence of rib notching.
Auscultation of his chest reveals a systolic murmur which is loudest at the
posterior aspect of the fourth intercostal space. What is the most likely diagnosis?
gathered by dr. elbarky.

Patent ductus arteriosus

Aortic coarctation

Aortic dissection

Cervical rib

Subclavian steal syndrome

Coarctation of the aorta may occur due to the remnant of the ductus arteriosus
acting as a fibrous constrictive band of the aorta. Weak arm pulses may be seen,
radiofemoral delay is the classical physical finding. Collateral flow through the
intercostal vessels may produce notching of the ribs, if the disease is long
standing.

Please rate this question:

 Discuss and give feedback

Next question 

Vascular disease

Aortic Chest pain (anterior chest pain- ascending aorta,


dissection back pain - descending aorta)
Widening of aorta on chest x-ray
Diagnosis made by CT scanning
Treatment is either medical (Type B disease) or
surgical (Type A disease)
Cervical rib Supernumery fibrous band arising from seventh
cervical vertebra
Incidence of 1 in 500
May cause thoracic outlet syndrome
Treatment involves surgical division of rib

Subclavian steal Due to proximal stenotic lesion of the subclavian


syndrome artery
Results in retrograte flow through vertebral or
internal thoracic arteries
The result is that decrease in cerebral blood flow
gathered by dr. elbarky.

may occur and produce syncopal symptoms


A duplex scan and/ or angiogram will delineate the
lesion and allow treatment to be planned

Takayasu's Large vessel granulomatous vasculitis


arteritis Results in intimal narrowing
Most commonly affects young asian females
Patients present with features of mild systemic
illness, followed by pulseless phase with symptoms
of vascular insufficiency
Treatment is with systemic steroids

Patent ductus Ductus arteriosus is a normal foetal vessel that


arteriosus closes spontaneously after birth
Results in high pressure, oxygenated blood entering
the pulmonary circuit
Untreated patients develop symptoms of congestive
cardiac failure

Coarctation of Aortic stenosis at the site of the ductus arteriosus


the aorta insertion
More prevalent in boys or females with Turners
syndrome
Patients may present with symptoms of arterial
insufficiency, such as syncope and claudication
Blood pressure mismatch may be seen, as may
mismatch of pulse pressure in the upper and lower
limbs
Treatment is either with angioplasty or surgical
resection (the former is the most common)

Next question 
Display my notes on this topic

          

Save my notes

Question stats
gathered by dr. elbarky.

A 16.8%
B 60.1%
C 7.4%
D 9.6%
E 6.1%

60.1% of users answered this question correctly

Search eMRCS

Search term Go

 Open MRCS Part A textbook (../review/textbook.php)

External links

+ Suggest a link

Dashboard

5

Question 112 of 364

 

A 56 year old man presents with lethargy, haematuria and haemoptysis. On


examination he is hypertensive and has a right loin mass. A CT scan shows a
lesion affecting the upper pole of the right kidney, it has a small cystic centre.
Which of the options below is the most likely diagnosis?
gathered by dr. elbarky.

Squamous cell carcinoma of the kidney

Nephroblastoma

Renal adenocarcinoma

Transitional cell carcinoma of the kidney

Polycystic kidney disease

Renal adenocarcinoma are the most common renal tumours. These will typically
affect the renal parenchyma. Transitional cell carcinoma will usually affect
urothelial surfaces. Nephroblastoma would be very rare in this age group. Renal
adenocarcinoma may produce cannon ball metastasis in the lung which cause
haemoptysis, this is not a feature of PKD.

Please rate this question:

 Discuss and give feedback

Next question 

Renal tumours

Renal cell carcinoma


Renal cell carcinoma is an adenocarcinoma of the renal cortex and is believed to
arise from the proximal convoluted tubule. They are usually solid lesions, up to
20% may be multifocal, 20% may be calcified and 20% may have either a cystic
component or be wholly cystic. They are often circumscribed by a pseudocapsule
of compressed normal renal tissue. Spread may occur either by direct extension
into the adrenal gland, renal vein or surrounding fascia. More distant disease
usually occurs via the haematogenous route to lung, bone or brain.
Renal cell carcinoma comprise up to 85% of all renal malignancies. Males are
more commonly affected than females and sporadic tumours typically affect
patients in their sixth decade.
Patients may present with a variety of symptoms including; haematuria (50%), loin
pain (40%), mass (30%) and up to 25% may have symptoms of metastasis.Less
than 10% have the classic triad of haematuria, pain and mass.

Investigation
Many cases will present as haematuria and be discovered during diagnostic work
up. Benign renal tumours are rare, so renal masses should be investigated with
multislice CT scanning. Some units will add an arterial and venous phase to the
scan to demonstrate vascularity and evidence of caval ingrowth.
gathered by dr. elbarky.

CT scanning of the chest and abdomen to detect distant disease should also be
undertaken.

Routine bone scanning is not indicated in the absence of symptoms.

Biopsy should not be performed when a nephrectomy is planned but is mandatory


before any ablative therapies are undertaken.

Assessment of the functioning of the contra lateral kidney.

Management
T1 lesions may be managed by partial nephrectomy and this gives equivalent
oncological results to total radical nephrectomy. Partial nephrectomy may also be
performed when there is inadequate reserve in the remaining kidney.

For T2 lesions and above a radical nephrectomy is standard practice and this may
be performed via a laparoscopic or open approach. Preoperative embolisation is
not indicated nor is resection of uninvolved adrenal glands. During surgery early
venous control is mandatory to avoid shedding of tumour cells into the circulation.

Patients with completely resected disease do not benefit from adjuvant therapy
with either chemotherapy or biological agents. These should not be administered
outside the setting of clinical trials.

Patients with transitional cell cancer will require a nephroureterectomy with


disconnection of the ureter at the bladder.

References
Lungberg B et al. EAU guidelines on renal cell carcinoma: The 2010 update.
European Urology 2010 (58): 398-406.

Next question 
Display my notes on this topic

          

Save my notes

Question stats
gathered by dr. elbarky.

A 9.7%
B 9.5%
C 55.4%
D 9.1%
E 16.4%

55.4% of users answered this question correctly

Search eMRCS

Search term Go

 Open MRCS Part A textbook (../review/textbook.php)

External links

+ Suggest a link

Dashboard

5

Question 113 of 364

 

A 78 year old man is referred to the clinic by his general practitioner. For many
years he noticed a smooth swelling approximately 2cm anterior to the tragus of his
right ear. Apart from being a heavy smoker he has no co-morbidities. What is the
most likely diagnosis?
gathered by dr. elbarky.

Pleomorphic adenoma

Liposarcoma

Warthins tumour

Adenocarcinoma

None of the above

Warthins tumours are most common in elderly smokers. They have a relatively
benign and indolent course. They are usually well circumscribed as illustrated
below:

(https://d2zgo9qer4wjf4.cloudfront.net

/images_eMRCS/swb088b.jpg)
Image sourced from Wikipedia
(https://d2zgo9qer4wjf4.cloudfront.net
(http://en.wikipedia.org
/images_eMRCS/swb088b.jpg)
/wiki/Warthin%27s tumor)

Please rate this question:

 Discuss and give feedback

Next question 
gathered by dr. elbarky.

Parotid gland clinical

Benign neoplasms
Up to 80% of all salivary gland tumours occur in the parotid gland and up to 80% of
these are benign. There is no consistent correlation between the rate of growth
and the malignant potential of the lesion. However, benign tumours should not
invade structures such as the facial nerve.
With the exception of Warthins tumours, they are commoner in women than men.
The median age of developing a lesion is in the 5th decade of life.

Benign tumour types

Tumour type Features

Benign pleomorphic Most common parotid neoplasm (80%)


adenoma or benign mixed Proliferation of epithelial and myoepithelial cells
tumor of the ducts and an increase in stromal
components
Slow growing, lobular, and not well encapsulated
Recurrence rate of 1-5% with appropriate
excision (parotidectomy)
Recurrence possibly secondary to capsular
disruption during surgery
Malignant degeneration occurring in 2-10% of
adenomas observed for long periods, with
carcinoma ex-pleomorphic adenoma occurring
most frequently as adenocarcinoma
Tumour type Features

Warthin tumor (papillary Second most common benign parotid tumor


cystadenoma lymphoma or (5%)
adenolymphoma) Most common bilateral benign neoplasm of the
parotid
Marked male as compared to female
predominance
Occurs later in life (sixth and seventh decades)
Presents as a lymphocytic infiltrate and cystic
epithelial proliferation
May represent heterotopic salivary gland
gathered by dr. elbarky.

epithelial tissue trapped within intraparotid


lymph nodes
Incidence of bilaterality and multicentricity of
10%
Malignant transformation rare (almost unheard
of)

Monomorphic adenoma Account for less than 5% of tumours


Slow growing
Consist of only one morphological cell type
(hence term mono)
Include; basal cell adenoma, canalicular
adenoma, oncocytoma, myoepitheliomas

Haemangioma Should be considered in the differential of a


parotid mass in a child
Accounts for 90% of parotid tumours in children
less than 1 year of age
Hypervascular on imaging
Spontaneous regression may occur and
malignant transformation is almost unheard of

Malignant salivary gland tumours


Types of malignancy

Mucoepidermoid 30% of all parotid malignancies


carcinoma Usually low potential for local invasiveness and
metastasis (depends mainly on grade)
Adenoid cystic Unpredictable growth pattern
carcinoma Tendency for perineural spread
Nerve growth may display skip lesions resulting in
incomplete excision
Distant metastasis more common (visceral rather than
nodal spread)
5 year survival 35%

Mixed tumours Often a malignancy occurring in a previously benign


parotid lesion

Acinic cell carcinoma Intermediate grade malignancy


gathered by dr. elbarky.

May show perineural invasion


Low potential for distant metastasis
5 year survival 80%

Adenocarcinoma Develops from secretory portion of gland


Risk of regional nodal and distant metastasis
5 year survival depends upon stage at presentation,
may be up to 75% with small lesions with no nodal
involvement

Lymphoma Large rubbery lesion, may occur in association with


Warthins tumours
Diagnosis should be based on regional nodal biopsy
rather than parotid resection
Treatment is with chemotherapy (and radiotherapy)

Diagnostic evaluation
Plain x-rays may be used to exclude calculi
Sialography may be used to delineate ductal anatomy
FNAC is used in most cases
Superficial parotidectomy may be either diagnostic of therapeutic
depending upon the nature of the lesion
Where malignancy is suspected the primary approach should be definitive
resection rather than excisional biopsy
CT/ MRI may be used in cases of malignancy for staging primary disease

Treatment
For nearly all lesions this consists of surgical resection, for benign disease this will
usually consist of a superficial parotidectomy. For malignant disease a radical or
extended radical parotidectomy is performed. The facial nerve is included in the
resection if involved. The need for neck dissection is determined by the potential
for nodal involvement.

Other parotid disorders


HIV infection
Lymphoepithelial cysts associated with HIV occur almost exclusively in the
parotid
Typically presents as bilateral, multicystic, symmetrical swelling
Risk of malignant transformation is low and management usually
conservative

Sjogren syndrome
Autoimmune disorder characterised by parotid enlargement, xerostomia and
keratoconjunctivitis sicca
90% of cases occur in females
gathered by dr. elbarky.

Second most common connective tissue disorder


Bilateral, non tender enlargement of the gland is usual
Histologically, the usual findings are of a lymphocytic infiltrate in acinar
units and epimyoepithelial islands surrounded by lymphoid stroma
Treatment is supportive
There is an increased risk of subsequent lymphoma

Sarcoid
Parotid involvement occurs in 6% of patients with sarcoid
Bilateral in most cases
Gland is not tender
Xerostomia may occur
Management of isolated parotid disease is usually conservative

Next question 

Display my notes on this topic

          

Save my notes

Question stats

A 35%
B 7%
C 42.1%
D 9.8%
E 6.1%

Question 114 of 364

 

A 44 year old lady presents with a pathological fracture of the left femur. She has
previously undergone a renal transplant for end stage renal failure. Her blood test
results are as follows:

Serum Ca2+ 2.80


gathered by dr. elbarky.

PTH 88pg/ml

Phosphate 0.30

A surgeon decides to perform a parathyroidectomy on the basis of these results.


When the glands are assessed histologically, which of the appearances is most
likely to be identified?

Metaplasia the gland

Hypertrophy of the gland

Hyperplasia of the gland

Parathyroid carcinoma

Necrosis of the parathyroid gland

This is likely to be a case of tertiary hyperparathyroidism (high Calcium, high PTH,


low phosphate). Therefore the glands will be hyperplastic. Hypertrophy is not
correct as this implies an increase in size without an increase in cellularity. This
mistake has cost many candidates marks in the MRCS exams over the years!

Please rate this question:

 Discuss and give feedback

Next question 
Parathyroid glands and disorders of calcium metabolism

Hyperparathyroidism

Clinical
Disease type Hormone profile features Cause

Primary PTH May be Most cases


hyperparathyroidism (Elevated) asymptomatic if due to
Ca2+ mild solitary
(Elevated) Recurrent adenoma
Phosphate abdominal pain (80%),
(Low) (pancreatitis, multifocal
gathered by dr. elbarky.

Urine renal colic) disease


calcium : Changes to occurs in
creatinine emotional or 10-15% and
clearance cognitive state parathyroid
ratio > 0.01 carcinoma in
1% or less

Secondary PTH May have few Parathyroid


hyperparathyroidism (Elevated) symptoms gland
Ca2+ (Low or Eventually may hyperplasia
normal) develop bone occurs as a
Phosphate disease, osteitis result of low
(Elevated) fibrosa cystica calcium,
Vitamin D and soft tissue almost
levels (Low) calcifications always in a
setting of
chronic renal
failure

Tertiary Ca2+ (Normal Metastatic Occurs as a


hyperparathyroidism or high) calcification result of
PTH Bone pain and / ongoing
(Elevated) or fracture hyperplasia of
Phosphate Nephrolithiasis the
levels Pancreatitis parathyroid
(Decreased glands after
or Normal) correction of
Vitamin D underlying
(Normal or renal disorder,
decreased) hyperplasia of
Alkaline all 4 glands is
phosphatase usually the
(Elevated) cause
Differential diagnoses
It is important to consider the rare but relatively benign condition of benign familial
hypocalciuric hypercalcaemia, caused by an autosomal dominant genetic disorder.
Diagnosis is usually made by genetic testing and concordant biochemistry (urine
calcium : creatinine clearance ratio <0.01-distinguished from primary
hyperparathyroidism).

Treatment

Primary hyperparathyroidism
Indications for surgery
Elevated serum Calcium > 1mg/dL above normal
gathered by dr. elbarky.

Hypercalciuria > 400mg/day


Creatinine clearance < 30% compared with normal
Episode of life threatening hypercalcaemia
Nephrolithiasis
Age < 50 years
Neuromuscular symptoms
Reduction in bone mineral density of the femoral neck, lumbar spine, or
distal radius of more than 2.5 standard deviations below peak bone mass (T
score lower than -2.5)

Secondary hyperparathyroidism
Usually managed with medical therapy.

Indications for surgery in secondary (renal) hyperparathyroidism:


Bone pain
Persistent pruritus
Soft tissue calcifications

Tertiary hyperparathyroidism
Allow 12 months to elapse following transplant as many cases will resolve
The presence of an autonomously functioning parathyroid gland may require
surgery. If the culprit gland can be identified then it should be excised. Otherwise
total parathyroidectomy and re-implantation of part of the gland may be required.

References
1. Pitt S et al. Secondary and Tertiary Hyperparathyroidism, State of the Art
Surgical Management. Surg Clin North Am 2009 Oct;89(5):1227-39.

2. MacKenzie-Feder J et al. Primary Hyperparathyroidism: An Overview. Int J


Endocrinol 2011; 2011: 251410.

Next question 
Display my notes on this topic

          

Save my notes

Question stats
gathered by dr. elbarky.

A 7.1%
B 18.4%
C 59.2%
D 9.7%
E 5.5%

59.2% of users answered this question correctly

Search eMRCS

Search term Go

 Open MRCS Part A textbook (../review/textbook.php)

External links

+ Suggest a link

Dashboard

5

Question 115 of 364

 

Which of the following is not a typical feature of neuropraxia?

Transient delay in neuronal transmission

Axonal degeneration distal to the site of injury


gathered by dr. elbarky.

Absence of neuroma formation

Preservation of autonomic function

Absence of axonal degeneration proximal to the site of injury

Full recovery may occur 6-8 weeks after nerve injury in neuropraxia.
Wallerian degeneration does not usually occur in simple neuropraxia.
Autonomic function is usually preserved.

Please rate this question:

 Discuss and give feedback

Next question 

Nerve injury

There are 3 types of nerve injury:

Neuropraxia Nerve intact but electrical conduction is affected


Full recovery
Autonomic function preserved
Wallerian degeneration does not occur

Axonotmesis Axon is damaged and the myelin sheath is preserved.


The connective tissue framework is not affected.
Wallerian degeneration occurs.
Neurotmesis Disruption of the axon, myelin sheath and surrounding
connective tissue.
Wallerian degeneration occurs.

Wallerian Degeneration
Axonal degeneration distal to the site of injury.
Typically begins 24-36 hours following injury.
Axons are excitable prior to degeneration occurring.
Myelin sheath degenerates and is phagocytosed by tissue macrophages.
gathered by dr. elbarky.

Nerve repair
Neuronal repair may only occur physiologically where nerves are in direct
contact. Where a large defect is present, the process of nerve regeneration
is hampered. It may not occur at all or result in the formation of a neuroma.
Where nerve regrowth occurs it is typically at a rate of 1mm per day.

Next question 

Display my notes on this topic

          

Save my notes

Question stats

A 7.7%
B 50.5%
C 12.9%
D 13.1%
E 15.8%

50.5% of users answered this question correctly

Search eMRCS

Question 116 of 364

 

A 64-year-old woman is reviewed due to multiple non-healing leg ulcers. She


reports feeling generally unwell for many months. Examination findings include a
blood pressure of 138/72 mmHg, pulse 90 bpm, pale conjunctivae and poor
dentition associated with bleeding gums. What is the most likely underlying
diagnosis?
gathered by dr. elbarky.

Thyrotoxicosis

Vitamin B12 deficiency

Vitamin C deficiency

Diabetes mellitus

Sarcoidosis

Bleeding gums and poor healing are suggestive of vitamin C deficiency.

Please rate this question:

 Discuss and give feedback

Next question 

Vitamin C deficiency

Vitamin C deficiency (scurvy) leads to defective synthesis of collagen resulting in


capillary fragility (bleeding tendency) and poor wound healing

Features
gingivitis, loose teeth
poor wound healing
bleeding from gums, haematuria, epistaxis
general malaise
Next question 

Display my notes on this topic

          

Save my notes
gathered by dr. elbarky.

Question stats

A 5.9%
B 14.6%
C 66.8%
D 6.7%
E 5.9%

66.8% of users answered this question correctly

Search eMRCS

Search term Go

 Open MRCS Part A textbook (../review/textbook.php)

External links

+ Suggest a link

Dashboard

3

Question 117 of 364

 

A 52 year old male attends for a preoperative assessment for an inguinal hernia
repair. You notice that the chest x-ray shows a loculated left pleural effusion. On
further questioning the patient reports that he worked as a builder 30 years ago.
What is the most likely cause for the effusion?
gathered by dr. elbarky.

Asbestosis

Pneumonia

Mesothelioma

Silicosis

Left ventricular failure

This patient has a risk of asbestos exposure through his occupation as a builder.
As there a is latent period of 30 years and a complicated effusion, the most likely
cause is mesothelioma.

Please rate this question:

 Discuss and give feedback

Next question 

Mesothelioma

Features
Dyspnoea, weight loss, chest wall pain
Clubbing
30% present as painless pleural effusion
Only 20% have pre-existing asbestosis
History of asbestos exposure in 85-90%, latent period of 30-40 years

Basics
Malignancy of mesothelial cells of pleura
Metastases to contralateral lung and peritoneum
Right lung affected more often than left

Management
Investigation: pleural biopsy, CT Scanning, (PET Scanning if surgery
considered)
Symptomatic
Industrial compensation
Chemotherapy, Surgery if operable
Prognosis poor, median survival 12 months
gathered by dr. elbarky.

Next question 

Display my notes on this topic

          

Save my notes

Question stats

A 27.4%
B 5.7%
C 49.2%
D 12.4%
E 5.3%

49.2% of users answered this question correctly

Search eMRCS

Search term Go

 Open MRCS Part A textbook (../review/textbook.php)


Question 118 of 364

 

Which of the tumour markers listed below is most likely to be elevated in a patient
with pancreatic cancer?

CEA
gathered by dr. elbarky.

CA19-9

AFP

PSA

CA15-3

Please rate this question:

 Discuss and give feedback

Next question 

Tumour markers

Tumour markers may be divided into:


monoclonal antibodies against carbohydrate or glycoprotein tumour
antigens
tumour antigens
enzymes (alkaline phosphatase, neurone specific enolase)
hormones (e.g. calcitonin, ADH)

It should be noted that tumour markers usually have a low specificity

Monoclonal antibodies

Tumour marker Association

CA 125 Ovarian cancer


Tumour marker Association

CA 19-9 Pancreatic cancer

CA 15-3 Breast cancer

NB: The breast cancer tumour marker is not specific or sensitive enough to be
used routinely.

Tumour antigens

Tumour marker Association


gathered by dr. elbarky.

Prostate specific antigen (PSA) Prostatic carcinoma

Alpha-feto protein (AFP) Hepatocellular carcinoma, teratoma

Carcinoembryonic antigen (CEA) Colorectal cancer

Next question 

Display my notes on this topic

          

Save my notes

Question stats

A 6.4%
B 77.5%
C 5.1%
D 5%
E 5.9%

77.5% of users answered this question correctly

Search eMRCS

Question 119 of 364

 

A 3 month old boy is suspected of having hypospadias. At which of the following


locations is the urethral opening most frequently located in boys suffering from the
condition?

On the distal ventral surface of the penis


gathered by dr. elbarky.

On the proximal ventral surface of the penis

On the distal dorsal surface of the penis

On the proximal dorsal surface of the penis

At the base of the scrotum

The defect is located ventrally and most often distally. Proximally located urethral
openings are well recognised. Circumcision may compromise reconstruction.

Please rate this question:

 Discuss and give feedback

Next question 

Hypospadias
The urethral meatus opens on the ventral surface of the penis. There is also a
ventral deficiency of the foreskin. The urethral meatus may open more proximally
in the more severe variants. However, 75% of the openings are distally located. The
incidence is 1 in 300 male births.

Features include:

Absent frenular artery


Ventrally opened glans
Skin tethering to hypoplastic urethra
Splayed columns of spongiosum tissue distal to the meatus
Deficiency of the foreskin ventrally
gathered by dr. elbarky.

Management:

No routine cultural circumcisions


Urethroplasty
Penile reconstruction

The foreskin is often utilised in the reconstructive process. In boys with very distal
disease no treatment may be needed.

Next question 

Display my notes on this topic

          

Save my notes

Question stats

A 53.2%
B 14.7%
C 18.1%
D 8.8%
E 5.2%

53.2% of users answered this question correctly


Question 120 of 364

 

What is the main risk factor for the development of anal cancer?

Smoking

Ano-receptive intercourse
gathered by dr. elbarky.

Immunosuppression

Infection with Epstein Barr virus

Infection with Human Papilloma virus

Anal cancer is strongly associated with HPV infection

Please rate this question:

 Discuss and give feedback

Next question 

Anal cancer

Cancers arising from the squamous epithelium of the anal canal


Arise inferior to the dentate line
Strongly linked to HPV type 16 infection
Other risk factors include ano-receptive intercourse, smoking and
immunosuppression
Presenting symptoms include anal discomfort, discharge or pruritus
Lymphatic spread typically occurs to the inguinal nodes
Diagnosis is made by EUA and biopsies
Staging is with CT scanning of the chest, abdomen and pelvis
First line treatment is typically with chemoradiotherapy
Second line treatment for non metastatic disease is with salvage radical
abdominoperineal excision of the anus and rectum
Next question 

Display my notes on this topic

          

Save my notes
gathered by dr. elbarky.

Question stats

A 8.6%
B 9.8%
C 6.8%
D 6.8%
E 68%

68% of users answered this question correctly

Search eMRCS

Search term Go

 Open MRCS Part A textbook (../review/textbook.php)

External links

+ Suggest a link

Dashboard

3

Question 121 of 364

 

An 18 year old female presents with 3 nodules in the right lobe of the thyroid.
Clinically she is euthyroid and there is associated cervical lymphadenopathy. She
has no family history of thyroid disease. What is the most likely cause?

Follicular thyroid cancer


gathered by dr. elbarky.

Follicular thyroid adenoma

Papillary thyroid cancer

Hashimotos thyroiditis

Toxic nodular goitre

Papillary thyroid cancers are the most common type of thyroid cancer and are the
more common in females (M:F=1:3). Papillary tumours are more likely to develop
lymphatic spread than follicular tumours.

Please rate this question:

 Discuss and give feedback

Next question 

Thyroid disease
Patients may present with a number of different manifestations of thyroid disease.
They can be broadly sub classified according to whether they are euthyroid or have
clinical signs of thyroid dysfunction. In addition it needs to be established whether
they have a mass or not.

Assessment
History
Examination including USS
If a nodule is identified then it should be sampled ideally via an image
guided fine needle aspiration
Radionucleotide scanning is of limited use
gathered by dr. elbarky.

Thyroid Tumours
Papillary carcinoma
Follicular carcinoma
Anaplastic carcinoma
Medullary carcinoma
Lymphoma's

Multinodular goitre
One of the most common reasons for presentation
Provided the patient is euthyroid and asymptomatic and no discrete nodules
are seen, they can be reassured.
In those with compressive symptoms surgery is required and the best
operation is a total thyroidectomy.
Sub total resections were practised in the past and simply result in recurrent
disease that requires a difficult revisional resection.

Endocrine dysfunction
In general these patients are managed by physicians initially.
Surgery may be offered alongside radio iodine for patients with Graves
disease that fails with medical management or in patients who would prefer
not to be irradiated (e.g. pregnant women).
Patients with hypothyroidism do not generally get offered a thyroidectomy.
Sometimes people inadvertently get offered resections during the early
phase of Hashimotos thyroiditis, however, with time the toxic phase passes
and patients can simply be managed with thyroxine.

Complications following surgery


Anatomical such as recurrent laryngeal nerve damage.
Bleeding. Owing to the confined space haematoma's may rapidly lead to
respiratory compromise owing to laryngeal oedema.
Damage to the parathyroid glands resulting in hypocalcaemia.

Further sources of information


1. http://www.acb.org.uk/docs/TFTguidelinefinal.pdf- Association of Clinical
Biochemistry guidelines for thyroid function tests.

2. British association of endocrine surgeons website- http://www.baets.org.uk

Next question 

Display my notes on this topic


gathered by dr. elbarky.

          

Save my notes

Question stats

A 11.5%
B 13%
C 61.4%
D 7.7%
E 6.5%

61.4% of users answered this question correctly

Search eMRCS

Search term Go

 Open MRCS Part A textbook (../review/textbook.php)

External links

+ Suggest a link

Question 122 of 364

 

A 73 year old man presents with pain in the right leg. It is most uncomfortable on
walking. On examination, he has a deformity of his right femur, which on x-ray is
thickened and sclerotic. His serum alkaline phosphatase is elevated, but calcium is
within normal limits. What is the most probable underlying diagnosis?
gathered by dr. elbarky.

Rickets

Pagets disease

Metastatic lung cancer

Osteoclastoma

Chondrosarcoma

Pagets is typically associated with thickened, sclerotic, long bones. Lung


carcinoma typically causes lytic bony lesions and hypercalcaemia.

Please rate this question:

 Discuss and give feedback

Next question 

Pagets disease
Paget's disease is a disease of increased but uncontrolled bone turnover and is
characterised by architecturally abnormal bones. It is thought to be primarily a
disorder of osteoclasts, with excessive osteoclastic resorption followed by
increased osteoblastic activity causing areas of sclerosis and deformity. Paget's
disease is common (UK prevalence 5%) but symptomatic in only 1 in 20 patients

Predisposing factors
increasing age
male sex
northern latitude
family history
gathered by dr. elbarky.

Clinical features
bone pain (e.g. pelvis, lumbar spine, femur)
classical, untreated features: bowing of tibia, bossing of skull
raised alkaline phosphatase (ALP) - calcium* and phosphate are typically
normal
skull x-ray: thickened vault, osteoporosis circumscripta

Indications for treatment include bone pain, skull or long bone deformity, fracture,
periarticular Paget's
bisphosphonate (either oral risedronate or IV zoledronate)
calcitonin is less commonly used now

Complications
deafness (cranial nerve entrapment)
bone sarcoma (1% if affected for > 10 years)
fractures
skull thickening
high-output cardiac failure

*usually normal in this condition but hypercalcaemia may occur with prolonged
immobilisation

Next question 

Display my notes on this topic

          
Save my notes

Question stats

A 6.1%
B 62.6%
C 14.4%
D 9.3%
E 7.6%
gathered by dr. elbarky.

62.6% of users answered this question correctly

Search eMRCS

Search term Go

 Open MRCS Part A textbook (../review/textbook.php)

External links

+ Suggest a link

Dashboard

9

Question 122 of 364

 

A 73 year old man presents with pain in the right leg. It is most uncomfortable on
walking. On examination, he has a deformity of his right femur, which on x-ray is
thickened and sclerotic. His serum alkaline phosphatase is elevated, but calcium is
within normal limits. What is the most probable underlying diagnosis?
gathered by dr. elbarky.

Rickets

Pagets disease

Metastatic lung cancer

Osteoclastoma

Chondrosarcoma

Pagets is typically associated with thickened, sclerotic, long bones. Lung


carcinoma typically causes lytic bony lesions and hypercalcaemia.

Please rate this question:

 Discuss and give feedback

Next question 

Pagets disease
Paget's disease is a disease of increased but uncontrolled bone turnover and is
characterised by architecturally abnormal bones. It is thought to be primarily a
disorder of osteoclasts, with excessive osteoclastic resorption followed by
increased osteoblastic activity causing areas of sclerosis and deformity. Paget's
disease is common (UK prevalence 5%) but symptomatic in only 1 in 20 patients

Predisposing factors
increasing age
male sex
northern latitude
family history
gathered by dr. elbarky.

Clinical features
bone pain (e.g. pelvis, lumbar spine, femur)
classical, untreated features: bowing of tibia, bossing of skull
raised alkaline phosphatase (ALP) - calcium* and phosphate are typically
normal
skull x-ray: thickened vault, osteoporosis circumscripta

Indications for treatment include bone pain, skull or long bone deformity, fracture,
periarticular Paget's
bisphosphonate (either oral risedronate or IV zoledronate)
calcitonin is less commonly used now

Complications
deafness (cranial nerve entrapment)
bone sarcoma (1% if affected for > 10 years)
fractures
skull thickening
high-output cardiac failure

*usually normal in this condition but hypercalcaemia may occur with prolonged
immobilisation

Next question 

Display my notes on this topic

          
Save my notes

Question stats

A 6.1%
B 62.6%
C 14.4%
D 9.3%
E 7.6%
gathered by dr. elbarky.

62.6% of users answered this question correctly

Search eMRCS

Search term Go

 Open MRCS Part A textbook (../review/textbook.php)

External links

+ Suggest a link

Dashboard

9

Question 123 of 364

 

Which of the conditions listed below is an example of heterotopia?

Barretts oesophagus

Squamous cell carcinoma of the lung


gathered by dr. elbarky.

Meckels diverticulum containing gastric mucosa

Colonic diverticulum

Ectopic testis

In heterotopia, the tissue type that is found in the abnormal location is present
there from birth and does not migrate to that site subsequently or arise as a result
of metaplasia. The tissue that lines a Meckels diverticulum is determined early in
development.

Please rate this question:

 Discuss and give feedback

Next question 

Heterotopia

Heterotopia is the presence of a specified normal tissue type at a non-


physiological site, but usually co-existing with original tissue in its correct
anatomical location. It implies ectopic tissue, in addition to retention of the original
tissue type. In neuropathology, for example, gray matter heterotopia is the
presence of gray matter within the cerebral white matter or ventricles. Heterotopia
within the brain is often divided into three groups: subependymal heterotopia, focal
cortical heterotopia and band heterotopia. Another example is a Meckel's
diverticulum, which may contain heterotopic gastric or pancreatic tissue.
It is the fact that the tissue is itself that distinguishes heterotopia from a malignant
process.
Next question 

Display my notes on this topic

          

Save my notes
gathered by dr. elbarky.

Question stats

A 8.9%
B 5.9%
C 60.6%
D 6.9%
E 17.7%

60.6% of users answered this question correctly

Search eMRCS

Search term Go

 Open MRCS Part A textbook (../review/textbook.php)

External links

+ Suggest a link

Dashboard

3

Question 124 of 364

 

A 78 year old lady presents with a tender swelling in her right groin. On
examination there is a tender swelling that lies below and lateral to the pubic
tubercle. It has a cough impulse. What is the most likely underlying diagnosis?

Thrombophlebitis of the great saphenous vein


gathered by dr. elbarky.

Femoral hernia

Thrombophlebitis of saphena varix

Inguinal hernia

Obturator hernia

Whilst a thrombophlebitis of a saphena varix may cause a tender swelling at this


site, it would not usually be associated with a cough impulse.

Please rate this question:

 Discuss and give feedback

Next question 

Femoral canal

The femoral canal lies at the medial aspect of the femoral sheath. The femoral
sheath is a fascial tunnel containing both the femoral artery laterally and femoral
vein medially. The canal lies medial to the vein.

Borders of the femoral canal

Laterally Femoral vein

Medially Lacunar ligament

Anteriorly Inguinal ligament


Posteriorly Pectineal ligament

Image showing dissection of femoral canal


gathered by dr. elbarky.

(https://d2zgo9qer4wjf4.cloudfront.net/images_eMRCS/swb036b.jpg)
Image sourced from Wikipedia
(https://d2zgo9qer4wjf4.cloudfront.net
(http://en.wikipedia.org/wiki/Femoral
/images_eMRCS/swb036b.jpg)
canal)

Contents
Lymphatic vessels
Cloquet's lymph node

Physiological significance
Allows the femoral vein to expand to allow for increased venous return from the
lower limbs.

Pathological significance
As a potential space, it is the site of femoral hernias. The relatively tight neck
places these at high risk of strangulation.
Next question 

Display my notes on this topic

          

Save my notes
gathered by dr. elbarky.

Question stats

A 5.2%
B 71.3%
C 8.2%
D 9%
E 6.4%

71.3% of users answered this question correctly

Search eMRCS

Search term Go

 Open MRCS Part A textbook (../review/textbook.php)

External links

+ Suggest a link

Dashboard

3

Question 125 of 364

 

Which one of the following is least associated with the development of colorectal
cancer in patients with ulcerative colitis?

Unremitting disease
gathered by dr. elbarky.

Disease duration > 10 years

Onset before 15 years old

Poor compliance to treatment

Disease confined to the rectum

Please rate this question:

 Discuss and give feedback

Next question 

Ulcerative colitis and colorectal cancer

Overview
risk of colorectal cancer is 10-20 times that of general population
the increased risk is mainly related to chronic inflammation
worse prognosis than patients without ulcerative colitis (partly due to
delayed diagnosis)
lesions may be multifocal

Factors increasing risk of cancer


disease duration > 10 years
patients with pancolitis
onset before 15 years old
unremitting disease
poor compliance to treatment
Next question 

Display my notes on this topic

          

Save my notes
gathered by dr. elbarky.

Question stats

A 12.5%
B 7.7%
C 12.1%
D 9.2%
E 58.6%

58.6% of users answered this question correctly

Search eMRCS

Search term Go

 Open MRCS Part A textbook (../review/textbook.php)

External links

+ Suggest a link

Dashboard

3

Question 126 of 364

 

What is the urinary diagnostic marker for carcinoid syndrome?

B-HCG

Histamine
gathered by dr. elbarky.

Chromogranin A

5-Hydroxyindoleacetic acid

5-Hydroxytryptamine

Urinary measurement of 5- HIAA is an important part of clinical follow up.

Please rate this question:

 Discuss and give feedback

Next question 

Carcinoid syndrome

Carcinoid tumours secrete serotonin


Originate in neuroendocrine cells mainly in the intestine (midgut-distal
ileum/appendix)
Can occur in the rectum, bronchi
Hormonal symptoms mainly occur when disease spreads outside the bowel

Clinical features
Onset: insidious over many years
Flushing face
Palpitations
Pulmonary valve stenosis and tricuspid regurgitation causing dyspnoea
Asthma
Severe diarrhoea (secretory, persists despite fasting)
Investigation
5-HIAA in a 24-hour urine collection
Somatostatin receptor scintigraphy
CT scan
Blood testing for chromogranin A

Treatment
Octreotide
Surgical removal
gathered by dr. elbarky.

Next question 

Display my notes on this topic

          

Save my notes

Question stats

A 6.5%
B 6.1%
C 17.2%
D 59.7%
E 10.5%

59.7% of users answered this question correctly

Search eMRCS

Search term Go

 Open MRCS Part A textbook (../review/textbook.php)

External links

Question 127 of 364

 

A 72 year old lady falls and lands on her left hip. She attends the emergency
department and is given some paracetamol by the junior doctor and discharged.
Several months later she presents with ongoing pain and discomfort of the hip.
Avascular necrosis of the femoral head is suspected. Which of the following
features is least likely to be present?
gathered by dr. elbarky.

Union of the fracture

Angiogenesis at the fracture site

Increased numbers of fibroblasts at the fracture site

Osteochondritis dissecans

Apoptosis of osteoblasts

Apoptosis is not a feature of necrotic cell death. By this stage there would usually
be attempted repair so angiogenesis and proliferation of fibroblasts would be
expected. These cells may differentiate further to become osteoblasts which in
turn will lay down new matrix.

Please rate this question:

 Discuss and give feedback

Next question 

Avascular necrosis

Cellular death of bone components due to interruption of the blood supply,


causing bone destruction
Main joints affected are hip, scaphoid, lunate and the talus.
It is not the same as non union. The fracture has usually united.
Radiological evidence is slow to appear.
Vascular ingrowth into the affected bone may occur. However, many joints
will develop secondary osteoarthritis.
Causes
P ancreatitis
L upus
A lcohol
S teroids
T rauma
I diopathic, infection
C aisson disease, collagen vascular disease
R adiation, rheumatoid arthritis
A myloid
G aucher disease
gathered by dr. elbarky.

S ickle cell disease

Presentation
Usually pain. Often despite apparent fracture union.

Investigation
MRI scanning will show changes earlier than plain films.

Treatment
In fractures at high risk sites anticipation is key. Early prompt and accurate
reduction is essential.

Non weight bearing may help to facilitate vascular regeneration.

Joint replacement may be necessary, or even the preferred option (e.g. Hip in the
elderly).

Next question 

Display my notes on this topic

          

Save my notes

Question stats

A 28.5%
B 24%
C 9.1%
D 11%
E 27.4%

27.4% of users answered this question correctly

Search eMRCS

Search term Go
gathered by dr. elbarky.

 Open MRCS Part A textbook (../review/textbook.php)

External links

+ Suggest a link

Dashboard

10

11

12

13

14

Question 128 of 364

 

A male infant is born by emergency cesarean section at 39 weeks gestation for


foetal distress. Soon after the birth the baby becomes progressively hypoxic and
on examination is found to have a scaphoid abdomen. What is the most likely
underlying diagnosis?
gathered by dr. elbarky.

Intestinal malrotation

Hiatus hernia

Foramen of Bochdalek hernia

Foramen of Morgagni hernia

Tracheo-oesphageal fistula

The finding of a scaphoid abdomen and respiratory distress suggests extensive


intra thoracic herniation of the abdominal contents. This is seen most frequently
with Bochdalek hernias. Morgagni hernias seldom present in such a dramatic
fashion. The other options do not typically present with the symptoms and signs
described.

Please rate this question:

 Discuss and give feedback

Next question 

Embryology of the diaphragm and diaphragmatic hernia

Embryology
The diaphragm is formed between the 5th and 7th weeks of gestation through the
progressive fusion of the septum transversum, pleuroperitoneal folds and via
lateral muscular ingrowth. The muscular origins of the diaphragm are somites
located in cervical segments 3 to 5, which accounts for the long path taken by the
phrenic nerve. The components contribute to the following diaphragmatic
segments:
Septum transversum - Central tendon
Pleuroperitoneal membranes - Parietal membranes surrounding viscera
Cervical somites C3 to C5 - Muscular component of the diaphragm

Diaphragmatic hernia

Type of hernia Features

Morgagni Anteriorly located


Minimal compromise on lung development
Minimal signs on antenatal ultrasound
Usually present later
Usually good prognosis
gathered by dr. elbarky.

Bochdalek hernia Posteriorly located


Larger defect
Often diagnosed antenatally
Associated with pulmonary hypoplasia
Poor prognosis

The posterior hernias of Bochdalek are the most common type and if not
diagnosed antenatally will typically present soon after birth with respiratory
distress. The classical finding is that of a scaphoid abdomen on clinical
examination because of herniation of the abdominal contents into the chest.
Bochdalek hernias are associated with a number of chromosomal abnormalities
such as Trisomy 21 and 18. Infants have considerable respiratory distress due to
hypoplasia of the developing lung. Historically this was considered to be due to
direct compression of the lung by herniated viscera. This view over simplifies the
situation and the pulmonary hypoplasia occurs concomitantly with the hernial
development, rather than as a direct result of it. The pulmonary hypoplasia is
associated with pulmonary hypertension and abnormalities of pulmonary
vasculature. The pulmonary hypertension renders infants at risk of right to left
shunting (resulting in progressive and worsening hypoxia).
Diagnostic work up of these infants includes chest x-rays/ abdominal ultrasound
scans and cardiac echo.
Surgery forms the mainstay of treatment and both thoracic and abdominal
approaches may be utilised. Following reduction of the hernial contents a careful
search needs to be made for a hernial sac as failure to recognise and correct this
will result in a high recurrence rate. Smaller defects may be primarily closed, larger
defects may require a patch to close the defect. Malrotation of the viscera is a
recognised association and may require surgical correct at the same procedure
(favoring an abdominal approach).
The mortality rate is 50-75% and is related to the degree of lung compromise and
age at presentation (considerably better in infants >24 hours old).

Next question 
Display my notes on this topic

          

Save my notes

Question stats
gathered by dr. elbarky.

A 10.8%
B 10.1%
C 55%
D 13.3%
E 10.8%

55% of users answered this question correctly

Search eMRCS

Search term Go

 Open MRCS Part A textbook (../review/textbook.php)

External links

+ Suggest a link

Dashboard

5

Question 129 of 364

 

A 58 year old man undergoes an upper GI endoscopy for the investigation of


odynophagia. At endoscopy a reddish area is seen to extend into the oesophagus
from the gastro-oesophageal junction. Which of the following pathological events
is most likely to explain this process?
gathered by dr. elbarky.

Metaplasia

Anaplasia

Dysplasia

Hypoplasia

Hyperplasia

This is most likely to represent Barretts oesphagus and is thus metaplasia.


Dysplasia is less likely in this setting although biopsies are mandatory.

Please rate this question:

 Discuss and give feedback

Next question 

Barrett's oesophagus

Barretts oesophagus is a condition characterised by the metaplastic


transformation of squamous oesophageal epithelium to columnar gastric type
epithelium. Three types of this metaplastic process are recognised; intestinal (high
risk), cardiac and fundic. The latter two categories may cause difficulties in
diagnosis. The most concrete diagnosis can be made when endoscopic features
of Barretts oesophagus are present together with a deep biopsy that demonstrates
not just goblet cell metaplasia but also oesophageal glands.

Barrett's can be sub divided into short (<3cm) and long (>3cm). The length of the
affected segment correlates strongly with the chances of identifying metaplasia.
The overall prevalence of Barrett's oesophagus is difficult to determine but may be
in the region of 1 in 20 and is identified in up to 12% of those undergoing
endoscopy for reflux.

A proportion of patients with metaplasia will progress to dysplasia and for this
reason individuals identified as having Barrett's should undergo endoscopic
surveillance (every 2-5 years). Biopsies should be quadrantic and taken at 2-3cm
intervals. Biopsies need to be adequate. Where mass lesions are present
consideration should be given to endoscopic sub mucosal resection. Up to 40% of
patients will be upstaged from high grade dysplasia to invasive malignancy with
such techniques.

Treatment
gathered by dr. elbarky.

Long term proton pump inhibitor


Consider pH and manometry studies in younger patients who may prefer to
consider an anti reflux procedure
Regular endoscopic monitoring (more frequently if moderate dysplasia).
With quadrantic biopsies every 2-3 cm
If severe dysplasia be very wary of small foci of cancer

References
A consensus statement of the British approach is provided by:
Bennett C et al Consensus Statements for Management of Barrett's Dysplasia and
Early-Stage Esophageal Adenocarcinoma, Based on a Delphi Process.
Gastroenterology Volume 143, Issue 2 , Pages 336-346, August 2012.

Next question 

Display my notes on this topic

          

Save my notes

Question stats

A 52.2%
B 6.3%
C 25.2%
D 5.3%
E 11%

Question 130 of 364

 

Which of the following changes are most likely to be identified in the aortic wall of
a 38 year old lady with a Marfans syndrome and a dissecting aortic aneurysm?

Transmural aortitis
gathered by dr. elbarky.

Cystic medial necrosis

Foamy macrophages

Dense dystrophic calcification

None of the above

Cystic medial necrosis ( or cystic medial degeneration) occurs when basophils and
mucoid material lie in between the intimal elastic fibres of the aorta. It is typically
found in the aortic degeneration of Marfans syndrome, but may also be seen in
aortic degeneration in older adults.

Please rate this question:

 Discuss and give feedback

Next question 

Aortic dissection

More common than rupture of the abdominal aorta


33% of patients die within the first 24 hours, and 50% die within 48 hours if
no treatment received
Associated with hypertension
Features of aortic dissection: tear in the intimal layer, followed by formation
and propagation of a subintimal hematoma. Cystic medial necrosis
(Marfan's)
Most common site of dissection: 90% occurring within 10 centimetres of the
aortic valve
Stanford Classification

Type Location Treatment

A Ascending aorta/ aortic root Surgery- aortic root replacement

B Descending aorta Medical therapy with antihypertensives

DeBakey classification

Type Site affected


gathered by dr. elbarky.

I Ascending aorta, aortic arch, descending aorta

II Ascending aorta only

III Descending aorta distal to left subclavian artery

Clinical features
Tearing, sudden onset chest pain (painless 10%)
Hypertension or Hypotension
A blood pressure difference (in each arm) greater than 20 mm Hg
Neurologic deficits (20%)

Investigations
CXR: widened mediastinum, abnormal aortic knob, ring sign, deviation of the
trachea/oesophagus
CT angiography of the thoracic aorta
MRI angiography
Conventional angiography (now rarely used diagnostically)

Management
Beta-blockers: aim HR 60-80 bpm and systolic BP 100-120 mm Hg
For type A dissections the standard of care is aortic root replacement

Next question 

Display my notes on this topic

          
Save my notes

Question stats

A 19.4%
B 40.2%
C 14.2%
D 15.8%
E 10.4%
gathered by dr. elbarky.

40.2% of users answered this question correctly

Search eMRCS

Search term Go

 Open MRCS Part A textbook (../review/textbook.php)

External links

+ Suggest a link

Dashboard

9

Question 131 of 364

 

What is the process that is most likely to account for a 73 year old lady presenting
with a cold pulseless hand 3 days following a myocardial infarct?

Foreign body embolus


gathered by dr. elbarky.

Rupture of existing atheromatous plaque

Steal syndrome

Vasospasm

Clot embolus

The development of mural or atrial appendage thrombi may occur following a


myocardial infarct and co-existing atrial fibrillation may contribute to the
formation. They tend to present with classical features of an embolic event.

Please rate this question:

 Discuss and give feedback

Next question 

Arterial occlusions/ insufficiency

Arterial occlusions may occur as a result of a number of processes. The typical


clinical scenarios are outlined below.

Cause of
occlusion Typical picture

Embolus Sudden onset


Depending upon level of occlusion; limb may show typical
features of pain, loss of pulses and pallor. Sensory perceptive
changes may also be present
Cause of
occlusion Typical picture

Thrombosis Usually known disease and prodromal symptoms e.g.


claudication
Disruption to flow may be incomplete
If background disease process present then collaterals may
be present and picture less dramatic

Vasospasm May be due to Raynauds and affect extremities


Symptoms are often temperature related
Discolouration of the hands may occur (pale, dark, red)
gathered by dr. elbarky.

Symptoms improve during pregnancy (hyperdynamic


circulation)

Steal Occur secondary to arteriovenous fistula, or partial arterial


syndromes occlusions (e.g. cervical rib)
Pain and diminished pulses distal to fistula are seen

Vasculitis
Vessel diameter and vasculitis classification

Aorta and branches Takayasu's arteritis


Buergers disease
Giant cell arteritis

Large and medium sized arteries Buergers disease


Giant cell arteritis
Polyarteritis nodosa

Medium sized muscular arteries Polyarteritis nodosa


Wegeners granulomatosis

Small muscular arteries Wegeners granulomatosis


Rheumatoid vasculitis

Specific conditions
Takyasu's arteritis Inflammatory, obliterative arteritis affecting aorta
and branches
Females> Males
Symptoms may include upper limb claudication
Clinical findings include diminished or absent
pulses

ESR often affected during the acute phase

Buergers disease Segmental thrombotic occlusions of the small


and medium sized lower limb vessels
gathered by dr. elbarky.

Commonest in young male smokers


Proximal pulses usually present, but pedal pulses
are lost
An acuter hypercellular occlusive thrombus is
often present
Tortuous corkscrew shaped collateral vessels
may be seen on angiography

Giant cell arteritis Systemic granulomatous arteritis that usually


affects large and medium sized vessels
Females > Males
Temporal arteritis is commonest type
Granulomatous lesions may be seen on biopsy
(although up to 50% are normal)

Polyarteritis nodosa Systemic necrotising vasculitis affecting small


and medium sized muscular arteries
Most common in populations with high
prevalence of hepatitis B
Renal disease is seen in 70% cases
Angiography may show saccular or fusiform
aneurysms and arterial stenoses

Wegeners Predominantly affects small and medium sized


granulomatosis arteries
Systemic necrotising granulomatous vasculitis
Cutaneous vascular lesions may be seen
(ulceration, nodules and purpura)
Sinus imaging may show mucosal thickening and
air fluid levels
Next question 

Display my notes on this topic

          

Save my notes
gathered by dr. elbarky.

Question stats

A 6.5%
B 13.5%
C 11%
D 6.7%
E 62.4%

62.4% of users answered this question correctly

Search eMRCS

Search term Go

 Open MRCS Part A textbook (../review/textbook.php)

External links

+ Suggest a link

Dashboard

3

Question 132 of 364

 

What is the commonest tumour type encountered in the colon?

Squamous cell carcinoma

Adenocarcinoma
gathered by dr. elbarky.

Lymphoma

Anaplastic carcinoma

Sarcoma

Adenocarcinoma are the most common and typically arise as a result of the
adenoma - carcinoma sequence.

Please rate this question:

 Discuss and give feedback

Next question 

Colorectal cancer

Annually, about 150,000 new cases are diagnosed and 50,000 deaths from
the disease
About 75% will have sporadic disease and 25% will have a family history
Colorectal tumours comprise a spectrum of disease ranging from
adenomas through to polyp cancers and frank malignancy.
Polyps may be categorised into: neoplastic polyps, adenomatous polyps and
non neoplastic polyps.
The majority of adenomas are polypoidal lesions, although flat lesions do
occur and may prove to be dysplastic.
Non-neoplastic polyps include hyperplastic, juvenile, hamartomatous,
inflammatory, and lymphoid polyps, which have not generally been thought
of as precursors of cancer.
Three characteristics of adenomas that correlate with malignant potential
have been characterised. These include increased size, villous architecture
and dysplasia. For this reason most polyps identified at colonoscopy should
be removed.
The transformation from polyp to cancer is described by the adenoma -
carcinoma sequence and its principles should be appreciated. Essentially
genetic changes accompany the transition from adenoma to carcinoma; key
changes include APC, c-myc, K RAS mutations and p53 deletions.

Next question 
gathered by dr. elbarky.

Display my notes on this topic

          

Save my notes

Question stats

A 7.8%
B 75.7%
C 6.1%
D 5.7%
E 4.7%

75.7% of users answered this question correctly

Search eMRCS

Search term Go

 Open MRCS Part A textbook (../review/textbook.php)

External links

+ Suggest a link

Question 133 of 364

 

At which of the following anatomical sites does dormant tuberculosis most


frequently reactivate?

Apex of the lung


gathered by dr. elbarky.

Base of the lung

Brain

Terminal ileum

Lumbar spine

TB reactivation most commonly occurs at the lung apex. This site is better
oxygenated than elsewhere allowing the mycobacteria to multiply more rapidly and
then spread both locally and distantly.

Please rate this question:

 Discuss and give feedback

Next question 

Tuberculosis pathology

Is a form of primary chronic inflammation, caused by the inability of


macrophages to kill the Mycobacterium tuberculosis.
The macrophages often migrate to regional lymph nodes, the lung lesion
plus affected lymph nodes is referred to as a Ghon complex.
This leads to the formation of a granuloma which is a collection of
epithelioid histiocytes.
There is the presence of caseous necrosis in the centre.
The inflammatory response is mediated by a type 4 hypersensitivity
reaction.
In healthy individuals the disease may be contained, in the
immunocompromised disseminated (miliary TB) may occur.
Diagnosis
Waxy membrane of mycobacteria prevents binding with normal stains. Ziehl
- Neelsen staining is typically used.
Culture based methods take far longer.

Image showing acid- alcohol fast mycobacteria stained using the Ziehl- Neelsen
method
gathered by dr. elbarky.

(https://d2zgo9qer4wjf4.cloudfront.net/images_eMRCS/swb107b.jpg)
Image sourced from Wikipedia
(https://d2zgo9qer4wjf4.cloudfront.net
(http://en.wikipedia.org/wiki/Ziehl%E2%80
/images_eMRCS/swb107b.jpg)
%93Neelsen stain)

Next question 

Display my notes on this topic

          

Save my notes

Question stats

A 69.8%
B 9.6%
C 5.9%
D 7%

Question 134 of 364

 

An 85 year old man presents with a cough and haemoptysis. He has a modest
smoking history of 15 pack years. He is found to have a tumour located in the right
main bronchus, with no evidence of metastatic disease. He decides not undergo
any treatment and he remains well for a further 12 months before developing
symptomatic metastasis. What is the most likely diagnosis?
gathered by dr. elbarky.

Adenocarcinoma

Small cell carcinoma

Squamous cell carcinoma

Metastatic renal cancer

Lymphoma

Squamous cell carcinomas are reported to be more slow growing and are typically
centrally located. Small cell carcinomas are usually centrally located. However,
small cell carcinomas would seldom be associated with a survival of a year
without treatment.

Please rate this question:

 Discuss and give feedback

Next question 

Lung cancer

Lung cancers may be classified according to histological subtypes. The main


distinction is between small cell and non small cell lung cancer. Non small cell
lung cancer is the most common variant and accounts for 80% of all lung cancers.

Non small cell lung cancer


These share common features of prognosis and management. They comprise the
following tumours:
Squamous cell carcinoma (25% cases)
Adenocarcinoma (40% cases)
Large cell carcinoma (10% cases)

Paraneoplastic features and early disease dissemination are less likely than with
small cell lung carcinoma. Adenocarcinoma is the most common lung cancer type
encountered in never smokers.

Small cell lung carcinoma


Small cell lung carcinomas are comprised of cells with a neuro endocrine
differentiation. The neuroendocrine hormones may be released from these cells
with a wide range of paraneoplastic associations. These tumours are strongly
associated with smoking and will typically arise in the larger airways. They
gathered by dr. elbarky.

disseminate early in the course of the disease and although they are usually
chemosensitive this seldom results in long lasting remissions.

Next question 

Display my notes on this topic

          

Save my notes

Question stats

A 20.4%
B 28.8%
C 42%
D 4.4%
E 4.4%

42% of users answered this question correctly

Search eMRCS

Search term Go

Question 135 of 364

 

An 18 month old boy presents with recurrent urinary tract infections. As part of the
diagnostic work-up he is noted to have abnormal renal function. An ultrasound
scan is performed and shows bilateral hydronephrosis and hydroureter. What is the
most likely underlying diagnosis?
gathered by dr. elbarky.

Posterior urethral valves

Meatal stenosis

Hydronephrosis

Pelvico-ureteric junction obstruction

Benign prostatic hyperplasia

A posterior urethral valve is an obstructive, developmental uropathy that usually


affects male infants (incidence 1 in 8000). Diagnostic features include bladder wall
hypertrophy, hydronephrosis and bladder diverticula.

Please rate this question:

 Discuss and give feedback

Next question 

Urethral valves

Posterior urethral valves are the commonest cause of infravesical outflow


obstruction in males. They may be diagnosed on ante natal ultrasonography.
Because the bladder has to develop high emptying pressures in utero, the child
may develop renal parenchymal damage. This translates to renal impairment noted
in 70% of boys at presentation. Treatment is with bladder catheterisation.
Endoscopic valvotomy is the definitive treatment of choice with cystoscopic and
renal follow up.
Next question 

Display my notes on this topic

          

Save my notes
gathered by dr. elbarky.

Question stats

A 61.5%
B 8.4%
C 5.3%
D 19.1%
E 5.7%

61.5% of users answered this question correctly

Search eMRCS

Search term Go

 Open MRCS Part A textbook (../review/textbook.php)

External links

+ Suggest a link

Dashboard

3

Question 136 of 364

 

Which of the following is not a pathological feature of breast cancer?

Resemblance to ductal epithelial cells

Angiogenesis
gathered by dr. elbarky.

Nuclear pleomorphism

Metastatic calcification

Vascular invasion

Dystrophic calcification may be present in breast malignancy and is the basis for
the breast screening programme. Metastatic calcification is calcification which
occurs in otherwise normal tissues, usually as a result of hypercalcaemia. Invasive
ductal carcinoma is the most common type of breast cancer, unless the tumour is
very poorly differentiated there is usually some resemblance to ductal epithelial
cells.

Please rate this question:

 Discuss and give feedback

Next question 

Breast cancer pathology

The histological features of breast cancer depend upon the underlying diagnosis.
The invasive component is usually comprised of ductal cells (unless it is an
invasive lobular cancer). In situ lesions may co-exist (such as DCIS).

Typical changes seen in conjunction with invasive breast cancer include:


1. Nuclear pleomorphism
2. Coarse chromatin
3. Angiogenesis
4. Invasion of the basement membrane
5. Dystrophic calcification (may be seen on mammography)
6. Abnormal mitoses
7. Vascular invasion
8. Lymph node metastasis

The primary tumour is graded on a scale of 1-3 where 1 is the most benign lesion
and 3 the most poorly differentiated.

Immunohistochemistry for oestrogen receptor and herceptin status is routinely


performed.

The grade, lymph node stage and size are combined to provide the Nottingham
prognostic index.
gathered by dr. elbarky.

Next question 

Display my notes on this topic

          

Save my notes

Question stats

A 39.4%
B 12.3%
C 7.7%
D 30.5%
E 10.1%

30.5% of users answered this question correctly

Search eMRCS

Search term Go

 Open MRCS Part A textbook (../review/textbook.php)


Question 137 of 364

 

A 24 year old man from Sub Saharan Africa presents with a lymphadenopathy and
weight loss. A diagnosis of tuberculosis is suspected and a lymph node biopsy is
performed. Staining with which of the agents below is most likely to facilitate
identification of the causative organism?
gathered by dr. elbarky.

Gram stain

Ziehl-Neelsen stain

Von Kossa stain

Van Gieson stain

Masson Trichrome stain

Ziehl-Neelsen stain is typically used to identify mycobacteria. They are not stained
in the Gram staining process. Van Gieson and Masson trichrome are histological
staining methods for identification of connective tissues. The Von Kossa
technique is useful for identifying tissue mineralisation.

Please rate this question:

 Discuss and give feedback

Next question 

Tuberculosis pathology

Is a form of primary chronic inflammation, caused by the inability of


macrophages to kill the Mycobacterium tuberculosis.
The macrophages often migrate to regional lymph nodes, the lung lesion
plus affected lymph nodes is referred to as a Ghon complex.
This leads to the formation of a granuloma which is a collection of
epithelioid histiocytes.
There is the presence of caseous necrosis in the centre.
The inflammatory response is mediated by a type 4 hypersensitivity
reaction.
In healthy individuals the disease may be contained, in the
immunocompromised disseminated (miliary TB) may occur.

Diagnosis
Waxy membrane of mycobacteria prevents binding with normal stains. Ziehl
- Neelsen staining is typically used.
Culture based methods take far longer.

Image showing acid- alcohol fast mycobacteria stained using the Ziehl- Neelsen
method
gathered by dr. elbarky.

(https://d2zgo9qer4wjf4.cloudfront.net/images_eMRCS/swb107b.jpg)
Image sourced from Wikipedia
(https://d2zgo9qer4wjf4.cloudfront.net
(http://en.wikipedia.org/wiki/Ziehl%E2%80
/images_eMRCS/swb107b.jpg)
%93Neelsen stain)

Next question 

Display my notes on this topic

          

Save my notes

Question stats

A 6.5%
B 77.2%

Question 138 of 364

 

Which of the following is not considered a risk factor for the development of
oesophageal malignancy?

Oesophageal metaplasia
gathered by dr. elbarky.

Smoking

Excessive intake of alcoholic spirits

Achalasia

Blood group O

Blood group O is not a risk factor for oesophageal cancer. Achalasia is associated
with the risk of developing squamous cell carcinoma of the oesophagus.

Please rate this question:

 Discuss and give feedback

Next question 

Oesophageal cancer

Incidence is increasing
In most cases in the Western world this increase is accounted for by a rise
in the number of cases of adenocarcinoma. In the UK adenocarcinomas
account for 65% of cases.
Barretts oesophagus is a major risk factor for most cases of oesophageal
adenocarcinoma.
In other regions of the world squamous cancer is more common and is
linked to smoking, alcohol intake, diets rich in nitrosamines and achalasia.
Surveillance of Barretts is important, as it imparts a 30 fold increase in
cancer risk and if invasive malignancy is diagnosed early then survival may
approach 85% at 5 years.
Diagnosis
Upper GI endoscopy is the first line test
Contrast swallow may be of benefit in classifying benign motility disorders
but has no place in the assessment of tumours
Staging is initially undertaken with CT scanning of the chest, abdomen and
pelvis. If overt metastatic disease is identified using this modality then
further complex imaging is unnecessary
If CT does not show metastatic disease, then local stage may be more
accurately assessed by use of endoscopic ultrasound.
Staging laparoscopy is performed to detect occult peritoneal disease. PET
CT is performed in those with negative laparoscopy. Thoracoscopy is not
routinely performed.
gathered by dr. elbarky.

Treatment
Operable disease is best managed by surgical resection. The most standard
procedure is an Ivor- Lewis type oesophagectomy. This procedure involves the
mobilisation of the stomach and division of the oesophageal hiatus. The abdomen
is closed and a right sided thoracotomy performed. The stomach is brought into
the chest and the oesophagus mobilised further. An intrathoracic
oesophagogastric anastomosis is constructed. Alternative surgical strategies
include a transhiatal resection (for distal lesions), a left thoraco-abdominal
resection (difficult access due to thoracic aorta) and a total oesophagectomy
(McKeown) with a cervical oesophagogastric anastomosis.
The biggest surgical challenge is that of anastomotic leak, with an intrathoracic
anastomosis this will result in mediastinitis. With high mortality. The McKeown
technique has an intrinsically lower systemic insult in the event of anastomotic
leakage.

In addition to surgical resection many patients will be treated with adjuvant


chemotherapy.

Next question 

Display my notes on this topic

          

Save my notes

Question stats

A 8%
B 6.8%
C 6.8%
D 11.4%
E 67%

67% of users answered this question correctly

Search eMRCS
gathered by dr. elbarky.

Search term Go

 Open MRCS Part A textbook (../review/textbook.php)

External links

+ Suggest a link

Dashboard

10

11

12

13

Question 139 of 364

 

Which of the following is not an extraintestinal feature of Crohns disease?

Iritis

Clubbing
gathered by dr. elbarky.

Aphthous ulcers

Erythema multiforme

Pyoderma gangrenosum

Extraintestinal manifestation of inflammatory bowel disease: A PIE SAC

Aphthous ulcers
Pyoderma gangrenosum
Iritis
Erythema nodosum
Sclerosing cholangitis
Arthritis
Clubbing

Please rate this question:

 Discuss and give feedback

Next question 

Crohns disease

Crohns disease is a chronic transmural inflammation of a segment(s) of the


gastrointestinal tract and may be associated with extra intestinal manifestations.
Frequent disease patterns observed include ileal, ileocolic and colonic disease.
Peri-anal disease may occur in association with any of these. The disease is often
discontinuous in its distribution. Inflammation may cause ulceration, fissures,
fistulas and fibrosis with stricturing. Histology reveals a chronic inflammatory
infiltrate that is usually patchy and transmural.

Ulcerative colitis Vs Crohns

Crohn's disease Ulcerative colitis

Distribution Mouth to anus Rectum and colon

Macroscopic Cobblestone appearance, Contact bleeding


changes apthoid ulceration
gathered by dr. elbarky.

Depth of Transmural inflammation Superficial inflammation


disease

Distribution Patchy Continuous


pattern

Histological Granulomas (non caseating Crypt abscesses,


features epithelioid cell aggregates with Inflammatory cells in the
Langhans' giant cells) lamina propria

Extraintestinal manifestations of Crohns

Related to disease extent Unrelated to disease extent

Aphthous ulcers (10%) Sacroiliiitis (10-15%)

Erythema nodosum (5-10%) Ankylosing spondylitis (1-2%)

Pyoderma gangrenosum (0.5%) Primary sclerosing cholangitis (Rare)

Acute arthropathy (6-12%) Gallstones (up to 30%)

Ocular complications (up to 10%) Renal calculi (up to 10%)

Diarrhoea in Crohns
Diarrhoea in Crohns may be multifactorial since actual inflammation of the colon is
not common. Causes therefore include the following:
Bile salt diarrhoea secondary to terminal ileal disease
Entero-colic fistula
Short bowel due to multiple resections
Bacterial overgrowth

Surgical interventions in Crohns disease


The commonest disease pattern in Crohns is stricturing terminal ileal disease and
this often culminates in an ileocaecal resection. Other procedures performed
include segmental small bowel resections and stricturoplasty. Colonic involvement
in patients with Crohns is not common and, where found, distribution is often
segmental. However, despite this distribution segmental resections of the colon in
patients with Crohns disease are generally not advocated because the recurrence
rate in the remaining colon is extremely high. As a result, the standard options of
colonic surgery in Crohns patients are generally; sub total colectomy,
panproctocolectomy and staged sub total colectomy and proctectomy. Restorative
procedures such as ileoanal pouch have no role in therapy.
Crohns disease is notorious for the developmental of intestinal fistulae; these may
form between the rectum and skin (peri anal) or the small bowel and skin.
Fistulation between loops of bowel may also occur and result in bacterial
gathered by dr. elbarky.

overgrowth and malabsorption. Management of enterocutaneous fistulae involves


controlling sepsis, optimising nutrition, imaging the disease and planning definitive
surgical management.

Next question 

Display my notes on this topic

          

Save my notes

Question stats

A 14%
B 20.6%
C 9.3%
D 43.5%
E 12.5%

43.5% of users answered this question correctly

Search eMRCS

Search term Go

Question 140 of 364

 

A splenectomy increases the risk of infection from all the following organisms
except?

Pneumococcus
gathered by dr. elbarky.

Klebsiella

Haemophilus influenzae

Staphylococcus aureus

Neisseria meningitidis

Mnemonic used to remember some encapsulated pathogens is: 'Even Some


Super Killers Have Pretty Nice Big Capsules'
Escherichia coli, Streptococcus pneumoniae, Salmonella, Klebsiella
pneumoniae, Haemophilus influenzae, Pseudomonas aeruginosa, Neisseria
meningitidis, Bacteroides fragilis, and the yeast Cryptococcus neoformans

Staphylococcus aureus infection following splenectomy is no more common than


in non splenectomised individuals. The other organisms are encapsulated, which is
why they are more likely to cause overwhelming post splenectomy sepsis.

Please rate this question:

 Discuss and give feedback

Next question 

Post splenectomy sepsis

Hyposplenism may complicate certain medical conditions where splenic atrophy


occurs or may be the result of medical intervention such as splenic artery
embolization and splenectomy for trauma. Diagnosis of hyposplenism is difficult
and whilst there may be peripheral markers of the splenectomised state (e.g.
Howell Jolly bodies) these are neither 100% sensitive or specific. The most
sensitive test is a radionucleotide labeled red cell scan.
Hyposplenism, by whatever mechanism it occurs dramatically increases the risk of
post splenectomy sepsis, particularly with encapsulated organisms. Since these
organisms may be opsonised, but this then goes undetected at an immunological
level due to loss of the spleen. For this reason individuals are recommended to be
vaccinated and have antibiotic prophylaxis.

Key recommendations
All those with hyposplenism or may become so (such as prior to an elective
splenectomy) should receive pneumococcal, haemophilus type b and
meningococcal type C vaccines. These should be administered 2 weeks
gathered by dr. elbarky.

prior to splenectomy or two weeks following splenectomy. The vaccine


schedule for meningococcal disease essentially consists of a dose of Men
C and Hib at 2 weeks and then a dose of the MenACWY vaccine one month
later. Those aged under 2 may require a booster at 2 years. A dose of
pneumococcal polyvalent polysaccharide vaccine (PPV) is given at two
weeks. A conjugated vaccine (PCV) is offered to young children. The PCV is
more immunogenic but covers fewer serotypes. Boosting PPV is either
guided by serological measurements (where available) or by routine
boosting doses at 5 yearly intervals.
Annual influenza vaccination is recommended in all cases
Antibiotic prophylaxis is offered to all. The risk of post splenectomy sepsis
is greatest immediately following splenectomy and in those aged less than
16 years or greater than 50 years. Individuals with a poor response to
pneumococcal vaccination are another high risk group. High risk individuals
should be counselled to take penicillin or macrolide prophylaxis. Those at
low risk may choose to discontinue therapy. All patients should be advised
about taking antibiotics early in the case of intercurrent infections.
Asplenic individuals traveling to malaria endemic areas are at high risk and
should have both pharmacological and mechanical protection.

Dosing
Penicillin V 500mg BD or amoxicillin 250mg BD

References
Davies J et al. Review of guidelines for the prevention and treatment of infection in
patients with an absent or dysfunctional spleen: Prepared on behalf of the British
Committee for Standards in Haematology by a Working Party of the Haemato-
Oncology Task Force. British Journal of Haematology 2011 (155): 308317.

Next question 
Display my notes on this topic

          

Save my notes

Question stats

A 6.4%
gathered by dr. elbarky.

B 22.5%
C 7.8%
D 53.6%
E 9.7%

53.6% of users answered this question correctly

Search eMRCS

Search term Go

 Open MRCS Part A textbook (../review/textbook.php)

External links

+ Suggest a link

Dashboard

5

Question 141 of 364

 

Which of the following is seen more commonly with Crohns disease rather than
ulcerative colitis?

Mucosal islands at endoscopy


gathered by dr. elbarky.

Goblet cell depletion on biopsy

Fat wrapping of the terminal ileum

Attenuated symptoms in smokers

Toxic megacolon

Crohns disease is worse in smokers and smoking is an independent risk


factor for disease recurrence following resection.

Fat wrapping of the terminal ileum is commonly seen in patients with ileal disease
(the commonest disease site). The mesenteric fat in patients with IBD is often
dense, hard and prone to considerable haemorrhage during surgery. At endoscopy,
the mucosa in patients with Crohns disease is said to resemble cobblestones,
mucosal islands (pseudopolyps) are seen in ulcerative colitis.

Please rate this question:

 Discuss and give feedback

Next question 

Crohns disease

Crohns disease is a chronic transmural inflammation of a segment(s) of the


gastrointestinal tract and may be associated with extra intestinal manifestations.
Frequent disease patterns observed include ileal, ileocolic and colonic disease.
Peri-anal disease may occur in association with any of these. The disease is often
discontinuous in its distribution. Inflammation may cause ulceration, fissures,
fistulas and fibrosis with stricturing. Histology reveals a chronic inflammatory
infiltrate that is usually patchy and transmural.

Ulcerative colitis Vs Crohns

Crohn's disease Ulcerative colitis

Distribution Mouth to anus Rectum and colon

Macroscopic Cobblestone appearance, Contact bleeding


changes apthoid ulceration

Depth of Transmural inflammation Superficial inflammation


gathered by dr. elbarky.

disease

Distribution Patchy Continuous


pattern

Histological Granulomas (non caseating Crypt abscesses,


features epithelioid cell aggregates with Inflammatory cells in the
Langhans' giant cells) lamina propria

Extraintestinal manifestations of Crohns

Related to disease extent Unrelated to disease extent

Aphthous ulcers (10%) Sacroiliiitis (10-15%)

Erythema nodosum (5-10%) Ankylosing spondylitis (1-2%)

Pyoderma gangrenosum (0.5%) Primary sclerosing cholangitis (Rare)

Acute arthropathy (6-12%) Gallstones (up to 30%)

Ocular complications (up to 10%) Renal calculi (up to 10%)

Diarrhoea in Crohns
Diarrhoea in Crohns may be multifactorial since actual inflammation of the colon is
not common. Causes therefore include the following:
Bile salt diarrhoea secondary to terminal ileal disease
Entero-colic fistula
Short bowel due to multiple resections
Bacterial overgrowth

Surgical interventions in Crohns disease


The commonest disease pattern in Crohns is stricturing terminal ileal disease and
this often culminates in an ileocaecal resection. Other procedures performed
include segmental small bowel resections and stricturoplasty. Colonic involvement
in patients with Crohns is not common and, where found, distribution is often
segmental. However, despite this distribution segmental resections of the colon in
patients with Crohns disease are generally not advocated because the recurrence
rate in the remaining colon is extremely high. As a result, the standard options of
colonic surgery in Crohns patients are generally; sub total colectomy,
panproctocolectomy and staged sub total colectomy and proctectomy. Restorative
procedures such as ileoanal pouch have no role in therapy.
Crohns disease is notorious for the developmental of intestinal fistulae; these may
form between the rectum and skin (peri anal) or the small bowel and skin.
Fistulation between loops of bowel may also occur and result in bacterial
overgrowth and malabsorption. Management of enterocutaneous fistulae involves
gathered by dr. elbarky.

controlling sepsis, optimising nutrition, imaging the disease and planning definitive
surgical management.

Next question 

Display my notes on this topic

          

Save my notes

Question stats

A 20.8%
B 11.7%
C 44%
D 14%
E 9.4%

44% of users answered this question correctly

Search eMRCS

Search term Go

Question 142 of 364

 

A 13 month old boy is brought to the surgical clinic by his mother because his left
testicle is not located in the scrotum. At which of the following sites would the
testicle be located if it were an ectopic testis?

Canalicular
gathered by dr. elbarky.

Inguinal

External inguinal ring

Superficial inguinal pouch

High scrotal

Ectopic testes are those that come to lie outside the normal range of
embryological descent (i.e. in the superficial inguinal pouch). Other sites of ectopic
testes include; base of penis, femoral triangle and perineum.

Please rate this question:

 Discuss and give feedback

Next question 

Testicular disorders-paediatric

Testicular disorders
Testicular disorders are some of the commonest conditions present in paediatric
urological practice.

Cryptorchidism
The embryological descent of the testicle from within the abdominal cavity
may be subject to a number of variations. Distinctions need to be made
clinically from a non descended testis and a testis that is retractile.
Testis that lie outside the normal path of embryological descent are termed
ectopic testis. Undescended testis occurs in 1% of male infants. Where the
testis does not lie in an intra scrotal location, its location should be
ascertained (by laparoscopy in first instance). Where both testes are absent,
the infant may be intersex.
MRI scanning may reveal intra-abdominal testes; however a GA is often
needed to perform this investigation in this age group.
Testes that are undescended should be placed in the scrotum after 1 year of
age, as the testosterone surge that may facilitate descent occurs at 6
months of age.
Where the testes lie distally e.g. Superficial inguinal pouch an open
orchidopexy is the procedure of choice.
With abdominal testes a laparoscopy should be performed. The risk of
seminoma is increased in individuals with a non descended testes and this
risk is not reduced by orchidopexy.
gathered by dr. elbarky.

Testicular torsion
Typically the patient has severe sudden onset of scrotal pain. The difficulty
in paediatric practice is the lack of clear history.
On examination the testis is tender and enlarged.
Management is by surgical exploration.
Delay beyond 6 hours is associated with low salvage rates.
A torted hyatid produces pain that is far more localised and the testis itself
should feel normal. However, diagnostic doubt often exists and in such
cases surgical exploration is warranted.

Hydrocele
Occur secondary to patent processus vaginalis
Present as fluid filling in scrotum or as cyst of the spermatic cord
Communicating hydroceles are treated by a trans inguinal ligation of the
PPV
Cystic hydroceles in older children may be treated with scrotal exploration

Next question 

Display my notes on this topic

          

Save my notes

Question stats

A 17.2%
B 22.4%
C 14.6%
D 37%
E 8.8%

37% of users answered this question correctly

Search eMRCS
gathered by dr. elbarky.

Search term Go

 Open MRCS Part A textbook (../review/textbook.php)

External links

+ Suggest a link

Dashboard

10

11

12

13

Question 143 of 364

 

A 2 year old boy presents with a right renal mass. On examination, he has an
irregular mass arising from the right flank and is hypertensive. A CT scan shows a
non calcified irregular lesion affecting the apex of the right kidney and the right
adrenal gland. What is the most likely diagnosis?
gathered by dr. elbarky.

Neuroblastoma

Nephroblastoma

Renal cell carcinoma

Transitional cell carcinoma

Lymphoma

Wilm's tumour of the kidney is the most common renal tumour in children. Both
nephroblastoma and neuroblastoma may occupy the adrenal and apex of the
kidney. In the case of neuroblastoma the lesion will have arisen from the adrenal, in
the case of nephroblastoma the lesion will have arisen from the kidney.
Hypertension is more commonly associated with nephroblastoma.
Neuroblastomas are usually calcified, whereas nephroblastomas are not and this
may be of diagnostic usefulness pre operatively.

Please rate this question:

 Discuss and give feedback

Next question 

Renal lesions

Lesion Disease specific features Treatment


Lesion Disease specific features Treatment

Renal cell Most present with Usually radical or partial


carcinoma haematuria (50%) nephrectomy
Common renal tumour
(85% cases)
Paraneoplastic
features include
hypertension and
polycythaemia
Most commonly has
haematogenous
gathered by dr. elbarky.

mestastasis

Nephroblastoma Rare childhood tumour Surgical resection


It accounts for 80% of combined with
all genitourinary chemotherapy (usually
malignancies in those vincristine, actinomycin D
under the age of 15 and doxorubicin)
years
Up to 90% will have a
mass
50% will be
hypertensive
Diagnostic work up
includes ultrasound
and CT scanning

Neuroblastoma Most common Surgical resection,


extracranial tumour of radiotherapy and
childhood chemotherapy
80% occur in those
under 4 years of age
Tumour of neural crest
origin (up to 50% occur
in the adrenal gland)
The tumour is usually
calcified and may be
diagnosed using MIBG
scanning
Staging is with CT
Lesion Disease specific features Treatment

Transitional cell Accounts for 90% of Radical


carcinoma lower urinary tract nephroureterectomy
tumours, but only 10%
of renal tumours

Males affected 3x
more than females
Occupational exposure
to industrial dyes and
rubber chemicals may
gathered by dr. elbarky.

increase risk
Up to 80% present with
painless haematuria
Diagnosis and staging
is with CT IVU

Angiomyolipoma 80% of these 50% of patients with


hamartoma type lesions >4cm will have
lesions occur symptoms and will require
sporadically, the surgical resection
remainder are seen in
those with tuberous
sclerosis
Tumour is composed
of blood vessels,
smooth muscle and fat
Massive bleeding may
occur in 10% of cases

Next question 

Display my notes on this topic

          

Save my notes

Question stats
A 22%
B 61.4%
61.4% of users answered this question correctly
C 7.1%
D 4.9%
E 4.5%
Search eMRCS

Search term Go
gathered by dr. elbarky.

 Open MRCS Part A textbook (../review/textbook.php)

External links

+ Suggest a link

Dashboard

10

11

12

13

14

15

Question 144 of 364

 

The pathogenesis of osteopetrosis is best explained by a defect in which of the


following?

Osteoclast function
gathered by dr. elbarky.

PTH receptors

Osteoblast function

Calcium resorption in proximal tubule

Calcium absorption

Please rate this question:

 Discuss and give feedback

Next question 

Osteopetrosis

Overview
Also known as marble bone disease
Rare disorder of defective osteoclast function resulting in failure of normal
bone resorption
Stem cell transplant and interferon-gamma have been used for treatment

Next question 

Display my notes on this topic

          
Save my notes

Question stats

A 49.1%
B 7.3%
C 28%
D 8%
gathered by dr. elbarky.

E 7.7%

49.1% of users answered this question correctly

Search eMRCS

Search term Go

 Open MRCS Part A textbook (../review/textbook.php)

External links

+ Suggest a link

Dashboard

8

Question 145 of 364

 

A 34 year old male presents with painful rectal bleeding and a fissure in ano is
suspected. On examination he has an epithelial defect at the mucocutaenous
junction that is located anteriorly. Approximately what proportion of patients with
fissure in ano will present with this pattern of disease?
gathered by dr. elbarky.

90%

10%

50%

25%

100%

Only a minority of patients with fissure in ano will have an anteriorly sited fissure.
They are particularly rare in males and an anterior fissure in a man should prompt
a search for an underlying cause.

Please rate this question:

 Discuss and give feedback

Next question 

Anal fissure

Anal fissures are a common cause of painful, bright red, rectal bleeding.
Most fissures are idiopathic and present as a painful mucocutaneous defect in the
posterior midline (90% cases). Fissures are more likely to be anteriorly located in
females, particularly if they are multiparous. Multiple fissures and those which are
located at other sites are more likely to be due to an underlying cause.
Diseases associated with fissure in ano include:
Crohns disease
Tuberculosis
Internal rectal prolapse
Diagnosis
In most cases the defect can be visualised as a posterior midline epithelial defect.
Where symptoms are highly suggestive of the condition and examination findings
are unclear an examination under anaesthesia may be helpful. Atypical disease
presentation should be investigated with colonoscopy and EUA with biopsies of
the area.

Treatment
Stool softeners are important as the hard stools may tear the epithelium and result
in recurrent symptoms. The most effective first line agents are topically applied
GTN (0.2%) or Diltiazem (2%) paste. Side effects of diltiazem are better tolerated.
gathered by dr. elbarky.

Resistant cases may benefit from injection of botulinum toxin or lateral internal
sphincterotomy (beware in females). Advancement flaps may be used to treat
resistant cases.
Sphincterotomy produces the best healing rates. It is associated with incontinence
to flatus in up to 10% of patients in the long term.

Next question 

Display my notes on this topic

          

Save my notes

Question stats

A 22.2%
B 43.6%
C 10.3%
D 17.2%
E 6.7%

43.6% of users answered this question correctly

Search eMRCS

Question 146 of 364

 

A 52 year old man with dyspepsia is found to have a duodenal ulcer. A CLO test is
taken and is positive. Which statement relating to the likely causative organism is
false?

It is a gram negative organism


gathered by dr. elbarky.

It preferentially colonises gastric type mucosa

It may occupy areas of ectopic gastric metaplasia

In patients who are colonised there is commonly evidence of fundal


gastritis on endoscopy

It produces a powerful urease that forms the basis of the Clo test

Helicobacter pylori accounts for >75% cases of duodenal ulceration. It may


be diagnosed with either serology, microbiology, histology or CLO testing.

Helicobacter pylori rarely produces any typical features on endoscopy. Where


infection is suspected the easiest course of action is to take an antral biopsy for
Clo testing in the endoscopy suite.

Please rate this question:

 Discuss and give feedback

Next question 

Helicobacter Pylori

Infection with Helicobacter Pylori is implicated in many cases of duodenal


ulceration and up to 60% of patients with gastric ulceration.

It is a gram negative, helical shaped rod with microaerophillic requirements. It has


the ability to produce a urease enzyme that will hydrolyse urea resulting in the
production of ammonia. The effect of ammonia on antral G cells is to cause
release of gastrin via a negative feedback loop.

Once infection is established the organism releases enzymes that disrupt the
gastric mucous layer. Certain subtypes release cytotoxins cag A and vac A gene
products. The organism incites a classical chronic inflammatory process of the
gastric epithelium. This accounts for the development of gastric ulcers. The mildly
increased acidity may induce a process of duodenal gastric metaplasia. Whilst
duodenal mucosa cannot be colonised by H-Pylori, mucosa that has undergone
metaplastic change to the gastric epithelial type may be colonised by H- Pylori with
subsequent inflammation and development of duodenitis and ulcers.
gathered by dr. elbarky.

In patients who are colonized, there is a 10-20% risk of peptic ulcer, 1-2% risk
gastric cancer and <1% risk MALT lymphoma.

Next question 

Display my notes on this topic

          

Save my notes

Question stats

A 16.7%
B 11.8%
C 15.9%
D 44.5%
E 11.1%

44.5% of users answered this question correctly

Search eMRCS

Search term Go

Question 147 of 364

 

A 43 year old man presents with a 3 week history of malaise, sore throat,
odynophagia and dysphagia. On examination he is found to have patchy white
spots in his oropharynx. An upper GI endoscopy is performed and similar lesions
are identified in the oesophagus. Which investigation is most likely to identify the
underlying pathology in this case?
gathered by dr. elbarky.

Serum urea and electrolytes

Oesophageal biopsy for culture

Oesophageal biopsy for histology

Glucose tolerance testing

Viral serology

Oesophageal candidiasis is associated with immunosupression; mainly in patients


on chemotherapy, with haematological malignancy, HIV or inhaled steroids. In
patients with HIV, oesophageal candidiasis is part of the spectrum of AIDS
defining illnesses and usually occurs when the CD4 count is less than 200. Others
include PCP pneumonia and CMV infections.

Please rate this question:

 Discuss and give feedback

Next question 

Oesophageal candidiasis

Characterised by white spots in the oropharynx with extension into the


oesophagus. Associated with broad spectrum antibiotic usage,
immunosupression and immunological disorders.
Patients may present with oropharyngeal symptoms, odynophagia and dysphagia.
Treatment is directed both at the underlying cause (which should be investigated
for) and with oral antifungal agents.
Next question 

Display my notes on this topic

          

Save my notes
gathered by dr. elbarky.

Question stats

A 6%
B 23.8%
C 14.6%
D 10.8%
E 44.8%

44.8% of users answered this question correctly

Search eMRCS

Search term Go

 Open MRCS Part A textbook (../review/textbook.php)

External links

+ Suggest a link

Dashboard

3

Question 148 of 364

 

A 43 year old female develops severe chest wall cellulitis following a mastectomy.
On examination, the skin is markedly erythematous. Which of the acute
inflammatory mediators listed below is least likely to produce vasodilation at this
site?
gathered by dr. elbarky.

Complement component C5a

Lysosomal compounds

Histamine

Serotonin

Prostaglandins

Erythema is a classical feature of acute inflammation. Potent mediators of


vascular dilatation include; histamine, prostaglandins, nitric oxide, platelet
activating factor, complement C5a (and C3a) and lysosomal compounds. Although
serotonin is associated with acute inflammation it is a vasoconstrictor. The effects
of serotonin are dependent upon the state of the vessels in the tissues. Intact and
healthy tissues and vessels will respond to a serotonin infusion with vasodilation
(hence the flushing seen in carcinoid syndrome). In contrast it worsens cardiac
ischaemia in myocardial infarcts when released from damaged platelets.

Please rate this question:

 Discuss and give feedback

Next question 

Acute inflammation

Inflammation is the reaction of the tissue elements to injury. Vascular changes


occur, resulting in the generation of a protein rich exudate. So long as the injury
does not totally destroy the existing tissue architecture, the episode may resolve
with restoration of original tissue architecture.
Vascular changes
Vasodilation occurs and persists throughout the inflammatory phase.
Inflammatory cells exit the circulation at the site of injury.
The equilibrium that balances Starlings forces within capillary beds is
disrupted and a protein rich exudate will form as the vessel walls also
become more permeable to proteins.
The high fibrinogen content of the fluid may form a fibrin clot. This has
several important immunomodulatory functions.

Sequelae

Resolution Typically occurs with minimal initial injury


gathered by dr. elbarky.

Stimulus removed and normal tissue


architecture results

Organisation Delayed removal of exudate


Tissues undergo organisation and usually
fibrosis

Suppuration Typically formation of an abscess or an


empyema
Sequestration of large quantities of dead
neutrophils

Progression to chronic Coupled inflammatory and reparative


inflammation activities
Usually occurs when initial infection or
suppuration has been inadequately managed

Causes
Infections e.g. Viruses, exotoxins or endotoxins released by bacteria
Chemical agents
Physical agents e.g. Trauma
Hypersensitivity reactions
Tissue necrosis

Presence of neutrophil polymorphs is a histological diagnostic feature of acute


inflammation

Next question 
Display my notes on this topic

          

Save my notes

Question stats

A 15.6%
gathered by dr. elbarky.

B 20.7%
C 14.1%
D 36.4%
E 13.3%

36.4% of users answered this question correctly

Search eMRCS

Search term Go

 Open MRCS Part A textbook (../review/textbook.php)

External links

+ Suggest a link

Dashboard

5

Question 149 of 364

 

A 48 year old lady with chronic hepatitis B infection is noted to have worsening
liver function tests and progressive jaundice. Her alpha feto protein levels are
grossly elevated. What is the most likely diagnosis?

Hepatocellular carcinoma
gathered by dr. elbarky.

Metastatic cancer

Cholangiocarcinoma

Yolk sac tumour

Liver cell adenoma

Please rate this question:

 Discuss and give feedback

Next question 

Liver tumours

Primary liver tumours


The most common primary tumours are cholangiocarcinoma and hepatocellular
carcinoma. Overall metastatic disease accounts for 95% of all liver malignancies
making the primary liver tumours comparatively rare.

Primary liver tumours include:


Cholangiocarcinoma
Hepatocellular carcinoma
Hepatoblastoma
Sarcomas (Rare)
Lymphomas
Carcinoids (most often secondary although primary may occur)
Hepatocellular carcinoma
These account for the bulk of primary liver tumours (75% cases). Its worldwide
incidence reflects its propensity to occur on a background of chronic inflammatory
activity. Most cases arise in cirrhotic livers or those with chronic hepatitis B
infection, especially where viral replication is actively occurring. In the UK it
accounts for less than 5% of all cancers, although in parts of Asia its incidence is
100 per 100,000.
The majority of patients (80%) present with existing liver cirrhosis, with a mass
discovered on screening ultrasound.

Diagnosis
CT/ MRI (usually both) are the imaging modalities of choice
gathered by dr. elbarky.

a-fetoprotein is elevated in almost all cases


Biopsy should be avoided as it seeds tumours cells through a resection
plane.
In cases of diagnostic doubt serial CT and αFP measurements are the
preferred strategy.

Treatment
Patients should be staged with liver MRI and chest, abdomen and pelvic CT
scan.
The testis should be examined in males (testicular tumours may cause
raised AFP). PET CT may be used to identify occult nodal disease.
Surgical resection is the mainstay of treatment in operable cases. In
patients with a small primary tumour in a cirrhotic liver whose primary
disease process is controlled, consideration may be given to primary whole
liver resection and transplantation.
Liver resections are an option but since most cases occur in an already
diseased liver the operative risks and post-operative hepatic dysfunction are
far greater than is seen following metastectomy.
These tumours are not particularly chemo or radiosensitive however, both
may be used in a palliative setting. Tumour ablation is a more popular
strategy.

Survival
Poor, overall survival is 15% at 5 years.

Cholangiocarcinoma
This is the second most common type of primary liver malignancy. As its name
suggests these tumours arise in the bile ducts. Up to 80% of tumours arise in the
extra hepatic biliary tree. Most patients present with jaundice and by this stage the
majority will have disease that is not resectable.
Primary sclerosing cholangitis is the main risk factor. In deprived countries typhoid
and liver flukes are also major risk factors.

Diagnosis
Patients will typically have an obstructive picture on liver function tests.
CA 19-9, CEA and CA 125 are often elevated
CT/ MRI and MRCP are the imaging methods of choice.

Treatment
Surgical resection offers the best chance of cure. Local invasion of peri hilar
tumours is a particular problem and this coupled with lobar atrophy will
often contra indicate surgical resection.
Palliation of jaundice is important, although metallic stents should be
avoided in those considered for resection.

Survival
gathered by dr. elbarky.

Is poor, approximately 5-10% 5 year survival.

Next question 

Display my notes on this topic

          

Save my notes

Question stats

A 79.7%
B 4.6%
C 5.7%
D 5.2%
E 4.9%

79.7% of users answered this question correctly

Search eMRCS

Search term Go

Question 151 of 364

 

A 28 year old man presents with right upper quadrant pain and hydatid disease is
suspected. Which of the following statements relating to the disease is untrue?

First line treatment is with pentavalent antimony.


gathered by dr. elbarky.

Peritoneal contamination with active daughter cysts may complicate


surgery.

CT scanning of the liver may show a floating membrane.

Biliary communication with the cysts may occur.

It is caused by Echinococcus granulosus.

Drug treatment is with albendazole or mebendazole. Praziquantzel may be used in


the pre operative stages.

Please rate this question:

 Discuss and give feedback

Next question 

Hydatid cysts
Hydatid cysts are endemic in Mediterranean and Middle Eastern countries. They
are caused by the tapeworm parasite Echinococcus granulosus. An outer fibrous
capsule is formed containing multiple small daughter cysts. These cysts are
allergens which precipitate a type 1 hypersensitivity reaction.

Clinical features are as follows:


Up to 90% cysts occur in the liver and lungs
Can be asymtomatic, or symptomatic if cysts > 5cm in diameter
Morbidity caused by cyst bursting, infection and organ dysfunction (biliary,
bronchial, renal and cerebrospinal fluid outflow obstruction)
In biliary ruputure there may be the classical triad of; biliary colic, jaundice,
and urticaria
gathered by dr. elbarky.

CT is the best investigation to differentiate hydatid cysts from amoebic and


pyogenic cysts.
Surgery is the mainstay of treatment (the cyst walls must not be ruptured during
removal and the contents sterilised first).

Next question 

Display my notes on this topic

          

Save my notes

Question stats

A 58.2%
B 7.5%
C 11.7%
D 10.3%
E 12.3%

58.2% of users answered this question correctly

Search eMRCS

Question 152 of 364

 

A 52 year old woman attends clinic for investigation of abdominal pain and
constipation. On examination, you note blue lines on the gum margin. She
mentions that her legs have become weak in the past few days. What is the most
likely diagnosis?
gathered by dr. elbarky.

Acute intermittent porphyria

Lead poisoning

Constipation

Guillan Barre syndrome

Rectal carcinoma

This would be an impressive diagnosis to make in the surgical out patient


department! The combination of abdominal pain and a motor periperal neuropathy,
should indicate this diagnosis. The blue line along the gum margin can occur in up
to 20% patients with lead poisoning.

Please rate this question:

 Discuss and give feedback

Next question 

Lead poisoning
Along with acute intermittent porphyria, lead poisoning should be considered in
questions giving a combination of abdominal pain and neurological signs

Features
abdominal pain
peripheral neuropathy (mainly motor)
fatigue
constipation
blue lines on gum margin (only 20% of adult patients, very rare in children)

Investigations
gathered by dr. elbarky.

The blood lead level is usually used for diagnosis. Levels greater than 10
mcg/dl are considered significant
Full blood count: microcytic anaemia. Blood film shows red cell
abnormalities including basophilic stippling and clover-leaf morphology
Raised serum and urine levels of delta aminolaevulinic acid may be seen
making it sometimes difficult to differentiate from acute intermittent
porphyria
Urinary coproporphyrin is also increased (urinary porphobilinogen and
uroporphyrin levels are normal to slightly increased)

Management - various chelating agents are currently used:


Dimercaptosuccinic acid (DMSA)
D-penicillamine
EDTA
Dimercaprol

Next question 

Display my notes on this topic

          

Save my notes

Question stats

A 20.3%
B 61.2%
C 5.2%

Question 153 of 364

 

A 23 year old lady undergoes a total thyroidectomy as treatment for a papillary


carcinoma of the thyroid. The pathologist examines histological sections of the
thyroid gland and identifies a psammoma body. What are these primarily
composed of?
gathered by dr. elbarky.

Clusters of calcification

Aggregations of neutrophils

Aggregations of macrophages

Giant cells surrounding the tumour

Clusters of oxalate crystals

Psammoma bodies consist of clusters of microcalcification. They are most


commonly seen in papillary carcinomas.

Please rate this question:

 Discuss and give feedback

Next question 

Thyroid malignancy

Papillary carcinoma
Commonest sub-type
Accurately diagnosed on fine needle aspiration cytology
Histologically, they may demonstrate psammoma bodies (areas of
calcification) and so called 'orphan Annie' nuclei
They typically metastasise via the lymphatics and thus laterally located
apparently ectopic thyroid tissue is usually a metastasis from a well
differentiated papillary carcinoma

Follicular carcinoma
Are less common than papillary lesions
Like papillary tumours, they may present as a discrete nodule. Although they
appear to be well encapsulated macroscopically there is invasion on
microscopic evaluation
Lymph node metastases are uncommon and these tumours tend to spread
haematogenously. This translates into a higher mortality rate
Follicular lesions cannot be accurately diagnosed on fine needle aspiration
cytology and thus all follicular FNA's (THY 3f) will require at least a hemi
thyroidectomy

Anaplastic carcinoma
gathered by dr. elbarky.

Less common and tend to occur in elderly females


Disease is usually advanced at presentation and often only palliative
decompression and radiotherapy can be offered.

Medullary carcinoma
These are tumours of the parafollicular cells ( C Cells) and are of neural
crest origin.
The serum calcitonin may be elevated which is of use when monitoring for
recurrence.
They may be familial and occur as part of the MEN -2A disease spectrum.
Spread may be either lymphatic or haematogenous and as these tumours
are not derived primarily from thyroid cells they are not responsive to
radioiodine.

Lymphoma
These respond well to combined chemoradiotherapy
Radical surgery is unnecessary once the disease has been diagnosed on
biopsy material. Such biopsy material is not generated by an FNA and thus a
core biopsy has to be obtained (with care!).

Next question 

Display my notes on this topic

          

Save my notes

Question stats
A 48.7%
B 9.7%
48.7% of users answered this question correctly
C 16.6%
D 15.6%
E 9.3%
Search eMRCS

Search term Go
gathered by dr. elbarky.

 Open MRCS Part A textbook (../review/textbook.php)

External links

+ Suggest a link

Dashboard

10

11

12

13

14

15

Question 154 of 364

 

A 23 year old man presents with diarrhoea and passage of mucous. He is


suspected of having ulcerative colitis. Which of the following is least likely to be
associated with this condition?

Superficial mucosal inflammation in the colon


gathered by dr. elbarky.

Significant risk of dysplasia in long standing disease

Epsiodes of large bowel obstruction during acute attacks

Haemorrhage

Disease sparing the anal canal

Large bowel obstruction is not a feature of UC, patients may develop megacolon.
However, this is a different entity both diagnostically and clinically. Ulcerative
colitis does not affect the anal canal and the anal transitional zone. Inflammation
is superficial. Dysplasia can occur in 2% overall, but increases significantly if
disease has been present over 20 years duration. Granulomas are features of
crohn's disease.

Other features:
Disease maximal in the rectum and may spread proximally
Contact bleeding
Longstanding UC crypt atrophy and metaplasia/dysplasia

Please rate this question:

 Discuss and give feedback

Next question 

Ulcerative colitis

Ulcerative colitis is a form of inflammatory bowel disease. Inflammation always


starts at rectum, does not spread beyond ileocaecal valve (although backwash
ileitis may occur) and is continuous. The peak incidence of ulcerative colitis is in
people aged 15-25 years and in those aged 55-65 years. It is less common in
smokers.

The initial presentation is usually following insidious and intermittent symptoms.


Features include:
bloody diarrhoea
urgency
tenesmus
abdominal pain, particularly in the left lower quadrant
extra-intestinal features (see below)
gathered by dr. elbarky.

Questions regarding the 'extra-intestinal' features of inflammatory bowel disease


are common. Extra-intestinal features include sclerosing cholangitis, iritis and
ankylosing spondylitis.

Common to both Crohn's Notes


disease (CD) and Ulcerative
colitis (UC)

Related to Arthritis: pauciarticular, Arthritis is the most


disease asymmetric common extra-intestinal
activity Erythema nodosum feature in both CD and UC
Episcleritis Episcleritis is more common
Osteoporosis in Crohns disease

Unrelated to Arthritis: polyarticular, Primary sclerosing


disease symmetric cholangitis is much more
activity Uveitis common in UC
Pyoderma gangrenosum Uveitis is more common in
Clubbing UC
Primary sclerosing
cholangitis

Pathology
Red, raw mucosa, bleeds easily
No inflammation beyond submucosa (unless fulminant disease)
Widespread superficial ulceration with preservation of adjacent mucosa
which has the appearance of polyps ('pseudopolyps')
Inflammatory cell infiltrate in lamina propria
Neutrophils migrate through the walls of glands to form crypt abscesses
Depletion of goblet cells and mucin from gland epithelium
Granulomas are infrequent

Barium enema
Loss of haustrations
Superficial ulceration, 'pseudopolyps'
Long standing disease: colon is narrow and short -'drainpipe colon'

Endoscopy
Superficial inflammation of the colonic and rectal mucosa
Continuous disease from rectum proximally
Superficial ulceration, mucosal islands, loss of vascular definition and
continuous ulceration pattern.

Management
gathered by dr. elbarky.

Patients with long term disease are at increased risk of development of


malignancy
Acute exacerbations are generally managed with steroids, in chronic
patients agents such as azathioprine and infliximab may be used
Individuals with medically unresponsive disease usually require surgery- in
the acute phase a sub total colectomy and end ileostomy. In the longer term
a proctectomy will be required. An ileoanal pouch is an option for selected
patients

References
Ford A et al. Ulcerative colitis. BMJ 2013 (346):29-34.

Next question 

Display my notes on this topic

          

Save my notes

Question stats

A 9.9%
B 7.2%
C 44.7%
D 10.3%
E 27.9%

Question 155 of 364

 

A patient presents with a facial nerve palsy. This occurred following repeat
excision of a facial lump. The histology report remarks on the biphasic appearance
of the lesion and mucinous connective tissue. What is the most likely underlying
lesion?
gathered by dr. elbarky.

Pleomorphic adenoma

Adenolymphoma

Anaplastic carcinoma

Adenoid cystic carcinoma

Schwannoma

The histological features are as described with a classic biphasic (mixed stromal
and epithelial elements), although benign local recurrence can complicate
incomplete excision. As this is a benign lesion direct extension into the facial nerve
is unlikely to occur. Facial nerve injury can happen during repeat parotid surgery.

Please rate this question:

 Discuss and give feedback

Next question 

Neck lumps

The table below gives characteristic exam question features for conditions
causing neck lumps:

Reactive By far the most common cause of neck swellings.


lymphadenopathy There may be a history of local infection or a
generalised viral illness
Lymphoma Rubbery, painless lymphadenopathy
The phenomenon of pain whilst drinking alcohol is very
uncommon
There may be associated night sweats and
splenomegaly

Thyroid swelling May be hypo-, eu- or hyperthyroid symptomatically


Moves upwards on swallowing

Thyroglossal cyst More common in patients < 20 years old


Usually midline, between the isthmus of the thyroid and
the hyoid bone
gathered by dr. elbarky.

Moves upwards with protrusion of the tongue


May be painful if infected

Pharyngeal pouch More common in older men


Represents a posteromedial herniation between
thyropharyngeus and cricopharyngeus muscles
Usually not seen, but if large then a midline lump in the
neck that gurgles on palpation
Typical symptoms are dysphagia, regurgitation,
aspiration and chronic cough

Cystic hygroma A congenital lymphatic lesion (lymphangioma) typically


found in the neck, classically on the left side
Most are evident at birth, around 90% present before 2
years of age

Branchial cyst An oval, mobile cystic mass that develops between the
sternocleidomastoid muscle and the pharynx
Develop due to failure of obliteration of the second
branchial cleft in embryonic development
Usually present in early adulthood

Cervical rib More common in adult females


Around 10% develop thoracic outlet syndrome

Carotid aneurysm Pulsatile lateral neck mass which doesn't move on


swallowing

Next question 

Display my notes on this topic


          

Save my notes

Question stats

A 42.3%
B 6.7%
C 6.6%
gathered by dr. elbarky.

D 38.9%
E 5.5%

42.3% of users answered this question correctly

Search eMRCS

Search term Go

 Open MRCS Part A textbook (../review/textbook.php)

External links

+ Suggest a link

Dashboard

7

Question 156 of 364

 

A 52 year old male is referred to urology clinic with impotence. He is known to


have hypertension. He does not have any morning erections. On further
questioning the patient reports pain in his buttocks, this worsens on mobilising. On
examination there is some muscle atrophy. The penis and scrotum are normal.
What is the most likely diagnosis?
gathered by dr. elbarky.

Leriche syndrome

S3-S4 cord lesion

Pudendal nerve lesion

Psychological impotence

Beta blocker induced impotence

Leriche syndrome

Classically, it is described in male patients as a triad of symptoms:

1. Claudication of the buttocks and thighs


2. Atrophy of the musculature of the legs
3. Impotence (due to paralysis of the L1 nerve)

Leriche syndrome, is atherosclerotic occlusive disease involving the abdominal


aorta and/or both of the iliac arteries. Management involves correcting underlying
risk factors such as hypercholesterolaemia and stopping smoking. Investigation is
usually with angiography.

Please rate this question:

 Discuss and give feedback

Next question 
Leriche syndrome

Atheromatous disease involving the iliac vessels. Blood flow to the pelvic viscera
is compromised. Patients may present with buttock claudication and impotence
(in this particular syndrome). Diagnostic work up will include angiography, where
feasible, iliac occlusions are usually treated with endovascular angioplasty and
stent insertion.

Next question 
gathered by dr. elbarky.

Display my notes on this topic

          

Save my notes

Question stats

A 53%
B 18.7%
C 14.3%
D 5.8%
E 8.3%

53% of users answered this question correctly

Search eMRCS

Search term Go

 Open MRCS Part A textbook (../review/textbook.php)


Question 157 of 364

 

What is the most common presentation of a parotid gland tumour?

Parapharyngeal mass

Mass at anterior border of masseter


gathered by dr. elbarky.

Mass inferior to the angle of the mandible

Mass posterior to the angle of the mandible

Mass anterior to the ear

Parotid tumours may present at any region in the gland. However, most lesions will
be located behind the angle of the mandible, inferior to the ear lobe. Tumours of
the deep lobe of the parotid may present as a parapharyngeal mass and large
lesions may displace the tonsil.

Please rate this question:

 Discuss and give feedback

Next question 

Parotid gland malignancy

Most parotid neoplasms (80%) are benign lesions


Most commonly present with painless mass around the mandible
Up to 30% may present with pain, when this is associated with a discrete
mass lesion in the parotid it usually indicates perineural invasion.
Perineural invasion is very unlikely to occur in association with benign
lesions
80% of patients with facial nerve weakness caused by parotid malignancies
will have nodal metastasis and a 5 year survival of 25%

Types of malignancy
Mucoepidermoid 30% of all parotid malignancies
carcinoma Usually low potential for local invasiveness and
metastasis (depends mainly on grade)

Adenoid cystic Unpredictable growth pattern


carcinoma Tendency for perineural spread
Nerve growth may display skip lesions resulting in
incomplete excision
Distant metastasis more common (visceral rather than
nodal spread)
5 year survival 35%
gathered by dr. elbarky.

Mixed tumours Often a malignancy occurring in a previously benign


parotid lesion

Acinic cell Intermediate grade malignancy


carcinoma May show perineural invasion
Low potential for distant metastasis
5 year survival 80%

Adenocarcinoma Develops from secretory portion of gland


Risk of regional nodal and distant metastasis
5 year survival depends upon stage at presentation,
may be up to 75% with small lesions with no nodal
involvement

Lymphoma Large rubbery lesion, may occur in association with


Warthins tumours
Diagnosis should be based on regional nodal biopsy
rather than parotid resection Treatment is with
chemotherapy (and radiotherapy)

Next question 

Display my notes on this topic

          

Save my notes

Question stats
A 4.8%
B 9.6%
30.6% of users answered this question correctly
C 16.7%
D 30.6%
E 38.4%
Search eMRCS

Search term Go
gathered by dr. elbarky.

 Open MRCS Part A textbook (../review/textbook.php)

External links

+ Suggest a link

Dashboard

10

11

12

13

14

15

Question 158 of 364

 

Which of the following is not true of gastric cancer?

There is an association with blood group A

Adenocarcinoma is the most common subtype


gathered by dr. elbarky.

Individuals with histological evidence of signet ring cells have a lower


incidence of lymph node metastasis

Lymphomas account for 5% cases

In Western Countries a more proximal disease distribution has been noted

Signet ring cells are features of poorly differentiated gastric cancer associated
with a increased risk of metastatic disease.

Please rate this question:

 Discuss and give feedback

Next question 

Gastric cancer

Overview
There are 700,000 new cases of gastric cancer worldwide each year. It is most
common in Japan and less common in western countries. It is more common in
men and incidence rises with increasing age. The exact cause of many sporadic
cancer is not known, however, familial cases do occur in HNPCC families. In
addition, smoking and smoked or preserved foods increase the risk. Japanese
migrants retain their increased risk (decreased in subsequent generations). The
distribution of the disease in western countries is changing towards a more
proximal location (perhaps due to rising obesity).

Pathology
There is some evidence of support a stepwise progression of the disease through
intestinal metaplasia progressing to atrophic gastritis and subsequent dysplasia,
through to cancer. The favoured staging system is TNM. The risk of lymph node
involvement is related to size and depth of invasion; early cancers confined to
submucosa have a 20% incidence of lymph node metastasis. Tumours of the
gastro-oesophageal junction are classified as below:

Type True oesophageal cancers and may be associated with Barrett's


1 oesophagus.

Type Carcinoma of the cardia, arising from cardiac type epithelium


2 or short segments with intestinal metaplasia at the oesophagogastric
junction.
gathered by dr. elbarky.

Type Sub cardial cancers that spread across the junction. Involve similar
3 nodal stations to gastric cancer.

Groups for close endoscopic monitoring


Intestinal metaplasia of columnar type
Atrophic gastritis
Low to medium grade dysplasia
Patients who have previously undergone resections for benign peptic ulcer
disease (except highly selective vagotomy).

Referral to endoscopy

Patients of any age with Patients without Worsening dyspepsia


dyspepsia and any of the dyspepsia
following

Chronic gastrointestinal Dysphagia Barretts oesophagus


bleeding

Dysphagia Unexplained Intestinal metaplasia


abdominal pain or
weight loss

Weight loss Vomiting Dysplasia

Iron deficiency anaemia Upper abdominal Atrophic gastritis


mass

Upper abdominal mass Jaundice Patient aged over 55 years


with unexplained or
persistent dyspepsia

Upper GI endoscopy performed for dyspepsia. The addition of dye spraying (as
shown in the bottom right) may facilitate identification of smaller tumours
gathered by dr. elbarky.

(https://d2zgo9qer4wjf4.cloudfront.net/images_eMRCS/swb067b.jpg)
Image sourced from Wikipedia
(https://d2zgo9qer4wjf4.cloudfront.net
(http://en.wikipedia.org/wiki/Gastric
/images_eMRCS/swb067b.jpg)
cancer)

Staging
CT scanning of the chest abdomen and pelvis is the routine first line staging
investigation in most centres.
Laparoscopy to identify occult peritoneal disease
PET CT (particularly for junctional tumours)

Treatment
Proximally sited disease greater than 5-10cm from the OG junction may be
treated by sub total gastrectomy
Total gastrectomy if tumour is <5cm from OG junction
For type 2 junctional tumours (extending into oesophagus)
oesophagogastrectomy is usual
Endoscopic sub mucosal resection may play a role in early gastric cancer
confined to the mucosa and perhaps the sub mucosa (this is debated)
Lymphadenectomy should be performed. A D2 lymphadenectomy is widely
advocated by the Japanese, the survival advantages of extended
lymphadenectomy have been debated. However, the overall
recommendation is that a D2 nodal dissection be undertaken.
Most patients will receive chemotherapy either pre or post operatively.
Prognosis

UK Data

Disease extent Percentage 5 year survival

All RO resections 54%

Early gastric cancer 91%

Stage 1 87%
gathered by dr. elbarky.

Stage 2 65%

Stage 3 18%

Operative procedure

Total Gastrectomy , lymphadenectomy and Roux en Y anastomosis

General anaesthesia
Prophylactic intravenous antibiotics
Incision: Rooftop.
Perform a thorough laparotomy to identify any occult disease.
Mobilise the left lobe of the liver off the diaphragm and place a large pack over it.
Insert a large self retaining retractor e.g. omnitract or Balfour (take time with this,
the set up should be perfect). Pack the small bowel away.
Begin by mobilising the omentum off the transverse colon.
Proceed to detach the short gastric vessels.
Mobilise the pylorus and divide it at least 2cm distally using a linear cutter stapling
device.
Continue the dissection into the lesser sac taking the lesser omentum and left
gastric artery flush at its origin.
The lymph nodes should be removed en bloc with the specimen where possible.
Place 2 stay sutures either side of the distal oesophagus. Ask the anaesthetist to
pull back on the nasogastric tube. Divide the distal oesophagus and remove the
stomach.
The oesphago jejunal anastomosis should be constructed. Identify the DJ flexure
and bring a loop of jejunum up to the oesophagus (to check it will reach). Divide
the jejunum at this point. Bring the divided jejunum either retrocolic or antecolic to
the oesophagus. Anastamose the oesophagus to the jejunum, using either
interrupted 3/0 vicryl or a stapling device. Then create the remainder of the Roux
en Y reconstruction distally.
Place a jejunostomy feeding tube.
Wash out the abdomen and insert drains (usually the anastomosis and duodenal
stump). Help the anaesthetist insert the nasogastric tube (carefully!)

Question 159 of 364

 

A 2 week infant has foul smelling material discharging from the umbilicus. What is
the underlying problem?

Proximal enterocutaneous fistula


gathered by dr. elbarky.

Distal enterocutaneous fistula

Entero-enteric fistula

Colo-cutaneous fistula

Vesico cutaneous fistula

This baby has an enterocutaneous fistula at the umbilicus due to complete failure
of the omphalomesenteric duct to obliterate. This is treated with resection.
Remember that vitello intestinal duct anomalies are common and these are always
distal.

Please rate this question:

 Discuss and give feedback

Next question 

Fistulas

A fistula is defined as an abnormal connection between two epithelial


surfaces.
There are many types ranging from Branchial fistulae in the neck to entero-
cutaneous fistulae abdominally.
In general surgical practice the abdominal cavity generates the majority and
most of these arise from diverticular disease and Crohn's.
As a general rule all fistulae will resolve spontaneously as long as there is no
distal obstruction. This is particularly true of intestinal fistulae.

The four types of fistulae are:


Enterocutaneous
These link the intestine to the skin. They may be high (>500ml) or low output
(<250ml) depending upon source. Duodenal /jejunal fistulae will tend to produce
high volume, electrolyte rich secretions which can lead to severe excoriation of the
skin. Colo-cutaneous fistulae will tend to leak faeculent material. Both fistulae may
result from the spontaneous rupture of an abscess cavity onto the skin (such as
following perianal abscess drainage) or may occur as a result of iatrogenic input.
In some cases it may even be surgically desirable e.g. mucous fistula following
sub total colectomy for colitis.

Suspect if there is excess fluid in the drain.


gathered by dr. elbarky.

Enteroenteric or Enterocolic
This is a fistula that involves the large or small intestine. They may originate in a
similar manner to enterocutaneous fistulae. A particular problem with this fistula
type is that bacterial overgrowth may precipitate malabsorption syndromes. This
may be particularly serious in inflammatory bowel disease.

Enterovaginal
Aetiology as above.

Enterovesical
This type of fistula goes to the bladder. These fistulas may result in frequent
urinary tract infections, or the passage of gas from the urethra during urination.

Management
Some rules relating to fistula management:
They will heal provided there is no underlying inflammatory bowel disease
and no distal obstruction, so conservative measures may be the best option
Where there is skin involvement, protect the overlying skin, often using a
well fitted stoma bag- skin damage is difficult to treat
A high output fistula may be rendered more easily managed by the use of
octreotide, this will tend to reduce the volume of pancreatic secretions.
Nutritional complications are common especially with high fistula (e.g. high
jejunal or duodenal) these may necessitate the use of TPN to provide
nutritional support together with the concomitant use of octreotide to
reduce volume and protect skin.
When managing perianal fistulae surgeons should avoid probing the fistula
where acute inflammation is present, this almost always worsens
outcomes.
When perianal fistulae occur secondary to Crohn's disease the best
management option is often to drain acute sepsis and maintain that
drainage through the judicious use of setons whilst medical management is
implemented.
Always attempt to delineate the fistula anatomy, for abscesses and fistulae
that have an intra abdominal source the use of barium and CT studies
should show a track. For perianal fistulae surgeons should recall Goodsall's
rule in relation to internal and external openings.

Next question 

Display my notes on this topic

          
gathered by dr. elbarky.

Save my notes

Question stats

A 18.8%
B 36%
C 6.6%
D 26.9%
E 11.7%

36% of users answered this question correctly

Search eMRCS

Search term Go

 Open MRCS Part A textbook (../review/textbook.php)

External links

+ Suggest a link

Dashboard

1

Question 160 of 364

 

A 75 year old lady is investigated for episodes of painless haematuria. Apart from
COPD from long term smoking she is otherwise well. She has no other urinary
symptoms. What is the most likely cause?

Transitional cell bladder cancer


gathered by dr. elbarky.

Squamous cell bladder cancer

Adenocarcinoma of the bladder

Stone disease

Arteriovenous malformation of the bladder

TCC commonly presents with painless haematuria that may be detected during
testing carried out for other reasons. Bladder AVM are very rare.

Please rate this question:

 Discuss and give feedback

Next question 

Bladder cancer

Bladder cancer is the second most common urological cancer. It most commonly
affects males aged between 50 and 80 years of age. Those who are current, or
previous (within 20 years), smokers have a 2-5 fold increased risk of the disease.
Exposure to hydrocarbons such as 2-Naphthylamine increases the risk. Although
rare in the UK, chronic bladder inflammation arising from Schistosomiasis
infection remains a common cause of squamous cell carcinomas, in those
countries where the disease is endemic.

Benign tumours
Benign tumours of the bladder including inverted urothelial papilloma and
nephrogenic adenoma are uncommon.
Bladder malignancies
Transitional cell carcinoma (>90% of cases)
Squamous cell carcinoma ( 1-7% -except in regions affected by
schistosomiasis)
Adenocarcinoma (2%)

Transitional cell carcinomas may arise as solitary lesions, or may be multifocal,


owing to the effect of 'field change' within the urothelium. Up to 70% of TCC's will
have a papillary growth pattern. These tumours are usually superficial in location
and accordingly have a better prognosis. The remaining tumours show either
mixed papillary and solid growth or pure solid growths. These tumours are typically
gathered by dr. elbarky.

more prone to local invasion and may be of higher grade, the prognosis is therefore
worse. Those with T3 disease or worse have a 30% (or higher) risk of regional or
distant lymph node metastasis.

TNM Staging

Stage Description

T0 No evidence of tumour

Ta Non invasive papillary carcinoma

T1 Tumour invades sub epithelial connective tissue

T2a Tumor invades superficial muscularis propria (inner half)

T2b Tumor invades deep muscularis propria (outer half)

T3 Tumour extends to perivesical fat

T4 Tumor invades any of the following: prostatic stroma, seminal vesicles,


uterus, vagina

T4a Invasion of uterus, prostate or bowel

T4b Invasion of pelvic sidewall or abdominal wall

N0 No nodal disease

N1 Single regional lymph node metastasis in the true pelvis (hypogastric,


obturator, external iliac, or presacral lymph node)

N2 Multiple regional lymph node metastasis in the true pelvis (hypogastric,


obturator, external iliac, or presacral lymph node metastasis)

N3 Lymph node metastasis to the common iliac lymph nodes

M0 No distant metastasis
Stage Description

M1 Distant disease

Presentation
Most patients (85%) will present with painless, macroscopic haematuria. In those
patients with incidental microscopic haematuria, up to 10% of females aged over
50 will be found to have a malignancy (once infection excluded).

Staging
gathered by dr. elbarky.

Most will undergo a cystoscopy and biopsies or TURBT, this provides histological
diagnosis and information relating to depth of invasion. Locoregional spread is
best determined using pelvic MRI and distant disease CT scanning. Nodes of
uncertain significance may be investigated using PET CT.

Treatment
Those with superficial lesions may be managed using TURBT in isolation. Those
with recurrences or higher grade/ risk on histology may be offered intravesical
chemotherapy. Those with T2 disease are usually offered either surgery (radical
cystectomy and ileal conduit) or radical radiotherapy.

Prognosis

T1 90%

T2 60%

T3 35%

T4a 10-25%

Any T, N1-N2 30%

Next question 

Display my notes on this topic

          

Save my notes
Question stats

A 66.3%
B 13.9%
C 10%
D 5%
E 4.8%

66.3% of users answered this question correctly


gathered by dr. elbarky.

Search eMRCS

Search term Go

 Open MRCS Part A textbook (../review/textbook.php)

External links

+ Suggest a link

Dashboard

10

Question 161 of 364

 

Which of the following is not a risk factor for developing tuberculosis?

Gastrectomy

Solid organ transplantation with immunosupression


gathered by dr. elbarky.

Intravenous drug use

Haematological malignancy

Amiodarone

Risk factors for developing active tuberculosis include:


silicosis
chronic renal failure
HIV positive
solid organ transplantation with immunosuppression
intravenous drug use
haematological malignancy
anti-TNF treatment
previous gastrectomy

Please rate this question:

 Discuss and give feedback

Next question 

Tuberculosis

Tuberculosis (TB) is an infection caused by Mycobacterium tuberculosis that most


commonly affects the lungs. Understanding the pathophysiology of TB can be
difficult - the key is to differentiate between primary and secondary disease.

Primary tuberculosis
A non-immune host who is exposed to M. tuberculosis may develop primary
infection of the lungs. A small lung lesion known as a Ghon focus develops. The
Ghon focus is composed of tubercle-laden macrophages. The combination of a
Ghon focus and hilar lymph nodes is known as a Ghon complex

In immunocompetent people the initial lesion usually heals by fibrosis. Those who
are immunocompromised may develop disseminated disease (miliary
tuberculosis).

Secondary (post-primary) tuberculosis


If the host becomes immunocompromised the initial infection may become
reactivated. Reactivation generally occurs in the apex of the lungs and may spread
locally or to more distant sites. Possible causes of immunocomprise include:
immunosuppressive drugs including steroids
gathered by dr. elbarky.

HIV
malnutrition

The lungs remain the most common site for secondary tuberculosis. Extra-
pulmonary infection may occur in the following areas:
central nervous system (tuberculous meningitis - the most serious
complication)
vertebral bodies (Pott's disease)
cervical lymph nodes (scrofuloderma)
renal
gastrointestinal tract

Next question 

Display my notes on this topic

          

Save my notes

Question stats

A 26.3%
B 6.6%
C 8.7%
D 6.2%
E 52.3%

Question 162 of 364

 

A 42 year old lady is investigated for symptoms of irritability and altered bowel
habit. On examination she is noted to have a smooth enlargement of the thyroid
gland. As part of her investigations thyroid function tests are requested, these are
as follows:

TSH 0.1 mug/l


gathered by dr. elbarky.

Free T4 35 pmol/l

The most likely underlying diagnosis is:

Multinodular goitre

Follicular carcinoma of the thyroid gland

Graves disease

Pregnancy

None of the above

TSH receptor antibodies will cause stimulation of the thyroid to synthesise T4.
However, this will have a negative feedback effect on the pituitary causing
decrease in TSH levels.
Where hyperthyroidism occurs secondary to pregnancy the TSH is typically
elevated.

Please rate this question:

 Discuss and give feedback

Next question 

Thyroid disease

Patients may present with a number of different manifestations of thyroid disease.


They can be broadly sub classified according to whether they are euthyroid or have
clinical signs of thyroid dysfunction. In addition it needs to be established whether
they have a mass or not.

Assessment
History
Examination including USS
If a nodule is identified then it should be sampled ideally via an image
guided fine needle aspiration
Radionucleotide scanning is of limited use

Thyroid Tumours
gathered by dr. elbarky.

Papillary carcinoma
Follicular carcinoma
Anaplastic carcinoma
Medullary carcinoma
Lymphoma's

Multinodular goitre
One of the most common reasons for presentation
Provided the patient is euthyroid and asymptomatic and no discrete nodules
are seen, they can be reassured.
In those with compressive symptoms surgery is required and the best
operation is a total thyroidectomy.
Sub total resections were practised in the past and simply result in recurrent
disease that requires a difficult revisional resection.

Endocrine dysfunction
In general these patients are managed by physicians initially.
Surgery may be offered alongside radio iodine for patients with Graves
disease that fails with medical management or in patients who would prefer
not to be irradiated (e.g. pregnant women).
Patients with hypothyroidism do not generally get offered a thyroidectomy.
Sometimes people inadvertently get offered resections during the early
phase of Hashimotos thyroiditis, however, with time the toxic phase passes
and patients can simply be managed with thyroxine.

Complications following surgery


Anatomical such as recurrent laryngeal nerve damage.
Bleeding. Owing to the confined space haematoma's may rapidly lead to
respiratory compromise owing to laryngeal oedema.
Damage to the parathyroid glands resulting in hypocalcaemia.

Further sources of information


1. http://www.acb.org.uk/docs/TFTguidelinefinal.pdf- Association of Clinical
Biochemistry guidelines for thyroid function tests.
2. British association of endocrine surgeons website- http://www.baets.org.uk

Next question 

Display my notes on this topic

          
gathered by dr. elbarky.

Save my notes

Question stats

A 9.7%
B 7.8%
C 70.1%
D 6.1%
E 6.3%

70.1% of users answered this question correctly

Search eMRCS

Search term Go

 Open MRCS Part A textbook (../review/textbook.php)

External links

+ Suggest a link

Dashboard

Question 163 of 364

 

Which one of the following may be associated with an increased risk of venous
thromboembolism?

Diabetes
gathered by dr. elbarky.

Cannula

Hyperthyroidism

Tamoxifen

Amiodarone

Consider thromboembolism in breast cancer patients on tamoxifen!

Please rate this question:

 Discuss and give feedback

Next question 

Venous thromboembolism: risk factors

Common predisposing factors include malignancy, pregnancy and the period


following an operation. The comprehensive list below is partly based on the 2010
SIGN venous thromboembolism (VTE) guidelines:

General
increased risk with advancing age
obesity
family history of VTE
pregnancy (especially puerperium)
immobility
hospitalisation
anaesthesia
central venous catheter: femoral >> subclavian
Underlying conditions
malignancy
thrombophilia: e.g. Activated protein C resistance, protein C and S deficiency
heart failure
antiphospholipid syndrome
Behcet's
polycythaemia
nephrotic syndrome
sickle cell disease
paroxysmal nocturnal haemoglobinuria
hyperviscosity syndrome
gathered by dr. elbarky.

homocystinuria

Medication
combined oral contraceptive pill: 3rd generation more than 2nd generation
hormone replacement therapy
raloxifene and tamoxifen
antipsychotics (especially olanzapine) have recently been shown to be a risk
factor

SIGN also state that the following are risk factors for recurrent VTE:
previous unprovoked VTE
male sex
obesity
thrombophilias

Next question 

Display my notes on this topic

          

Save my notes

Question stats

A 14.6%
B 13.9%
C 8.2%
D 54.4%
E 9%

54.4% of users answered this question correctly

Search eMRCS

Search term Go
gathered by dr. elbarky.

 Open MRCS Part A textbook (../review/textbook.php)

External links

+ Suggest a link

Dashboard

10

11

12

13

14

15

Question 164 of 364

 

A 78 year old man presents with symptoms of headaches and deteriorating vision.
He notices that there is marked pain on the right hand side of his face when he
combs his hair. What is the most likely diagnosis?

Giant cell arteritis


gathered by dr. elbarky.

Wegeners granulomatosis

Polyarteritis nodosa

Takayasu's arteritis

Buergers disease

Temporal arteritis may present acutely with symptoms of headache and visual
loss, or with a less acute clinical picture. Sight may be threatened and treatment
with immunosuppressants should be started promptly. The often requested
temporal artery biopsy (which can be the bane of many surgeons) is often non
diagnostic and unhelpful.

Please rate this question:

 Discuss and give feedback

Next question 

Vasculitis

The vasculitides are a group of conditions characterised by inflammation of the


blood vessel walls. This may, in turn, compromise vessel integrity. Constitutional
symptoms may be present. Whilst certain disease subtypes are reported to affect
specific vessels, there is often a degree of overlap clinically.

Vessel diameter and vasculitis classification


Aorta and branches Takayasu's arteritis
Buergers disease
Giant cell arteritis

Large and medium sized arteries Buergers disease


Giant cell arteritis
Polyarteritis nodosa

Medium sized muscular arteries Polyarteritis nodosa


Wegeners granulomatosis
gathered by dr. elbarky.

Small muscular arteries Wegeners granulomatosis


Rheumatoid vasculitis

Specific conditions

Takyasu's arteritis Inflammatory, obliterative arteritis affecting aorta


and branches
Females> Males
Symptoms may include upper limb claudication
Clinical findings include diminished or absent
pulses
ESR often affected during the acute phase

Buergers disease Segmental thrombotic occlusions of the small


and medium sized lower limb vessels
Commonest in young male smokers
Proximal pulses usually present, but pedal pulses
are lost
An acuter hypercellular occlusive thrombus is
often present
Tortuous corkscrew shaped collateral vessels
may be seen on angiography

Giant cell arteritis Systemic granulomatous arteritis that usually


affects large and medium sized vessels
Females > Males
Temporal arteritis is commonest type
Granulomatous lesions may be seen on biopsy
(although up to 50% are normal)
Polyarteritis nodosa Systemic necrotising vasculitis affecting small
and medium sized muscular arteries

Most common in populations with high


prevalence of hepatitis B
Renal disease is seen in 70% cases
Angiography may show saccular or fusiform
aneurysms and arterial stenoses

Wegeners Predominantly affects small and medium sized


granulomatosis arteries
gathered by dr. elbarky.

Systemic necrotising granulomatous vasculitis


Cutaneous vascular lesions may be seen
(ulceration, nodules and purpura)
Sinus imaging may show mucosal thickening and
air fluid levels

Treatment
Conditions such as Buergers disease are markedly helped by smoking cessation.
Immunosupression is the main treatment for vasculitides.

Next question 

Display my notes on this topic

          

Save my notes

Question stats
A 68.5%
B 7.4%
C 8.5%
D 10%
E 5.6%

68.5% of users answered this question correctly


gathered by dr. elbarky.

Search eMRCS

Search term Go

 Open MRCS Part A textbook (../review/textbook.php)

External links

+ Suggest a link

Dashboard

10

11

12

Question 165 of 364

 

A 55 year old lady presents with discomfort in the right breast. On clinical
examination a small lesion is identified and clinical appearances suggest
fibroadenoma. Imaging confirms the presence of a fibroadenoma alone. A core
biopsy is taken, this confirms the presence of the fibroadenoma. However, the
pathologist notices that a small area of lobular carcinoma in situ is also present in
gathered by dr. elbarky.

the biopsy. What is the best management?

Whole breast irradiation

Simple mastectomy

Mastectomy and sentinal lymph node biopsy

Wide local excision and sentinel lymph node biopsy

Breast MRI scan

Lobular carcinoma in situ has a low association with invasive malignancy. It is


seldom associated with microcalcification and therefore MRI is the best tool for
determining disease extent. Resection of in situ disease is not generally
recommended and most surgeons would simply pursue a policy of close clinical
and radiological follow up.

Please rate this question:

 Discuss and give feedback

Next question 

Lobular carcinoma of the breast

Lobular breast cancers are less common than their ductal counterparts. They
typically present differently, the mass is usually more diffuse and less obvious on
the usual imaging modalities of ultrasound and mammography. This is significant
since the disease may be understaged resulting in inadequate treatment when
wide local excision is undertaken.
In women with invasive lobular carcinoma it is usually safest to perform an MRI
scan of the breast, if breast conserving surgery is planned.
Lobular carcinomas are also more likely to be multifocal and metastasise to the
contralateral breast.
Lobular carcinoma in situ is occasionally diagnosed incidentally on core biopsies.
Unlike DCIS, lobular carcinoma in situ is far less strongly associated with foci of
invasion and is usually managed by close monitoring.

Lobular carcinoma stained using haematoxylin and eosin


gathered by dr. elbarky.

(https://d2zgo9qer4wjf4.cloudfront.net/images_eMRCS/swb217b.png)
Image sourced from (https://d2zgo9qer4wjf4.cloudfront.net/images_eMRCS
Wikipedia () /swb217b.png)

Next question 

Display my notes on this topic

          

Save my notes

Question stats

A 5.9%
B 9.6%
C 11.9%
D 43.4%
E 29.3%

29.3% of users answered this question correctly

Search eMRCS

Search term Go
gathered by dr. elbarky.

 Open MRCS Part A textbook (../review/textbook.php)

External links

+ Suggest a link

Dashboard

10

11

12

13

14

15
286

287

288

289

290

291

292

293
gathered by dr. elbarky.

294

295

296

297

298

299

300

301

302

303

304

305

306

307

308

309

310

311

312

313

314

315

Question 166 of 364

 

A 45 year old lady presents with a pathological fracture of her femoral shaft. She is
a poor historian, but it transpires that she underwent a thyroidectomy 1 year
previously. She has no other illness or co-morbidities. What is the most likely
underlying diagnosis?
gathered by dr. elbarky.

Hyperparathyroidism

Metastatic papillary carcinoma of the thyroid

Metastatic medullary carcinoma of the thyroid

Metastatic follicular carcinoma of the thyroid

None of the above

Follicular carcinomas are a recognised cause of bone metastasis. Papillary lesions


typically spread via the lymphatics.

Please rate this question:

 Discuss and give feedback

Next question 

Thyroid malignancy

Papillary carcinoma
Commonest sub-type
Accurately diagnosed on fine needle aspiration cytology
Histologically, they may demonstrate psammoma bodies (areas of
calcification) and so called 'orphan Annie' nuclei
They typically metastasise via the lymphatics and thus laterally located
apparently ectopic thyroid tissue is usually a metastasis from a well
differentiated papillary carcinoma

Follicular carcinoma
Are less common than papillary lesions
Like papillary tumours, they may present as a discrete nodule. Although they
appear to be well encapsulated macroscopically there is invasion on
microscopic evaluation
Lymph node metastases are uncommon and these tumours tend to spread
haematogenously. This translates into a higher mortality rate
Follicular lesions cannot be accurately diagnosed on fine needle aspiration
cytology and thus all follicular FNA's (THY 3f) will require at least a hemi
thyroidectomy

Anaplastic carcinoma
gathered by dr. elbarky.

Less common and tend to occur in elderly females


Disease is usually advanced at presentation and often only palliative
decompression and radiotherapy can be offered.

Medullary carcinoma
These are tumours of the parafollicular cells ( C Cells) and are of neural
crest origin.
The serum calcitonin may be elevated which is of use when monitoring for
recurrence.
They may be familial and occur as part of the MEN -2A disease spectrum.
Spread may be either lymphatic or haematogenous and as these tumours
are not derived primarily from thyroid cells they are not responsive to
radioiodine.

Lymphoma
These respond well to combined chemoradiotherapy
Radical surgery is unnecessary once the disease has been diagnosed on
biopsy material. Such biopsy material is not generated by an FNA and thus a
core biopsy has to be obtained (with care!).

Next question 

Display my notes on this topic

          

Save my notes

Question stats
A 10.1%
B 9.4%
58.9% of users answered this question correctly
C 13.9%
D 58.9%
E 7.8%
Search eMRCS

Search term Go
gathered by dr. elbarky.

 Open MRCS Part A textbook (../review/textbook.php)

External links

+ Suggest a link

Dashboard

10

11

12

13

14

15

Question 167 of 364

 

A 56 year old man is investigated for anaemia and a large exophytic growth is
identified in the distal stomach. What pathological finding is most likely?

Squamous cell carcinoma


gathered by dr. elbarky.

Adenocarcinoma

Lymphoma

Leiomyosarcoma

Rhadomyosarcoma

Most gastric malignancies are adenocarcinomas.

Please rate this question:

 Discuss and give feedback

Next question 

Gastric cancer

Overview
There are 700,000 new cases of gastric cancer worldwide each year. It is most
common in Japan and less common in western countries. It is more common in
men and incidence rises with increasing age. The exact cause of many sporadic
cancer is not known, however, familial cases do occur in HNPCC families. In
addition, smoking and smoked or preserved foods increase the risk. Japanese
migrants retain their increased risk (decreased in subsequent generations). The
distribution of the disease in western countries is changing towards a more
proximal location (perhaps due to rising obesity).

Pathology
There is some evidence of support a stepwise progression of the disease through
intestinal metaplasia progressing to atrophic gastritis and subsequent dysplasia,
through to cancer. The favoured staging system is TNM. The risk of lymph node
involvement is related to size and depth of invasion; early cancers confined to
submucosa have a 20% incidence of lymph node metastasis. Tumours of the
gastro-oesophageal junction are classified as below:

Type True oesophageal cancers and may be associated with Barrett's


1 oesophagus.

Type Carcinoma of the cardia, arising from cardiac type epithelium


2 or short segments with intestinal metaplasia at the oesophagogastric
junction.

Type Sub cardial cancers that spread across the junction. Involve similar
gathered by dr. elbarky.

3 nodal stations to gastric cancer.

Groups for close endoscopic monitoring


Intestinal metaplasia of columnar type
Atrophic gastritis
Low to medium grade dysplasia
Patients who have previously undergone resections for benign peptic ulcer
disease (except highly selective vagotomy).

Referral to endoscopy

Patients of any age with Patients without Worsening dyspepsia


dyspepsia and any of the dyspepsia
following

Chronic gastrointestinal Dysphagia Barretts oesophagus


bleeding

Dysphagia Unexplained Intestinal metaplasia


abdominal pain or
weight loss

Weight loss Vomiting Dysplasia

Iron deficiency anaemia Upper abdominal Atrophic gastritis


mass

Upper abdominal mass Jaundice Patient aged over 55 years


with unexplained or
persistent dyspepsia

Upper GI endoscopy performed for dyspepsia. The addition of dye spraying (as
shown in the bottom right) may facilitate identification of smaller tumours
gathered by dr. elbarky.

(https://d2zgo9qer4wjf4.cloudfront.net/images_eMRCS/swb067b.jpg)
Image sourced from Wikipedia
(https://d2zgo9qer4wjf4.cloudfront.net
(http://en.wikipedia.org/wiki/Gastric
/images_eMRCS/swb067b.jpg)
cancer)

Staging
CT scanning of the chest abdomen and pelvis is the routine first line staging
investigation in most centres.
Laparoscopy to identify occult peritoneal disease
PET CT (particularly for junctional tumours)

Treatment
Proximally sited disease greater than 5-10cm from the OG junction may be
treated by sub total gastrectomy
Total gastrectomy if tumour is <5cm from OG junction
For type 2 junctional tumours (extending into oesophagus)
oesophagogastrectomy is usual
Endoscopic sub mucosal resection may play a role in early gastric cancer
confined to the mucosa and perhaps the sub mucosa (this is debated)
Lymphadenectomy should be performed. A D2 lymphadenectomy is widely
advocated by the Japanese, the survival advantages of extended
lymphadenectomy have been debated. However, the overall
recommendation is that a D2 nodal dissection be undertaken.
Most patients will receive chemotherapy either pre or post operatively.

Prognosis
UK Data

Disease extent Percentage 5 year survival

All RO resections 54%

Early gastric cancer 91%

Stage 1 87%

Stage 2 65%
gathered by dr. elbarky.

Stage 3 18%

Operative procedure

Total Gastrectomy , lymphadenectomy and Roux en Y anastomosis

General anaesthesia
Prophylactic intravenous antibiotics
Incision: Rooftop.
Perform a thorough laparotomy to identify any occult disease.
Mobilise the left lobe of the liver off the diaphragm and place a large pack over it.
Insert a large self retaining retractor e.g. omnitract or Balfour (take time with this,
the set up should be perfect). Pack the small bowel away.
Begin by mobilising the omentum off the transverse colon.
Proceed to detach the short gastric vessels.
Mobilise the pylorus and divide it at least 2cm distally using a linear cutter stapling
device.
Continue the dissection into the lesser sac taking the lesser omentum and left
gastric artery flush at its origin.
The lymph nodes should be removed en bloc with the specimen where possible.
Place 2 stay sutures either side of the distal oesophagus. Ask the anaesthetist to
pull back on the nasogastric tube. Divide the distal oesophagus and remove the
stomach.
The oesphago jejunal anastomosis should be constructed. Identify the DJ flexure
and bring a loop of jejunum up to the oesophagus (to check it will reach). Divide
the jejunum at this point. Bring the divided jejunum either retrocolic or antecolic to
the oesophagus. Anastamose the oesophagus to the jejunum, using either
interrupted 3/0 vicryl or a stapling device. Then create the remainder of the Roux
en Y reconstruction distally.
Place a jejunostomy feeding tube.
Wash out the abdomen and insert drains (usually the anastomosis and duodenal
stump). Help the anaesthetist insert the nasogastric tube (carefully!)
Close the abdomen and skin.
Enteral feeding may commence on the first post-operative day. However, most
surgeons will leave patients on free NG drainage for several days and keep them nil
by mouth.

Next question 

Display my notes on this topic

          
gathered by dr. elbarky.

Save my notes

Question stats

A 12.4%
B 61.6%
C 8.2%
D 12.1%
E 5.8%

61.6% of users answered this question correctly

Search eMRCS

Search term Go

 Open MRCS Part A textbook (../review/textbook.php)

External links

+ Suggest a link

Dashboard

Question 168 of 364

 

A 43 year old lady presents with severe chest pain. Investigations demonstrate a
dissecting aneurysm of the ascending aorta which originates at the aortic valve.
What is the optimal long term treatment?

Endovascular stent
gathered by dr. elbarky.

Medical therapy with beta blockers

Medical therapy with ACE inhibitors

Sutured aortic repair

Aortic root replacement

Proximal aortic dissections are generally managed with surgical aortic root
replacement. The proximal origin of the dissection together with chest pain (which
may occur in all types of aortic dissection) raises concerns about the possibility of
coronary ostial involvement (which precludes stenting). There is no role for
attempted suture repair in this situation.

Please rate this question:

 Discuss and give feedback

Next question 

Aortic dissection
More common than rupture of the abdominal aorta
33% of patients die within the first 24 hours, and 50% die within 48 hours if
no treatment received
Associated with hypertension
Features of aortic dissection: tear in the intimal layer, followed by formation
and propagation of a subintimal hematoma. Cystic medial necrosis
(Marfan's)
Most common site of dissection: 90% occurring within 10 centimetres of the
aortic valve

Stanford Classification
gathered by dr. elbarky.

Type Location Treatment

A Ascending aorta/ aortic root Surgery- aortic root replacement

B Descending aorta Medical therapy with antihypertensives

DeBakey classification

Type Site affected

I Ascending aorta, aortic arch, descending aorta

II Ascending aorta only

III Descending aorta distal to left subclavian artery

Clinical features
Tearing, sudden onset chest pain (painless 10%)
Hypertension or Hypotension
A blood pressure difference (in each arm) greater than 20 mm Hg
Neurologic deficits (20%)

Investigations
CXR: widened mediastinum, abnormal aortic knob, ring sign, deviation of the
trachea/oesophagus
CT angiography of the thoracic aorta
MRI angiography
Conventional angiography (now rarely used diagnostically)

Management
Beta-blockers: aim HR 60-80 bpm and systolic BP 100-120 mm Hg
For type A dissections the standard of care is aortic root replacement
Next question 

Display my notes on this topic

          

Save my notes
gathered by dr. elbarky.

Question stats

A 13.1%
B 9.8%
C 6.8%
D 12.1%
E 58.3%

58.3% of users answered this question correctly

Search eMRCS

Search term Go

 Open MRCS Part A textbook (../review/textbook.php)

External links

+ Suggest a link

Dashboard

3

Question 169 of 364

 

A 20 year old man is suspected of having an inflamed Meckels diverticulum. At


which of the following locations is it most likely to be found?

Approximately 60 cm distal to the ileo-caecal valve


gathered by dr. elbarky.

Approximately 60 cm proximal to the ileocaecal valve

Approximately 200cm distal to the ileocaecal valve

Approximately 200cm proximal to the ileocaecal valve

50cm distal to the DJ flexure

Rule of 2's

2% of population
2 inches (5cm) long
2 feet (60 cm) from the ileocaecal valve
2 x's more common in men
2 tissue types involved

They are typically found 2 feet proximal to the ileocaecal valve (or approximately
60cm).

Please rate this question:

 Discuss and give feedback

Next question 

Meckel's diverticulum

Congenital abnormality resulting in incomplete obliteration of the vitello-


intestinal duct
Normally, in the foetus, there is an attachment between the vitello-intestinal
duct and the yolk sac.This disappears at 6 weeks gestation.
The tip is free in majority of cases.
Associated with enterocystomas, umbilical sinuses, and omphaloileal
fistulas.
Arterial supply: omphalomesenteric artery.
2% of population, 2 inches long, 2 feet from the ileocaecal valve.
Typically lined by ileal mucosa but ectopic gastric mucosa can occur, with
the risk of peptic ulceration. Pancreatic and jejunal mucosa can also occur.

Clinical
Normally asymptomatic and an incidental finding.
gathered by dr. elbarky.

Complications are the result of obstruction, ectopic tissue, or inflammation.


Removal if narrow neck or symptomatic. Options are between wedge
excision or formal small bowel resection and anastomosis.

Next question 

Display my notes on this topic

          

Save my notes

Question stats

A 16.1%
B 58.6%
C 8%
D 10.2%
E 7.2%

58.6% of users answered this question correctly

Search eMRCS

Search term Go

Question 170 of 364

 

A 69 year old male presents with haematuria. He worked in the textile industry. He
has a left flank mass. A CT IVU shows a lesion of the left renal pelvis. What is the
most likely lesion?

Adenocarcinoma
gathered by dr. elbarky.

Transitional cell carcinoma

Nephroblastoma

Neuroblastoma

Angiomyolipoma

TCC is a rare form of renal cancer, accounting for approximately 7% of all renal
tumours. Risk factors include exposure to chemicals in the textile, plastic and
rubber industry.

Please rate this question:

 Discuss and give feedback

Next question 

Renal lesions

Lesion Disease specific features Treatment


Lesion Disease specific features Treatment

Renal cell Most present with Usually radical or partial


carcinoma haematuria (50%) nephrectomy
Common renal tumour
(85% cases)
Paraneoplastic
features include
hypertension and
polycythaemia
Most commonly has
haematogenous
gathered by dr. elbarky.

mestastasis

Nephroblastoma Rare childhood tumour Surgical resection


It accounts for 80% of combined with
all genitourinary chemotherapy (usually
malignancies in those vincristine, actinomycin D
under the age of 15 and doxorubicin)
years
Up to 90% will have a
mass
50% will be
hypertensive
Diagnostic work up
includes ultrasound
and CT scanning

Neuroblastoma Most common Surgical resection,


extracranial tumour of radiotherapy and
childhood chemotherapy
80% occur in those
under 4 years of age
Tumour of neural crest
origin (up to 50% occur
in the adrenal gland)
The tumour is usually
calcified and may be
diagnosed using MIBG
scanning
Staging is with CT
Lesion Disease specific features Treatment

Transitional cell Accounts for 90% of Radical


carcinoma lower urinary tract nephroureterectomy
tumours, but only 10%
of renal tumours

Males affected 3x
more than females
Occupational exposure
to industrial dyes and
rubber chemicals may
gathered by dr. elbarky.

increase risk
Up to 80% present with
painless haematuria
Diagnosis and staging
is with CT IVU

Angiomyolipoma 80% of these 50% of patients with


hamartoma type lesions >4cm will have
lesions occur symptoms and will require
sporadically, the surgical resection
remainder are seen in
those with tuberous
sclerosis
Tumour is composed
of blood vessels,
smooth muscle and fat
Massive bleeding may
occur in 10% of cases

Next question 

Display my notes on this topic

          

Save my notes

Question stats
A 14.8%
B 64.8%
64.8% of users answered this question correctly
C 6.6%
D 5%
E 8.8%
Search eMRCS

Search term Go
gathered by dr. elbarky.

 Open MRCS Part A textbook (../review/textbook.php)

External links

+ Suggest a link

Dashboard

10

11

12

13

14

15

Question 171 of 364

 

Which of the tumour types listed below is found most frequently in a person with
aggressive fibromatosis?

Medullary thyroid cancer


gathered by dr. elbarky.

Basal cell carcinoma of the skin

Desmoid tumours

Dermoid tumours

Malignant melanoma

Please rate this question:

 Discuss and give feedback

Next question 

Aggressive fibromatosis

Aggressive fibromatosis is a disorder consisting of desmoid tumours, which


behave in a locally aggressive manner. Desmoid tumours may be identified in both
abdominal and extra-abdominal locations. Metastatic disease is rare. The main
risk factor (for abdominal desmoids) is having APC variant of familial
adenomatous polyposis coli. Most cases are sporadic.
Treatment is by surgical excision.

Next question 

Display my notes on this topic


          

Save my notes

Question stats

A 18.3%
B 9.6%
C 49.9%
gathered by dr. elbarky.

D 13%
E 9.2%

49.9% of users answered this question correctly

Search eMRCS

Search term Go

 Open MRCS Part A textbook (../review/textbook.php)

External links

+ Suggest a link

Dashboard

7

Question 172 of 364

 

Which of the following conditions is least likely to exhibit the Koebner


phenomenon?

Vitiligo
gathered by dr. elbarky.

Molluscum contagiosum

Lichen planus

Psoriasis

Lupus vulgaris

Lupus vulgaris is not associated with the Koebner phenomenon.

Please rate this question:

 Discuss and give feedback

Next question 

Koebner phenomenon

The Koebner phenomenon describes skin lesions which appear at the site of injury.
It is seen in:
Psoriasis
Vitiligo
Warts
Lichen planus
Lichen sclerosus
Molluscum contagiosum

Next question 
Display my notes on this topic

          

Save my notes

Question stats
gathered by dr. elbarky.

A 21.2%
B 22.3%
C 13.7%
D 12.8%
E 30%

30% of users answered this question correctly

Search eMRCS

Search term Go

 Open MRCS Part A textbook (../review/textbook.php)

External links

+ Suggest a link

Dashboard

5

Question 173 of 364

 

Which of the following statements relating to malignant mesothelioma is false?

It may be treated by extrapleural pneumonectomy.

It is linked to asbestos exposure.


gathered by dr. elbarky.

It is linked to cigarette smoking independent of asbestos exposure.

It may occur intra abdominally.

It is relatively resistant to radiotherapy

It is not linked to cigarette smoking. When identified at an early stage a radical


resection is the favored option. Radiotherapy is often given perioperatively.
However, it is not a particularly radiosensitive tumour. Combination chemotherapy
gives some of the best results and most regimes are cisplatin based.

Please rate this question:

 Discuss and give feedback

Next question 

Occupational cancers

Occupational cancers accounted for 5.3% cancer deaths in 2005.


In men the main cancers include:
Mesothelioma
Bladder cancer
Non melanoma skin cancer
Lung cancer
Sino nasal cancer

Occupations with high levels of occupational tumours include:


Construction industry
Working with coal tar and pitch
Mining
Metalworkers
Working with asbestos (accounts for 98% of all mesotheliomas)
Working in rubber industry

Shift work has been linked to breast cancer in women (Health and safety executive
report RR595).

The latency between exposure and disease is typically 15 years for solid tumours
and 20 for leukaemia.
gathered by dr. elbarky.

Many occupational cancers are otherwise rare. For example sino nasal cancer is
an uncommon tumour, 50% will be SCC. They are linked to conditions such as
wood dust exposure and unlike lung cancer is not strongly linked to cigarette
smoking. Another typical occupational tumour is angiosarcoma of the liver which
is linked to working with vinyl chloride. Again in the non occupational context this
is an extremely rare sporadic tumour.

Next question 

Display my notes on this topic

          

Save my notes

Question stats

A 14.2%
B 6.3%
C 53.5%
D 11.2%
E 14.8%

53.5% of users answered this question correctly

Search eMRCS

Question 174 of 364

 

Which of the following tumours are most likely to give rise to para-aortic nodal
metastasis early?

Ovarian
gathered by dr. elbarky.

Bladder

Rectal

Caecal

Cervical

Ovarian tumours are supplied by the ovarian vessels, these branch directly from
the aorta. The cervix drains to the internal and external iliac nodes.

Please rate this question:

 Discuss and give feedback

Next question 

Para-aortic lymphadenopathy

Organ sites that may metastasise (early) to the para-aortic lymph nodes:
Testis
Ovary
Uterine fundus

Many other organs may result in para-aortic nodal disease. However, these
deposits will represent a much later stage of the disease, since other nodal
stations are involved earlier.

Next question 
Display my notes on this topic

          

Save my notes

Question stats
gathered by dr. elbarky.

A 65.5%
B 8.4%
C 9%
D 6.8%
E 10.3%

65.5% of users answered this question correctly

Search eMRCS

Search term Go

 Open MRCS Part A textbook (../review/textbook.php)

External links

+ Suggest a link

Dashboard

5

Question 175 of 364

 

A 4 year old boy is brought to the clinic by his mother who has noticed a small
lesion at the external angle of his eye. On examination there is a small cystic
structure which has obviously been recently infected. On removal of the scab, there
is hair visible within the lesion. What is the most likely diagnosis?
gathered by dr. elbarky.

Dermoid cyst

Desmoid cyst

Sebaceous cyst

Epidermoid cyst

Keratoacanthoma

Dermoid cysts occur at sites of embryonic fusion and may contain multiple
cell types. They occur most often in children.

The lesion is unlikely to be a desmoid cyst as these are seldom located either at
this site or in this age group. In addition they do not contain hair. Sebaceous cysts
will usually have a punctum and contain a cheesy material. Epidermoid cysts
contain keratin plugs.

Please rate this question:

 Discuss and give feedback

Next question 

Dermoid cysts

A cutaneous dermoid cyst may develop at sites of embryonic developmental


fusion. They are most common in the midline of the neck, external angle of the eye
and posterior to the pinna of the ear. They typically have multiple inclusions such
as hair follicles that bud out from its walls. They may develop at other sites such
as the ovary and in these sites are synonymous with teratomas.

A desmoid tumour is a completely different entity and may be classified either as


low grade fibrosarcomas or non aggressive fibrous tumours. They commonly
present as large infiltrative masses. They may be divided into abdominal, extra
abdominal and intra abdominal. All types share the same biological features. Extra
abdominal desmoids have an equal sex distribution and primarily arise in the
musculature of the shoulder, chest wall, back and thigh. Abdominal desmoids
usually arise in the musculoaponeurotic structures of the abdominal wall. Intra
abdominal desmoids tend to occur in the mesentery or pelvic side walls and occur
most frequently in patients with familial adenomatous polyposis coli syndrome.
gathered by dr. elbarky.

Next question 

Display my notes on this topic

          

Save my notes

Question stats

A 61.9%
B 10.8%
C 11.1%
D 9.1%
E 7.1%

61.9% of users answered this question correctly

Search eMRCS

Search term Go

 Open MRCS Part A textbook (../review/textbook.php)


Question 176 of 364

 

A 72 year old woman with back pain and chronic renal failure has the following
blood test results:

Reference range
gathered by dr. elbarky.

Ca2+ 2.03 2.15-2.55 mmol/l

Parathyroid hormone 10.4 1-6.5 pmol/l

Phosphate 0.80 0.6-1.25 mmol/l

What is the most likely diagnosis?

Hypoparathyroidism

Primary hyperparathyroidism

Secondary hyperparathyroidism

Tertiary hyperparathyroidism

Pseudohypoparathyroidism

In relation to secondary hyperparathyroidism; there is a HIGH PTH and the Ca 2+ is


NORMAL or LOW. In secondary hyperparathyroidism there is hyperplasia of the
parathyroid glands in response to chronic hypocalcaemia (or
hyperphosphataemia) and is a normal physiological response. Calcium is released
from bone, kidneys and the gastrointestinal system.

Please rate this question:

 Discuss and give feedback

Next question 
Parathyroid glands and disorders of calcium metabolism

Hyperparathyroidism

Clinical
Disease type Hormone profile features Cause

Primary PTH May be Most cases


hyperparathyroidism (Elevated) asymptomatic if due to
Ca2+ mild solitary
(Elevated) Recurrent adenoma
Phosphate abdominal pain (80%),
(Low) (pancreatitis, multifocal
gathered by dr. elbarky.

Urine renal colic) disease


calcium : Changes to occurs in
creatinine emotional or 10-15% and
clearance cognitive state parathyroid
ratio > 0.01 carcinoma in
1% or less

Secondary PTH May have few Parathyroid


hyperparathyroidism (Elevated) symptoms gland
Ca2+ (Low or Eventually may hyperplasia
normal) develop bone occurs as a
Phosphate disease, osteitis result of low
(Elevated) fibrosa cystica calcium,
Vitamin D and soft tissue almost
levels (Low) calcifications always in a
setting of
chronic renal
failure

Tertiary Ca2+ (Normal Metastatic Occurs as a


hyperparathyroidism or high) calcification result of
PTH Bone pain and / ongoing
(Elevated) or fracture hyperplasia of
Phosphate Nephrolithiasis the
levels Pancreatitis parathyroid
(Decreased glands after
or Normal) correction of
Vitamin D underlying
(Normal or renal disorder,
decreased) hyperplasia of
Alkaline all 4 glands is
phosphatase usually the
(Elevated) cause
Differential diagnoses
It is important to consider the rare but relatively benign condition of benign familial
hypocalciuric hypercalcaemia, caused by an autosomal dominant genetic disorder.
Diagnosis is usually made by genetic testing and concordant biochemistry (urine
calcium : creatinine clearance ratio <0.01-distinguished from primary
hyperparathyroidism).

Treatment

Primary hyperparathyroidism
Indications for surgery
Elevated serum Calcium > 1mg/dL above normal
gathered by dr. elbarky.

Hypercalciuria > 400mg/day


Creatinine clearance < 30% compared with normal
Episode of life threatening hypercalcaemia
Nephrolithiasis
Age < 50 years
Neuromuscular symptoms
Reduction in bone mineral density of the femoral neck, lumbar spine, or
distal radius of more than 2.5 standard deviations below peak bone mass (T
score lower than -2.5)

Secondary hyperparathyroidism
Usually managed with medical therapy.

Indications for surgery in secondary (renal) hyperparathyroidism:


Bone pain
Persistent pruritus
Soft tissue calcifications

Tertiary hyperparathyroidism
Allow 12 months to elapse following transplant as many cases will resolve
The presence of an autonomously functioning parathyroid gland may require
surgery. If the culprit gland can be identified then it should be excised. Otherwise
total parathyroidectomy and re-implantation of part of the gland may be required.

References
1. Pitt S et al. Secondary and Tertiary Hyperparathyroidism, State of the Art
Surgical Management. Surg Clin North Am 2009 Oct;89(5):1227-39.

2. MacKenzie-Feder J et al. Primary Hyperparathyroidism: An Overview. Int J


Endocrinol 2011; 2011: 251410.

Next question 
Display my notes on this topic

          

Save my notes

Question stats
gathered by dr. elbarky.

A 7%
B 14.8%
C 53.1%
D 17.7%
E 7.5%

53.1% of users answered this question correctly

Search eMRCS

Search term Go

 Open MRCS Part A textbook (../review/textbook.php)

External links

+ Suggest a link

Dashboard

5

Question 177 of 364

 

A newborn infant is noted to have a unilateral cleft lip only. What is the most likely
explanation for this process?

Incomplete fusion of the second branchial arch


gathered by dr. elbarky.

Incomplete fusion of the nasolabial muscle rings

Incomplete fusion of the first branchial arch

Incomplete fusion of the third branchial arch

Incomplete fusion of the secondary palate

Unilateral isolated cleft lip represents a failure of nasolabial ring fusion. It is not
related to branchial arch fusion. Arch disorders have a far more profound
phenotype and malformation sequences.

Please rate this question:

 Discuss and give feedback

Next question 

Cleft lip and palate

Cleft lip and palate are the most common congenital deformity affecting the
orofacial structures. Whilst they may be an isolated developmental malformation
they are also a recognised component of more than 200 birth defects. The
incidence is as high as 1 in 600 live births. The commonest variants are:
Isolated cleft lip (15%)
Isolated cleft palate (40%)
Combined cleft lip and palate (45%)

The aetiology of the disorder is multifactorial; both genetic (affected first degree
relative increases risk) and environmental factors play a role.

Cleft lip
Cleft lip occurs as a result of disruption of the muscles of the upper lip and
nasolabial region. These muscles comprise a chain of muscles viz; nasolabial,
bilabial and labiomental. Defects may be unilateral or bilateral.

Cleft palate
The primary palate consists of all anatomical structures anterior to the incisive
foramen. The secondary palate lies more posteriorly and is sub divided into the
hard and soft palate. Cleft palate occurs as a result of non fusion of the two
palatine shelves. Both hard and soft palate may be involved. Complete cases are
associated with complete separation of the nasal septum and vomer from the
palatine processes.
gathered by dr. elbarky.

Treatment
Surgical reconstruction is the mainstay of management. The procedures are
planned according to the extent of malformation and child age. Simple defects are
managed as a single procedure. Complex malformations are usually corrected in
stages. Affected individuals have a higher incidence of hearing and speech
problems.

Next question 

Display my notes on this topic

          

Save my notes

Question stats

A 17%
B 49.3%
C 11.3%
D 8.3%
E 14%

49.3% of users answered this question correctly

Search eMRCS

Question 178 of 364

 

A 56 year old man from Egypt has suffered from recurrent attacks of haematuria
for many years. He presents with suprapubic discomfort and at cystoscopy is
found to have a mass lesion within the bladder. What is the most likely diagnosis?

Squamous cell carcinoma


gathered by dr. elbarky.

Transitional cell papilloma

Adenocarcinoma

Leiomyosarcoma

Rhabdomyosarcoma

In Egypt, Schistosomiasis is more common than in the UK and may cause


recurrent episodes of haematuria. In those affected with the condition who
develop a bladder neoplasm, an SCC is the most common type.

Please rate this question:

 Discuss and give feedback

Next question 

Bladder cancer

Bladder cancer is the second most common urological cancer. It most commonly
affects males aged between 50 and 80 years of age. Those who are current, or
previous (within 20 years), smokers have a 2-5 fold increased risk of the disease.
Exposure to hydrocarbons such as 2-Naphthylamine increases the risk. Although
rare in the UK, chronic bladder inflammation arising from Schistosomiasis
infection remains a common cause of squamous cell carcinomas, in those
countries where the disease is endemic.

Benign tumours
Benign tumours of the bladder including inverted urothelial papilloma and
nephrogenic adenoma are uncommon.
Bladder malignancies
Transitional cell carcinoma (>90% of cases)
Squamous cell carcinoma ( 1-7% -except in regions affected by
schistosomiasis)
Adenocarcinoma (2%)

Transitional cell carcinomas may arise as solitary lesions, or may be multifocal,


owing to the effect of 'field change' within the urothelium. Up to 70% of TCC's will
have a papillary growth pattern. These tumours are usually superficial in location
and accordingly have a better prognosis. The remaining tumours show either
gathered by dr. elbarky.

mixed papillary and solid growth or pure solid growths. These tumours are typically
more prone to local invasion and may be of higher grade, the prognosis is therefore
worse. Those with T3 disease or worse have a 30% (or higher) risk of regional or
distant lymph node metastasis.

TNM Staging

Stage Description

T0 No evidence of tumour

Ta Non invasive papillary carcinoma

T1 Tumour invades sub epithelial connective tissue

T2a Tumor invades superficial muscularis propria (inner half)

T2b Tumor invades deep muscularis propria (outer half)

T3 Tumour extends to perivesical fat

T4 Tumor invades any of the following: prostatic stroma, seminal vesicles,


uterus, vagina

T4a Invasion of uterus, prostate or bowel

T4b Invasion of pelvic sidewall or abdominal wall

N0 No nodal disease

N1 Single regional lymph node metastasis in the true pelvis (hypogastric,


obturator, external iliac, or presacral lymph node)

N2 Multiple regional lymph node metastasis in the true pelvis (hypogastric,


obturator, external iliac, or presacral lymph node metastasis)

N3 Lymph node metastasis to the common iliac lymph nodes


Stage Description

M0 No distant metastasis

M1 Distant disease

Presentation
Most patients (85%) will present with painless, macroscopic haematuria. In those
patients with incidental microscopic haematuria, up to 10% of females aged over
50 will be found to have a malignancy (once infection excluded).
gathered by dr. elbarky.

Staging
Most will undergo a cystoscopy and biopsies or TURBT, this provides histological
diagnosis and information relating to depth of invasion. Locoregional spread is
best determined using pelvic MRI and distant disease CT scanning. Nodes of
uncertain significance may be investigated using PET CT.

Treatment
Those with superficial lesions may be managed using TURBT in isolation. Those
with recurrences or higher grade/ risk on histology may be offered intravesical
chemotherapy. Those with T2 disease are usually offered either surgery (radical
cystectomy and ileal conduit) or radical radiotherapy.

Prognosis

T1 90%

T2 60%

T3 35%

T4a 10-25%

Any T, N1-N2 30%

Next question 

Display my notes on this topic

          
Save my notes

Question stats

A 55.4%
B 23.6%
C 7.9%
D 7.5%
E 5.7%
gathered by dr. elbarky.

55.4% of users answered this question correctly

Search eMRCS

Search term Go

 Open MRCS Part A textbook (../review/textbook.php)

External links

+ Suggest a link

Dashboard

9

Question 179 of 364

 

A 22 year old man presents with symptoms of headache, lethargy and confusion.
On examination, he is febrile and has a right sided weakness. A CT scan shows a
ring enhancing lesion affecting the left motor cortex. Which of the following is the
most likely diagnosis?
gathered by dr. elbarky.

Arteriovenous malformation

Cerebral abscess

Herpes simplex encephalitis

Metastatic renal adenocarcinoma

Glioblastoma multiforme

The combination of rapidly progressive neurology, fever and headache is highly


suggestive of cerebral abscess. CT scanning will show a ring enhancing lesion
because the intravenous contrast cannot penetrate the centre of the abscess
cavity. HSV encephalitis does not produce ring enhancing lesions.

Please rate this question:

 Discuss and give feedback

Next question 

Brain abscess

CNS abscesses may result from a number of causes including, extension of


sepsis from middle ear or sinuses, trauma or surgery to the scalp,
penetrating head injuries and embolic events from endocarditis.
The presenting symptoms will depend upon the site of the abscess (those in
critical areas e.g.motor cortex) will present earlier. Abscesses have a
considerable mass effect in the brain and raised intra cranial pressure is
common.
Although fever, headache and focal neurology are highly suggestive of a
brain abscess the absence of one or more of these does not exclude the
diagnosis, fever may be absent and even if present, is usually not the
swinging pyrexia seen with abscesses at other sites.
Assessment of the patient includes imaging with CT scanning.
Treatment is usually surgical, a craniotomy is performed and the abscess
cavity debrided. The abscess may reform because the head is closed
following abscess drainage.

Next question 
gathered by dr. elbarky.

Display my notes on this topic

          

Save my notes

Question stats

A 6.9%
B 64%
C 11.9%
D 5.5%
E 11.7%

64% of users answered this question correctly

Search eMRCS

Search term Go

 Open MRCS Part A textbook (../review/textbook.php)

External links

+ Suggest a link

Question 180 of 364

 

A 56 year old man with long standing ulcerative colitis and a DALM lesion in the
rectum is admitted with jaundice. On CT scanning the liver has 3 nodules in the
right lobe and 1 nodule in the left lobe. Carcinoembryonic antigen levels are
elevated. What is the most likely diagnosis?
gathered by dr. elbarky.

Yolk sac tumour

Hepatocellular carcinoma

Metastatic lesion

Cholangiocarcinoma

Rhabdomyosarcoma

This is likely to be due to metastatic lesions from a colonic primary. DALM lesions
should be excised by oncological colectomy for this reason. This burden of
metastatic disease is unlikely to precipitate jaundice directly and nodal disease at
the porta hepatis is the most likely cause in this case.

Please rate this question:

 Discuss and give feedback

Next question 

Liver tumours

Primary liver tumours


The most common primary tumours are cholangiocarcinoma and hepatocellular
carcinoma. Overall metastatic disease accounts for 95% of all liver malignancies
making the primary liver tumours comparatively rare.

Primary liver tumours include:


Cholangiocarcinoma
Hepatocellular carcinoma
Hepatoblastoma
Sarcomas (Rare)
Lymphomas
Carcinoids (most often secondary although primary may occur)

Hepatocellular carcinoma
These account for the bulk of primary liver tumours (75% cases). Its worldwide
incidence reflects its propensity to occur on a background of chronic inflammatory
activity. Most cases arise in cirrhotic livers or those with chronic hepatitis B
infection, especially where viral replication is actively occurring. In the UK it
accounts for less than 5% of all cancers, although in parts of Asia its incidence is
100 per 100,000.
gathered by dr. elbarky.

The majority of patients (80%) present with existing liver cirrhosis, with a mass
discovered on screening ultrasound.

Diagnosis
CT/ MRI (usually both) are the imaging modalities of choice
a-fetoprotein is elevated in almost all cases
Biopsy should be avoided as it seeds tumours cells through a resection
plane.
In cases of diagnostic doubt serial CT and αFP measurements are the
preferred strategy.

Treatment
Patients should be staged with liver MRI and chest, abdomen and pelvic CT
scan.
The testis should be examined in males (testicular tumours may cause
raised AFP). PET CT may be used to identify occult nodal disease.
Surgical resection is the mainstay of treatment in operable cases. In
patients with a small primary tumour in a cirrhotic liver whose primary
disease process is controlled, consideration may be given to primary whole
liver resection and transplantation.
Liver resections are an option but since most cases occur in an already
diseased liver the operative risks and post-operative hepatic dysfunction are
far greater than is seen following metastectomy.
These tumours are not particularly chemo or radiosensitive however, both
may be used in a palliative setting. Tumour ablation is a more popular
strategy.

Survival
Poor, overall survival is 15% at 5 years.

Cholangiocarcinoma
This is the second most common type of primary liver malignancy. As its name
suggests these tumours arise in the bile ducts. Up to 80% of tumours arise in the
extra hepatic biliary tree. Most patients present with jaundice and by this stage the
majority will have disease that is not resectable.
Primary sclerosing cholangitis is the main risk factor. In deprived countries typhoid
and liver flukes are also major risk factors.

Diagnosis
Patients will typically have an obstructive picture on liver function tests.
CA 19-9, CEA and CA 125 are often elevated
CT/ MRI and MRCP are the imaging methods of choice.

Treatment
Surgical resection offers the best chance of cure. Local invasion of peri hilar
tumours is a particular problem and this coupled with lobar atrophy will
often contra indicate surgical resection.
gathered by dr. elbarky.

Palliation of jaundice is important, although metallic stents should be


avoided in those considered for resection.

Survival
Is poor, approximately 5-10% 5 year survival.

Next question 

Display my notes on this topic

          

Save my notes

Question stats

A 6.9%
B 16.2%
C 61.6%
D 10.6%
E 4.8%

61.6% of users answered this question correctly

Search eMRCS

Question 181 of 364

 

A 22 year old man is having a blood transfusion after losing blood from a peptic
ulcer. He is normally fit and well. Four hours after the transfusion; he complains of
sudden onset shortness of breath and chest pain. On examination his temperature
is 37.2, saturations are 88% on air, blood pressure 100/55 mmHg and HR 110 bpm.
He has crepitations bilaterally up to the midzones. He is given IV furosemide, but
gathered by dr. elbarky.

deteriorates and is admitted to ITU. Invasive monitoring shows normal right heart
pressures. What is the most likely underlying process?

Acute haemolytic transfusion reaction

Delayed haemolytic transfusion reaction

Fluid overload

Transfusion associated lung injury

Graft versus host disease

The normal right heart pressures indicate this is not fluid overload. Transfusion
associated lung injury is a rare reaction causing neutrophilic mediated allergic
pulmonary oedema. Patient's have antibodies to donor leukocytes. It is important
to consider this as a diagnosis when patients don't respond to treatment for
pulmonary oedema. Patients normally respond to supportive therapy including
fluids and oxygen.

Please rate this question:

 Discuss and give feedback

Next question 

Blood transfusion reactions

Acute transfusion reactions present as adverse signs or symptoms during or


within 24 hours of a blood transfusion. The most frequent reactions are fever,
chills, pruritus, or urticaria, which typically resolve promptly without specific
treatment or complications. Other signs occurring in temporal relationship with a
blood transfusion, such as severe dyspnoea, pyrexia, or loss of consciousness may
be the first indication of a more severe potentially fatal reaction.
The causes of adverse reactions are multi-factorial. Immune mediated reactions,
some of the most feared, occur as a result of component mismatch, the
commonest cause of which is clerical error. More common, non immune mediated,
complications may occur as a result of product contamination, this may be
bacterial or viral.
Transfusion related lung injury is well recognised and there are two proposed
mechanisms which underpin this. One involves the sequestration of primed
neutrophils within the recipient pulmonary capillary bed. The other proposed
mechanism suggests that HLA mismatches between donor neutrophils and
recipient lung tissue is to blame.
gathered by dr. elbarky.

The table below summarises the main types of transfusion reaction.

Immune mediated Non immune mediated

Pyrexia Hypocalcaemia

Alloimmunization CCF

Thrombocytopaenia Infections

Transfusion associated lung injury Hyperkalaemia

Graft vs Host disease

Urticaria

Acute or delayed haemolysis

ABO incompatibility

Rhesus incompatibility

Next question 

Display my notes on this topic

          

Save my notes
Question stats

A 16.8%
B 7.7%
C 6.7%
D 62%
E 6.8%

62% of users answered this question correctly


gathered by dr. elbarky.

Search eMRCS

Search term Go

 Open MRCS Part A textbook (../review/textbook.php)

External links

+ Suggest a link

Dashboard

10

Question 182 of 364

 

A 45 year old man undergoes a surgical excision of a carotid body tumour.


Histological analysis is most likely to demonstrate which of the following?

Paraganglionoma
gathered by dr. elbarky.

Fibromatosis colli

Schwannoma

Lipoma

Neuroma

Carotid body tumours are the commonest type of head and neck
paraganglionoma.

Please rate this question:

 Discuss and give feedback

Next question 

Carotid body tumour

These are rare tumours. However, they typically account for around 60% of head
and neck paraganglionomas. They are usually tumours of middle age. Around 5%
are bilateral and around 5% are malignant. They are rarely found as part of the
MEN II or neurofibromatosis type I disease complexes.

They typically present as an asymptomatic neck mass in the anterior triangle of


the neck. They are typically slow growing lesions.

Types of carotid body tumour


Sporadic - Accounts for 85% of cases
Familial - Seen in around 10% of cases and usually in younger patients
Hyperplastic - Seen in those at high altitude or in those with COPD

Imaging
They are readily imaged using duplex ultrasonography. CT angiography is
sometimes helpful.

Treatment
Typically this comprises surgical resection. This is preceded by embolization in
gathered by dr. elbarky.

selected cases.

Next question 

Display my notes on this topic

          

Save my notes

Question stats

A 54.2%
B 11.8%
C 13.6%
D 5.3%
E 15%

54.2% of users answered this question correctly


Question 183 of 364

 

A 56 year old man presents with epigastric discomfort and episodes of migratory
thrombophlebitis. On examination he is mildly jaundiced. A CT scan shows
lymphadenopathy at the porta hepatis and a mass in the pancreatic head. Which
of the following is the most likely underlying diagnosis?
gathered by dr. elbarky.

Squamous cell carcinoma of the pancreas

Adenocarcinoma of the pancreas

Insulinoma

Glucagonoma

Gastrinoma

Adenocarcinoma of the pancreas is the most likely diagnosis and migratory


thrombophlebitis is associated with the condition. Squamous cells carcinoma is
extremely uncommon in the pancreas. Gastrinoma are extremely rare and thus not
the most likely diagnosis.

Please rate this question:

 Discuss and give feedback

Next question 

Pancreatic cancer

Adenocarcinoma
Risk factors: Smoking, diabetes, adenoma, familial adenomatous polyposis
Mainly occur in the head of the pancreas (70%)
Spread locally and metastasizes to the liver
Carcinoma of the pancreas should be differentiated from other
periampullary tumours with better prognosis

Clinical features
Weight loss
Painless jaundice
Epigastric discomfort (pain usually due to invasion of the coeliac plexus is a
late feature)
Pancreatitis
Trousseau's sign: migratory superficial thrombophlebitis

Investigations
USS: May miss small lesions
CT Scanning (pancreatic protocol). If unresectable on CT then no further
staging needed
gathered by dr. elbarky.

PET/CT for those with operable disease on CT alone


ERCP/ MRI for bile duct assessment
Staging laparoscopy to exclude peritoneal disease

Management
Head of pancreas: Whipple's resection (SE dumping and ulcers). Newer
techniques include pylorus preservation and SMA/ SMV resection
Carcinoma body and tail: poor prognosis, distal pancreatectomy, if operable
Usually adjuvent chemotherapy for resectable disease
ERCP and stent for jaundice and palliation
Surgical bypass may be needed for duodenal obstruction

Next question 

Display my notes on this topic

          

Save my notes

Question stats

A 7.9%
B 58.4%
C 8.1%
D 16.2%
E 9.4%

Question 184 of 364

 

Which of these lesions is most closely associated with Barretts oesophagus?

Squamous cell carcinoma

Gastro intestinal stromal tumours


gathered by dr. elbarky.

Carcinoid tumours

Leiomyosarcoma

Adenocarcinoma

Barretts oesophagus is most closely associated with adenocarcinoma, and it


confers a 30 fold increased risk of developing the condition.

Please rate this question:

 Discuss and give feedback

Next question 

Oesophageal cancer

Incidence is increasing
In most cases in the Western world this increase is accounted for by a rise
in the number of cases of adenocarcinoma. In the UK adenocarcinomas
account for 65% of cases.
Barretts oesophagus is a major risk factor for most cases of oesophageal
adenocarcinoma.
In other regions of the world squamous cancer is more common and is
linked to smoking, alcohol intake, diets rich in nitrosamines and achalasia.
Surveillance of Barretts is important, as it imparts a 30 fold increase in
cancer risk and if invasive malignancy is diagnosed early then survival may
approach 85% at 5 years.

Diagnosis
Upper GI endoscopy is the first line test
Contrast swallow may be of benefit in classifying benign motility disorders
but has no place in the assessment of tumours
Staging is initially undertaken with CT scanning of the chest, abdomen and
pelvis. If overt metastatic disease is identified using this modality then
further complex imaging is unnecessary
If CT does not show metastatic disease, then local stage may be more
accurately assessed by use of endoscopic ultrasound.
Staging laparoscopy is performed to detect occult peritoneal disease. PET
CT is performed in those with negative laparoscopy. Thoracoscopy is not
routinely performed.
gathered by dr. elbarky.

Treatment
Operable disease is best managed by surgical resection. The most standard
procedure is an Ivor- Lewis type oesophagectomy. This procedure involves the
mobilisation of the stomach and division of the oesophageal hiatus. The abdomen
is closed and a right sided thoracotomy performed. The stomach is brought into
the chest and the oesophagus mobilised further. An intrathoracic
oesophagogastric anastomosis is constructed. Alternative surgical strategies
include a transhiatal resection (for distal lesions), a left thoraco-abdominal
resection (difficult access due to thoracic aorta) and a total oesophagectomy
(McKeown) with a cervical oesophagogastric anastomosis.
The biggest surgical challenge is that of anastomotic leak, with an intrathoracic
anastomosis this will result in mediastinitis. With high mortality. The McKeown
technique has an intrinsically lower systemic insult in the event of anastomotic
leakage.

In addition to surgical resection many patients will be treated with adjuvant


chemotherapy.

Next question 

Display my notes on this topic

          

Save my notes

Question stats

A 19.8%
B 6.5%
C 5.2%
D 5.6%
E 62.9%

62.9% of users answered this question correctly

Search eMRCS
gathered by dr. elbarky.

Search term Go

 Open MRCS Part A textbook (../review/textbook.php)

External links

+ Suggest a link

Dashboard

10

11

12

13

Question 185 of 364

 

A 50 year old male presents with painless frank haematuria. Clinical examination is
unremarkable. Routine blood tests reveal a haemoglobin of 18g/dl but are
otherwise normal. What is the most likely underlying diagnosis?

Squamous cell carcinoma of the bladder


gathered by dr. elbarky.

Adenocarcinoma of the prostate

Adenocarcinoma of the kidney

Wilms tumour

Transitional cell carcinoma of the renal pelvis

Polycythaemia is a recognised feature of renal cell carcinoma. Wilms tumours


most commonly occur in children.

Please rate this question:

 Discuss and give feedback

Next question 

Haematuria

Causes of haematuria

Trauma Injury to renal tract


Renal trauma commonly due to blunt injury
(others penetrating injuries)
Ureter trauma rare: iatrogenic
Bladder trauma: due to RTA or pelvic fractures

Infection Remember TB
Malignancy Renal cell carcinoma (remember paraneoplastic
syndromes): painful or painless
Urothelial malignancies: 90% are transitional cell
carcinoma, can occur anywhere along the urinary
tract. Painless haematuria.
Squamous cell carcinoma and adenocarcinoma:
rare bladder tumours
Prostate cancer
Penile cancers: SCC

Renal disease Glomerulonephritis


gathered by dr. elbarky.

Stones Microscopic haematuria common

Structural Benign prostatic hyperplasia (BPH) causes


abnormalities haematuria due to hypervascularity of the
prostate gland
Cystic renal lesions e.g. polycystic kidney disease
Vascular malformations
Renal vein thrombosis due to renal cell carcinoma

Coagulopathy Causes bleeding of underlying lesions

Drugs Cause tubular necrosis or interstitial nephritis:


aminoglycosides, chemotherapy
Interstitial nephritis: penicillin, sulphonamides,
and NSAIDs
Anticoagulants

Benign Exercise

Gynaecological Endometriosis: flank pain, dysuria, and


haematuria that is cyclical

Iatrogenic Catheterisation
Radiotherapy; cystitis, severe haemorrhage,
bladder necrosis

Pseudohaematuria For example following consumption of beetroot

References
Http://bestpractice.bmj.com/best-practice/monograph/316/overview
/aetiology.html
Next question 

Display my notes on this topic

          

Save my notes
gathered by dr. elbarky.

Question stats

A 13.6%
B 7.3%
C 50%
D 8.4%
E 20.7%

50% of users answered this question correctly

Search eMRCS

Search term Go

 Open MRCS Part A textbook (../review/textbook.php)

External links

+ Suggest a link

Dashboard

3

Question 186 of 364

 

A 32 year old lady presents with a 1.5cm pigmented lesion on her back. The
surgeon is concerned that this may be a melanoma. What is the most appropriate
course of action?

2mm punch biopsy from the centre of the lesion


gathered by dr. elbarky.

4mm punch biopsy from the centre of the lesion

Wide excision of the lesion with 3cm margins

Excisional biopsy of the lesion

Wide excision of the lesion with 1cm margins

Suspicious naevi should NOT be partially sampled as histological


interpretation is severely compromised. Complete excision is mandatory
where lesions fulfil diagnostic criteria. However, wide excision for margins
may be deferred until definitive histology is available.

Lesions that are suspicious for melanoma should be excised with complete
margins. Radical excision is not routinely undertaken for diagnostic purposes and
therefore if subsequent histopathological assessment determines that the lesion
is a melanoma a re-exicision of margins may be required. Incisional punch
biopsies of potential melanomas makes histological interpretation difficult and is
best avoided.

Please rate this question:

 Discuss and give feedback

Next question 

Malignant melanoma
The main diagnostic features (major Secondary features (minor

Treatment
Suspicious lesions should undergo excision biopsy. The lesion should be
removed in completely as incision biopsy can make subsequent
gathered by dr. elbarky.

histopathological assessment difficult.


Once the diagnosis is confirmed the pathology report should be reviewed to
determine whether further re-excision of margins is required (see below):

Margins of excision-Related to Breslow thickness

Lesions 0-1mm thick 1cm

Lesions 1-2mm thick 1- 2cm (Depending upon site and pathological features)

Lesions 2-4mm thick 2-3 cm (Depending upon site and pathological features)

Lesions >4 mm thick 3cm

Marsden J et al Revised UK guidelines for management of Melanoma. Br J


Dermatol 2010 163:238-256.

Further treatments such as sentinel lymph node mapping, isolated limb perfusion
and block dissection of regional lymph node groups should be selectively applied.

Next question 

Display my notes on this topic

          

Save my notes

Question stats
A 7.9%
B 7.6%
38.8% of users answered this question correctly
C 17.3%
D 38.8%
E 28.4%
Search eMRCS

Search term Go
gathered by dr. elbarky.

 Open MRCS Part A textbook (../review/textbook.php)

External links

+ Suggest a link

Dashboard

10

11

12

13

14

15

Question 187 of 364

 

At which of the following sites is the development of diverticulosis least likely?

Caecum

Ascending colon
gathered by dr. elbarky.

Transverse colon

Sigmoid colon

Rectum

Rectal involvement with diverticular disease almost never occurs.

Because the rectum has a longitudinal muscle coat (blending of of the tenia marks
the recto-sigmoid junction), diverticular disease almost never occurs here. Right
sided colonic diverticular disease is well recognised (though less common than
left sided).

Please rate this question:

 Discuss and give feedback

Next question 

Diverticular disease

Diverticular disease is a common surgical problem. It consists of herniation of


colonic mucosa through the muscular wall of the colon. The usual site is between
the taenia coli where vessels pierce the muscle to supply the mucosa. For this
reason, the rectum, which lacks taenia, is often spared.

Symptoms
Altered bowel habit
Bleeding
Abdominal pain

Complications
Diverticulitis
Haemorrhage
Development of fistula
Perforation and faecal peritonitis
Perforation and development of abscess
Development of diverticular phlegmon

Diagnosis
gathered by dr. elbarky.

Patients presenting in clinic will typically undergo either a colonoscopy, CT


cologram or barium enema as part of their diagnostic work up. All tests can
identify diverticular disease. It can be far more difficult to confidently exclude
cancer, particularly in diverticular strictures.

Acutely unwell surgical patients should be investigated in a systematic way. Plain


abdominal films and an erect chest x-ray will identify perforation. An abdominal CT
scan (not a CT cologram) with oral and intravenous contrast will help to identify
whether acute inflammation is present but also the presence of local
complications such as abscess formation.

Severity Classification- Hinchey

I Para-colonic abscess

II Pelvic abscess

III Purulent peritonitis

IV Faecal peritonitis

Treatment
Increase dietary fibre intake.
Mild attacks of diverticulitis may be managed conservatively with
antibiotics.
Peri colonic abscesses should be drained either surgically or radiologically.
Recurrent episodes of acute diverticulitis requiring hospitalisation are a
relative indication for a segmental resection.
Hinchey IV perforations (generalised faecal peritonitis) will require a
resection and usually a stoma. This group have a very high risk of post
operative complications and usually require HDU admission.

Next question 
Display my notes on this topic

          

Save my notes

Question stats
gathered by dr. elbarky.

A 21.4%
B 9.1%
C 9.6%
D 8.5%
E 51.3%

51.3% of users answered this question correctly

Search eMRCS

Search term Go

 Open MRCS Part A textbook (../review/textbook.php)

External links

+ Suggest a link

Dashboard

You might also like